Download CONTENTS

Document related concepts

Drosophila embryogenesis wikipedia , lookup

Vertebra wikipedia , lookup

Cell nucleus wikipedia , lookup

Primate basal ganglia system wikipedia , lookup

Myocyte wikipedia , lookup

Anatomical terminology wikipedia , lookup

Nervous system wikipedia , lookup

Anatomical terms of location wikipedia , lookup

Transcript
CONTENTS
Chapter 1 Osteo1ogy.
.
.
.
.
.
.
.
.
.
.
.
..
.
.
.
.
.
.
.
.
.
.
.
.
.
......
....
....
....
....
.
......
..1
Chapter 2 Arthrology.
.
.
.
.
.
.
.
.
.
.
..
.
.
.
.
.
.
.
.
.
.
.
.
.
.
.
.
.
.
.
.
...
....
....
....
.
......
..14
Chapter 3 Myo1ogy.
.
.
.
.
.
.
.
.
.
.
.
..
.
.
.
.
.
.
.
.
.
.
.
.
.
.
.
.
.
.
.
.
.
.
.
.
.
.
.
.
.
.
.
.
.
...
.
......
.23
Chapter 4-5 The General Description of Splanchnology & The Alimentary System.
.
....
.
.34
Chapter 6 The Respiratory System..
.
.
.
.
.
.
.
.
.
.
.
.
.
.
.
.
.
.
.
.
.
.
.
.
.
.
.
.
.
.
.
.
.
.
..
.
......
..45
Chapter 7 The Urinary System.
.
.
..
.
.
.
.
.
.
.
.
.
.
.
.
.
.
.
.
.
.
.
.
.
.
.
.
.
.
.
.
.
.
.
.
.
...
.
......
..56
Chapter 8 The Male Reproductive System.
.
.
.
.
.
.
.
.
.
.
.
.
.
.
.
.
.
.
.
.
.
.
.
.
.
.
.
.
.
..
.
......
.
.70
Chapter 9 The Female Reproductive Organs...
....
.
......
....
....
....
..
..
.
......
..85
Chapter 10 The Peritoneum.
.
.
.
.
.
..
.
.
.
.
.
.
.
.
.
.
.
.
.
。
.
.
.
.
.
.
.
.
.
.
.
.
.
.
.
.
.
....
.
......
.101
Chapter 11 The Cardiovascular System.
.
.
.
.
.
.
.
.
.
.
.
.
.
.
.
.
.
.
.
.
.
.
.
.
.
.
.
.
..
....
.
.....
.107
Chapter 12 The Lymphatic System..
.
.
.
.
.
.
.
.
.
.
.
.
.
.
.
.
.
.
.
.
.
.
.
.
.
.
.
.
.
.
.
.
.
.
.
.
.
.
....
.128
Chapter 13-15 The Sensory Organs.
....
....
....
...
......
....
....
....
....
.
.....
.134
Chapter 16 The General Description of Nervous System.
.
.
.
.
.
.
.
.
.
.
.
.
.
.
.
.
.
..
......
..144
Chapter 17 The Central Nervous System.
.
.
.
.
.
.
.
.
.
.
.
.
.
.
.
.
.
.
.
.
.
.
.
.
.
.
.
.
....
.
......
.152
Chapter 18 The Peripheral Nervous System.
.
.
.
.
.
.
.
.
.
.
.
.
.
.
.
.
.
.
.
.
.
.
.
.
.
.
.
.
.
.
.
......
.208
Chapter 19 The Nervous Pathways.......
....
....
......
....
....
....
....
.
......
.234
Chapter 20 Meninges and B1ood Vessels of Brain and Spinal Cord,and the Cerebrospinal
Fluid.
.
.
.
.
.
.
.
.
.
.
.
.
.
.
..
..
......
....
....
....
....
.
......
....
....
....256
Chapter 21 The Endocrine System..
.
.
.
.
.
.
.
.
.
.
.
.
.
.
.
.
.
.
..
....
....
..
..
....
.
......
.268
Chapter 1
Osteology
Multiple Choice Questions
A1
1.In adults,the radius is to the ulna,as the
.
A.tibia is to the femur
B.fibula is to the ulna
C.fibula is to the tibia
D.humerus is to the femur
E.None of the above
2.Which of the following bones is/are not part of the pectoral girdle and its appendages?
A.humerus
B.carpals
C.phalanges
D.ulna
E.coccyx
3.The anatomical neck of the humerus 1ies
.
A.where the expanded upper end joins the shaft
B.between the greater and lesser tuberosities
C.at the deltoid tuberosity
D.immediately below the head
E.just above the bicipital groove
4.The medial surface of the calcaneus has a large shelf-like process called the
A.peroneal tubercle
B.anterior tubercle
C.sustentaculum tali
D.medial tubercle
E.lateral tubercle
5.The styloid process projects downward and forward from the
.
A.occipital bone
B.temporal bone
C.sphenoid bone
D.palatine bone
E.hyoid bone
6.The optic canal is an opening in the
.
A.frontal bone
B.occipital bone
C.petrous part of the temporal bone
D.lesser wing of the sphenoid bone
E.squamous part of the temporal bone
7.The hypoglossal canal is located in the
.
A.squamous part of the temporal bone
B.frontal bone
C.occipital bone
.
D.sphenoid bone
E.parietal bone
8.The foramen rotundum is located in the
.
A.greater wing of the sphenoid bone
B.frontal bone
C.petrous part of the temporal bone
D.occipital bone
E.lesser wing of the sphenoid bone
9.The orbital margins are bounded by the frontal bone superiorly,the____________bone
laterally,the maxilla and zygomaric bone inferiorly,and the processes of the maxilla
and frontal bone medially.
A.greater wing of the sphenoid
B.maxilla
C.parietal
D.lesser wing of the sphenoid
E.zygomatic
10.The carotid canal is located in the
.
A.frontal bone
B.petrous part of the temporal bone
C.occipital bone
D.greater wing of the sphenoid bone
E.parietal bone
11.The ophthalmic division of the trigeminal nerve leaves the skull through the
.
A.pterygopalatine foramen
B.foramen ovale
C.foramen rotundum
D.superior orbital fissure
E.inferior orbital fissure
12.The maxillary division of the trigeminal nerve leaves the skull through the
.
A.foramen spinosum
B.foramen rotundum
C.superior orbital fissure
D.foramen ovale
E.jugular foramen
13.The foramen magnum is located in the
.
A.sphenoid bone
B.temporal bone
C.parietal bone
D.frontal bone
E.occipital bone
14.The seventh cervical vertebra is characterized by having
.
A.an odontoid process
B.a large transverse foramen
C.a heart-shaped body
D.a massive body
E.the longest spinous process
15.The sixth thoracic vertebra is characterized by
.
A.its heart-shaped body
B.its bifid spinous process
C.its massive body
D.having the superior articular processes facing medially and those of the inferior
articular process facing laterally
E.its thick lamina
16.The characteristic feature of the second cervical vertebra is its
.
A.absent body
B.odontoid process
C.heart-shaped body
D.massive body
E.bifid spinous process
17.The characteristic feature of the first cervical vertebra is its
.
A.odontoid process
B.massive body
C.absent body
D.long spinous process
E.absent foramen transversarium
18.The characteristic feature of the fifth lumbar vertebra is its
.
A.heart-shaped body
B.rounded vertebral foramen
C.small pedicles
D.massive body
E.short and thick transverse process
19.The costal margin is formed by the
.
A.sixth,eighth,and tenth ribs
B.inner margins of the first ribs
C.edge of the xiphoid process
D.costal cartilages of the seventh,eighth,ninth,and tenth ribs
E.costal cartilages of the seventh to the tenth ribs and the ends of the cartilages of the
eleventh and twelfth ribs
20.The clavicle articulates with the
laterally.
A.coracoid process
B.superior angle of the scapula
C.acromion
D.base of the spine of the scapula
E.glenoid fossa
21.The foramen spinosum is located in the
.
A.sphenoid bone
B.occipital bone
C.frontal bone
D.petrous part of the temporal bone
E.squamous part of the temporal bone
22.The lateral end of the spine of scapula forms the
.
A.acromion
B.glenoid fossa
C.coracoid process
D.suprascapular notch
E.superior angle of the scapula
23.The scapula is a flat triangular bone that lies on the posterior chest wall between the
.
A.first and sixth ribs
B.second and seventh ribs
C.third and eighth ribs
D.fourth and ninth ribs
E.fifth and tenth ribs
24.Above the trochlea of the humerus anteriorly is the
.
A.coronoid fossa
B.radial fossa
C.capitulum
D.olecranon fossa
E.deltoid tuberosity
25.The dorsal tubercle of the radius is situated
.
A.just below the head
B.on the posterior surface of the distal end
C.on the lateral margin of the distal end
D.on the interosseous border
E.halfway down on the lateral side of the shaft
26.The olecranon process of the ulna lies
.
A.at the distal end of the bone
B.halfway down the lateral border
C.halfway down the medial border
D.at the proximal end
E.below the radial notch
27.The following bones form the proximal row of carpal bones except
.
A.lunate
B.pisiform
C.scaphoid
D.triquetral
E.trapezium
28.Which one of the following is not a part of the appendicular skeleton?
A.ribs
B.fibula
C.humerus
D.scapula
E.radius
29.The following statements concening the bony pelvis are correct except which one?
A.when the patient is in the standing position,the anterior superior iliac spines lie
vertically above the anterior surface of the symphysis pubis
B.very little movement is possible at the sacrococcygeal joint
C.the false pelvis helps guide the fetus into the true pelvis during labor
D.the female sex hormones cause a relaxation of the ligaments of the pelvis
during pregnancy
E.obliteration of the cavity of the sacroiliac joint often occurs in both sexes after
middle age
30.Clinicians define the thoracic outlet as
.
A.the lower opening in the thoracic cage
B.the gap between the crura of the diaphragm
C.the esophageal opening in the diaphragm
D.the upper opening in the thoracic cage
E.the gap between the sternal and costal origins of the diaphragm
31.The posterior surface of the tibia shows an oblique line for the attachment of the soleus muscle.
The line is referred to as the
.
A.interosseous line
B.popliteal line
C.marginal 1ine
D.soleal line
E.malleolar 1ine
32.At the junction of the anterior border of the tibia with the upper end is the
for the attachment of the patellar ligament
A.tibial plateau
B.medial condyle
C.intercondylar eminence
D.medial malleolus
E.tibial tuberosity
33.The lower end of the fibula forms the triangular
.
A.lateral malleolus
B.medial malleolus
C.styloid process
D.interosseous border
E.malleolar fossa
34.The head of the talus has an oval convex articular surface for articulation with the
.
A.cuboid bone
B.medial cuneiform bone
C.intermediate cuneiform bone
D.navicular bone
E.calcaneum
35.The two parietal bones articulate with each other in the midline at the
.
A.lambdoid suture
B.sagittal suture
C.coronal suture
D.squamotympanic suture
E.pterion
36.The fifth metatarsal bone has a prominent tubercle on its base for the attachment of the
.
A.peroneus brevis muscle
B.flexor digitorum longus muscle
C.flexor digiti minimi brevis
D.peroneus longus muscle
E.peroneus tertius muscle
37.Which of the following is not a function of bones?
A.providing protection
B.providing support
C.allowing f1exible and contro1led movement
D.providing a storehouse of inorganic salts
E.providing a non living "concrete" architecture
38.Which statement is not true about the development of bone in humans?
A.bone breakdown and replacement occur throughout life
B.a primary ossification center forms in the middle of a long bone
C.a secondary ossification center forms at the ends of a 1ong bone
D.bones may form within cartilage model or within a membrane matrix
E.a cartilaginous disk remains between the primary and secondary ossification centers
until old age
39.The bones of the wrist are called the
.
A.tarsal
B.carpals
C.metatarsals
D.metacarpals
E.phalanges
40.Which of the following bones is/are not part of the axial skeleton?
A.ribs
B.skull
C.sternum
D.scapula
E.vertebrae
41.Which of the following is not true about the vertebrae?
A.They help protect the spinal cord
B.They are part of the appendicular skeleton
C.They form the dorsal backbone
D.They help form 4 curvatures of the spine
E.They are separated by intervertebral discs
42.The correct order of regions of vertebra is
.
A.thoracic-lumbar-sacrum-cervical-coccyx
B.cervical-lumbar-sacrum-thoracic-coccyx
C.cervical-thoracic-lumbar-sacrum-coccyx
D.lumbar-sacrum-cervical-thoracic-coccyx
E.sacrum-cervical-thoracic-lumbar-coccyx
A2
1.A1though both tympanic membranes appear normal,fluid is observed in both middle ears.The
right ear also suffers a moderate hearing loss. Ear involvement in this case may best be
explained by spread of infection from the tonsils via the
.
A. facial canal
B. foramen ovale
C. pharyngotympanic tube
D. external auditory meatus
E. internal auditory meatus
2.A 26-year-old accountant trips over a brief case and falls onto his outstretched hand. You
suspect a fracture of a carpal bone. Which of the following is most likely fractured?
A. pisiform
B. lunate
C. triquetrum
D. scaphoid
E. capitate
3.An 18-year-old boy was suspected of having leukemia. It was decided to confirm the diagnosis
by performing a bone marrow biopsy. The following statements concerning this procedure are
correct except which one?
A. Red bone marrow specimens can be obtained from the sternum or the iliac crests
B. The biopsy was taken from the lower end of the tibia
C. At birth, the marrow of all bones of the body is red and hematopoietic
D. The blood-forming activity of bone marrow in many long bones gradually lessens
with age, and the red marrow is gradually replaced by yellow marrow
E. By the time the person becomes adult, red marrow is restricted to the bones of the skull, the
vertebral column, the thoracic cage, the girdle bones, and the head of the humerus and
femur
4.During a little league game one of the players accidently struck her teammate with her bat.
When the bat hit the boy' s arm there was an audible crack and the boy fell to the ground in pain.
Luckily, one of the mothers watching was a physician and she rushed to the boy' s aid. Quick
examination showed that the boy had a closed fracture in his upper arm. After watching him
abduct and rotate the arm, she diagnosed damage to the axillary nerve. Damage to this nerve
normally occurs when injury is located at the
.
A. lateral epicondyle of the humerus
B. medial epicondyle of the humerus
C. anatomical neck of the humerus
D. surgical neck of the humerus
E. deltoid tuberosity of the humerus
5.A heavily built, middle-aged man running down a flight of stone steps misjudged the position of
one of the steps and fell suddenly onto his buttocks. Following the fall, he complained of severe
bruising of the area of the cleft between the buttocks and persistent pain in this area. The
following statements concerning this patient are correct except which?
A. The lower end of the vertebral column was traumatized by the stone step
B. The coccyx can be palpated beneath the skin in the natal cleft
C. The anterior surface of the coccyx cannot be felt clinically
D. The coccyx is usually severely bruised or fractured
E. The pain is felt in the distribution of dermatomes S4 and S5
6.An elderly woman was run over by an automobile as she was crossing the road. Radiographic
examination of the pelvis in the Emergency Department of the local hospital revealed a fracture
of the ilium and iliac crest on the left side. The following statements about fractures of the
pelvis are correct except which?
A. Fractures of the ilium have little displacement
B. Displacement is prevented by the presence of the iliacus and the gluteal muscles on the inner
and outer surfaces of this bone, respectively
C. If two fractures occur in the ring forming the true pelvis, the fracture will be unstable and
displacement will occur
D. Fractures of the true pelvis do not cause injury to the pelvic viscera
E. The post-vertebral and abdominal muscles are responsible for elevating the lateral part of the
pelvis should two fractures occur
7.A pregnant woman visited an antenatal clinic. A vaginal examination revealed that the sacral
promontory could be easily palpated and that the diagonal conjugate measured less than 10 cm.
The following statements concerning this examination are correct except which?
A. Normally it is difficult or impossible to feel the sacral promontory by means of a vaginal
examination
B. The normal diagonal conjugate measures about 25 cm
C. This patient's pelvis was flattened anteroposteriorly, and the sacral promontory projected too
far forward
D. It is likely that this patient would have an obstructed labor
E. This patient was advised to have a cesarean section
8.A 26-year-old man was running across a field when he caught his right foot in a rabbit hole. As
he fell, the right foot was violently rotated laterally and overeverted. On attempting to stand, he
could place no weight on his right foot. On examination by a physician, the right ankle was
considerably swollen, especially on the lateral side. After further examination, including a
radiograph of the ankle, a diagnosis of severe fracture dislocation of the ankle joint was made.
The following statements concerning this patient are correct except which?
A. This type of fracture dislocation is caused by forced external rotation and overexertion of the
foot.
B. The talus is externally rotated against the lateral malleolus of the fibula, causing it to
fracture.
C. The torsion effect on the lateral malleolus produces a spiral fracture.
D. The medial ligament of the ankle joint is strong and never ruptures.
E. If the talus is forced to move farther laterally and continues to rotate, the posterior inferior
margin of the tibia will be sheared off.
9.Infection may spread from the nasal cavity to the meninges along the olfactory nerves.
Olfactory fibers pass from the mucosa of the nasal cavity to the olfactory bulb via the
.
A. semilunar hiatus
B. cribriform plate of the ethmoid bone
C. anterior and posterior ethmoidal foramina
D. sphenopalatine foramen
E. nasociliary nerve
10.A 23-year-old college student is being seen for possible sinusitis. The physician sees purulent
drainage arising from the superior nasal meatus. Which of the following sinuses is likely to be
infected?
A. frontal
B. left maxillary
C. sphenoidal
D. ethmoidal
E. right maxillary
B1
A. compact bone
B. periosteum
C. medullary cavity
D. articular cartilage
E. endosteum
1. The membrane covering a bone, that assumes a role in fracture repair is the
.
2. The portion of a long bone that stores yellow marrow in adults is the
.
A. ulna
B. radius
C. humerus
D. tibia
E. fibula
3. The medial bone of the forearm, in the anatomical position, is the
.
4. The lateral bone of the leg is the
.
A. mandible
B. sternum
C. rib
D. clavicle
E. frontal bone
5. Which of the above is not axial?
.
6. Which of the above belongs to the facial bone?
A. inferior
B. proximal
C. medial
D. distal
E. lateral
7. The clavicle is
to the head.
8. The tibia is
to the fibula.
A. spongy bone
B. periosteum
C. marrow
D. compact bone
E. medullary cavity
9. The second structure encountered by a pin entering the shaft of a long bone would be
the
.
10. Flat bones lack
.
True or False Questions
1. The axial portion of the body includes the arms and legs.
(
)
2. Yellow bone marrow functions in the formation of blood cells.
(
)
3. The long bone in the arm forms by intramembranous ossification.
(
)
4. The appendicular skeleton consists of the parts that support and protect the head, neck, and
trunk.
(
)
5. Red bone marrow is normally present in the epiphyses of all long bones.
(
)
6. The hollow cylindrical portion of a long bone is called the diaphysis.
(
)
7. Bone is a hard, rigid tissue because of calcium salts deposited in its matrix.
(
)
8. Each individual bone contains either red marrow or yellow marrow, but not both..
(
)
Explanation of Terms
1. anatomical position
2. sternal angle
3. pterion
4. carrying angle
5. anterior fontanelle
6. terminal line
Answer the Following Questions
1. List the bones of the skull that are paired. Which are unpaired?
2. List four types of bones based on shape and give an example of each type.
ANSWERS
Multiple Choice Questions
A1
1. C 2. E 3. D 4. C 5. B 6. D 7. C 8. A 9. E 10. B 11. D 12. B
13. E 14. E
15. A 16. B 17. C
18. D 19. E
20. C 21. A 22. A
23. B
24. A 25. B 26. D 27. E 28. A 29. B 30. D 31. D 32. E 33. A 34. D
35. B 36. A 37. E 38. E 39. B 40. D
41. B 42. C
A2
1. C 2. D
3. B
4. D
5. C
6. D
7. B
8. D
9. B 10. D
B1
1. B
2. C
3. A
4. E
5. D
6. A
7. A
8. C
9. D
10. E
True or False Questions
1. F
2. F
3. F
4. F
5. F
6. T
7. T
8. F
Explanation of Terms
1. This is a reference position that allows for the use of consistent directional terminology. All
descriptions of location are made from within anatomical position. The person is standing erect,
palms facing anteriorly, feet together and facing forward.
2. The sternal angle is the angle formed by the junction of the manubrium and the body of the
sternum in the form of a secondary cartilaginous joint (symphysis). This is also called the
manubriosternal joint or Angle of Louis. The sternal angle is a palpable clinical landmark. It
marks the approximate level of the 2nd pair of costal cartilages and the level of the
intervertebral disc between T4 and T5. The angle is 140 degrees.
3. The point corresponding with the posterior end of the sphenoparietal suture is named the
pterion. It is situated about 3 cm behind, and a little above the level of the zygomatic process
of the frontal bone. It marks the junction between four bones: parietal bone, temporal bone,
sphenoid bone and frontal bone. The pterion is known as the weakest part of the skull.
Clinically, the pterion is relevant because the middle meningeal artery runs beneath it, on the
inner side of the skull, which is quite thin at this point.
4. When the arm is extended, with the palm facing forward or up, the bones of the humerus and
。
forearm are not perfectly aligned. The deviation from a straight line (generally on the order of 5
。
-10 ) occurs in the direction of the thumb, and is referred to as the carrying angle. In females the
carrying angle is greater than in males. The carrying angle can influence how objects are held
by individuals--those with a more extreme carrying angle may be more likely to supinate the
forearm when holding objects in the hand to keep the elbow closer to the body.
5. The largest one of all fontanelles.
Location: the junction of the sagittal and coronal sutures
Shape: diamond-shaped
Size: about 4cm anteroposteriorly and 2. 5cm transversely.
The fontanelle is closed at about the middle of the second year.
6. It is a circular line, formed by the promontory of the sacrum, the anterior border of the sacrum
ala, the arcuate line of ilium, the pecten pubis, the pubic crest and the upper border of the pubic
symphysis. Whole pelvis is divided into a greater pelvis and a lesser pelvis by the terminal line.
Answer the Following Questions
1. Paired: the parietal, temporal, maxillae, palatine, zygomatic, nasal, lacrimal bones and the
inferior nasal conchae, pair
Unpaired: the frontal, occipital, sphenoid, ethmoid, vomer, mandible and the hyoid
2. Long bone: humerus
Short bone: scaphoid bone
Flat bone: parietal bone
Irregular bone: maxilla
(Pan Feng)
Chapter2
Arthrology
Multiple Choice Questions
A1
1. The joint that links the bones of trunk and upper limb is
A. pubic symphysis
B. shoulder joint
.
C. acromioclavicular joint
D. sternoclavicular joint
E. hip joint
2. A patient is performing the movement of flexion of the hip joint when she
.
A. moves the lower limb away from the midline in the coronal plane
B. moves the lower limb posteriorly in the paramedian plane
C. moves the lower limb anteriorly in the paramedian plane
D. rotates the lower limb so that the anterior surface faces medially
E. moves the lower limb toward the median sagittal plane
3. Inversion of the foot is the movement so that the sole faces
.
A. downward and posteriorly
B. medially
C. laterally
D. downward
E. downward and laterally
4. The joint possessing an articular disc is the
.
A. shoulder joint
B. sternoclavicular joint
C. elbow joint
D. hip joint
E. ankle joint
5. The bone which is not consisted in the wrist joint is
.
A. lunate bone
B. triquetral bone
C. scaphoid bone
D. pisiform bone
E. inferior extremity of radius
6. Which one of the following joints has no articular disc or meniscus?
A. knee joint
B. sternoclavicular joint
C. temporomandibular joint
D. shoulder joint
E. radiocarpal joint
7. The following statements concerning an intervertebral disc are correct except which?
A. The atlantoaxial joint possesses no discs.
B. The discs are the thickest in the lumbar region.
C. The nucleus pulposus is moist likely to herniate in an anterolateral direction.
D. The discs play major role in the development of the curvatures of the vertebral column.
E. During aging, the fluid within the nucleus pulposus is replaced by fibrocartilage.
8. The bones consisted of knee joint are
.
A. femur and tibia
B. femur, tibia and fibula
C. femur, tibia, fibula and patella
D. femur, tibia and patella
E. femur, fibula and patella
9. The following facts concerning the joints between two vertebral arches are correct except
which?
A. The articular surfaces are devoid of hyaline cartilage.
B. The joints between the superior and inferior articular surfaces are devoid of hyaline
cartilage.
C. The joints are synovial joints.
D. The joints are surrounded by a capsular ligament.
E. In the cervical region, the supraspinous and interspinous ligaments are greatly thickened
to form the ligamentum nuchae.
10. The sixth thoracic vertebra articulates by means of synovial joints with all the following
structures except which?
A. the body of the fifth thoracic vertebra
B. the head of the sixth rib
C. the tubercle of the sixth rib
D. the inferior articular process of the fifth thoracic vertebra
E. the superior articular process of the seventh thoracic vertebra
11. Which of the following costal cartilages do not directly articulate with the body of the
Sternum ?
A. second
B. fourth
C. fifth
D. eighth
E. third
12. Which of the following nerves is related to the inferior aspect of the shoulder joint and may be
injured in dislocations of the shoulder joint?
A. radial
B. ulnar
C. axillary
D. median
E. musculocutaneous
13. The tendon passing through shoulder articular capsule is the
.
A. supraspinous muscle tendon
B. infraspinous muscle tendon
C. long head tendon of triceps brachii
D. long head tendon of biceps brachii
E. short head tendon of biceps brachii
14. The following muscles are responsible for flexion of the elbow joint except which?
A. The biceps brachii
B. The brachioradialis
C. The pronator teres
D. The anconeus
E. The brachialis
15. The medial ligament of the elbow joint is closely related to the
.
A. brachial artery
B. radial nerve
C. ulnar artery
D. basilic vein
E. ulnar nerve
16. Select the structure that is most important in strengthening the wrist joint
.
A. the capsule
B. the tone of the flexor and extensor muscles of the wrist joint
C. the anterior and posterior ligaments
D. the synovial membrane
E. the medial and lateral ligaments
17. The calcaneus participates in the formation of which arch(es) of the foot?
A. medial longitudinal arch only
B. transverse arch only
C. medial and lateral longitudinal arches
D. medial longitudinal and transverse arches
E. lateral longitudinal and transverse arches
18. The following facts concerning the movements of the metacarpophalangeal joints are correct
except which?
A. The lumbricals and the interossei, assisted by the flexor digitorum superficialis and
profundus muscles, produce flexion.
B. The movement away from the midline of the third finger is performed by the palmar
interossei muscles.
C. Extension is performed by the extensor digitorum, extensor indicis, and extensor digiti
minimi.
D. Adduction movement toward the midline of the third finger is performed by the palmar
interossei muscles.
E. No rotation is possible at these joints.
19. The following statements concerning the joints of the bony pelvis are correct except which?
A. Very little movement is possible at the sacroiliac joint.
B. The female sex hormones cause a relaxation of the ligaments of the pelvis during pregnancy.
C. Obliteration of the cavity of the sacroiliac joint often occurs in both sexes after middle age.
D. The sacroiliac joint is supplied by the lumbar plexus of nerves.
E. The symphysis pubis has a fibrocartilaginous disc.
20. Hyperextension of the hip joint is prevented by
.
A. obturator internus tendon
B. ischiofemoral ligament
C. tensor fasciae latae
D. iliotibial tract
E. ligamentum teres
21. The
prevents abduction of the tibia at the knee joint.
A. posterior cruciate ligament
B. anterior cruciate ligament
C. lateral collateral ligament
D. lateral meniscus
E. medial collateral ligament
22. The
is attached to the head of the fibula.
A. lateral meniscus
B. lateral collateral ligament
C. anterior cruciate ligament
D. posterior cruciate ligament
E. medial meniscus
23.The talus participates in the formation of which arch (es) of the foot?
A. transverse arch only
B. lateral longitudinal arch only
C. medial longitudinal arch only
D. medial and lateral longitudinal arches
E. transverse and medial longitudinal arches
24. Articulation of the talus with the malleoli and lower tibia forms the ankle or talocrural joint.
Factors that contribute to the stability of this joint include all the following except the
.
A. deltoid or medial ligament, which joints the medial malleolus of the tibia to the talus,
navicular, and calcaneus bones
B. posterior tibiofibular ligament
C. lateral ligament, which joins the lateral malleolus of the talus and calcaneus
D. calcaneonavicular ("spring") ligament
E. trapezoidal shape of the talar articular surface (wider anteriorly than posteriorly), which
results in greater stability in dorsiflexion than in plantar flexion
25. The following statements concerning the ankle joint are correct except which?
A. It is strengthened by the deltoid(medial collateral) ligaments.
B. It is a hinge joint.
C. It is formed by the articulation of the talus and the distal ends of the tibia and the fibula.
D. It is most stable in the fully plantar-flexed position.
E. It is a synovial joint.
26. The
prevents dislocation of the femur backward at the knee joint.
A. anterior cruciate ligament
B. posterior cruciate ligament
C. medial collateral ligament
D. lateral collateral ligament
E. tendon of the popliteus muscle
27. The thinnest and the least-supported part of the shoulder joint capsule are situated
.
A. anteriorly and superiorly
B. anteriorly and inferiorly
C. posteriorly and superiorly
D. posteriorly and inferiorly
E. none of the above
28. About sternoclavicular joint, which statement is right?
A. It is consisting of clavicle and sternum.
B. The articular capsule is loose.
C. The articular cavity is divided into two parts.
D. It is ball-and-socket joint.
E. The above descriptions are all wrong.
29. The accessory structure of joint is the
.
A. articular cavity
B. articular capsule
C. articular surface
D. articular disc
E. synovial fluid
30. Which is the right description about hip joint?
A. It is the biggest and most complex joint in human body.
B. It belongs to the biaxial joint.
C. Neck of femur is out of the articular capsule completely.
D. Femoral head is easy to dislocate downwards.
E. There is pubic ligament in the articular cavity.
B1
A. metacarpophalangeal joint of index finger
B. shoulder joint
C. wrist joint
D. carpometacarpal joint of the thumb
E. atlantoaxial joint
1. ball-and-socket joint
2. saddle joint
True or False Questions
1. The intervertebral discs are thickest in the sacral regions, where the movements of the vertebral
column are greatest.
(
)
2. In the adult in standing position, the vertebral column exhibits in the sagittal plane the following
regional curves: cervical, anterior concavity; thoracic, anterior convexity; umbar, anterior
concavity; and sacral, anterior convexity.
(
)
3. The flat fibrocartilaginous disc lies within the sternoclavicular joint and divides the joint's
interior into two compartments.
(
)
4. Articulation forms between the distal end of the radius and the articular disc above and the
trapezium, trapezoid, hamate and triquetral bones below.
(
)
5. The cruciate ligaments are two strong extracapsular ligaments that cross each other within the
knee joint cavity.
(
)
6. A segmented structure can hold up weight only if it is built in the form of an arch. The foot has
three such arches, which are present at birth: the medial longitudinal, lateral longitudinal, and
transverse arches.
(
)
Explanation of Terms
1 .intervertebral disc
2. ligamenta flava (yellow ligament)
3. menisci
4. anterior cruciate ligament
5. posterior cruciate ligament
Answer the Following Questions
1. Describe the curvature of the vertebral column. What do the terms primary curves and
secondary curves refer to?
2. What are the three structural classes of joints? Describe the characteristics of each.
3. What are the structural elements of a synovial joint that determine its range of movement?
4. Identify four types of synovial joints found in the wrist and hand regions, and state the types of
movement permitted by each.
ANSWERS
Multiple Choice Questions
A1
1. D 2. C 3. B
4. B
5. C
6. C
7. A
8. D
9. A
10. A 11. D
12.C 13.D 14.D 15.E 16. E 17. C
18. B 19. D 20. B
21. E 22. B
23.C 24.D 25. D 26. A 27. A 28. C 29. D
30. D
B1
1. B
2. D
True or False Questions
1. F
2. F
3. T
4. F
5. F
6. T
Explanation of Terms
1. The intervertebral discs make up one fourth of the spinal column's length. There are no discs
between the Atlas (C1), Axis (C2), and Coccyx. Discs are not vascular and therefore depend
on the end plates to diffuse needed nutrients. The cartilaginous layers of the end plates anchor
the discs in place. Intervertebral discs are composed of an annulus fibrosus and a nucleus
pulposus. A sudden increase in the compression load on the vertebral column causes the
semifluid nucleus pulposus to become flattened. The outward thrust of the nucleus is
accommodated by the resilience of the surrounding annulus fibrosus and it ruptures, allowing
the nucleus pulposus to herniate and protrude into the vertebral canal, where it may press on
the spinal nerve roots, the spinal nerve, or even the spinal cord.
2. The paired bands of yellow elastic tissue that connect adjoining laminae of the vertebrae. With
the laminae, it forms the posterior wall of the spinal canal and helps hold the body erect.
3. Two crescentic fibrocartilaginous plates which are interposed between the femoral and tibial
condyles and triangular in cross-section. Action: They deepen the articular surfaces of the tibia
for the reception of the femoral condyles.
4. Location: arises in front of the intercondylar eminence of the tibia, extends upward, backward
and laterally to attach to the posterior part of the medial side of the lateral condyle of the femur.
Action: it prevents backward displacement of the femur on the tibia and hyper extention of the
knee joint. When the knee joint is flexed to a right angle, the tibia cannot be pulled forward.
5. Location: arises from behind the intercondylar eminence and passes upward and forward on the
medial side of the anterior cruciate ligament to attach to the anterior part of the lateral surface of
the medial condyle of the femur. Action: prevents forward displacement of the femur or
backward displacement of the tibia.
Answer the Following Questions
1. Curves in the sagittal plane:
In the adult in the standing position, the vertebral column exhibits in the sagittal plane the
following regional curves: cervical, posterior concavity; thoracic, posterior convexity;
lumbar, posterior concavity; and sacral, posterior convexity.
Curves in the coronal plane:
In late childhood, it is common to find the development of minor lateral curves in the thoracic
region of the vertebral column. This is normal and is usually caused by the predominant use of
one of the upper limbs.
The primary curves: thoracic and sacral curves
The secondary curves: cervical and lumbar curves
2. (1) Fibrous joints. In fibrous joints, the articulating bones are held together by fibrous
connective tissue. These joints lack joint cavities.
(2) Cartilaginous joints. In cartilaginous joints, the articulating bones are held together by
cartilage. These joints also lack joint cavities.
(3) Synovial joints. In synovial joints, the articulating bones are capped with cartilage, and
ligaments frequently help support them. These joints are distinguished by fluid-filled joint
cavities.
3. A synovial joint's range of motion is determined by three factors:
(1) the structure of the hones involved in the articulation (for example, the olecranon of the
ulna limits hyperextension of the elbow joint);
(2) the strength of the joint capsule and the strength and tautness of the associated ligaments
and tendons;
(3) the size, arrangement, and action of the muscles that span the joint. Range of motion at
synovial joints is characterized by tremendous individual variation, most of which is related to
body conditioning. Excessive obesity may also limit the range of movement at synovial joints.
Although some people can perform remarkable contortions and are said to be
"double-jointed," they have no extra joints that help them do this. Rather, through conditioning,
they are able to stretch the ligaments that normally inhibit movement.
4. (1) Hinge. Hinge joints are monaxial--like the hinge of a door, they permit movement in only
one plane. In this type of articulation, the surface of one bone is always concave, and the
other convex. Hinge joints are the most common type of synovial joints. Examples include the
joints between the phalanges.
(2) Pivot. The movement at a pivot joint is limited to rotation about a central axis. In this type
of articulation, the articular surface on one bone is conical or rounded and fits into a
depression on another bone. Examples are the proximal articulation of the radius and ulna for
rotation of the forearm, as in turning a doorknob.
(3) Condyloid (ellipsoid) .A condyloid articulation is structured so that an oval, convex
articular surface of one bone fits into a concave depression on another bone. This permits
angular movement in two directions, as in up-and-down and side-to-side motions. Condyloid
joints are therefore said to be biaxial joints. The radiocarpal join of the wrist and the
metacarpophalangeal joints are examples.
(4) Saddle. Each articular process of a saddle joint has a concave surface in one direction and a
convex surface in another. This articulation is a modified condyloid joint that allows a wide
range of movement. There are two places in the body where a saddle joint occurs. A typical
example is articulation of the trapezium of the carpus with the first metacarpal bone. This
carpometacarpal joint is the one responsible for the opposable thumb--a hallmark of primate
anatomy.
(Ying Dajun, Pan Feng)
Chapter3
Myology
Multiple Choice Questions
A1
1. The connective tissue component of a skeletal muscle that surrounds fasciculi is called the
.
A. perimysium
B. epimysium
C. endomysium
D. deep fascia
E. superficial fascia
2. The sheath of connective tissue that surrounds the individual fibers of skeletal muscle is called
the
.
A. perimysium
B. epimysium
C. deep fascia
D. endomysium
E. superficial fascia
3. The trapezius muscle is named on the basis of
.
A. shape
B. size
C. location
D. action
E. orientation
4. What is not the action of the muscles located in the anterior compartment of the forearm?
A. flexes the elbow joint
B. adducts the wrist joint
C. supinates the forearm.
D. flexes the wrist
E. pronates forearm
5. Which of the following hyoid muscles is an important landmark in posterior triangle of the
neck?
A. geniohyoid
B. mylohyoid
C. omohyoid
D. sternohyoid
E. stylohyoid
6. A muscle that inserts to both the spine of the scapula and the clavicle is the
.
A. pectoralis major
B. erector spinae
C. deltoid
D. sternocleidomastoid
E. trapezius
7. Which muscle is involved in moving the vertebral column?
A. platysma
B. erector spinae
C. sartorius
D. diaphragm
E. pectoralis minor
8. After being thrown from a motorcycle moving at high speed, a 16-year-old female was found to
have a paralyzed right pectoralis major muscle. Which set of movements at the shoulder joint
would be found greatly weakened?
A. abduction and extension
B. abduction and lateral rotation
C. adduction and flexion
D. lateral rotation and extension
E. lateral rotation and extension
9. The inguinal ligament is formed by
.
A. the obliquus externus abdominis
B. the cremaster
C. the transversus abdominis
D. the obliquus internus abdominis
E. the rectus abdominis
10. The inguinal falx is formed by
.
A. the obliquus internus abdominis and the transversus abdominis
B. the cremaster
C. the transversus abdominis
D. the obliquus externus abdominis and the transversus abdominis
E. the rectus abdominis
11. Which muscle of the posterior abdominal wall pulls the thoracic cage toward the pelvis and
bends the vertebral column toward the side that is being contracted?
A. transversus abdominis
B. quadratus lumborum
C. obliquus internus abdominis
D. rectus abdominis
E. obliquus externus abdominis
12. A muscle that originates on both the spine of the scapula and the clavicle is the
.
A. pectoralis maior
B. trapezius
C. deltoid
D. sternocleidomastoid
E. erector spinae
13. Several deficits in muscle function of the right upper limb were noted, including inability to
abduct the arm. This was caused by denervation of which muscle?
A. deltoid
B. infraspinatus
C. latissimus dorsi
D. teres minor
E. trapezius
14. Which of the following is not used as a means of naming muscles?
A. location
B. action
C. shape
D. attachment
E. strength of contraction
15. The following muscles belong to the posterior group of leg except the
.
A. peroneus longus
B. popliteus
C. flexor digitorum longus
D. flexor hallucis longus
E. gastrocnemius
16. The rotator cuff is composed of all of the following muscles except the
.
A. infraspinatus
B. subscapularis
C. supraspinatus
D. teres major
E. teres minor
17. All of the following muscles are both the adductor and medial rotator of the arm except the
A. pectoralis major
B. teres major
C. infraspinatus
D. subscapularis
E. latissimus dorsi
18. Which of the following muscles does not have either an origin or insertion on the femur?
A. the sartorius
B. the gluteus medius
C. the vastus medialis
D. the adductor magnus
E. the iliopsoas
19. Which muscle is the strongest medial rotator of the arm?
A. coracobrachialis
B. infraspinatus
C. subscapularis
D. supraspinatus
E. teres minor
20. Which muscle contributes to the “rotator cuff”?
A. deltoid
B. latissimus dorsi
C. pectoralis minor
D. supraspinatus
E. teres major
.
21. The muscle of the thigh that flexes the hip and knee joints is the
.
A. sartorius
B. quadriceps femoris
C. biceps femoris
D. pectineus
E. adductor brevis
22. The muscle in the posterior compartment of arm is the
.
A. deltoid
B. biceps brachii
C. triceps brachii
D. teres major
E. teres minor
23. What muscle tendon is enclosed within its own synovial sheath in the carpal canal?
A. flexor carpi ulnaris
B. flexor digitorum profundus to 2nd digit
C. flexor digitorum superficialis to 2nd digit
D. flexor pollicis longus
E. palmaris longus
24. Which of the following muscles does not have either an origin or insertion on the humerus?
A. the teres minor
B. the biceps brachii
C. the supraspinatus
D. the brachialis
E. the pectoralis major
25. What is the action of the muscles located in the anterior compartment of the leg?
A. dorsi-flex the foot and/or extend the toes
B. flex the leg
C. plantar flex the foot and flex the toes
D. extend the leg
E. flex the leg
26. The muscle of the thigh that flexes the leg and extends the thigh is the
.
A. sartorius
B. quadriceps femoris
C. biceps femoris
D. adductor longus
E. gracilis
27. Which muscle is the flexor and adductor of the wrist joint?
A. brachioradialis
B. pronator teres
C. palmaris longus
D. flexor carpi ulnaris
E. flexor carpi radialis
28. The muscle in the posterior compartment of forearm is the
.
A. abductor pollicis longus
B. brachioradialis
C. palmaris longus
D. brachialis
E. pronator teres
29. Which of the following muscles belongs to the hypothenar eminence of the hand?
A. dorsal interossei muscles
B. abductor digiti minimi
C. flexor pollicis brevis
D. adductor pollicis muscles
E. lumbricales
30. What is the action of the muscles located in the posterior compartment of the thigh?
A. adduct the thigh
B. rotate the thigh
C. flex the leg
D. extense the leg
E. flex the thigh
31. Which of the following muscles flexes elbow joint and supinates forearm and also helps to
flex the shoulder joint?
A. the triceps brachii
B. the brachioradialis
C. the biceps brachii
D. the coracobrachialis
E. the brachialis
32. What muscle passes through the lesser sciatic foramen?
A. gluteus minimus
B. obturator internus
C. piriformis
D. quadratus femoris
E. superior gemellus
33. Which movement would fail in case of paralysis of the quadriceps femoris muscle?
A. adduction at the hip
B. extension at the hip
C. extension at the knee
D. flexion at the knee
E. medial rotation at the knee
34. All of the following muscles are both the adductor and flexor of the thigh except the
.
A. pectineus
B. adductor longus
C. gracilis
D. adductor magnus
E. rectus femoris
35. Which of the following muscles belongs to the anterior group of thigh?
.
A. adductor longus
B. biceps femoris
C. semitendinosus
D. quadriceps femoris
E. gluteus maximus
A2
1. A patient presented to his physician with chronic shoulder pain. It was noted that when asked to
abduct his arm, he initially leaned laterally, and then straightened up. When iodinated contrast
was injected into his shoulder joint it was found to be in the subdeltoid bursa as well as in the
joint cavity. Which structure was damaged to produce the shoulder pain?
A. acromioclavicular ligament
B. long head of the biceps brachii
C. subscapularis
D. superior glenohumeral ligament
E. supraspinatus muscle
2. An elderly patient complains of shoulder pain and has difficulty abducting his arm. Arthroscopy
is done in which a dye is injected into the shoulder joint and an X-ray taken. The radiologist
notes that the dye has leaked from the shoulder joint into the subacromial bursa. What tendon
would need to be ruptured for this to occur?
A. deltoid
B. infraspinatus
C. latissimus dorsi
D. supraspinatus
E. teres minor
3. An elderly man complained of pain in his shoulder when he brought his forearm and hand
behind his back while dressing. It was determined that stretching of the lateral rotators of his
arm during this motion caused the pain. Which muscle was most likely involved?
A. infraspinatus
B. latissimus dorsi
C. subscapularis
D. supraspinatus
E. teres major
4. During a strenuous game of tennis a 55-year-old woman complained of severe shoulder pain
that forced her to quit the game. During physical examination it was found that she could not
initiate abduction of her arm, but if her arm was elevated to 45 degrees from the vertical (at her
side) position, she had no trouble fully abducting it. Which muscle was injury responsible for?
A. deltoid
B. supraspinatus
C. infraspinatus
D. teres major
E. trapezius
5. A six-year-old child, whose medical history includes a rather difficult birth, has a permanently
tilted head posture, with the right ear near the right shoulder and the face turned upward and to
the left. Which of the following muscles was involved?
A. anterior scalene
B. omohyoid
C. sternocleidomastoid
D. trapezius
E. platysma
6. In the lumbar region, tuberculosis may spread from the vertebrae into an adjacent space to
produce an abscess. Pus from the abscess may travel downwards within this space. A patient
presents with pus surfacing in the super medial part of the thigh. To which muscle was the
tuberculosis most likely involved?
A. internal oblique
B. obturator internus
C. psoas major
D. quadratus lumborum
E. rectus abdominis
7. A patient with painful swelling in the distal calf cannot plantar flex at the ankle with any power.
Which tendon was likely to rupture?
A. tibialis anterior
B. extensor digitorum longus
C. extensor hallucis longus
D. plantaris
E. tendon calcaneus
B1
A. the phrenic nerve
B. the inferior vena cava
C. the thoracic duct
D. the superior vena cava
E. the anterior and posterior vagal trunks
1. The esophageal hiatus transmits
.
2. The vena caval foramen transmits
.
3. The aortic hiatus transmits
.
A. the crest of greater tubercle of humerus
B. intertubercular sulcus of humerus
C. the crest of lesser tubercle of humerus
D. the lesser tubercle of humerus
E. the greater tubercle of humerus
4. The insertion of pectoralis major is
.
5. The insertion of the infraspinatus is
.
6. The insertion of the teres major is
.
7. The insertion of the subscapularis is
.
A. the rectus femoris
B. the biceps femoris
C. the gastrocnemius
D. the adductor magnus
E. the sartorius
8. Which of the above muscles flexes hip joint and extends knee joint?
9. Which of the above muscles flexes hip joint and knee joint?
10. Which of the above muscles extends hip joint and flexes knee joint?
11. Which of the above muscles flexes knee joint and ankle joint?
True or False Questions
1. Skeletal muscles are named on the basis of sharp, location, attachment, orientation of fibers,
relative position, or function.
(
)
2. The masseter arises from the inferior border and medial surface of the zygomatic arch and is
inserted on the lateral aspect of the ramus and angle of the mandible.
( )
3. The intercostales interni elevate the ribs, and are therefore considered to be muscles of
inspiration.
(
)
4. The deltoid caps the shoulder joint and forms the rounded contour of the shoulder.
( )
5. The quadriceps femoris can act on both the hip and knee joints to flex and rotate the hip
laterally, and also to assist in flexing the knee joint and rotating it medially.
(
)
Explanation of Terms
1. linea alba
2. inguinal ligament
3. scalene fissure
4. synergistic muscles
5. endomysium
Answer the Following Questions
1. Which muscles of the neck either originate from or insert on the hyoid bone.
2. List all the muscles that either originate from or insert on the scapula.
3. Which muscles of the lower extremity span two joints, and therefore have two different
actions?
4. List the supplementary structures of muscles.
5. Describe the flexor and extensor compartments of the muscles of the forearm.
6. Describe the inguinal canal.
7. Describe the structural arrangement of the muscle fibers and fasciculi within muscle.
8. Describe the openings in the diaphragm.
9. On the basis of function, categorize the muscles that move the humerus at the shoulder joint.
ANSWERS
Multiple Choice Questions
A1
1.A
2.D
3.A
4.C
5.C
6.E
7.B
8.C
9.A
10.A 11.B
12.C
13.A 14.E
15.A 16.D
17.C
18.A
19.C
20.D 21.A
22.C 23.D
24.B 25.A 26.C 27.D 28. A 29. B 30. C
31.C
32.B
33.C
34.E
35.D
A2
1. E
2. D
3. A
4. B
5. C
6. C
7. E
B1
1. E 2. B 3. C 4. A 5. E 6. C
7. D
8. A
9. E 10. B
11. C
True or False Questions
1. T
2. T
3. F
4. T
5. F
Explanation of Terms
1. It is placed on the anterior median line of the abdomen, extending from the xiphoid process to
the pubic symphysis. It is formed by the blending of the fibers of the sheath of rectus abdominis
of two sides.
2. The lower part of the aponeurosis of the obliquus externus abdominis forms the inguinal
ligament attached to the anterior superior lilac spine and the pubic tubercle.
3. Above the first rib, there is a triangular space (scalene fissure) between the scalenus anterior and
the scalenus medius. The brachial plexus and the subclavian artery emerge from this space.
4. Muscles that contract together in accomplishing a particular movement are said to be
synergistic muscles.
5. The individual fibers of skeletal muscle are surrounded by a fine sheath of connective tissue
called endomysium. The endomysium binds adjacent fibers together and supports capillaries
and nerve endings serving the muscle.
Answer the Following Questions
1. The digastric, mylohyoid, stylohyoid, geniohyoid, sternohyoid, sternothyroid, thyrohyoid, and
omohyoid muscles.
2. The trapezius, omohyoid, supraspinous, infraspinous, deltoid, teres minor, major teres, biceps
brachii, triceps brachii, coracobrachialis subscapularis, and levator scapulae muscles.
3. The long head of biceps femoris, semitendinosus, semimembranosus, rectus femoris, sartorius
and gastrocnemius muscles.
4. The fascia, synovial bursa, sesamoid bones and synovial sheath of tendon.
5. The flexors compartment include the brachioradialis, pronator teres, flexor carpi radialis,
palmaris longus, flexor carpi ulnaris, flexor digitorum superficialis, flexor pollicis longus,
pronator quadratus and flexor digitorum profundus. The extensors compartment include the
extensor carpi radialis longus and brevis, extensor digitorum, extensor digiti minimi, extensor
carpi ulnaris, supinator, abductor pollicis longus, extensor pollicis brevis, extensor indicis and
extensor pollicis longus.
6. The inguinal canal is an oblique cleft or passage about 4-5cm long in the adult. It runs obliquely
between the muscles, aponeurosis and fasciae of the abdominal wall above the medial part of
the inguinal ligament and passes downwards and medially from the deep to superficial inguinal
rings. The inguinal canal contains the spermatic cord in male or the round ligament of uterus in
female and in both sexes the ilioinguinal nerve.
7. The individual fibers of skeletal muscle are surrounded by a fine sheath of connective tissue
called endomysium. The endomysium binds adjacent fibers together and supports capillaries
and nerve endings serving the muscle. Another connective tissue, the perimysium, binds
groups of muscle fibers together into bundles called fasciculi . The perimysium supports blood
vessels and nerve fibers serving the various fasciculi. The entire muscle is covered by the
epimysium, which in turn is continuous with a tendon.
8. There are three openings in the diaphragm:
(1) The aortic hiatus lies in front of the 12th thoracic vertebra. It transmits the abdominal aorta
and the thoracic duct.
(2) The esophageal hiatus is located in the right crus of the diaphragm at the level of T10. It
transmits the esophagus and the anterior and posterior vagal trunks.
(3) The vena caval foramen is located in the central tendon of the diaphragm at the level of T8.
The inferior vena cava is adherent to the margin of this foramen.
9. Flexors: The coracobrachialis and biceps brachii;
Extensors: The latissimus dorsi muscle and the long head of triceps brachii;
Adductors: The pectoralis major, latissimus dorsi and teres major;
Abductors. The deltoid and supraspinatus;
Medial rotators: The latissimus dorsi, pectoralis major, deltoid and teres major;
Lateral rotators. The infraspinatus, deltoid, and teres minor.
(Liu Xuezheng, Pan Feng)
Chapter4-5
The General Description of Splanchnology & The Alimentary System
Multiple Choice Questions
A1
1. The palatine tonsil lies
.
A. in oral cavity
B. in piriform recess
C. in pharyngeal recess
D. between palatoglossal arch and palatopharyngeal arch
E. in posterior wall of pharynx
2. The major digestive gland is
.
A. buccal glands
B. esophageal gland
C. submandibular gland
D. gastric gland
E. intestinal gland
3. The major duodenal papilla lies in
.
A. superior part of duodenum
B. descending part of duodenum
C. horizontal part of duodenum
D. ascending part of duodenum
E. duodenojejunal flexure
4. The structure under inferior surface of tongue is
.
A. foramen cecum of tongue
B. lingual tonsil
C. terminal sulcus
D. vallate papilla
E. sublingual caruncle
5. Which structure doesn't belong to stomach?
A. angular notch
B. pyloric antrum
C. antrum of stomach
D. pyloric canal
E. fundus of stomach
6. Pharyngeal opening of auditory tube lies in
.
A. nasopharynx
B. oropharynx
C. laryngopharynx
D. epiglottic vallecula
E. inferior meatus
7. The beginning of jejunum is
.
A. duodenojejunal flexure
B. angular notch
C. transverse part of duodenum
D. pylorus
E. left colic flexure
8. The shortest segment of large intestinal is
.
A. ascending colon
B. descending colon
C. sigmoid colon
D. anal canal
E. cecum
9. The piriform recess lies in
.
A. nasopharynx
B. oropharynx
C. laryngopharynx
D. isthmus of fauces
E. vestibule of larynx
10. The dentate line is formed by
.
A. anal canal
B. transverse fold of rectum
C. anal columns
D. anal valves
E. anal sinus
11. The opening of hepatopancreatic ampulla lied in
.
A. superior portion of duodenum
B. major duodenal papilla
C. Calot's triangle
D. pyloric antrum
E. minor duodenal papilla
12. The limitation of pyloric antrum and pyloric canal is
.
A. angular notch
B. pyloric valve
C. cardiac incisure
D. intermedia sulcus
E. lesser curvature of stomach
13. Which one is right about the rectum?
A. It continues upwards with the sigmoid colon at the level of pelvic inlet.
B. It continues downwards with the anal canal at the level of dentate line.
C. The upper part is not covered by peritoneum.
D. It has sacral and perineal flexures in the sagittal plane.
E. It is an intraperitoneal viscus.
14. Concerning Hilton's line, the wrong description is
.
A. lies at the interval between the sphincter ani internus and externus
B. is about lcm below the dentate line
C. is also called white line
D. is continuous upwards with the anal pectin
E. can not be felt in digital examination of the anal canal
15. Anal canal
.
A. continues with the rectum at the level of pelvic diaphragm
B. terminates at the dentate line
C. is about 7-8 cm in length
D. has two flexures in sagittal plane
E. has a dilated upper part called ampulla of rectum
16. Below the dentate line or
.
A. the epidermis is skin
B. the epidermis is innervated by somatic nerve
C. the lymph is drained into the superficial inguinal lymph node
D. the blood is drained into the internal iliac vein
E. all of the above are right
17. Which of the following structures is not comprised of forming the isthmus of fauces?
A. uvula
B. free margin of the palatine velum
C. palatopharyngeal arch
D.palatoglossal arch
E. root of tongue
18. All of the deciduous teeth normally erupt by the age of
.
A.6 months
B.1 year
C.2.5years
D.3.5years
E.6 years
19. About the teeth, which of the following statements is not true?
A. Teeth are the hardest organs in human body.
B. Normally two sets of teeth develop in a person’s lifetime.
C. The central region of the tooth contains the pulp cavity.
D. The third molars are the last to erupt.
E. Both of the dentine, enamel and cement have no innervation.
20. About the parotid gland, which of the following statements is not true?
A. Parotid gland is the largest salivary gland.
B. It can be divided into two parts in shape: the superficial and the deep.
C. The deep extends and lies between the ramus of the mandible and the sternoclei-domastoid.
D. The parotid duct opens on the oral surface of the cheek opposite the crown of the third
upper molar tooth.
E. About 35% of the people have an accessory parotid gland.
21. Which of following is not the structure of nasopharynx?
A. pharyngeal opening of the auditory tube
B. piriform recess
C. tubal torus
D. pharyngeal recess
E. pharyngeal tonsil
22. Which of the following distances is right from the third esophageal constrictions to the upper
central incisor teeth?
A. 5cm
B. 15cm
C. 25cm
D. 40cm
E. 45cm
23. About the duodenum, which of the following statements is not right?
A. It has mesentery, and is thus only partially covered with peritoneum.
B. It is the shortest, widest and most fixed part of small intestine.
C. It extends from the pylorus to the jejunum.
D. It receives the openings of the bile and pancreatic ducts.
E. It is divided into four parts.
24. About the vermiform appendix, which of the following statements is right?
A. It has colic bands, haustra of colon and epiploic appendices.
B. It has its own mesentery, the mesoappendix.
C. The appendix is stable in position because of the cecum.
D. Most commonly, its tip lies in front of the ileum.
E. McBurney's point is variable in position
25. Which of the following structures is not contained in the visceral surface of liver?
A. falciform ligament
B. porta hepatis
C. ligamentum venosum
D. ligamentum teres hepatis
E. fossa for gallbladder
A2
1. A 60-year-old male executive who had a history of a chronic duodenal ulcer was admitted to the
ER exhibiting signs of a severe internal hemorrhage- He was quickly diagnosed with
perforation of the posterior wall of the first part of the duodenum and erosion of an artery
behind it by the gastric expellent. The artery is most likely the
.
A. common hepatic
B. gastroduodenal
C. left gastric
D. proper hepatic
E. superior mesenteric
2. An ulcer near the pyloro-duodenal junction perforated and eroded a large artery immediately
posterior to the duodenum. The ligation of the eroded vessel at its origin would least affect the
arterial supply to the
A. first part of the duodenum
B. second part of the duodenum
C. greater curvature of the stomach
D. head of the pancreas
E. tail of the pancreas
3. You are observing an operation to remove the left suprarenal gland. To expose the gland the
surgeon mobilizes the descending colon by cutting along its lateral attachment to the body wall
and dissecting medial-ward in the fusion fascia behind it. Suddenly the operative field is filled
with blood. The surgeon realizes he has failed to cut a mesenteric attachment between the left
colic flexure and another organ, As a result of the traction, the surface of the organ tore- Which
organ was injured?
A. duodenum
B. kidney
C. liver
D. spleen
E. suprarenal grand
4. A patient was admitted with symptoms of an upper bowel obstruction. Upon CT examination, it
was found that the third (transverse) portion of the duodenum was compressed by a large vessel
causing the obstruction. The vessel involved is most likely to be the
.
A. inferior mesenteric artery
B. superior mesenteric artery
C. inferior mesenteric vein
D. portal vein
E. splenic vein
5. During emergency surgery, it was found that a chronic ulcer had perforated the posterior wall of
the stomach and eroded a large artery running immediately posterior to the stomach. The artery
is the
.
A. gastroduodenal
B. common hepatic
C. left gastroepiploic
D. splenic
E. superior mesenteric
6. In order to approach the area posterior to the stomach, a surgeon decided to go through the
lesser omentum Before incising the mesentery she was careful to find and preserve a nerve
lying in the upper portion of the hepatogastric ligament, i. e, the
.
A. celiac branch of the anterior vagal trunk
B. celiac branch of the posterior vagal trunk
C. greater splanchnic branch to the right suprarenal gland
D. hepatic branch of the anterior vagal trunk
E. hepatic branch of the posterior vagal trunk
7. A 50-year-old female patient with severe jaundice was diagnosed with pancreatic cancer. You
suspect that the tumor is located in which portion of the pancreas?
A. head
B. neck
C. body
D. tail
E. uncinate process
8. To stop hemorrhaging from a ruptured spleen, it was necessary to temporarily ligate the splenic
artery near the celiac trunk. Which structure of the blood supply is least likely to be affected by
the ligation?
A. duodenum
B. greater omentum
C. body of pancreas
D. tail of pancreas
E. stomach
9. A patient was admitted with symptoms of bowel obstruction. Further examination revealed
that the obstruction was caused by the nutcracker-like compression of the bowel between the
superior mesenteric artery and the aorta. The compressed bowel is most likely the
.
A. duodenum
B. jejunum
C. ileum
D. ascending colon
E. transverse colon
B1
A. pharyngeal recess
B. piriform recess
C. anal sinuses
D. major duodenal papilla
E. cardia
1. Where is carcinoma more frequently found?
2. Where can foreign bodies lodge more frequently?
3. Where do the common bile duct and the pancreatic duct open together?
A. duodenal bulb
B. major duodenal papilla
C. minor duodenal papilla
D. suspensory muscle of duodenum
E. longitudinal fold of duodenum
4. Where does most duodenal ulcer occur?
5. opening of hepatopancreatic ampulla
6. opening of accessory pancreatic duct
True or False Questions
1. Duodenum belongs to the inferior digestive tube.
(
)
2. Palatoglossal arch descends from both sides of the base of the uvula to the side of the
root of the tongue.
(
)
3. Deciduous teeth begin to erupt at about 6 months of age.
(
)
4. The enamel is the hardest substance in the body and covers the surface of all the teeth.
( )
5. The filiform papillae have a color of white and plenty of taste buds.
(
)
6. Parotid duct opens upon a small papilla on the oral surface of the cheek opposite the crown of
the second upper molar tooth.
(
)
7. The oropharynx and laryngopharynx are the common passages of the alimentary and
respiratory canals.
(
)
8. On the lateral wall of oropharynx presents the pharyngeal opening of the auditory tube.
( )
9. The esophagus is a collapsible muscular tube and is the narrowest portion of the
alimentary canal.
( )
10. The first constriction of esophagus is at its commencement, 15 cm from the upper central
incisor teeth.
(
)
11. The pyloric orifice is where the esophagus enters the stomach.
(
)
12. The fundus of stomach is in direct contact with the diaphragm.
(
)
13. The stomach lies in the epigastric, umbilical, and left hypochondriac regions of the abdomen.
(
)
14. The duodenum has mesentery, and is partially covered with peritoneum.
(
)
15. The duodenum extends from the pylorus to the jejunum, about 20-25cm long. It is the shortest,
widest and most fixed part of small intestine.
(
)
16. The duodenum is important because it receives the openings of the bile and pancreatic ducts.
(
)
17. The common bile duct and the pancreatic duct together perforate the medial side wall of
ascending part of the duodenum obliquely.
(
)
18. The ligament of Treitz, which is the mark of the commencement of small intestine clinically.
(
)
19. The solitary lymphatic follicles are scattered along the length of the jejunum and ileum
mucosa.
(
)
20. The colic bands are formed by two longitudinal muscular fibers.
(
)
21. The tip of the appendix may be in variable position, but its base is relatively constant.
(
)
22. The sigmoid colon is closely surrounded by peritoneum, which forms the sigmoid mesocolon.
(
)
23. The middle fold of rectum is the largest and most constant, about 7 cm away from anus.
(
)
24. White line represents the junction of the endodermal and ectodermal parts of the canal.
(
)
25. According to the Glisson distribution of the liver, it can be divided into two half parts and then
subdivided into some segments in each half.
(
)
26. The gallbladder is completely covered by peritoneum continued from the hepatic surface
(
)
27. Two hepatic ducts leave the porta hepatis and join together to form the common bile duct.
(
)
28. The pancreas is the biggest alimentary gland of the body. It is both an exocrine and an
endocrine gland.
(
)
29. The pancreatic duct opens into the major duodenal papilla via the hepatopancreatic ampulla.
(
)
Explanation of Terms
1. isthmus of fauces
2. pyloric antrum
3. ligament of Treitz
4. Meckel' s diverticulum
5. ileocecal valve
6. dentate line
7. porta hepatic
8. hepatopancreatic ampulla
Answer the Following Questions
1. Briefly describe the types of papillae of the tongue.
2. Briefly describe the types of major salivary glands.
3. Briefly describe the shape of stomach.
4. Briefly describe the parts of pharynx.
5. Briefly describe the composition of extrahepatic parts of biliary system.
6. Briefly describe the parts of duodenum.
7. Briefly describe the features of colon and cecum.
8. Briefly describe the surface marking of the root of vermiform appendix.
9. Briefly describe the parts of colon.
10. Briefly describe the parts of gallbladder and surface markings of fundus of gallbladder.
11. Briefly describe the location and structures of pancreas.
12. Briefly describe the three constrictions of esophagus.
ANSWERS
Multiple Choice Questions
A1
1. D
2. C
3. B
4. E
5. C
6. A
7. A
8. D
9. C
10. D
11. B
12. D
13. D
14. E
15. A
16. E
17. C
18. C
19. E
20. D
21. B
22. D
23. A
24. B
25. A
A2
1. B
2. E
3. D
4. B
5. D
6. D
7. A
8. A
9. A
B1
1. A
2. B
3. D
4. A
5. B
6. C
True or False Questions
1. F 2. T 3. T
4. F
5. F 6. T 7. T 8. F 9. T 10.T 11. F 12. T
13. T
14. F
15. T
16. T 17. F
18. F 19. F 20. F
21. T 22. T
23. T 24. F 25. T 26. T 27. F 28. F 29. T
Explanation of Terms
1. A little narrow passage from the mouth to the pharynx, named the isthmus of fauces,
surrounded by the uvula, the free margin of the palatine velum, the palatoglossal arch and the
root of tongue.
2. Pylorus is subdivided by the intermediate groove into a pyloric antrum on the left and a pyloric
canal and is the distal end of the stomach. Gastric ulcer and cancer usually occur in pyloric
antrum where close to the lesser curvature.
3. The du0denojejunal flexure is held in position by a peritoneal fold, the ligament of Treitz,
which is the mark of the commencement of jejunum clinically.
4. In about .2% of individuals, Meckel's diverticulum projects from the antimesenteric border of
the distal ileum, its average length about 5cm. It represents the persistent portion of the vitelline
stalk (yolk stalk) in early fetal life.
5. Ileocecal orifice consists of two semilunar shaped flaps projecting into the lumen of the large
intestine, named the ileocecal valve that not only prevents the reflux from cecum to the ileum
but is probably also a sphincter regulating the passage of ileal contents into cecum.
6. The circular line along which the base of the anal valves and the lower part of the anal columns
are situated is termed the dentate line, thus representing the junction of the endodermal and
ectodermal parts of the canal.
7. The cross-bar of the "H" is the porta hepatis. Through the porta hepatis, the portal vein
posteriorly, the left and right branches of the proper hepatic artery on the left side and the
hepatic nerve plexus enter the liver.
8. The common bile duct reaches the pancreatic duct together they enter the duodenal wall where
they usually unite to form the hepatopancreatic ampulla.
Answer the Following Questions
1. Filiform papillae, fungiform papillae, foliate papillae and vallate papillae.
2. The parotid, submandibular and sublingual glands.
3. The stomach has two surfaces, an anterior and posterior surface; two curvatures, greater and
lesser curvature; and two orifices, cardiac and pyloric orifice.
4. By the levels of the soft palate and the superior border of the epiglottis, the pharynx may be
divided into three parts: nasal, oral and laryngeal.
5. Common hepatic duct, cystic duct, common bile duct.
6. It is divided into four parts: superior, descending, horizontal and ascending part.
7. The typical characteristics of the colon and cecum are colic bands, haustra of colon and epiploic
appendices.
8. McBurney's point, the usual surface marking for appendicular base is the junction of the middle
and lateral thirds of a line that joins the right anterior superior iliac spine to the umbilicus.
9. The colon is divided into four parts: the ascending, transverse, descending and sigmoid.
10. It consists of the fundus, body, neck and cystic duct, the usual surface marking for fundus of
gallbladder is the junction of costal arch and the lateral border of the right rectus abdominis.
11. The pancreas is an elongated structure that lies in the posterior abdominal wall, and
transversely situated in the epigastric and left hypochondriac regions. The pancreas can be
divided into head, neck, body and tail.
12. The first constriction is at its commencement, 15 cm away from the upper central incisor teeth
The second is where the left bronchus crossed its anterior surface, 25 cm away from the upper
central incisor teeth. The third occurs where the esophagus passes through the diaphragm into
the stomach, 40 cm away from the upper central incisor teeth. Foreign bodies can lodge and
carcinoma of esophagus is more frequently found in these constrictions.
(Zhang
Yafang)
Multiple Choice Questions
A1
1. Which muscle group combination permits inspiration?
A. diaphragm muscle, abdominal complex
B. intercostales interni muscle, diaphragm muscle
C. intercostales externi muscle, intercostales interni muscle
D. intercostales externi muscle, diaphragm muscle
E. transversus thoracis muscle, diaphragm muscle
2. Several surface landmarks on the thoracic wall correspond to the pleural borders. Relationships
between these landmarks and the pleura on the left side include
.
A. posteroinferior extension of the left costodiaphragmatic recess above the twelfth rib
B. lateral divergence of the left costomediastinal recess at the level of the fourth to the sixth rib
to form the cardiac notch
C. superior extension of the apex of the pleura below the level of the left clavicle
D. anteroinferior extension of the left pleural cavity below the costal margin in the region of
the xiphoid process
E. later inferior extension of the left costodiaphragmatic recess in the seventh rib
3. Paired air spaces in certain bones of the skull are called paranasal sinuses. Which of following
answer is not belong to paranasal sinuse?
A. maxillary sinus
B. ethmoidal sinus
C. frontal sinus
D. cavernous sinus
E. sphenoidal sinus
4. Which of following cartilage compose of the framework of the larynx?
A. thyroid cartilage
B. cricoid cartilage
C. epiglottic cartilage
D. arytenoid cartilage
E. all of above
5. Which of the paranasal sinuses listed below does not drain by gravity with the body in the erect
position?
A. frontal sinus
B. sphenoidal sinus
C. ethmoidal sinus
D. maxillary sinus
E. B and C
6. The right lung is different from the left lung in being
.
A. subdivided into ten bronchopulmonary segments
B. generally longer and narrower
C. divided by the oblique fissure
D. composed of three lobes
E. none of above
7. Structures located in the posterior mediastinum do not include the
.
A. thoracic duct
B. esophagus
C. hemiazygous vein
D. phrenic nerves
E. thoracic aorta
8. Which of the description about the lung listed follow is not correct?
A. Each lung extends from the diaphragm to a point just above the clavicle.
B. Each lung's surfaces are bordered by the ribs to the front and back.
C. Each lung has four surfaces that match the contour of the thoracic cavity.
D. The left lung is somewhat smaller than the right and has a cardiac impression on its medial
surface to accommodate the heart.
E. The right lung is subdivided into a superior lobe and an inferior lobe by a single fissure
9. Which of the following is a false statement?
A. The term respiration can be used in reference to ventilation (breathing) or oxygen
utilization by body cells.
B. The incoming (inhaled) air that contacts the pulmonary alveoli is unchanged from that
which surrounds the body.
C. As a resonating chamber, the nasal cavity is important in sound production.
D. It is only through the wails of the pulmonary alveoli that gaseous exchange occurs.
E. none of above
10. Which is not a component of the nasal septum?
A. the palatine bone
B. nasal septal cartilage
C. the ethmoid bone
D. the vomer
E. mucous membrane
11. Which of the following is not characteristic of the left lung?
A. a superior lobe
B. an inferior lobe
C. a cardiac impression
D. a single fissure
E. a middle lobe
12. The epithelial lining of the wall of the thoracic cavity is called the
.
A. costal pleura
B. mediastinal pleura
C. pleural peritoneum
D. parietal pleura
E. visceral pleura
13. The vocal folds (cords) are attached to the
.
A. cricoid and thyroid cartilages
B. cuneiform and cricoid cartilages
C. arytenoid and thyroid cartilages
D. corniculate and thyroid cartilages
E. cricoid and the corniculate
14. Which of the description about the pharynx is not correct?
A. The pharynx is a funnel-shaped organ. It contains a passageway that connects the oral and
nasal cavities with the esophagus and larynx.
B. The pharynx is divided on the basis of location and function into three regions.
C. The oropharynx is the middle portion, extending from the soft palate to the level of the
hyoid bone, contains the palatine and lingual tonsil.
D. The nasopharynx, connected by the auditory tubes to the tympanic cavities, contains the
lingual tonsils.
E. The laryngopharynx extends from the hyoid bone to the larynx and esophagus.
15. The trachea, right principal bronchus, left principal bronchus, lobar bronchi, segmental
bronchi and associated structures compose of the
A. bronchial tree
B. respiratory division
C. upper respiratory system
D. lower respiratory system
E. none of above
16. What structures support the very tip of the nose?
A. inferior nasal concha
B. lateral cartilages
C. lesser alar cartilages
D. greater alar cartilages
E. supreme nasal concha
17. The lower end of the larynx is formed by the
.
A. cricoid cartilage
B. thyroid cartilage
C. cuneiform cartilage
D. arytenoids cartilage
E. the first tracheal cartilage
18. Cartilage gives way to smooth muscle fiber at
.
A. respiratory bronchioles
B. segmental bronchi
C. lobar bronchi
D. alveolar ducts
E. principal bronchus
19. What is the force that causes air to flow into the lungs during inspiration and out if the lungs
during expiration?
A. intercostales externi muscle contraction
B. intercostales interni muscle contraction
C. surface tension
D. diaphragm movements
E. atmospheric pressure
20. The groove inferior to a nasal concha is known as a
.
A. papilla
B. meatus
C. canal
D. suture
E. Eustachian groove
21. Which of the following is true?
A. The laryngeal prominence is part of the cricoid cartilage.
B. The laryngeal prominence is larger in females than in males.
C. The cricoid cartilage is inferior to the thyroid cartilage.
D. The thyroid cartilage is the smallest of the laryngeal cartilages.
E. There are pairs of the epiglottic cartilage.
22. The nose serves all the following functions except:
A. providing resonance for voice production
B. passageway for air movement
C. warming and humidifying inspired air
D. cleansing inhaled air
E. absorbing oxygen
23. The description about the paranasal sinuses below which is wrong?
A. There are 4 pairs of paranasal sinuses.
B. Maxillary sinus communicates via drainage ducts within the oral cavity.
C. Paranasal sinuses may help warm and moisture the inspired air.
D. Paranasal sinuses are responsible for some sound resonance.
E. Paranasal sinuses function to decrease the weight of the skull while providing structural
strength.
24. Which structure listed below close the glottis during swallowing to prevent the foreign bodies
coming in the trachea?
A. hyoid bone
B. epiglottis
C. thyroid cartilage
D. cricoid cartilage
E. arytenoid cartilage
25. Which is not correct in the following description?
A. Pleurae are serous membranes surrounding the lungs and lining the thoracic cavity.
B. The visceral pleura adhere to the outer surface of the lung and extend into each of the
interlobar fissures.
C. The parietal pleura line the thoracic walls and the thoracic surface of the diaphragm.
D. Between the visceral and parietal pleurae is the slitlike pleural cavity.
E. The pleural cavity between the two moistened membranes contains large amount of fluid
to lubricate each other.
26. Normal rate of respiration in an adult human being is
times per minute.
A. 10-12
B. 12-14
C. 16-18
D. 22-24
E. 26-28
27. Which structure listed below doesn't appear in the laryngeal cavity?
A. aperture of larynx
B. vestibular fold
C. cricothyroid muscle
D. ventricle of larynx
E. vocal cord
28. Which structure is included in the hilum (root of lung)?
A. pulmonary artery
B. pulmonary vein
C. principal bronchus
D. associated nerve
E. all of above
29. Which is not correctly described below?
A. The alveolar ducts open into pulmonary alveoli as outpouching along their length.
B. Alveolar sacs are clusters of pulmonary alveoli.
C. The alveolar ducts, pulmonary alveoli, and alveolar sacs make up the respiratory
division of the lungs.
D. Gas exchange occurs across the walls of the alveolar ducts and the tiny pulmonary
alveoli.
E. Pulmonary alveoli are polyhedral in shape and are usually clustered together.
30. Which is not correct about the mediastinum described below?
A. The mediastinum lies between the right and left pleura in and near the median sagittal
plane of the chest.
B. It extends from the sternum in front to the vertebral column behind, and contains all the
thoracic viscera excepting the lungs.
C. May be divided for purpose of description into four parts.
D. The superior mediastinum is that portion of the interpleural space which lies between the
manubrium sternum in front, and thel-4 thoracic vertebrae behind.
E. The heart located in the anterior mediastinum.
A2
1. In case of chest trauma, an examining physician checks for increasing venous pressure, falling
arterial pressure, and increasing heart rate with diminishing heart sounds. These signs are
indicative of heart failure due to tamponade and necessitate immediate pericardiocentesis. At
which of the following sites may a needle effectively be inserted into the pericardial cavity
without risking a pneumothorax?
A. adjacent to the xiphoid process angling upward and to the left
B. adjacent to the sternum in the right fourth intercostal space
C. adjacent to the sternum in the left second intercostal space
D. in the right fourth intercostal space immediately below the nipple
E. in the left fourth intercostal space immediately below the nipple
2. Foreign bodies that are aspirated tend to be located in the right inferior lobar bronchus because
the
.
A. right main stem or principal bronchus is of greater diameter than the left
B. left principal bronchus is more horizontal than the right
C. right inferior lobar bronchus nearly continues the direction of the trachea
D. right lung has no middle lobe
E. all of above except D
3. Pulmonary disease can sometimes be localized to a bronchopulmonary segment, in which event
surgical resection may be feasible. Which of the following is not characteristic of a
bronchopulmonary segment that assists in its surgical definition?
A. an apex directed toward the hilum of the lung
B. a central segment artery
C. a central tertiary or segmental bronchus
D. a central vein
E. separated from each other by connective tissue septa
4. Which of the structure listed below is the marker of bifurcation of trachea during tracheoscope
examination?
A. tracheal cartilages
B. vestibular fold
C. carina of trachea
D. vocal fold
E. hilum of lung
5. When a patient has pleural effusion, usually the fluid aggregate in which part of the pleural
cavity in the erect position?
A. phrenicomediastinal recess
B. costomediastinal recess
C. costodiaphragmatic recess
D. retrocecal recess
E. intersigmoid recess
6. In clinical patient with lobar pneumonia, the inflammation usually localizes in
.
A. all the left or right lung
B. pulmonary lobes
C. bronchopulmonary segment
D. pulmonary lobule
E. pulmonary alveoli
BI
Movements of the chest wall and diaphragm produce changes in thoracic volume and
pulmonary ventilation. For each respiratory muscle action described below, select the factor to
which it is most nearly related.
A. a factor in quiet expiration
B. a factor in quiet inspirationC. a factor in exertional expiration
D. an accessory factor in exertional inspiration
E. not a factor in respiration
1. contraction of the intercostales externi muscles
2. contraction of the rectus abdominis muscle
3. contraction of the diaphragmatic muscle
4. contraction of the sternocleidomastoid muscle
5. contraction of the obliquus internus abdominis muscle
True or False Questions
1. The upper respiratory tract consists of the nose, pharynx and larynx and trachea.
(
)
2. The nose mucosa can be divided into two parts: the olfactory region and the respiratory region.
(
)
3. The sphenoidal sinus opens their mouth at semilunar hiatus of middle nasal meatus.
(
)
4. Thyroid cartilage is the largest cartilage in the larynx, the superior extremity of the anterior horn
is called laryngeal prominence and that is obvious in adult males.
(
)
5. The laryngeal cavity can be divided into laryngeal vestibule, infraglottic cavity and intermediate
cavity.
(
)
6. The left principal bronchus is short, straight and thick, and the subcarinal angle is small, so the
foreign particles are more likely to lodge here.
(
)
7. The apex of lung usually is 2. 5 cm below the level of the medial 1/3 of the clavicle.
( )
8. Pleura are serous membrane surrounding the lungs and lining the thoracic cavity. The visceral
pleura adhere to outer surface of the lung and extend into each of the interlobar fissures.
(
)
9. On the anterior aspect of pericardium of this region, there are no pleura, so the left xiphocostal
angle is the safe region for pericardiopuncture in clinic.
(
)
10. The anterior mediastinum is very narrow, which contains thymus or its remnant, principal
bronchi, anterior mediastinal lymph nodes, mediastinal branches of internal thoracic artery,
loose connective tissue, and stern pericardial ligament, etc.
(
)
Explanation of Terms
1. paranasal sinuses
2. conus elasticus
3. vocal fold
4. carina of trachea
5. bronchial tree
6. pleural cavity
7. root of lung
Answer the Following Questions
1. What is the composition of the nasal septum?
2. Explain the difference between the lung of fetus and adult?
3. Describe the Blood supply of lungs?
4. Describe the composition of the costodiaphragmatic recess?
5. Describe the projection of the lung?
ANSWERS
Multiple Choice Questions
A1
1. D
2. B
3. D
4. E
5. E
6. D
7. D
8. E
9. B
10. A
11. E
12. A
13. C
14. D
15. A
16. D
17. E
18. A
19. E
20. B
21. C
22. E
23. B
24. B
25. E
26. C
27. C
28. E
29. D
30. E
A2
1. A
2. E
3. D
4. C
5. C
6. B
B1
1. B 2. C 3. B 4. D 5. C
True or False Questions
1. F
2. T
3. F
4. T
5. T
6. F 7. F 8. T 9. T 10. F
Explanation of Terms
1. The paranasal sinuses are the gas cavities in cranium around the nasal cavity, and debouch at
nasal cavity. There are four pairs of paranasal sinuses including frontal sinus, ethmoidal sinus,
sphenoidal sinus and maxillary sinus. The paranasal sinuses have the effects of warming and
moistening the inspired air, generating resonance of pronunciation as well as decreasing the
weight of skull.
2. The conus elasticus is an elastic fibrous membrane, which arises from the posterior aspect of
thyroid cartilage's anterior horn and shows fan-shape, then stops at the vocal process of the
arytenoids cartilage and the superior border of the cricoid cartilage outwards and posteriorly.
3. The vocal fold is constructed by vocal ligament, voealis and laryngeal mucous membrane.
4. There is a carina of trachea of half-moon on the inner aspect of bifurcation of trachea, and it is
of anteroposterior axes and upward, and deviates to the left side, so it is an important sign of
judging the bifurcation of trachea by bronchoscope test.
5. The bronchial tree is so named because it is composed of a series of respiratory tubes that
branch into progressively narrower tubes as they are extending into the lung.
6. Between the visceral and parietal pleurae is the slitlike pleural cavity. It contains a lubricating
fluid that allows the membranes to slide each other easily during respiration.
7. All the pulmonary vessels, bronchi, nerves and lymphatic vessels are wrapped by connective
tissues, called root of lung.
Answer the Following Questions
1. The nasal septum is constructed by the perpendicular plate of the ethmoid bone, vomer and
septal cartilage, with mucous membrane covering, and usually deviates on one side.
2. The lungs of fetus and newborn which have never respired have no air and with a bigger
specific gravity and can sink into the water. On the contrary, the lungs of respirators have a
smaller specific gravity, and can float on the water. The weight of fetus's lung is 1/70 of their
body weight, and the volume of which is about a half of that of thoracic cavity. In the
development process of lung, the fetus’s lung of 3 months before birth grows fastest, and the
volume of lung after birth occupies about 2/3 of thoracic cavity's volume. The lungs of a fetus
are pink but may become discolored in an adult as a result of smoking or air pollution. In the
normal condition, the fetus's lung will mature in the 36th week of duration of pregnancy. Before
that, the lung's type II alveolar cells in premature infant before the 36th week duration of
pregnancy isn't mature and can't secret enough pulmonary surfactant, so respiratory distress
syndrome of newborn (RDS) easily takes place and the treatment of promoting the mature of
fetuses lung is needed.
3. The pulmonary artery is the functional blood vessels of transferring blood for gas exchange, the
branches of which situate at the anterior aspect of bronchus at first, then change direction to
posterior aspect. The branches of pulmonary artery in the lung are usually accompanied with the
branches of bronchus until they enter into the interalveolar septum and wrap the alveolar wall to
form alveolar capillary network. The left and right bronchial arteries are blood vessels of
trophism and usually have 1 to 4 branches. The left bronchial artery arises from thoracic aorta
and aortic arch, while the right one arises from the 3rd to the 5 th posterior intercostal arteries.
In the hilum of lung, the pulmonary arteries coincide with each other and form a network, then
enter into the lung and accompany closely with the bronchus, and enter into pulmonary segment
via the hilum of bronchopulmonary segment, then form 1 to 3 segmental bronchi arteries.
Finally, the segmental bronchial arteries form capillary network in the outer membrane and
submucosa of bronchial wall respectively to supply bronchus.
4. The costodiaphragmatic recess is formed by the returning of costal pleura and diaphragmatic
pleura, and is the part with lowest position and largest volume of all the pleural recesses. There
are two costodiaphragrnatic recesses, and the depth of which can reach the two intercostal
spaces. The fluid of pleural cavity often accumulates in the costodiaphragraatic recess at first.
5. The inferior borders of the two lungs have the similar projection on body surface, and the
inferior border of lung usually exceeds a distance of 2 ribs over the inferior border of pleura in
the related position. The lung intersect with the 6th rib on the mid-clavicular-line, with the 8th
rib on the mid-axillary line, with the 10th rib on the scapular line, then migrate with the
posterior border on the outside of the 11th spinous process of thoracic vertebra.
(Liu Fang, Fan Jun, Bai Shuling)
Multiple Choice Questions
A1
1. In its normal course, the right ureter passes
.
A. anteriorly to the renal artery
B. posteriorly to the ovarian artery
C. superiorly to the vas deferens
D. superiorly to the uterine artery
E. anteriorly to the ovarian artery
2. As a result d anatomic considerations, a nephrolith (stone) usually lodges at
.
A. the abdominal part of ureter
B. the pelvic part of ureter
C. the intramural part of the ureter
D. the apex of bladder
E. the trigone of bladder
3. In tracing the abdominal part of ureter, the ureter is observed to intersect the testicular vessel or
ovarian vessel and iliac blood vessels. In the following descriptions, which is correct?
A. anterior to the testicular vessel
B. anterior to the ovarian vessel
C. posterior to the ovarian vessel
D. posterior to the left common lilac artery
E. posterior to the right common iliac artery
4. Which isn't belongs to the capsule of kidney in the following?
A. the fibrous capsule
B. the fatty renal capsule
C. the renal fascia
D. the prerenal fascia
E. the muscular tunica
5. Which isn't correct about the description of the renal position?
A. It lies on the two sides of spinal column.
B. It lies at the upper part of posterior wall of abdomen.
C. The liver situates on the top of the right kidney.
D. The right kidney is lower than the left kidney.
E. It belongs to the intraperitoneal organ.
6. Which is incorrect about the positional relationship of kidney and vertebrae?
A. The superior extremity of left kidney lies at the inferior margin of the llth body of thoracic
vertebra.
B. The superior extremity of right kidney lies at the superior margin of the 12th thoracic vertebra.
C. The superior extremities of the two kidney approach each other, and their inferior extremities
are away from each other.
D. The kidney has the height of about two vertebrae.
E. The inferior extremity of the right kidney lies at the superior margin of the 3rd body of
lumbar vertebra.
7. Which is correct about the positional relationship of the 12th rib and kidney?
A. It goes through the upper part of right renal posterior surface.
B. ft goes through the upper part of left renal posterior surface.
C. It goes through the middle part of left renal anterior surface.
D. It goes through the middle part of right renal anterior surface.
E. It goes through the middle part of left renal posterior surface.
8. The structure of renal hilum doesn't include
.
A. renal blood vessels
B. nerves
C. renal pelvis
D. lymphatic vessel
E. minor renal calices
9. Which is incorrect about the description of renal shape?
A. It is a parenchymatous organ with the appearance of pea.
B. It can be divided into medial and lateral borders, anterior and posterior surfaces, superior and
inferior extremities.
C. Renal hilum is the depression of the middle part in renal medial border.
D. The connective tissue wrapping all the structures of renal hilum is called renal pedicle.
E. The right renal pedicle is longer than left renal pedicle.
10. Which is correct about the arranging relationship of all the structures in renal pedicle?
A. From front to behind, there are renal vein, renal artery and the end of renal pelvis.
B. From front to behind, there are renal artery, renal vein and the end of renal pelvis.
C. From up to down, there are renal vein, renal artery and the end of renal pelvis.
D. From up to down, there are the end of renal pelvis, renal artery and renal vein.
E. All the above aren't correct.
11. Which is incorrect about the description of renal structure?
A. Renal substance can be divided into renal cortex of surface layer and renal medulla of
deep layer.
B. The renal cortex can be divided into renal corpuscles and renal tubules.
C. The renal medulla is constituted of 15 to 20 renal pyramids.
D. The minor renal calices converge into major renal pelvis, then the latter converges and
forms renal pelvis.
E. The renal cortex has darker color than renal medulla.
12. Which of the following isn't correct about the description of ureter?
A. It arises from the end of renal pelvis, and terminates at urinary bladder.
B. It can be divided into abdominal part, pelvic part and intramural part.
C. There are three strictures in the whole course.
D. The left ureter steps over the anterior aspect of external iliac artery.
E. It descends along the anterior aspect of psoas major muscle.
13. Which is incorrect about the description of bladder?
A. The bladder volume of normal adult is about 350 to 500mi.
B. The bladder presents the shape of vertebral body with three edges.
C. It can be divided into four parts of apex, fundus, body and neck..
D. The fundus of bladder in male is adjoined with prostate gland.
E. The lowest part of bladder was called fundus of bladder.
14. Which is incorrect about the description of trigone of bladder?
A. It lies at the inner surface of fundus of bladder.
B. It is a triangular area formed by two ureteric orifices and an interureteric fold.
C. Its submucous layer and muscular layer are connected compactly with each other.
D. It maintains smooth all the time no matter extending or contraction.
E. It is the predilection site of tumor, concretion and inflammation.
15. Which is incorrect about the description of female urethra?
A. It is about 3 to 5cm long.
B. The character is short, straight and wide.
C. The internal orifice of urethra is surrounded by sphincter muscle of urinary bladder, which
is constructed by striated muscle.
D. The external orifice of urethra lies in the front of vaginal orifice.
E. The internal orifice of urethra crosses the accessory diaphragm and debouches at the
external orifice of vestibule of vagina.
16. Which is correct about the description of renal sinus?
A. It is the depression of renal hilum extended to renal parenchyma.
B. It is connected with the upper extremity of ureter.
C. It is constituted by renal medulla.
D. It contains renal artery and renal vein.
E. It is connected with the bottom of renal papilla.
17. Which of the following structures don't belong to renal medulla?
A. renal pyramid
B. renal papilla
C. papillary foramina
D. straight tubules
E. minor renal calices
18. From inside to outside, the arranging order of renal envelop is
.
A. fibrous capsule, fatty renal capsule and renal fascia
B. fibrous capsule, renal fascia and fatty renal capsule
C. renal fascia, fatty renal capsule and fibrous capsule
D. renal fascia, fibrous capsule and fatty renal capsule
E. fatty renal capsule, fibrous capsule and renal fascia
19. The quantity of renal pyramids in renal medulla is
.
A. 8-10
B. 10-12
C. 15-20
D. 20-25
E. 25-30
20. Which of the following belongs to renal cortex?
A. minor renal calices
B. major renal calices
C. renal pelvis
D. renal pyramid
E. renal column
21. Which is correct about the description of renal superior and inferior extremities?
A. The former is wide and thin, the latter is narrow and thick.
B. The former is narrow and thick, the latter is wide and thin.
C. The former is wide and thick, the latter is narrow and thin.
D. The former is narrow and thin, the latter is wide and thick.
E. All the above are false.
22. The renal region refers to
.
A. the included angle of the medial border of erector spinae and the 12th rib
B. the included angle of the exterior border of erector spinae and the 12th rib
C. the included angle of the medial border of erector spinae and the llth rib
D. the included angle of the exterior border of erector spinae and the llth rib
E. the included angle of the exterior border of latissimus dorsi muscle and the 12th rib
23. What is renal segment?
A. a renal vein and its supplying renal tissue
B. a renal pelvis and its affiliating renal tissue
C. a minor renal calix and its collecting renal tissue
D. a major renal calix and its collecting renal tissue
E. a renal artery distributes to the renal parenchyma of some region
24. Each kidney can be divided into
.
A. 3 renal segments
B. 4 renal segments
C. 5 renal segments
D. 6 renal segments
E. 7 renal segments
25. The renal segments of each kidney include
.
A. superior segment, superior anterior segment, inferior anterior segment, inferior segment and
posterior segment
B. superior segment, superior anterior segment, superior posterior segment, inferior segment
and inferior anterior segment
C. superior segment, superior anterior segment and inferior segment
D. superior segment, inferior segment, superior anterior segment and posterior segment
E. superior segment, superior anterior segment, superior posterior segment, inferior segment,
inferior anterior segment and posterior segment
26. The narrowest aperture of ureter is about
.
A. 0. 1-0. 2cm
B. 0.2-0. 3cm
C. 0. 3-0.4cra
D. 0.4-0. 5cm
E. 0. 5-0.6cm
27. The whole length of ureter is about
.
A. 10-15cm
B. 15-20cm
C. 20-30cm
D. 30-35cm
E. 35-40cm
28. The average caliber of ureter is about
.
A. 0.3-0.5cra
B. 0. 5-1.0cm
C. 1.0-1.5cm
D. 0.8cm
E. 1.0cm
29. In the ingress of small pelvis, the left ureter steps over
.
A. the anterior aspect of the end of left common iliac artery
B. the posterior aspect of the end of left common iliac artery
C. the anterior aspect of the end of left external iliac artery
D. the posterior aspect of the end of left external iliac artery
E. the anterior aspect of the end of left internal iliac artery
30. In the ingress of small pelvis, the right ureter steps over
.
A. the anterior aspect of the end of right common iliac artery
B. the posterior aspect of the end of right common iliac artery
C. the anterior aspect of the end of right external iliac artery
D. the posterior aspect of the end of right external iliac artery
E. the anterior aspect of the end of right internal iliac artery
31. The courser of abdominal part of ureter is
.
A. descends along the anterior aspect of abdominal aorta
B. descends along the anterior aspect of iliopsoas muscle
C. descends along the anterior aspect of psoas major muscle
D. descends along the anterior aspect of quadratus lumborum muscle
E. descends along the anterior aspect of erector spinae
32. The male pelvic part of ureter steps over which structure of the following?
A. ductus deferens
B. bulb of urethra
C. bulbourethral gland
D. ejaculatory duct
E. contorted seminiferous tubules
33. The female ureter lies at
.
A. the anterior aspect of uterine artery
B. the posteroinferior aspect of uterine artery
C. the anterolateral of uterine artery
D. the posterior aspect of uterine artery
E. the superior aspect of uterine artery
34. The first stricture of ureter lies at
.
A. the migration part of renal pelvis and ureter
B. the superior pelvic aperture
C. the crossing part of ureter and iliac blood vessels
D. the intramural part of ureter
E. all the above aren't correct
35. The second stricture of ureter lies at
.
A. the migration part of renal pelvis and ureter
B. the crossing part of ureter and iliac blood vessels
C. the intramural part of ureter
D. 3cra below renal pelvis
E. the pelvic part of ureter
36. The third stricture of ureter lies at
.
A. the migration part of renal pelvis and ureter
B. the intramural part of ureter
C. the crossing part of ureter and lilac blood vessels
D. the superior pelvic aperture
E. the outlet of pelvis
37. Which isn't correct about the description of urinary bladder?
A. The urinary bladder is an organ of capsular.
B. Its shape, size, position and the thickness of wall vary with the engorging extent of urine.
C. The normal volume of adult urinary bladder is 350 to 500ml.
D. The biggest volume of urinary bladder is 800ml.
E. The volume of newborn is about 1/5 of that of adult.
38. Where is the predilection site of bladder tumor?
A. internal orifice of urethra
B. ureteric orifice
C. trigone of bladder
D. interureteric fold
E. vesical uvula
39. The anterior adjacent of urinary bladder is
.
A. articulation of pubis
B. ileum
C. perineum
D. womb
E. colon
40. The posterior adjacent of urinary bladder is
.
A. articulation of pubis
B. colon
C. womb
D. perineum
E. ileum
41. Which of the following structure doesn't involve in cystoptosis?
A. puboprostatic ligament
B. pubovesical ligament
C. median umbilical fold
D. lateral umbilical fold
E. vesical plica
42. When the urinary bladder is engorged, which of the following statement is right?
A. It lies at the inner pelvic cavity totally.
B. The reflexion curve can migrate upward to the superior aspect of articulation of pubis.
C. The fundus of it can migrate upward to the articulation of pubis.
D. The neck of it migrates downward.
E. The trigone of bladder can migrate upward.
43. Which of the following isn't correct about the position of urinary bladder?
A. The position of urinary bladder in newborn is higher than that in adult.
B. The position of urinary bladder in old people is lower.
C. The position of urinary bladder in newborn is lower than that in adult.
D. The pricking usually processes at the superior aspect of articulation of pubis.
E. The urinary bladder lies at the pelvic cavity totally when it is void.
44. Which are the abnormity of kidney?
A. horseshoe kidney, polycystic kidney, double renal pelvis and ureters
B. polycystic kidney, double renal pelvis and ureters
C. mono-kidney, low-position kidney, double renal pelvis and ureters
D. mono-kidney, low-position kidney, horseshoe kidney
E. All of the above are correct.
45. Which of the following is the anatomic basement for renal transplantation?
A. The kidney has affluent blood circulation.
B. The donor's age is below 30.
C. The transplanted kidney usually is put into the abdominal cavity.
D. The renal veins can be coincided or not.
E. The immunosuppressive drug isn't very important for renal transplantation.
46. Which of the following doesn't belong to the posterior adjacent of kidney?
A. iliacus
B. psoas major
C. quadratus lumborum
D. transversus abdominis
E. diaphragm
A2
1. A male adult fall on a thick branch when he climbed on the tree. As a result, the scrotum
swelled, and urine was obstructed. After examination, we found that the abdominal wall was
inflated on the superior aspect of pubic symphysis, and there was a dullness region near
umbilical part by percussion. The urethral catheterization failed because of the swelling of
the private. Question. Where should the puncture be implemented to release urine?
A. on the superior aspect of pubic symphysis
B. on the inferior, aspect of pubic symphysis
C. on the superior aspect of prostate gland
D. on the inferior aspect of prostate gland
E. on the posterior aspect of prostate gland
2. A male patient, 50 years old, go to clinic for paroxysmal gross hematuria. Body examination
found that a lump can be palpated on the left abdomen and had light tenderness. The
pyelography found that the left kidney is lengthened, narrow and deformed. Question: the most
probable disease is
.
A. renal carcinoma
B. renal pelvic carcinoma
C. nephrohydrosis
D. renal cyst
E. kidney stone
3. A male patient, 40 years old, occurred gross hemamria without pain, obstructed urination and
frequent micturition repeatedly in recent one month. There were also deciduous tumor cells.
Besides, his left kidney, ureter and partial urinary bladder wall had been cut for left renal pelvic
carcinoma two years ago. Question: the probable diagnosis is
.
A. right kidney carcinoma
B. right renal pelvic carcinoma
C. right ureter carcinoma
D. urinary bladder carcinoma
E. urinary bladder tubercle
4. A male patient, 25 years old, undertook KUB one day. The result showed that both the two renal
pelvis had a dense shadow, with the size of lxl. 5crn, and the two ureters were obstructed. The
stone of two sides were shattered by ESWL. After operation, renal colic, fever and suppression
of urine occurred the second day. Question: the most probable diagnosis is
.
A. the obstruction of blood clot
B. contusion of kidney
C. the obstruction of gallet in the two ureters
D. acute tubular necrosis
E. acute pyelonephritis
5. A male patient, 36 years old, right solitary kidney, right waist colic pain, and anuresis for 3 days.
The examination showed that there was blood urine. Question: the most probable diagnosis
is
.
A. urinary calculus
B. urinary fistula
C. urethral stricture
D. contusion of kidney
E. acute pyelonephritis
6. A female patient, 40 years old, had right waist pain for I year. The examination found that her
right kidney occurred space occupying, and there was no blood urine or irritation sign of
bladder. The inferior extremity of right kidney could be palpated, and the right kidney region
had percussion pain. Besides, the urine routine and IVP were normal. CT indicated that the
inferior extremity of right kidney had space occupying, the size of which was about 3 X 3cra,
and the CT value was negative value. Question: the most probable diagnosis is
.
A. renal carcinoma
B. renal cyst
C. renal pelvic carcinoma
D. embryoma of kidney
E. angiomyolipoma of kidney
7. A male patient, 58 years old, occurred gross hematuria without pain many times in the recent 3
months. The urine routine showed that RBC+ + + +/HP. The ultrasonic B showed that there
seemed a lump in the right renal pelvis. Question: the most probable diagnosis is
.
A. the stone of renal pelvis
B. renal tuberculosis
C. renal pelvic carcinoma
D. renal carcinoma
E. embryoma of kidney
BI
A. adrenal gland
B. liver
C. diaphragm
D. the descending part of duodenum
E. the posterior aspect of the {undus of stomach
1. The upper 1/3 of the posterior kidney adjoins with
.
2. The superior extremity of kidney adjoins with
.
3. The anterosuperior part of right kidney adjoins with
.
A. the inferior border of the llth body of thoracic vertebra
B. the superior border of the 12th body of thoracic vertebra
C. the plane of the 1st body of lumber vertebra
D. the 7.nd to 3rd intervertebral disc
E. the superior border of the 3rd lumber vertebra
4. The superior extremity of right kidney is on the level with
.
5. The superior extremity of left kidney is on the level with
.
6. The inferior extremity of right kidney is on the level with
.
7. The inferior extremity of left kidney is on the level with
.
8. The renal hilum lies at
.
A. 1
B. 2 to 3
C. 5
D. 7 to 8
E. 15 to 20
9. Every kidney has
renal segments.
10. Every kidney has
minor renal calices.
11. Every kidney has
major renal calices.
12. Every kidney has
renal pelvis.
13. Every kidney has
renal pyramids.
True or False Questions
1. The left renal artery is shorter than the right.
(
)
2. The renal arteries are enclosed in renal fascia and surrounded with fatty renal capsule.
(
)
3. The autonomic nerve fiber isn't concluded in renal sinus.
(
)
4. The urinary bladder is covered by peritoneum on all surfaces.
(
)
5. The superior extremity of each kidney is capped by the suprarenal gland.
(
)
6. The perirenal space lies between the prerenal fascia and postrenal fascia.
(
)
7. The normal volume of adult's urinary bladder is 500 to 800ml.
(
)
8. The space between the bladder and symphysis pubis is called prevesical space or retropubic
space.
(
)
9. The female urethra has the length of 6 to 8cm.
(
)
10. The position of infant's bladder is higher than that of adult's, while that of old man's bladder is
lower.
(
)
Explanation of Terms
1. renal sinus
2. trigone of bladder
3. renal hilum
4. renal pedicle
5. fibrous capsule
6. segmental arteries
7. zone devoid of vessel
8. horseshoe kidney
Answer the Following Questions
1. What are the differences on position between the left and right kidney?
2. Try to state the position and adjacent of urinary bladder.
3. The quantity and names of renal segments.
4. The frequent malformation and abnormality of kidney.
5. The division of ureters.
6. Try to state the positions of strictures in the whole length of ureters.
7. The division of an empty bladder.
8. The position and meaning of interureteric fold.
ANSWERS
Multiple Choice Questions
A1
1. B
2. C
3. C
4. E
5. E
6. D
7. A
8. E
9. E
10. A
11. E
12. D
13. E
14. C
15. C
16. A
17. E
18. A
19. C
20. E
21. A
22. B
23. D
24. C
25. A
26. B
27. C
28. B
29. A
30. C
31. C
32. A
33. B
34. A
35. B
36. B
37. E
38. C
39. A
40. C
41. E
42. B
43. C
44. E
45. A
46. A
A2
1. A
2. A
3. D
4. C
5. A
6. E
7. C
B1
1. C
2. A
3. B
4. B
5. A
6. E
7. D
8. C
9. C
10. D
11. B
12. A
13. E
True or False Questions
1. T 2. T 3. F 4. F 5. T 6. T 7. F 8. T 9. F 10.:T
Explanation of Terms
1. A lacouna of blind sac, which enters into renal cortex from renal hilum. There are major renal
calices, minor renal calices, renal pelvis, branch of renal artery and renal veins, nerves,
lymphatic vessel and adipose tissue.
2. A triangular area known as the trigone of bladder is formed on the mucosa between the two
ureteric orifices and the single internal urethral orifice. The internal trigone lacks folds; it is
therefore smooth in-appearance and remains relatively fixed in position as the urinary bladder
changes shape during distension and contraction. The trigone of bladder is the predilection site
of tumor, tubercle and inflammation, and special attention should be paid when cystoscopy.
3. The hilum of the kidney is the depression along the medial border through which the renal
blood vessels, nerves, lymphatic vessels and renal pelvis enter or exit.
4. The structures which pass through the renal hilum are enclosed together by the connective
tissue, to form the renal pedicle.
5. The fibrous capsule, the innermost layer, is a strong, transparent fibrous attachment to the
surface of the kidney, and consists of dense connective tissue and elastic fiber.
6. The 5 secondary branches of renal artery often present segmental distribution, which are called
segmental arteries.
7. Each renal segment is supplied by its homonymous artery, and each renal segment is separated
by some connective tissue that is lack of blood vessel, which is called zone devoid of vessel.
8. The horseshoe kidney refers to the lower extremity of amphi-kidney connect to each other and
shows up a horseshoe form, and this malformation can cause hydronephrosis, infection or
calculus.
Answer the Following Questions
1. The upper extremity of the left kidney is on the level with the lower border of the body of the
eleventh thoracic vertebra, the lower extremity of the left kidney is between the second
intervertebral disc of lumbar vertebra and the third one. The upper extremity of the right kidney
is on the level with the upper border of the body of the twelfth thoracic vertebra, and the lower
extremity is on the upper border of the body of the third lumbar vertebra.
2. The symphysis pubis is in front of urinary bladder, and the space between the bladder and
symphysis pubis is called prevesical space or retropubic space. In male, there are medial and
lateral puboprostatic ligaments in this space. In female, the similar bands are termed
pubovesical ligament instead. The two connective tissue bands are behind pubic bone as well as
the tendinous arch of the pelvic fascia and link with the neck of bladder. In male, the posterior
part of bladder is adjoined with ampullae ductus deferentis, seminal vesicles and rectum. In
female, the posterior part of bladder is adjoined with uterus and vagina. The intermediate zone
of ampulla ductus deferentis of male is called triangle of ampulla ductus deferentis, which link
with ampulla of rectum via connective tissue, which is called rectovesical fascia. When the
bladder is empty it is placed entirely within the pelvis, while the bladder and line of peritoneal
reflection can move upward above symphysis pubis. The position of infant's bladder is higher
than that of adult's, and the internal orifice of urethra is at the level of superior border of pubis.
On the contrary, the position of old man's bladder is lower.
3. There are 5 renal segments, which are superior segment, anterior superior segment, anterior
inferior segment, inferior segment and posterior segment.
4. In the development process, the kidneys may appear malformation or the abnormality of
position and quantity, such as horseshoe kidney, polycystic kidney, double renal pelvis and
ureters, mono-kidney as well as low-position of kidney.
5. The subdivision of ureter includes three parts: the abdominal part of ureter, the pelvic part of
ureter and the intramural part of ureter.
6. There are three strictures in the whole length of ureters: the superior stricture, the middle
stricture and the inferior stricture. The superior stricture lies in the migration part of renal
pelvis and ureter, while the middle stricture locates in superior pelvic aperture where ureters
cross over iliac vessels, and the inferior stricture is situated at the ureteric orifice of intramural
part of ureter.
7. An empty urinary bladder is pyramidal; and can be divided into four parts, apex, fundus, body
and neck.
8. The fold between the'two ureteric orifices is' called interureterie fold, which is a pale band
observed by cystoscope, and it is a sign for finding ureteric orifice in clinic.
(Tian Xiaohong, Bai Shuling)
Multiple Choice Questions
A1
1. Which description is false about the male reproductive organs?
A. The testes are two glandular organs.
B. The epididymis consists of the head, body and tail.
C. The ductus deferens traverses the inguinal canal.
D. The ejaculatory duct opens into the urinary bladder.
E. The prostate is around the commencement of the urethra.
2. The internal genital organs of male do not include
.
A. testis
B. epididymis
C. ductus deferens
D. penis
E. seminal vesicle
3. The gonad of male reproductive system is
.
A. testis
B. prostate
C. epididymis
D. seminal vesicle
E. bulbourethral gland
4. The organ which can produce sperms is
.
A. testis
B. prostate
C. epididymis
D. seminal vesicle
E. bulbourethral gland
5. The organ which secretes male sex hormone is
.
A. testis
B. prostate
C. epididymis
D. seminal vesicle
E. bulbourethral gland
6. Sperms are produced in
.
A. straight seminiferous tubules
B. contorted seminiferous tubules
C. efferent ductules of testis
D. duct of epididymis
E. ductus deferens
7. The interstitial cells (cells of Leydig)
.
A. nourish sperms
B. produce testosterone
C. produce sperms
D. secrete alkaline fluid
E. produce testosterone and produce sperms
8. Where are the testes in a normal adult?
A. peritoneal cavity
B. abdominal cavity
C. scrotum
D. inguinal canal
E. pelvic cavity
9. All of the following are true except the
.
A. average weight of each testis is from 10 to 15 grams
B. anterior border of testis is free
C. epididymis is attached to anterior border of testis
D. blood vessels, nerves and lymphatics pass through the posterior border to enter or leave the
testis
E. tunica albuginea encapsulates each testis directly
10. All of the following are true except that
.
A. the function of testes is declined in old people
B. cryptorchidism is the condition that the testes do not descend into the scrotum after birth
C. the testes are contained in the abdominal cavity in early fetal life
D. the testes produce testosterone after birth
E. the descent of testes may be associated with the shortening and differential growth of the
gubernaculum
11. The efferent ductules of testis empty into the
.
A. rete testis
B. epididymis
C. ductus deferens
D. straight tubules
E. seminiferous tubules
12. Which is continuous with the duct of epididymis?
A. straight seminiferous tubules
B. contorted seminiferous tubules
C. efferent ductules of testis
D. ductus deferens
E. ejaculatory duct
13. Which statement is false about the testes?
A. Testes are paired organs.
B. Testes are located in the scrotum.
C. There are two tissue layers, or tunics, cover the testes.
D. The testis may be arrested or delayed in its transit to the scrotum.
E. The contorted seminiferous tubules produce sex hormone.
14. The ducts of the male reproductive system include the following except
.
A. testis
B. epididymis
C. ductus deferens
D. urethra
E. ejaculatory duct
15. Where is sperm stored?
A. testis
B. prostate
C. epididymis
D. seminal vesicle
E. bulbourethral gland
16. Which one belongs to the portion of the epididymis?
A. apex
B. base
C. tail
D. root
E. neck
17. Which description is false about the epididymis?
A. Epididymis is attached to the superior extremity and posterior surface of the testis.
B. Epididymis may be divided into head, body and tail.
C. The terminal ductule of epididymis is continuous to the ejaculatory duct.
D. The sperms are stored in epididymis.
E. The secretions nourish the sperms.
18. The spermatic cord may be divided into the following part except
.
A. testicular part
B. funicular part
C. inguinal part
D. pelvic part
E. prostatic part
19. The spermatic cord contains the following structures except the
.
A. testicular artery
B. pampiniform plexus
C. ejaculatory duct
D. nervous plexus
E. remnants of the vaginal process
20. The end of the ductus deferens is a dilated sac called the
.
A. spermatic cord
B. seminal vesicle
C. ampulla
D. ejaculatory duct
E. efferent ductules
21. Where is the place that the ligation of ductus deferens or vasectomy is performed usually?
A. the testicular part of ductus deferens
B. the funicular part of ductus deferens
C. the inguinal part of ductus deferens
D. the pelvic part of ductus deferens
E. the ampulla ductus deferentis
22. The part of the male reproductive tract which only carries semen within the prostate is
the
.
A. prostatic urethra
B. membranous urethra
C. seminal vesicle
D. ductus deferens
E. ejaculatory duct
23. The ejaculatory duct ejects the sperms with its additives into
.
A. membranous part of urethra
B. cavernous part of urethra
C. bulbar part of urethra
D. prostatic part of urethra
E. navicular fossa of urethra
24. The ejaculatory duct
.
A. courses through membranous part of urethra
B. courses through prostate
C. has function of storing sperms
D. opens into the bulbar part of urethra
E. is about 10cm long
25. Which statement is not true about the ejaculatory duct?
A. It is formed by the union of the ampulla ductus deferentis and the duct of the seminal
vesicle
B. It pierces the capsule of the prostate on its anterior surface.
C. It receives secretions from the seminal vesicles.
D. It ejects the sperms with its additives into the prostatic urethra.
E. It is about 2cm long.
26. Which of the following does not conduct sperms?
A. ampulla the ductus deferentis
B. duct of the seminal vesicle
C. epididymis
D. prostatic urethra
E. all of the above
27. Which belongs to the male accessory glands?
A. testis
B. epididymis
C. spermatic cord
D. scrotum
E. seminal vesicle
28. Which male accessory gland lies inferior to the urinary bladder and surrounds the urethra like
a donut?
A. epididymis
B. bulbourethral glands
C. ductus deferens
D. prostate
E. seminal vesicles
29. Seminal vesicles
.
A. are posterior to the prostate
B. can store the sperms
C. are unpaired
D. secrete fluid to make up semen
E. excrete their fluid into the bulbar part of urethra
30. The prostate
.
A. contains upper, middle and lower lobes
B. encircles the urethra
C. is well imaged radiologically using an intravenous urogram
D. is extraperitoneal
E. Both B and D are correct.
31. The prostate
.
A. is anterior to pubic symphysis
B. excretes its fluid into the bulbar part of urethra
C. is the male gonad
D. can secrete a little sex hormone
E. is passed by male urethra
32. The prostate
.
A. are paired parenchymatous organs
B. is chestnut-shaped organ with the apex is upward and base is downward
C. may be felt through the rectum
D. presents a sulcus of prostate in the front surface of the body
E. is usually divided into 3 lobes
33. The duct of prostate opens into
.
A. prostatic part of urethra
B. membranous part of urethra
C. bulbar part of urethra
D. cavernous part of urethra
E. anterior urethra
34. Which of the following produce(s) a secretion that helps maintain the motility and viability of
sperms?
A. prostate
B. penis
C. bulbourethral glands
D. ejaculatory duct
E. urethral glands
35. The ducts of bulbourethral glands open into
.
A. prostatic part of urethra
B. membranous part of urethra
C. bulbar part of urethra
D. cavernous part of urethra
E. posterior urethra
36. Whose secretions do not make up semen?
A. bulbourethral glands
B. epididymis
C. seminal vesicle
D. prostate
E. scrotum
37. The accessory reproductive gland(s) that contribute(s) the most volume to the semen is/are
the
.
A. seminal vesicles
B. bulbourethral glands
C. urethral glands
D. greater vestibular glands
E. prostate gland
38. The dartos coat is a layer of smooth muscle fibers found within
.
A. the scrotum
B. the penis
C. the epididymis
D. the prostate
E. the testis
39. Where do the testes originate in a fetus?
A. scrotum
B. pelvic cavity
C. abdominal cavity
D. retroperitoneal space
E. inguinal canal
40. The failure of a testis to descend is referred to as
.
A. inguinal hernia
B. strangulated hernia
C. vaginal process
D. cryptorchidism
E. hydrocele
41. Concerning the penis, the
.
A. root (fixed portion) of the penis is attached to the ischiopubic rami
B. cavernous body of urethra is found dorsal to the cavernous body of penis
C.tunica albuginea is a thickened layer of connective tissue surrounding the
ischiocavernosus muscles
D. membranous part of urethra contains a dilated portion called the navicular fossa
E. the glans penis is the expanded distal portion of the cavernous body of penis
42. Which is the correct division of the penis?
A. apex
B. base
C. body
D. tail
E. neck
43. Which portion belongs to the penis?
A. apex
B. base
C. root
D. tail
E. neck
44. The bulb and crus of the penis are located at
.
A. the glans penis
B. the corona glandis
C. the body of the penis
D. the prepuce
E. the root of the penis
45. The narrowest portion of the male urethra is the
.
A. internal urethral orifice
B. membranous part
C. external urethral orifice
D. cavernous part
E. prostatic part
46. Which is not correct about male urethra?
A. serves as a common tube for both the urinary and reproductive systems
B. presents 3 dilations
C. presents 2 curvatures
D. presents 3 narrows
E. is divided into prostatic part, cavernous part and bulbar part
47. The dilation part of male urethra is the
.
A. prostatic part
B. membranous part
C. cavernous part
D. internal urethral orifice
E. external urethral orifice
48. Which is the anterior urethra in the clinic?
A. prostatic part of urethra
B. membranous part of urethra
C. cavernous part of urethra
D. bulbar part of urethra
E. prostatic and membranous part of urethra
49. Which is the posterior urethra in the clinic?
A. prostatic part of urethra /
B. membranous part of urethra
C. cavernous part of urethra
D. bulbar part of urethra
E. prostatic and membranous part of urethra
50. Urethral glands function to
.
A. secrete mucus
B. produce nutrients
C. secrete hormones
D. regulate sperms production
E. secrete acid phosphatase
A2
1. After agreeing to have no more children, a man and his wife decided he should have a
vasectomy. Which structure would then be surgically ligated?
A. ductus deferens
B. ejaculatory duct
C. epididymis
D. navicular fossa
E. seminal vesicle
2. During a vasectomy, the ductus deferens is ligated in the superior part of the scrotum. Two months
following this sterilization procedure, the subsequent ejaculate contains mainly
.
A. prostatic fluid only
B. seminal fluid and prostatic fluid
C. sperms only
D. sperms and seminal fluid
E. sperms, seminal fluid, and prostatic fluid
3. The prostate is often imaged using an ultrasound transducer placed in which location?
A. penis
B. perineum
C. rectum
D. urethra
E. urinary bladder
4. An elderly male patient presents with dysuria and urgency. You suspect benign prostatic
hypertrophy which has caused an enlargement of the
.
A. interureteric crest
B. prostatic utricle
C. seminal colliculus
D. urethral sphincter
E. uvula
5. An elderly patient is having difficulty in voiding (urinating)- He complains that after voiding,
he still feels as though he needs "to go' again. You suspect that this patient suffers from benign
prostatic hypertrophy, which has caused enlargement of the
of the bladder.
A. seminal colliculus
B. interureteric crest
C. ampulla
D. trigone
E. uvula
6. A 65 years old male patient complains of difficulty in urinating. When the doctor touches inside
his anus, which structure may be enlarged?
A. seminal vesicle
B. prostate
C. bulbourethral gland
D. ampulla ductus deferentis
E. urinary bladder
7. When the doctor catheterizes for a male patient because of dysuria and retention, which
statement is not correct?
A. The penis should be raised to eliminate the prepubic curvature.
B. The catheter courses through the cavernous body of urethra.
C. The urethra courses through the prostate.
D. The catheter passes through three narrow of the urethra in turn, which are external orifice,
membranous part and prostatic part.
E. The urethra is 16 to 22cm long
BI
A. prostatic part of the urethra
B. membranous part of the urethra
C. cavernous part of the urethra
D. internal orifice of the urethra
E. external orifice of the urethra
1. The shortest part of the male urethra is
.
2. The longest part of the male urethra is
.
3. The anterior urethra in the clinic is
.
4. The ejaculatory duct opens into
.
A. testis
B. epididymis
C. scrotum
D. prostate
E. bulbourethral gland
5. The organ which may be touched inside the rectum
6.The organ stores the sperms
7. The largest male accessory gland
8.The organ which opens into bulbar part of the urethra
A. tunica albuginea
B. dartos coat
C. tunica vaginalis
D. contorted seminiferous tubules
E. straight seminiferous tubules
9. The sperms are produced in
.
10. The structure which contains smooth muscles is
.
11. The structure which encloses testis and epididymis is
.
True or False Questions
1. The functions of the testis are to produce sperms and male sex hormones.
(
)
2. The left testis hanging somewhat higher than its fellow.
(
)
3. There are 300-400 lobules per testis
(
)
4. The tunica albuginea is a capsule of connective tissue associated with the testis.
(
)
5. The seminiferous tubules are highly coiled duct in which sperms are stored for maturation.
(
)
6. The interstitial endocrinecytes are located between seminiferous tubules and secrete
testosterone.
(
)
7. The epididymis is one of the male accessory glands.
(
)
8. The epididymis is an elongated organ attached to the posterior surface of the testis.
(
)
9. The duct of epididymis tightly coiled tubes, composed of cells that develop into sperms. ( )
10. The tail of epididymis is continuous with the beginning portion of the ductus deferens. (
)
11. The funicular part of the ductus deferens is the best portion for ligation.
(
)
12. The ductus deferens enters the pelvic cavity through the inguinal canal.
(
)
13. The spermatic cord consists of the structures to testis except the cremaster.
(
)
14. From the external to internal, the three layers of tunicae enclosed the spermatic cord are
external spermatic fascia, cremaster and internal spermatic fascia.
(
)
15. The ejaculatory duct ejects sperms and fluid into the urethra just before ejaculation. (
)
16. The ejaculatory duct is formed by the union of the ampulla ductus deferentis and the duct of
the bulbourethral gland.
(
)
17. The seminal vesicles lie at the apex of the urinary bladder.
(
)
18. The prostate is situated in the pelvic cavity, below the lower part of the pubic symphysis. ( )
19. The base of prostate is directed downward, and is applied to the inferior surface of the bladder.
(
)
20. The prostate is immediately enveloped by a thin but firm fibrous capsule.
(
)
21. The discharge from the prostate makes up about 40 % of the volume of the semen.
(
)
22. The secretions of seminal vesicle contribute to the motility and viability of sperms.
(
)
23. The semen contains sperms and the fluid form seminal vesicles, prostate and bulbourethral
glands.
(
)
24. The scrotum is subdivided into two longitudinal compartments by a fibrous scrotal septum.
(
)
25. Contraction of the dartos muscle is responsible for elevation of the testes during sexual arousal
or on exposure to cold.
(
)
26. The root of penis is attached to the ischiopubic rami and expands posteriorly to form the bulb
of the penis and the crus of the penis.
(
)
27. The mass of tissue that surrounds the cavernous part of the urethra is called the cavernous
body of the penis.
(
)
28. The prostatic part of the urethra is the widest and most dilatable part of the canal.
(
)
29. The membranous part of the urethra is the narrowest part of the canal.
(
)
30. The prostatic part and membranous part of urethra are often called anterior urethra in the clinic.
(
)
Explanation of Terms
1. interstitial cells
2. spermatic cord
3. vaginal cavity
4. ampulla ductus deferentis
5. ejaculatory duct
6. urethral crest
7. bulb of urethra
8. prepubic curvature
9. seminal vesicle
10. sulcus of prostate
11. cryptorchidism
12. prepuce of penis
Answer the Following Questions
1. What are the divisions of ductus deferens?
2. What is the penis composed of?
3. What are the lobes of the prostate?
4. What do the male accessory reproductive glands include?
5. What are the divisions of male urethra?
6. Which muscles help regulate the temperature of the testes?
7. What are the groups of reproductive organs in males, and what are the functions of each group?
8. Where are the sperms produced? Which ducts are the sperms transported to the outside?
9. What are the morphology and location of the prostate?
10. Where are the narrows and dilations of male urethra located?
11. What are the structures of the testis?
12. How does the testis descend to the scrotum?
ANSWERS
Multiple Choice Questions
A1
1. D
2. D
3. A
4. A
5. A
6. B
7. B
8. C
9. C
10. D
11. B
12. D
13. E
14. A
15. C
16.C
17. C
18.E
19. C
20. C 21. B
22. E
23. D
24. B
25.B
26. B
27. E
28. D
29.D
30. E
31.E
32.C
33. A
34.A
35.C
36.E
37. A 38. A
39. D
40. D
41. A
42.C
43. C
44.E
45. C
46. E
47. A
48. C
49.E
50. A
A2
1. A
2. B
3. C
4. C
5. E
6. B
7. D
BI
1. B
2. C
3. C
4. A
5. D
6. B
7. D
8. E
9. D
10. B
11. C
True or False Questions
1. T
2. F
3. F
4. T 5. F
6. T
7. F
8. T
9. F
10. T
11. T 12. T 13. F
14. T
15. T
16. F
17. F
18. T 19. F 20. T
21. T
22. T
23. T
24. T
25. F 26. T 27. F 28. T 29. F 30. F
Explanation of Terms
1. The interstitial cells (cells of Leydig) are specialized endocrine cells between the seminiferous
tubules. The function of these cells is to produce and secrete the male sex hormones.
2. The spermatic cord extends from the testis to the inguinal canal and consists of the ductus
deferens, the testicular artery and pampiniform plexus, nerves, the cremaster muscle, lymph
vessels, remnants of the vaginal process and connective tissue.
3. The tunica vaginalis is a thin serous sac derived from the peritoneum during the descent of the
testes, which may be divided into parietal layer and visceral layer. The space between the two
layers is called vaginal cavity.
4. The ampulla ductus deferentis is the dilated terminal portion that joins the ejaculatory duct.
5. The ejaculatory duct is formed by the union of the ampulla ductus deferentis and the duct of the
seminal vesicle. The ejaculatory duct then pierces the capsule of the prostate on its posterior
surface and continues through this gland.
6. The urethral crest is a narrow longitudinal ridge upon the posterior wall or floor of the
prostatic urethra.
7. The posterior portion of the cavernous body of urethra expands to form the bulb of urethra.
8. The prepubic curvature lies anteroinferior to the pubic symphysis, which may straighten or
disappear upon erection of the penis.
9. The seminal vesicles are convoluted club-shaped glands lying at the base of the urinary bladder,
in front of the rectum. The seminal vesicle is one of the male accessory reproductive glands.
10. The lower larger portion of prostate presents a shallow median sulcus on its posterior surface.
11. If the testis is arrested or delayed in its transit to the scrotum, it is called cryptorchidism.
12. The skin of the body of the penis is loosely attached over the glans penis as a protective
retractable sheath called the prepuce.
Answer the Following Questions
1. The ductus deferens is divided into four parts, which are testicular part, funicular part, inguinal
part and pelvic part.
2. The penis is composed of three cylindrical columns of erectile tissue, which are the paired
cavernous bodies of penis and single cavernous body of urethra.
3. The prostate is usually divided into five lobes, which are anterior, posterior, middle and two
lateral lobes.
4. The accessory reproductive glands of the male include the seminal vesicles, the prostate, and
the bulbourethral glands.
5. The male urethra includes prostatic part, membranous part and cavernous part.
6. The cremaster and dartos muscles help regulate the temperature of the testes.
7. The gonads (testes) produce sperms and hormones; the ducts transport, store, and receive
sperms; and the accessory sex glands secrete materials that support sperms.
8. The sperms are produced in contorted seminiferous tubules, and transported to the epididymis
for storage via a series of efferent ductules of testes. Then the sperms are transferred to the
female through ductus deferens, ejaculatory duct and urethra in turn during ejaculation.
9. The prostate is about the size of a chestnut and somewhat conical in shape, and presents for
examination a base, an apex, an anterior, a posterior and two lateral surfaces. The part between
base the apex is also named body of prostate. The lower larger portion of prostate presents a
shallow median sulcus on its posterior surface. The prostate is situated in the pelvic cavity,
below the lower part of the pubic symphysis, above the superior fascia of the urogenital
diaphragm, and in front of the rectum, through which it may be distinctly felt, especially when
enlarged.
10. There are three narrows in male urethra, which are the internal orifice, the membranous part
and the external orifice. There are also three dilations, which are located in prostatic part, the
bulb of urethra and the navicular fossa of the urethra.
11. Two tissue layers, or tunics, cover the testes. The outer is tunica vaginalis and inner is tunica
albuginea. The later partitions the testis into 100 to 200 wedge-shaped testicular lobules.
Each lobule of the testis contains 2 to 4 tightly convoluted seminiferous tubules which
produce sperms. Between the contorted seminiferous tubules are specialized endocrine cells
called interstitial cells. Once the sperms are produced in the contorted seminiferous tubules,
they move through the contorted seminiferous tubules and enter a series of very short ducts
called straight seminiferous tubules. The straight tubules lead to the rete testis. The sperms are
transported out of the testis and into the epididymis through a series of efferent ductules.
12. The process by which a testis descends is not well understood, but it seems to be associated
with the shortening and differential growth of the gubernaculum, which is attached to the
testis and extends through the inguinal canal to the wall of the scrotum. As the testis descends
about week 28, it passes to the side of the urinary bladder and anterior to the pubic symphysis.
It carries with it the ductus deferens, the testicular vessels and nerve, a portion of the internal
abdominal oblique muscle, and lymph vessels. All of these structures remain attached to the
testis and form what is known as the spermatic cord. By the time the testis has taken its
position in the scrotal sac, the gubernaculum is no more than a remnant of scar-like tissue.
(Yang Xiangqun)
Multiple Choice Questions
A1
1. The suspensory ligament of the ovary
.
A. connects the ovary to the uterus
B. is continuous with the round ligament
C. passes inferiorly to the ureter
D. contains the ovarian artery and vein
E. is firmly attached to the bifurcation of the aorta
2. The ovary
.
A. lies against the anterior pelvic wall
B. lies within the ovarian fossa, which is bounded superiorly by the internal iliac vessels
C.receives its blood supply from the ovarian artery,which courses through the ovarian ligament
to reach the ovary
D. has lymphatic drainage which goes to lumbar lymph nodes
E. releases ova directly into the isthmus of the uterine tube
3. The duct of the greater vestibular gland opens
.
A. at the posterior-lateral edge of the introitus of the vagina
B. just superior to the hymen
C. at the posterior labial commissure
D. between the labium minus and labium majus
E. between the introitus and the urethra
4. Anterior extensions of the labia minima meet in the midline to form the
.
A. mons pubis
B. introitus
C. body of the clitoris
D. frenulum of clitoris
E. opening of the urethra
5. The extension of the vaginal lumen around the intravaginal part of the uterine cervix is
the
.
A. cervical canal
B. uterine lumen
C. fornix
D. rectouterine pouch
E. uterovesical pouch
6. Under normal conditions, fertilization occurs in which part of the female reproductive tract.'?
A. infundibulum of the uterine tube
B. ampulla of the uterine tube
C. isthmus of the uterine tube
D. uterine lumen
E. cervical canal
7. Which of the following is considered a part of the broad ligament?
A. mesovarium
B. ovarian ligament
C. round ligament of the uterus
D. suspensory ligament of the ovary
E. uterosacral ligament
8. The most inferior extent of the peritoneal cavity in the female is the
.
A. pararectal fossa
B. paravesical fossa
C. rectouterine pouch
D. parauterine fossa
E. vesicouterine pouch
9. The vestibular bulbs/bulb of the corpus spongiosum are firmly attached to the
.
A. perineal membrane
B. sacrotuberous ligament
C. ischiopubic rami
D. pubic symphysis
E. ischial tuberosities
10. A structure which takes the form of a hood anterosuperior to the clitoris?
A. frenulum of the clitoris
B. labia majora
C. labia minora
D. prepuce
E. all of the above
11. A structure which is homologous to the male scrotum?
A. labia minora
B. labia majora
C. glans
D. shaft of corpus cavernosum
E. all of the above
12. The part of the internal lining of the uterus which is NOT shed during menstruation is
the
.
A. endometrium
B. myometrium
C. mesometrium
D. cervical mucosa
E. rugae
13. During ovulation, a(n)
is released from the ovary.
A. ovum
B. secondary oocyte
C. primary oocyte
D. oogonium
E. follicle
14. Which layer of the uterus is shed once monthly during menstruation?
A. the myometrium
B. the perimetrium
C. the endometrium
D. the cervix of uterus
E. the middle layer
15. What is the term for the primary human sex organs?
A. gonads
B. genitals
C. ova
D. gametes
E. uterus
16. How many mature, functional follicles will a female produce in a lifetime?
A. about 1 million
B. 400
C. 4,000
D. 350,000
E. 1,000
17. What is the source of female sex steroids?
A. pituitary
B. ovary
C. uterus
D. hypothalamus
E. brain
18. What does an ovulated oocyte first come into contact with?
A. uterine tube
B. fimbriae
C. uterus
D. oviduct
E. vagina
19. The largest ovarian ligament is the
ligament.
A. ovarian
B. broad
C. suspensory
D. sacral
E. proper
20. Which cell has been fertilized by a sperm cell?
A. primary oocyte
B. secondary oocyte
C. ovum
D. oogonia
E. follicle
21. During ovulation, the oocyte passes into the
.
A. uterine tube
B. uterus
C. pelvic cavity
D. ovary
E. vagina
22. What is the first structure to receive the oocyte?
A. fimbriae
B. uterine tube
C. infundibulum
D. uterus
E. vagina
23. The largest component of the uterus by weight is the
A. broad ligament
B. myometrium
C. round ligament
D. endometrium
E. perimetrium
24. The superior portion of the uterus is the
.
A. body
B. cervix
C. fundus
D. infundibulum
E. isthmus of uterus
25. The outer serosal layer of the uterus is the
.
A. perimetrium
B. cervix
C. endometrium
D. myometrium
E. isthmus of uterus
26. The superior region of the vagina is the
.
.
A. vaginal orifice
B. cervix
C. hymen
D. vaginal rugae
E. isthmus of uterus
27. Which of these includes the other terms?
A. labia
B. clitoris
C. vulva
D. vestibular glands
E. mons pubis
28. What structure corresponds to the scrotum in the male?
A. labia major
B. labia minor
C. pudendal cleft
D. mons pubis
E. clitoris
29. The
is the space between the labia minor.
A. vulva
B. vestibule
C. vaginal orifice
D. pudendal deft
E. mons pubis
30. The female greater vestibular glands correspond to the male
A. prostate
B. Cowper' s
C. seminal vesicle
D. glans
E. bulbourethral glands
31. The notable structure of uterine tube or recognition is
.
A. mesosalpinx
B. infundibulum of uterine tube
C. fimbriae of uterine tube
D. suspensory ligament of ovary
E. broad ligament of uterus
32. What area experiences the greatest changes in a menstrual cycle?
A. vagina
B. perimetrium
C. cervix
D. endometrium
E. myometrium
33. Where does fertilization normally occur?
A. uterus
B. uterine tube
.
C. ovary
D. pelvic cavity
E. vagina
34. Where does implantation of the blastocyst occur?
A. uterine tubes
B. endometrium
C. cervix
D. ovary
E. pelvic cavity
35. Which structure will actually become the fetus?
A. morula
B. blastocyst
C. inner cell mass
D. zygote
E. ova
36. The uterine vessels cross the
superiorly.
A. ureter
B. round ligament of the uterus
C. ovarian artery
D. lumbosacral trunk
E. inferior hypogastric plexus
37. The uterine and ovarian vessels can be found in which ligaments?
A. broad and ovarian
B. broad and suspensory
C. round and ovarian
D. round and suspensory
E. suspensory and ovarian
38. Which one is formed by peritoneal plica?
A. round ligament of uterus
B. cardinal ligament of uterus
C. broad ligament of uterus
D. sacrouterine ligament of uterus
E. suspensory ligament of ovary
39. About uterus, which statement is right?
A. It lies between urinary bladder and rectum.
B. It belongs to the intraperitoneal organ.
C. It contains fundus, body, neck and isthmus.
D. Neck of uterus lies superiorly to vagina.
E. The lower third of the cervix has been termed the isthmus of uterus.
40. Uterus appendix refers to
.
A. urinary bladder and rectum
B. urinary bladder and ovary
C. ovary and uterine tube
D. uterine tube and rectum
E. uterine tube and urinary bladder
41. Which one is the main structure to keep anteversion of uterus?
A. broad ligament of uterus
B. round ligament of uterus
C. cardinal ligament of uterus
D. uterosacral ligament of uterus
E. proper ligament of ovary
42. Which one is the main structure to avoid metroptosis?
A. broad ligament of uterus
B. round ligament of uterus
C. cardinal ligament of uterus
D. uterosacral ligament of uterus
E. proper ligament of ovary
43. The accessory gland of the female internal genital organs is
A. ovary
B. uterine tube
C. uterus
D. vagina
E. greater vestibular gland
44. The common site for tubal ligation is
.
A. uterine part
B. isthmus
C. ampulla
D. infundibulum
E. fimbriae
45. Which organ produces ovum?
A. ovary
B. uterine tube
C. uterus
D. vagina
E. greater vestibular gland
46. Which organ secretes female sex hormone?
A. ovary
B. uterine tube
C. uterus
D. vagina
E. greater vestibular gland
47. Which one belongs to the intraperitoneal organ?
A. vermiform appendix
B. sigmoid colon
C. uterus
D. adrenal gland
E. superior part of duodenum
48. The tumor predilection site of uterus is
.
.
A. fundus of uterus
B. body of uterus
C. cervix of uterus
D. isthmus of uterus
E. horn of uterus
A2
1. Which structure is outlined with contact on a CT using intraperitoneal contact material?
A. ovary
B. prostate
C. rectum
D. seminal vesicles
E. vagina
2. During a hysterectomy, the uterine vessels are ligated.However, the patient's uterus continues to
bleed. The most likely source of blood still supplying the uterus is from which artery?
A. inferior vesical
B. internal pudendal
C. middle rectal
D. ovarian
E. superior vesical
3. While recovering from an open abdominal hysterectomy (i.e., using a midline abdominal
incision to gain entry to the pelvis), a patient realizes that she has lost sensation to the skin of
her anterior thigh and cannot extend her knee. Retractors holding the incision open and
pressing against the posterior abdominal wall most likely caused injury to which nerve?
A. femoral
B. genitofemoral
C. iliohypogastric
D. lateral femoral cutaneous
E. obturator
4. You are observing a doctor perform an abdominal hysterectomy. He notes that it is vitals protect
the ureter which is found in the base of the
.
A. mesometrium
B. mesovarium
C. mesosalpinx
D. round ligament of the uterus
E. suspensory ligament of the ovary
5. Following pregnancy and delivery, a 32-year-old woman continued to have problems with
urinary incontinence, which developed during pregnancy. Her obstetrician counseled her to
strengthen the muscle bordering the vagina and urethra, increasing its tone and exerting
pressure on the urethra. This physical therapy was soon adequate to restore urinary continence.
What muscle was strengthened?
A. coccygeus
B. ischiocavernosus
C. obturator internus
D. piriformis
E. puborectalis
6. After giving birth, a patient complains of urinary stress incontinence characterized by dribbling
of urine with an increase in intraabdominal pressure. Her physician suspects injury to the
pelvic floor during delivery, which may have altered the position of the neck of bladder and the
urethra. Which muscle was most likely damaged during the vaginal delivery?
A. bulbospongiosus
B. coccygeus
C. levator ani
D. obturator internus
E. piriformis
7. A gynecologist examines a 27-year-old woman. Upon rectal examination, a firm structure,
directly in front of the rectum in the midline, is palpated through the anterior wall of the
rectum. This structure is the
.
A. bladder
B. body of uterus
C. cervix of uterus
D. pubic symphysis
E. vagina
8. During a vaginal delivery, a surgeon performed medial episiotomy in which he cut too far (i. e. ,
through the perineal body into the structure immediately posterior). Which perineal structure
did he cut?
A. bulbocavernosus
B. external anal sphincter muscle
C. ischiocavernosus
D. sacrospinous ligament
E. sphincter urethrae
9. A patient complains of a boil located on her labia majora. Lymphatic spread of the infection
would most likely enlarge which nodes?
A. lumbar nodes
B. sacral nodes
C. external iliac nodes
D. superficial inguinal nodes
E. internal iliac nodes
10. A female patient is found to have an ectopic (tubal) pregnancy (embryo develops in the uterine
tube). In order to gain access to the peritoneal cavity endoscopically to remove the embryo,
the instrument can be passed into the vagina and through the
.
A. anterior fornix
B. cervix
C. posterior fornix
D. retropubic space
E. vesicouterine pouch
BI
A. uterine part of uterine tube
B. isthmus of uterine tube
C. ampulla of uterine tube
D. infundibulum of uterine tube
E. fimbriae of uterine tube
1. The tubal ligation site is
.
2. The fertilization site is
.
3. The terminal structure of uterine tube is
.
A. fundus of uterus
B. body of uterus
C. cervix of uterus
D. horn of uterus
E. isthmus of uterus
4. The part superior to the uterine orifice of uterine tube is
.
5. The tumor predilection site is
.
6. The lower uterine segment can be formed by
.
A. ovary
B. uterine tube
C. cervix of uterus
D. round ligament of uterus
E. proper ligament of ovary
7. Digital rectal examination can touch
.
8. Structure maintains anteversion and anteflexion of uterus.
9. Duct transports ovum or spermatovum.
10. In vitro fertilization, to obtain ovum from
.
True or False Questions
1. Ovarian follicles are located in the ovarian cortex.
(
)
2. The ovarian medulla contains cells that will differentiate into the oocytes.
(
)
3. Females produce hundreds of oocytes each month in comparison to the millions of sperm cells
produced in the same age male.
(
)
4. The vagina is anterior to the bladder and posterior to the rectum.
(
)
5. Although prolactin levels are high during pregnancy, milk production does not occur until after
delivery.
(
)
6. Like the testes, the female gonads-ovaries are endocrine glands that produce gametes. (
)
7. The primary reproductive organ of the female is the uterus.
(
)
8. Puberty in females starts with changes in the hypothalamus that causes the release of FSH and
LH from the pituitary gland.
(
)
9. The mixture of blood and the cells that make up the lining of the vagina is called menstrual
fluid.
(
)
10. The lower entrance to the uterus is called the cervix. Leading from the cervix to the outside of
the body is a muscular tube called the vagina or birth canal.
(
)
11. The female genital gland is the vestibular glands.
(
)
12. Ovary can produce ova and secrete sex hormones.
(
)
13. The inferior extremity of the ovary, also called tubal extremity, is positioned near the open end
of the uterine tube.
(
)
14. The principal supporting membrane of the female reproductive tract is the ovarian
ligament.
(
)
15. The uterine tube consists of 4 parts and 2 orifice. The 4 parts are the uterine part, the isthmus,
the ampulla and the infundibulum. The 2 orifice are the uterine orifice and the abdominal orifice.
(
)
16. The uterus receives the follicle and provides a site for implantation.
(
)
17. The uterus contains 3 parts, the fundus of uterus, the body of uterus and the isthmus of uterus.
(
)
18. The canal of cervix of uterus is a part of the cavity of uterus.
(
)
19. The whole perimetrium is continuous with the broad ligament.
(
)
20. The vesicouterine pouch is the lowest point in the pelvic cavity and provides a site for
puncture entry into the peritoneal cavity via vagina.
(
)
21. Anteversion is formed between the body of uterus and the cervix.
(
)
22. Anteflexion is formed between the uterus and vagina.
(
)
23. The broad ligament of uterus is divided into three portions, the mesovarium, mesosalpinx, and
the mesometrium, which support their corresponding organs.
(
)
24. The round ligament of uterus lies in the broad ligament of uterus.
(
)
25. The vagina is situated at the center of the pelvis between the urinary bladder and the rectum,
and is continuous with uterine tube.
(
)
26. The fornix of vagina is the deep recess surrounding the protrusion of the cervix into the vagina.
(
)
27. The pudendal cleft lies between the labia minora.
(
)
28. The clitoris is an erectile structure, homologous with the penis in male.
(
)
29. The greater vestibular glands are the homologues of prostate in male.
(
)
30. Suspensory ligaments extend from the skin to the deep fascia overlying the pectoralis major
muscle and support the breasts.
(
)
Explanation of Terms
1. mesovarium
2. fimbriae of uterine tube
3. isthmus of uterus
4. vesicouterine pouch
5. rectouterine pouch
6. cardinal ligaments
7. anteversion
8. anteflexion
9. fornix of vagina
10. suspensory ligament of mammary glands
Answer the Following Questions
1. Describe the relationship between the cervix, the vagina, and the uterus.
2. Describe the division of uterine tube.
3. Which structures in females are homologous to the testes, the glans penis, the prostate gland,
and the bulbourethral gland in male?
4. Describe the composition of vulva.
5. How does the fornix of vagina form?
6. Describe the composition and functions of female reproductive system.
7. Describe the position and support of ovaries.
8. Describe the position, division, cavity, and support of uterus.
9. Describe the pathway of ovum or fertilized ovum discharge and normal fetal disengagement.
10. Describe the structure of mammary glands.
ANSWERS
Multiple Choice Questions
A1
1. D
2. D
3. A
4. D
5. C
6. B
7. A
8. C
9. B
10. D
11. B
12. B
13. B
14. C
15. A
16. B 17. B
18. B
19. C
20. B
21. C
22. A
23. B
24. C
25. A
26. B
27. C
28. D
29. B
30. E
31. C
32. D
33. B
34. B
35. B
36. A
37. B
38. C
39. A
40. C
41. B
42. C 43. E
44. B
45. A
46. A
47. C
48. C
A2
1. A 2. D 3. A
4. A
5. E
6. C
7. C
8. A
9. B
10. C
BI
1. B 2. C
3. E 4. A 5. C
6. E 7. C
8. D
9. B
10. A
True or False Questions
1. T 2. F 3. F 4. F 5. T 6. T 7. F 8. T 9. F 10. T 11. F 12. T 13. F
14. F 15. T 16. F 17. F 18. F 19. F 20. F
21. F 22. F 23. T 24. T 25. F
26. T 27. F 28. T 29. F 30. T
Explanation of Terms
1. The mesovarium is a specialized posterior extension of the broad ligament that attaches to an
ovary.
2. Many slender digitationes formed around the terminal end of uterine tube near its abdominal
orifice.
3. Approximately the upper third of the cervix has been termed the isthmus of uterus.
4. The vesicouterine pouch is a shallow pouch and formed as the peritoneum is reflected over the
urinary bladder. It lies between urinary bladder and uterus.
5. The rectouterine pouch formed as the peritoneum is reflected onto the rectum. It lies between
rectum and uterus. The rectouterine pouch is the lowest point in the pelvic cavity and provides a
site for puncture entry into the peritoneal cavity via vagina.
6. The cardinal ligaments are fibrous bands located at the base of the broad ligaments that extend
laterally from the cervix and vagina across the pelvic floor, where they attach to the wall of the
pelvis.
7. About 90~ angle formed between the uterus and vagina.
8. About 170~ angle formed between the body of uterus and the cervix.
9. The deep recess surrounding the protrusion of the cervix into the vagina is called the fornix of
vagina.
10. Suspensory ligaments lie between the lobules, extend from the skin to the deep fascia
overlying the pectoralis major muscle and support the breasts.
Answer the Following Questions
1. The anatomical regions of the uterus include fundus of uterus, body of uterus and neck or cervix
of uterus. The narrow canal of cervix of uterus extends through the cervix and opens into the
lumen of the vagina.
2. The uterine tubes are described as consisting of four portions. (1) The uterine part, which lies in
the wall of the uterus; (2) the isthmus, where is the place of tubosterilization; (3) the ampulla,
a dilated portion for fertilization normally, which curves over the ovary; and (4) the
infundibulum with its abdominal orifice, surrounded by fimbriae, one of which, the ovarian
fimbria is attached to the ovary.
3. The ovaries are homologous to the testes; the clitoris is homologous to the glans penis; the
paraurethral glands are homologous to the prostate; and the greater vestibular glands are
homologous to the bulbourethral gland.
4. The vulva includes the mons pubis, labia majora, labia minora, clitoris, vaginal vestibule,
vestibular bulbs of vestibule, and vestibular glands. The structures of the vulva surround the
vaginal vestibule.
5. The cervix projects posteriorly and inferiorly, joining the anterior wall of the vagina at nearly a
right angle, which divides it into upper, supravaginal part and lower, vaginal part.
The deep recess surrounding the protrusion of the cervix into the vagina is called the fornix of
vagina. To the recess behind the cervix the term posterior fornix (part) is applied, while the
anterior and lateral parts are smaller called the anterior and lateral fornices as well.
6. The female genital organs consist of internal and external organs. The internal organs are
situated within the pelvis, and consist of the ovaries, the uterine tubes, the uterus, and the vagina.
The external organs comprise the mons pubis, the great lip of pudendum (labia majora), the
lesser lip of pudendum (labia minora), the clitoris, the vaginal vestibule, the bulb of vestibule,
the vestibular glands and the mammary glands.
The functions of the female reproductive system are (1) to produce ova; (2) to secrete sex
hormones; (3) to receive the semen from the male during coitus; (4) to provide sites for
fertilization, embryonic and fetal development; (5) to facilitate parturition, or delivery of the
baby; and (6) to provide nourishment for the baby through the mammary glands in the breasts.
7. Each ovary lies in a shallow depression between internal and external iliac arteries, named the
ovarian fossa, on the posterior wall of the pelvis.
Each ovary is secured by several membranous attachments. The principal supporting membrane
of the female reproductive tract is the broad ligament. The mesovarium is a specialized
posterior extension of the broad ligament that attaches to an ovary. Each ovary is additionally
supported by a proper ligament of ovary (ovarian ligament), which is anchored to the uterus,
and a suspensory ligament of ovary, which is attached to the pelvic wall.
8. The uterus is situated at the center of the pelvis between the urinary bladder and the rectum, and
forms anteversion (about 90°angle) between the uterus and vagina; besides, the body of uterus
and the cervix forms an angle about 170°called anteflexion. The anatomical regions of the
uterus include 3 parts: (1) the uppermost dome-shaped portion superior to the entrance of the
uterine tubes, called the fundus of uterus; (2) the enlarged main portion, called the body of
uterus; and (3) the inferior constricted portion opening into the vagina, called the neck (cervix)
of uterus. Approximately the upper third of the cervix has been termed the isthmus of uterus.
The cavity of the uterus is small in comparison with the size of the organ, partly due to its thick
wall. The cavity of uterus is the space within the fundus and body of the uterus. The narrow
canal of cervix of uterus extends through the cervix and opens into the lumen of the vagina. The
opening of the canal of cervix of uterus into the vagina is called the uterine orifice.
Four paired ligaments support the uterus in position within the pelvic cavity. (!) Broad
ligaments, extend from the pelvic walls and floor to the lateral walls of the uterus. (2)
Uterosacral ligaments, curve along the lateral pelvic wall on both sides of the rectum to connect
the uterus to the sacrum. (3) Cardinal ligaments, locate at the base of the broad ligaments that
extend laterally from the cervix and vagina across the pelvic floor to the wall of the pelvis. (4)
round ligaments, extend from the lateral border of the uterus just below the point where the
uterine tube attaches to the greater lips of pudendum.
9. ovum→peritoneal cavity → abdominal orifice of uterine tube → infundibulum of uterine
tube → ampulla of uterine tube → fertilized ovum → isthmus of uterine tube →uterine part
of uterine tube → uterine orifice of uterine tube → cavity of uterus → endometrium →
fetus → orifice of uterus → vaginal orifice → outside of body.
10. Each mammary gland is composed of 15 to 20 lobes, each with its own drainagepathway to
the outside. The lobes are separated by varying amounts of adipose tissue.The amount of
adipose tissue determines the size and shape of the breast but has nothing to do with the ability
ralveoli. The mammary alveoli are the structures that produce the milk of a lactating female.
Suspensory ligaments between the lobules extend from the skin to the deep fascia overlying the
pectoralis major muscle and support the breasts. The clustered mammary alveoli secrete milk
into a series of mammary ducts that converge to form lactiferous ducts. The lumen of each
lactiferous duct expands near the nipple to form a lactiferous sinus. Milk is stored in the
lactiferous sinuses before draining at the tip of the nipple.
(Liu Fang)
Appendix: The Perineum
Multiple Choice Questions
A1
1. Hepatoduodenal ligament contains
.
A. hepatic veins
B. splenic vein
C. hepatic portal vein
D. left gastric vein
E. celiac trunk
2. Which structure divides the peritoneal cavity into supracolic and infracolic compartment?
A. lesser omentum
B. transverse mesocolon
C. greater omentum
D. greater omentum
E. mesentery
3. Which of the following organs doesn't belong to intraperitoneal organ?
A. stomach
B. uterus
C. jejunum
D. transverse colon
E. spleen
4. Which of the following organs doesn't belong to intraperitoneal organ?
A. ovary
B. liver
C. urinary bladder
D. ascending and descending colon
E. urinary bladder
5. Which of the following organs doesn't belong to retroperitoneal organ?
A. ureters
B. suprarenal glands
C. kidneys
D. lower part of the rectum
E. transverse colon
6. About the lesser omentum, which statement is not true?
A. It is a double layer of peritoneum.
B. It contains hepatogastric ligament, hepatoduodenal ligament and gastrocolic ligament.
C. The hepatogastric ligament contains the right and left gastric arteries.
D. The hepatoduodenal ligament encloses the bile duct, the proper hepatic artery, hepatic
portal vein.
E. It connects the inferior surface of the liver to the lesser curvature of the stomach and the
upper part of the duodenum.
7. About the greater omentum, which statement is not true?
A. It is a biggest fold of the peritoneum.
B. It always contains some adipose tissue, blood vessels and macrophages.
C. It reaches inferiorly as an apron-like double layer of the peritoneum.
D. It contains hepatogastric ligament, hepatoduodenal ligament.
E. The anterior and posterior leaves usually fuse.
8. Which of the following organs has no mesentery?
A. jejunum
B. ileum
C. vermiform appendix
D. sigmoid colon
E. ascending colon
9. Which of the following muscles is not contained in the urogenital triangle?
A. superficial transverse muscle of perineum
B. bulbocavernosus
C. sphincter of urethra
D. coccygeus
E. ischiocavernosus
A2
1. While performing a splenectomy (removal of the spleen) following an automobile accident, the
surgeons were especially attentive to locate and preserve the tail of the pancreas, which is
closely associated with the spleen. This they found in the
.
A. gastrocolic ligament
B. gastrosplenic ligament
C. phrenicocolic ligament
D. splenorenal ligament
E. transverse mesocolon
2. During an emergency splenectomy, the surgeon accidentally tore the gastrosplenic ligament and
its contents. The artery that is likely to be damaged in this event is(are) the
.
A. left gastric
B. splenic
C. middle colic
D. short gastric
E. caudal pancreatic
BI
A. suspensory ligament of duodenum
B. hepatogastric ligament
C. falciform ligament of liver
D. ligamentum teres hepatis
E. gastrocolic ligament
1. The mark of the commencement of jejunum
2. Formed by lesser omentum
3. Formed by greater omentum
A. falciform ligament
B. coronary ligament
C. ligamentum teres hepatis
D. ligamentum venosum
E. hepatoduodenal ligament
4. It extends from the umbilicus to the notch for the ligamentum teres hepatic.
5. It contains the bile duct, the proper hepatic artery and hepatic portal vein.
6. Lower free border contains the ligamentum teres hepatic.
True or False Questions
1. The subphrenic peritoneum has greater absorptive capacity than other portions of the
peritoneum.
(
)
2. The stomach, jejunum, liver, vermiform appendix are all intraperitoneal organs.
(
)
3. Kidneys, suprarenal glands, ureters are all retroperitoneal organs.
(
)
4. The effect of levator ani is supporting pelvic floor and organs in the pelvic cavity. They also
have an important sphincter action on the anal canal and vagina.
(
)
5. Hepatogastric ligament encloses the bile duct, the proper hepatic artery, hepatic portal vein.
(
)
6. The greater omentum is usually thin, and presents a cribriform appearance, but always contains
some adipose tissue, blood vessels and macrophages.
(
)
7. By the omental foramen the omental bursa communicates with the (greater) peritoneal cavity.
(
)
8. The sigmoid mesocolon is a triangular peritoneal fold that attaches the sigmoid colon to the
abdominal wall.
(
)
9. The anterior layer of coronary ligament is continuous with the falciform ligament.
(
)
10. Duodenal folds include the anterior duodenal fold and posterior duodenal fold.
(
)
11. The rectouterine pouch in female or the rectovesical pouch in male is the lowest part of the
peritoneal cavity in anatomical position.
(
)
12. The suprahepatic space is divided into right and left suprahepatic spaces by the coronary
ligament.
(
)
13. Mesenteric sinuses lie between the radix of the mesentery and the ascending and descending
colons.
(
)
14. The right paracolic sulcus lies along the lateral side of the ascending colon, inferiorly it
continues with the right subhepatic space.
(
)
15. Suprahepatic space lies between the superior surface of liver and the diaphragm.
(
)
16. In anal triangle there are the levator ani, coccygeus, bulbocavernosus and sphincter ani
externus.
(
)
Explanation of Terms
1. peritoneal cavity
2. intraperitoneal organ
3. intraperitoneal organ(2、3 重复)
4. retroperitoneal organ
5. lesser omentum
6. greater omentum
7. omental bursa
8. mesentery
9. ischioanal fossae
10. pelvic diaphragm
11. urogenital diaphragm
Answer the Following Questions
1. Give at least 5 examples of intraperitoneal organ..
2. Give at least 5 examples of intraperitoneal organ.(1、2 重复)
3. Give at least 5 examples of retroperitoneal organ.
4. Describe briefly the division of the levator ani.
5. Describe briefly the walls of omental bursa.
6. What is mesentery? And take some examples.
7. Describe briefly the divisions of suprahepatic space.
8. Describe briefly the divisions of subhepatic space.
9. Describe briefly the borders of the broad sense of the perineum.
ANSWERS
Multiple Choice Questions
A1
1. C
2. B
3. B
4. A
5. E
6. B
7. D
8. E
9. D
A2
1. D 2. D
BI
1. A 2. B 3. E 4. C 5. E 6. A
True or False Questions
1. T 2. T 3. T 4. T 5. F 6. T 7. T 8. F 9. T 10. F
11. T
12. F
13. T
14. F
15. T
16. F
Explanation of Terms
1. The potential space between the parietal and visceral peritonea is the peritoneal cavity.
2. If an organ is almost completely surrounded with peritoneum which in the abdominal and pelvic
cavity, it is called the intraperitoneal organ.
3. The intraperitoneal organ is covered by peritoneum on their three aspects.
4. Only the anterior part of these organs covered by the peritoneum is called the retroperitoneal
organ.
5. It is a double-layer of peritoneum that connects the inferior surface of the liver to the lesser
curvature of the stomach and the upper part of the duodenum.
6. It is a biggest fold of the peritoneum extending downwards from the greater curvature of the
stomach to lie anteriorly within the abdominal cavity.
7. There is a potential, narrow space between the lesser omentum, the posterior wall of the
stomach and the posterior wall of the abdomen, which is called omental bursa.
8. The mesentery is a large, fan-shaped fold of peritoneum connecting the coils of jejunum and
ileum to the posterior abdominal wall.
9. In anal triangle there are two 'large spaces on both sides of the anal canal, which are called the
ischioanal fossae.
10. The levator ani and coccygeus together with the superior and inferior fascia of pelvic
diaphragm form the pelvic diaphragm which encloses the anal triangle. The anus passes through
the center of pelvic diaphragm.
11. The superior and inferior fasciae of urogenital diaphragm, the deep transverse muscle of
perineum, and the urethral sphincter muscle form the urogenital diaphragm, which encloses the
urogenital triangle.
Answer the Following Questions
l. Stomach, superior part of duodenum, jejunum, cecum, vermiform appendix, transverse colon,
sigmoid colon, spleen, ovary, uterine tube.
2. Liver, gallbladder, urinary bladder, uterus, upper part of the rectum, ascending and descending
colons.
3. Kidneys, suprarenal glands, ureters, major part of duodenum, pancreas (except for its tail), and
lower part of the rectum.
4. Levator ani is divided into pubovaginalis (in female), puborectalis, pubococcygeus and
iliococcygeus.
5. Its anterior wall is formed by the lesser omentum, the posterior wall of the stomach and the
gastrocolic ligament. The posterior wall is formed by the peritoneum covering the pancreas, the
left kidney and left suprarenal gland. The superior wall is the inferior surface of the caudate
lobe of the liver and the peritoneum covering diaphragm. The inferior wall is the transverse
mesocolon and the site that the anterior and posterior layers of the greater omentum fuse. On
the left, it is bounded by he spleen, the gastrosplenic and splenorenal ligaments. On the right,
the omental bursa communicates with the greater sac through the omental foramen.
6. Mesenteries are peritoneal folds that attach viscera to the posterior abdominal wall. They allow
some movement and provide a conduit for vessels, nerves and lymphatics to reach the viscera
and include the mesentery, mesoappendix, transverse mesocolon and sigmoid mesocolon.
7. This space is divided into suprahepatic space and subhepatic space by the liver. The
suprahepatic space is divided into right and left suprahepatic spaces by the falciform ligament.
The right and left suprahepatic spaces are subdivided into right anterior and posterior
suprahepatic spaces, left anterior and posterior suprahepatic spaces by the coronary ligament.
8. This space is divided into right and left subhepatic spaces by the ligamentum teres hepatis, the
right subhepatic space is usually called hepatorenal recess, the left subhepatic space is
subdivided into the left anterior and left posterior subhepatic spaces by the lesser omentum and
stomach.
9. Its anterior margin is formed by the inferior border of the pubic symphysis, the posterior margin
is the apex of coccyx, the lateral margin is surrounded by the inferior rami of pubis, rami of
ischium, ischial tuberosities, sacrotuberous ligaments.
(Zhang Yafang, Li Xiaodong)
Multiple Choice Questions
A1
1. The components of vascular system are
.
A. the cardiovascular system and the lymphatic system
B. the heart, the artery, the capillary and the vein
C. the cardiovascular system and the lymphatic organ
D. the heart, the artery, the vein and the lymphatic vessel
E. the cardiovascular system and the lymphatic vessel
2. Which is the accurate describing about the shape of the heart?
A. the cardiac apex consists of left and right ventricles
B. the whole cardiac base consists of the left atrium
C. the posterior of the aortic root is the left atrium
D. the coronary sulcus locate between right and left ventricles
E. the sulcus terminalis separates the left atrium and right atrium
3. The valves forcing antidromic blood during the ventricular diastole are
.
A. the aortic valve and the tricuspid valve
B. the aortic valve and the mitral valve
C. the tricuspid valve and the mitral valve
D. the aortic valve and valve of pulmonary trunk
E. the valve of pulmonary trunk and the tricuspid valve
4. Which consists of the fight border of the heart?
A. the right ventricle
B. the right atrium
C. the right ventricle and right atrium
D. the right ventricle and the inferior vena cava
E. the right atrium and the inferior vena cava
5. Which consists of the cardiac valve?
A. the endocardium and fibrous connective tissue
B. the epicardium
C. the cardiac muscle
D. the fibrous membrane
E. the connective tissue
6. The left fibrous trigone is between
.
A. the left fibrous ring of aortic valve and the fibrous ring of tricuspid valve
B. the left fibrous ring of aortic valve and the fibrous ring of mitral valve
C. the fibrous ring of tricuspid valve and the fibrous ring of mitral valve
D. the left fibrous ring of the aortic valve, the fibrous ring of tricuspid valve and the fibrous
ring of mitral valve
E. none of the above
7. The heart muscle contractions are coordinated by
.
A. mechanical timing devices throughout the heart
B. chemical signals.
C. electrical impulses through the conduction system
D. a single electrical signal since the entire heart contracts at once
E. none of the above
8. The sinoatrial node in the heart receives ks blood supply principally from
.
A. the right coronary artery
B. the circumflex branch of the left coronary artery
C. the posterior interventricular branch of the right coronary artery
D. the anterior interventricular branch of the left coronary
E. none of the above
9. In a normal cardiac cycle, the first site to receive an impulse is
.
A. the sinuatrial node
B. the atrioventricular node
C. the atrioventricular bundle
D. the bundle branch
E. the Purkinje fiber
10. The atrioventricular node receives its blood supply principally from
.
A. the initial segment of the right coronary artery
B. the initial segment of the left coronary artery
C. the right coronary artery (in atrioventricular junction)
D. the anterior interventricular branch of the left coronary artery
E. the anterior interventricular branch of the right coronary artery
11. The pulmonary trunk arises from
.
A. the left atrium
B. the right atrium
C. the left ventricle
D. the right ventricle
E. none of the above
12. The atrioventricular node
.
A. is the pacemaker when the cardiac muscle contract
B. is supplied by the anterior interventricular branch of the left coronary artery
C. locates in the membranous part of interventricular septum
D. locates in the muscular part of interventricular septum
E. none of the above
13. The coronary sinus enters
.
A. the left atrium
B. the right atrium
C. the right ventricle
D. the left ventricle
E. the inferior yens cava
14. Which is the first branch from the right of the aortic arch?
A. the right common carotid artery
B. the left subclavian artery
C. the brachiocephalic trunk
D. the left common carotid artery
E. the right subclavian artery
15. The complex of the heart valve doesn't include
.
A. the mitral annulus and the tricuspid annulus
B. the tip of valve
C. the chordae tendineae
D. the papillary muscles
E. the trabeculae carneae
16. The function of the papillary muscles is
.
A. to open the atrioventricular valve when the ventricles diastole
B. to close the atrioventricular valve when the ventricles contract
C. to fix the chordae tendineae on the atrioventricular valve
D. to participate the construction of the complex 0f the heart valve
E. none of the above
17. The transverse sinus of pericardium locates
.
A. in the antero-lower part of the pericardial cavity
B. between the inferior yen& cava and the posterior wall of the pericardium
C. between the superior vena cava and right pulmonary artery
D. between the ascending aorta and the pulmonary artery
E. none of the above
18. The branches of the aortic arch from right to left are
.
A. the brachiocephalic trunk, the right common carotid artery and the right subclavian artery
B. the brachiocephalic trunk, the right common carotid artery and the left subclavian artery
C. the right subclavian artery, the right common carotid artery and the brachiocephalic trunk
D. the brachiocephalic trunk, the left common carotid artery and the left subclavian artery
E. the left mn carotid artery, the left subclavian artery and the brachiocephalic trunk
19. The internal carotid artery
.
A. arises from the brachiocephalic trunk
B. nutrition the brain and visual organ
C. enter the cranial cavity through the foramen magnum
D. has branches in the outside of the cranium
E. none of the above
20. The middle meningeal artery arises from
.
A. the common carotid artery
B. the internal carotid artery
C. the maxillary artery
D. the mandibular artery
E. the superficial temporal artery
21. Which is the branch of the maxillary artery?
A. the ascending pharyngeal artery
B. the superior thyroid artery
C. the lingual artery
D. the facial artery
E. the middle meningeal artery
22. The branches of the external carotid artery do not include
.
A. the vertebral artery
B. the occipital artery and the posterior auricular artery
C. the lingual artery and the facial artery
D. the superior thyroid artery
E. the superficial temporal artery and the maxillary artery
23. When the temporal region is bleeding, in order to stop bleeding we can press
.
A. the maxillary artery
B. the superficial temporal artery
C. the facial artery
D. the external carotid artery
E. the angular artery
24. Which is correct as follows about the internal thoracic artery?
A. The last branch is the inferior epigastric artery.
B. The internal thoracic artery is the continuation of the subclavian artery.
C. The branches of the internal thoracic artery divisions to the pericardium and the
diaphragm.
D. The internal thoracic artery arises from the subclavian artery through the spatium
intermusculare of the scalenus muscle.
E. The internal thoracic artery arises from the external carotid artery.
25. Which is correct as follows about the brachial artery?
A. The brachial artery is the continuation of the subclavian artery.
B. The brachial artery courses on the back side of the radius.
C. The brachial artery closely follows the medial border of the biceps brachii.
D. The brachial artery locates in the lateral of the biceps brachii in the cubital fossa.
E. The brachial artery accompanies the median nerve to the cubital fossa.
26. The components of the conduction system are
.
A. the sinoatrial node, atrioventricular node, atrioventricular bundle
B. the sinoatrial node, atrioventricular node
C. the sinoatrial node, atrioventricular bundle
D. the sinoatrial node, atrioventricular node, conduction myofibers
E. the sinoatrial node, internodal tracts, atrioventricular node, atrioventricular bundle, left and
right bundle branches, and Purkinje fibers
27. Which of the following pairs of veins is considered as a portal-caval anastomosis?
A. hepatic veins and inferior vena cava
B. superior and inferior rectal veins
C. left and right gastric veins
D. suprarenal and renal veins
E. superior and inferior epigastric veins
28. The constitutions of superficial palmar arch are
.
A. the terminal part of the ulnar artery and the terminal part of the radial artery
B. the terminal part of the radial artery and the superficial palmar branch of the ulnar artery
C. the terminal part of the radial artery and the deep palmar branch of the ulnar artery
D. the terminal part of the ulnar artery and the superficial palmar branch of the radial artery
E. the superficial palmar branch of the ulnar artery and the superficial palmar branch of the
radial artery
29. The right gastroepiploic artery arises from
.
A. the celiac trunk
B. the splenic artery
C. the proper hepatic artery
D. the common hepatic artery
E. the gastroduodenal artery
30. The artery which serve the fundus of stomach is/are
.
A. the right gastric artery
B. the left gastric artery
C. the right gastroepiploic artery
D. the left gastroepiploic artery
E. the short gastric arteries
31. An obstruction of the inferior mesenteric vein just before joining the splenic vein is most likely
to enlarge which of the following veins?
A. middle colic vein
B. right colic vein
C. inferior pancreaticoduodenal vein
D. ileocolic vein
E. left colic vein
32. The defect of the interventricular septum always takes place at
.
A. the fossa ovalis
B. the muscular part of interventricular septum
C. the membranous part of interventricular septum
D. the supraventricular crest
E. none of the above
33. Which is the following vessels transporting oxygen-poor blood?
A. the renal arteries
B. the aorta
C. the pulmonary arteries
D. the coronary arteries
E. none of the above
34. The artery that we can touch its impulse on the anterior part of auricle is
.
A. the facial artery
B. the superficial temporal artery
C. the maxillary artery
D. the common carotid artery
E. the external carotid artery
35. The cystic artery arises from
.
A. the proper hepatic artery
B. the common hepatic artery
C. the left branch of the proper hepatic artery
D. the right branch of the proper hepatic artery
E. the celiac trunk
36. Which is correct about the carotid glomus?
A. The carotid glomus is located on the deep aspect of the bifurcation of the common carotid
artery.
B. The carotid glomus locate the anterior artery wall of the crotch of the common carotid
artery.
C. The carotid glomus locates at the end of the common carotid artery and the initial segment
of the external carotid artery.
D. The carotid glomus is the baroreceptor.
E. none of the above
37. Which vein accompanies the common carotid artery?
A. the external jugular vein
B. the facial vein
C . the internal jugular vein
D. the anterior jugular vein
E. the retromandibular vein
38. Which is the biggest superficial vein as follows?
A. the retromandibular vein
B. the great saphenous vein
C. the internal jugular vein
D. the anterior jugular vein
E. the external jugular vein
39. Which of the following veins lacks the venous valve?
A. the great saphenous vein
B. the small saphenous vein
C. the facial vein
D. the cephalic vein
E. the basilic vein
40. The external jugular veins empty into
.
A. the anterior jugular veins
B. the internal jugular veins
C. the facial veins
D. the brachiocephalic veins
E. the subclavian veins
41. The venous angle locates at
.
A. the junction of the external and internal jugular veins
B. the junction of the left and right the brachiocephalic veins
C. the junction of the internal jugular vein and the subclavian vein
D. the junction of the internal iliac vein and the external iliac vein
E. none of the above
42. Which of the following pairs of veins typically terminates in the same vein?
A. left and right ovarian veins
B. left and right hepatic veins
C. left and right suprarenal veins
D. left and right colic veins
E. left and right gastroepiploic veins
43. Which vein is not empty into the inferior vena cava?
A. the hepatic portal vein
B. the hepatic vein
C. the lumbar veins
D. the right adrenal vein
E. the renal veins
44. In a patient with hepatic portal hypertension, which of the following veins is most likely to be
dilated?
A. right colic vein
B. inferior epigastric vein
C. suprarenal vein
D. inferior phrenic vein
E. ovarian vein
45. Which is not the lymphatic organ?
A. the lymph node
B. the aggregated lymphatic follicle
C. the palatine tonsil
D. the spleen
E. the thymus
46. Which is not correct about the lymph nodes?
A. The lymph nodes are one part of the lymphatic organs.
B. The lymph nodes always locate the shelter of the body.
C. The lymph nodes always gather together.
D. The efferent lymphatic vessel of the lymph node is less than the afferent lymphatic vessel.
E. The efferent lymphatic vessel of the lymph node is connected with its prominence side.
47. Which of the following lymph trunks empties into the thoracic duct?
A. the left jugular trunk
B. the right jugular trunk
C. the intestinal trunk
D. the right lumbar trunk
E. the left lumbar trunk
48. Which is not correct about the thoracic duct?
A. The thoracic duct originates from the cisterna chili.
B. The thoracic duct assembles the lymph of the right and left lumbar trunk.
C. The thoracic duct enters the thoracic cavity through the esophageal hiatus.
D. The thoracic duct is the capital collecting duct.
E. The thoracic duct empties into the left venous angle.
49. The cisterna chyli is located at
.
A. the anterior of the body of first lumbar vertebra
B. the anterior of the body of second lumbar vertebra
C. the anterior of the body of third lumbar vertebra
D. the anterior of the body of twelfth thoracic vertebra
E. none of the above
50. The spleen is located at
.
A. the left lumbar region
B. the epigastric region
C. the left hypochondriac region
D. the internal surface from the left eighth to the tenth ribs
E. none of the above
51. Which blood vessd or group of vessels carry richly oxygenated blood to the heart?
A. superior vena cava
B. pulmonary arteries
C. pulmonary veins
D. ascending aorta
E. coronary sinus
52. which of the following structures shunts blood from the pulmonary trunk to the aorta, partially
bypassing the lungs?
A. placenta
B. umbilical artery
C. ductus arteriosus
D. foramen ovale
E. ductus venosus
53. The aortic arch gives rise to
.
A. right common carotid artery
B. left subclavian artery
C. posterior intercostal artery
D. left internal thoracic artery
E. costocervical trunk
54. Which of the following characteristics is associated with the hepatic portal vein or the hepatic
portal venous system?
A. lower blood pressure than in the inferior vena cava
B. least risk of venous varices at the tower end of the esophagus as a result of portal
hypertension
C. distention of the hepatic portal vein due to its numerous valves
D. caput medusae and hemorrhoids caused by hepatic portal hypertension
E. less blood flow than in the hepatic artery
55. The subclavian artery
.
A. arises from the brachiocephalic trunk at the left sternoclavicular joint
B. passes in front of the scalenus anterior
C. gives rise to the superior thyroid artery
D. arches in front of the pleura and lung
E. becomes the axillary artery at the medial border of the first rib
56. The subclavian artery gives rise to
.
A. inferior thyroid artery
B. vertebral artery
C. supreme intercostal artery
D. suprascapular artery
E. axillary artery
57. Which of the following arteries supplies the shoulder joint?
A. superior thoracic artery
B. lateral thoracic artery
C. thoracoacromial artery
D. thoracodorsal artery
E. circumflex scapular artery
58. Occlusion of the radial artery just distal to its origin is most likely to cause which of the
following conditions?
A. a marked decrease in the blood flow in the superficial palmar arterial arch
B. decreased pulsation in the superficial palmar branch of the radial artery
C. ischemia of the entire extensor muscles of the forearm
D. a marked decrease in the blood flow in the principal artery of thumb
E. a low blood pressure in the anterior interosseous artery
59. The brachial artery
.
A. follows the medial border of the biceps brachii
B. is in the front of the humerus in the upper part of the arm
C. gives rise to the anterior circumflex humeral artery
D. supplies the anterior wall of the axilla
E. gives rise to common interosseous artery
60. Which of the following statements is correct?
A. The left common carotid artery is one of the terminal branch of the hrachiocephafic trunk.
B. The common carotid artery divides into the internal and external carotid arteries at the
level of the upper border of the thyroid cartilage.
C. The common carotid artery can be palpated in its whole course.
D. The external carotid artery, at its commencement, is anterior and medial to the internal
carotid artery.
E. Branches of the internal carotid artery are less than the external carotid artery.
61. The thoracic aorta
.
A. extends to lower border of the twelfth thoracic vertebra
B. gives rise to eleven paired posterior intercostal arteries
C. crosses behind the esophagus
D. is situated to the left of the vertebral column in the lower part of the thorax
E. gives origin to the internal thoracic artery
62. Which of the following arteries takes origin from the common hepatic artery?
A. right gastric artery
B. right gastroepiploic artery
C. superior pancreaticoduodenal artery
D. cystic artery
E. gastroduodenal artery
63. Which of the following medes runs along the upper border of the pancreas?
A. inferior pancreaticoduodenal artery
B. left gastroepiploic artery
C. gastroduodenal artery
D. splenic artery
E. short gastric artery
64. During gastrocolostomy, a surgeon is ligating the branches of the arteries that supply the
stomach..Which of the following arteries may be spared?
A. inferior pancreaticoduodenal artery
B. splenic artery
C. gastroduodenal artery
D. left gastroepiploic artery
E. proper hepatic artery
65. Rapid occlusion of direct branches of which of the following arteries results in ischemia of the
suprarenal glands?
A. aorta; inferior phrenic and renal arteries
B. renal, splenic, and inferior mesenteric arteries
C. aorta; splenic and inferior phrenic arteries
D. superior mesenteric, inferior mesenteric, and renal arteries
E. aorta; hepatic and renal arteries
66. Which of the following statements is correct?
A. The superior rectal artery arises from the sigmoid artery.
B. The uterine artery crosses under the lower end of the ureter.
C. The inferior rectal artery arises from the inferior vesical artery.
D. The two common iliac arteries diverge at the lower border of the fifth lumbar vertebra.
E. The anal artery arises from the internal pudendal artery.
67. Which of the following arteries passes through the lesser sciatic foramen?
A. internal pudendal artery
B. inferior rectal artery
C. superior gluteal artery
D. obturator artery
E. inferior gluteal artery
68. Which of the following arteries arise from the femoral artery?
A. lateral femoral circumflex artery
B. inferior epigastric artery
C. obturator artery
D. deep circumflex iliac
E. deep femoral artery
69. The posterior tibial artery
.
A. takes origin from the femoral artery
B. ends about midway between the medial malleolus and the calcaneal tuberosity
C. inclines beneath the deep calf muscles
D. continues as the dorsal artery of foot
E. gives origin to the medial and lateral inferior genicular arteries
70. Hypertrophy of the extensor muscles of the leg may cause ischemia due to compression of
which of the following arteries?
A. popliteal artery
B. peroneal artery
C. anterior tibial artery
D. deep femoral artery
E. posterior tibial artery
71. The external jugular vein
.
A. receives posterior auricular, occipital and facial veins
B. passes downwards and backwards behind the sternocleidomastoid
C. drains into the internal jugular vein
D. may readily be seen during moderately severe or prolonged coughing
E. communicates with the pterygoid venous plexus
72. The internal jugular vein
.
A. receives veins from the tongue, pharynx, larynx and thyroid gland
B. descends immediately medial to the internal and common carotid arteries
C. is joined by the subclavian vein to form the superior vena cava
D. mainly receives venous blood of the neck
E. is situated outside of the carotid sheath
73. The facial vein
.
A. contains many valves
B. communicates with the superior and inferior ophthalmic veins
C. crosses the lower border of the mandible near the posterior margin of the masseter
D. drains into the external jugular vein
E. receives the anterior jugular vein
74. Which of the following statements is correct?
A. The cephalic vein passes upwards on the ulnar side of forearm.
B. The basilic vein enters the median cubital vein.
C. The cephalic vein ascends along the lateral border of the biceps brachii.
D. The median cubital vein is a deep vein.
E. The median antebrachial vein drains into the brachial vein.
75. The subclavian vein
.
A. curves behind the subclavian artery
B. passes behind the scalenus anterior
C. joins the external jugular vein to form the brachiocephalic vein
D. lies on in front of the cupula of the pleura
E. receives the internal thoracic vein
76. The azygos vein
.
A. arises from the right suprarenal vein
B. enters the thorax through the vena caval foramen of the diaphragm
C. arches forwards below the root of the right lung
D. receives accessory hemiazygos vein
E. joins the superior vena cava
77. The hemiazygos vein
.
A. receives the lower left posterior intercostal veins
B. receives the left superior intercostal veins
C. is connected to the inferior vena cava
D. empties into the superior vena cava
E. enters the thoracic cavity through the esophageal opening
78. The great saphenous vein runs
.
A. posterior to the medial malleolus
B. into the popliteal vein
C. anterior to the medial condyles of the tibia and femur
D. superficial to the deep fascia of the thigh
E. along with the femoral vessels
79. Which of the following statements is correct?
A. The small saphenous vein arises from the medial side of the dorsal venous arch.
B. The great saphenous vein receives the inferior epigastric vein.
C. The small saphenous vein drains into the femoral vein.
D. The great saphenous vein contains less valves.
E. The small saphenous vein passes behind the lateral malleolus.
80. Which of the following veins drains into the left renal vein?
A. left testicular vein
B. left ascending lumbar vein
C. left inferior phrenic vein
D. left lumbar vein
E. paraumbilical vein
A2
1. A 26-year-old female is requested to have a venous stripping operation because of the varicose
veins of lower extremity. Which of the branches can not be ligated?
A. the superficial iliac circumflex vein
B. the superficial medial femoral vein
C. the superficial lateral femoral vein
D. the inferior epigastric vein
E. the superficial epigastric vein
2. While attempting to suture the distal end of a coronary bypass onto the anterior interventricular
artery, the surgeon accidentally passed the needle through the adjacent vein. Which vein was
damaged?
A. the anterior cardiac vein
B. the coronary sinus
C. the great cardiac vein
D. the middle cardiac vein
E. the small cardiac vein
3. The myocardial infarction took place in the lateral wall and posterior wall of the left ventricle.
Which branch of the left coronary artery is blocked?
A. the anterior interventricular branch
B. the circumflex branch
C. the posterior interventricular branch
D. the right circumflex branch
E. the branch of sinuatrial node
4. A male, 65 years old, is brought into the emergency room because of the accidental
haematemesis. Examination: The liver is lightly intumescent and hard. The spleen can be
touched. The patient has the cirsomphalos and the abdominal dropsy. Which vein has nothing to
do with the cirsomphalos?
A. the hepatic vein
B. the inferior epigastric vein
C. the superficial epigastric vein
D. the paraumbilical veins
E. the internal thoracic artery
5. When a patient suffered from the hydrops pericardii (hydropericardium), we can puncture in
the
.
A. transverse pericardial sinus
B. oblique pericardial sinus
C. anterior inferior sinus of pericardium
D. fibrous pericardium
E. serous pericardium
6. A male, 24 years old, is brought into the emergency room after having been kicked in the chest
by a horse. After examination, it is concluded that the most likely immediate danger is cardiac
tamponade (bleeding into the pericardial sac). You prepare to draw off some of the blood from
the sac to relieve the pressure on the heart. The safest site at which to insert the needle of the
syringe in order to miss the pleura would be
.
A. just below the nipple on the left
B. just to the left of the xiphisternal junction
C. 4th left intercostal space in the midaxillary line
D. through the jugular notch
E. near the sternal angle
7. A 12-year-old girl falls off her bike and fractures the surgical neck of her humerus. This
accident most likely leads to damage of which of the following arteries?
A. axillary artery
B. deep brachial artery
C. posterior circumflex humeral artery
D. superior ulnar collateral artery
E. circumflex scapular artery
8. In an attempt to obtain a blood sample from an individual's median cubital vein, a nurse
inadvertently obtains arterial blood. The blood most likely comes from which of the following
arteries?
A. brachial artery
B. radial artery
C. ulnar artery
D. common interosseous artery
E. superior ulnar collateral artery
9. A 70-year-old man suffers a fracture of the shaft of the humerus as the result of an automobile
accident. Which of the following arteries may be damaged?
A. posterior circumflex humeral artery
B. deep brachial artery
C. brachial artery
D. radial artery
E. superior ulnar collateral artery
10. A radiograph of a 3 5-year-old woman reveals a perforation in the posterior wall of the
stomach in which the gastric contents have spilled into the lesser sac. The abdominal surgeon
who opened the gastrosplenic ligament to reach the lesser sac cut an artery accidentally. Which
of the following vessels is most likely to be injured?
A. splenic artery
B. gastroduodenal artery
C. left gastric artery
D. left gastroepiploic artery
E. right gastric artery
11. During surgery, a surgeon notices profuse bleeding from the deep cervical artery. Which of the
following arteries should be ligated immediately?
A. transverse cervical artery
B. thyrocervical trunk
C. costocervical trunk
D. inferior thyroid artery
E. ascending cervical artery
12. A tumor located just superior to the root of the right lung may block blood flow in which of the
following vein?
A. hemiazygos vein
B. right subclavian vein
C. azygos vein
D. right brachiocephalic vein
E. accessory hemiazygos vein
13. A knife wound penetrates the superficial vein that terminates in the popliteal vein. Bleeding
occurs from which of the following vessels?
A. posterior tibial vein
B. anterior tibial vein
C. great saphenous vein
D. peroneal vein
E. lesser saphenous vein
14. A thrombosis in the popliteal vein most likely causes reduction of blood flow in which of the
following veins?
A. lesser saphenous vein
B. femoral vein
C. great saphenous vein
D. anterior tibial vein
E. posterior tibial vein
15. A 45-year-old man with portal hypertension resulting from cirrhosis of the liver presents to
Surgery Department. The most practical method of shunting portal blood involves which of the
following surgical connections?
A. superior mesenteric vein to the inferior mesenteric vein
B. portal vein to the left renal vein
C. portal vein to the superior vena cava
D. splenic vein to the left renal vein
E. superior rectal vein to the left colic vein
B1
A. superior mesenteric artery
B. celiac trunk
C. common hepatic artery
D. gastroduodenal artery
E. splenic artery
1. The inferior pancreaticoduodenal artery originates from
.
2. The short gastric arteries originate from
.
3. The right gastroepiploic artery originates from
.
4. The proper hepatic artery originates from
.
5. The left gastric artery originates from
.
A. the 5th left intercostal space in the midclavicular line
B. from the 4th to the 5th intercostal space in the left limit of sternal border
C. the 2nd intercostal space in the right limit of sternal border
D. the 3rd intercostal space in the left limit of sternal border
E. the 2nd intercostal space in the left limit of sternal border
6. the mitral area
7. the aortic area
8. the tricuspid area
9. the 2nd aortic area
10. the pulmonary valve area
True or False Questions
1. The common interosseous artery arises from the ulnar artery about 2. 5 cm below its
commencement, and divides in to the anterior and posterior interosseous arteries
(
)
2. The peroneal artery takes origin from the popliteal artery.
(
)
3. The maxillary artery passes forwards deep to the neck of the mandible, runs through or deep to
the lateral pterygoid, then enters the infratemporal fossa through the pterygomaxillary fissure.
(
)
4. The azygos vein arises from the right lumbar veins, and enters the thorax through the right crus
of the diaphragm. It ends by arching forwards above the root of the right lung to join the right
brachiocephalic vein.
(
)
5. The left testicular vein drains into the left renal vein, the right testicular vein drains into the
inferior vena cava.
(
)
6. The external jugular, the anterior jugular and external pudendal veins and the jugular venous
arch are superficial veins.
(
)
Explanation of terms
1. Carotid sinus
2. Carotid glomus
3. Middle meningeal artery
4. Venous angle
5. Hepatic portal vein
6. Superior vena cava
Answer the following Questions
1. What are branches of the external carotid artery and their distributions?
2. What are branches of the axillary artery and their distributions?
3. Which arteries constitute the stomach's arterial supply?
4. What are location and drainage of the superficial veins of the upper limb?
5. What are location and drainage of the superficial veins of the lower limb?
6. Describe location, tributaries and drainage of the hepatic portal vein and anastomoses with the
superior and inferior vena cavae.
ANSWERS
Multiple Ghoice Questions
A1
1. D 2. C 3. D 4. B 5. A 6. B 7. C 8. A 9. A 10. C
11. D 12. E
13. B 14. C 15. E 16. C 17. E 18. D
19. B 20. C
21. E 22. A
23. B
24. C
25. C
26. E 27. B
28. D
29. E
30. E
31. E
32. C
33. C 34. B 35. D 36. A 37. C 38. B 39. C 40. E
41. C
42. B 43. A
44. A 45.B 46. E 47. A
48. C
49. A 50. C 51. C 52. C 53. B
54. D
55.D
56. B 57.C
58. D
59. A 60.D
61. C 62. E
63. D 64. A
65. A
66. E 67. A
68. E
69. B 70. B
71. D 72. A 73. B 74. C
75. D
76. E
77. A
78.D
79. E 80. A
A2
1. D 2. C
3. B
4. A
5. C
6. B
7. C
8. A
9. B
10. D
11. E 12. C 13. E 14. B 15. D
B1
1. A 2. E 3. D 4. C 5. B 6. A 7. C 8. B
9. D 10. E
True or False Questions
1. T
2. F
3. F
4. F
5. T
6. T
Explanation of terms
1. The carotid sinus is a slight dilatation of the upper end of the common carotid artery and the
commencement of the internal carotid artery. The wall of carotid sinus is thinner than the wall
of the artery elsewhere, and contains a large number of Sensory nerve endings which are
pressoreceptors. These receptors monitor blood pressure.
2. The carotid glomus lies on the deep aspect of the bifurcation of the common carotid artery. It is
a small oval structure, about 5 mm long, and contains ramification of nerve fibers and a rich
network of large sinusoidal capillaries. The carotid glomus is a chemoreceptor which responds
to changes of 02 and CO2 in blood.
3. The middle meningeal artery is the largest branch of the maxillary artery, which enters the skull
through the foramen spinosum, runs forwards for a short distance in a groove on the greater
wing of the sphenoid bone, and then divides into the anterior and posterior branches. The
anterior branch lies usually in a bone groove or canal on the inner aspect of the pterion. When
the pterion is fractured, it is easy to be injured, causing the epidural hematoma.
4. The junction between the subclavian and internal jugular veins forms the venous angle. The
thoracic duct opens into the left venous angle. The right lymphatic duct opens into the right
venous angle.
5. The hepatic portal vein is formed by the junction of the superior mesenteric and splenic veins.
Through these two veins and their tributaries the hepatic portal vein receives the venous blood
from most of the digestive tract in the abdomen, the spleen and the pancreas.
6. The superior vena cava is formed by the left and right brachiocephalic veins at the back of the
sternocostal synchondrosis of the first rib and opens into the right atrium at the level of the third
sternocostal joint. Its lower half is enclosed within the fibrous pericardium. The chief tributary
of the superior vena cava is the azygos vein.
Answer the following Questions
1. The external carotid artery gives rise to eight branches and supplies the neck and head.
(1) The superior thyroid artery supplies the thyroid gland and larynx.
(2) The lingual artery supplies the tongue and the sublingual gland.
(3) The facial artery supplies the submandibular gland, the muscles of facial expression,
palatine tonsil, the soft palate, and the tissues of the face.
(4) The occipital artery supplies the posterior part of the scalp.
(5) The posterior auricular artery supplies structures of ear and the adjacent scalp area.
(6) The ascending pharyngeal artery supplies the area of the pharynx.
(7) The superficial temporal artery supplies the parotid gland and temporoparietal region of the
scalp.
(8) The maxillary artery supplies the external acoustic meatus, tympanic cavity, upper teeth,
gum, nasal cavity, palate, masticatory muscles and cerebral dura mater.
2. The axiUary artery gives rise to six branches and supplies the contents and walls of the axilla.
(1) The superior thoracic artery is distributed to the upper part of the medial wall of the axilla.
(2) The thoracoacromial artery is distributed to the pectoralis major, the pectoralis minor, the
deltoid and the shoulder joint.
(3) The lateral thoracic artery is distributed to the serratus anterior, the pectoralis major, the
pectoralis minor and the breast.
(4) The subscapular artery divides into the thoracodorsal artery and the circumflex scapular
artery. The thoracodorsal artery supplies the latissimus dorsi and the serratus anterior.
The circumflex scapular artery supplies the muscles and the tissue in the infraspinous
fossa.
(5) The posterior and anterior circumflex humeral arteries supply the deltoid.
3. The stomach's arterial supply is as following.
(1) The left gastric artery arises from the celiac trunk and supplies the part of the stomach
along its lesser curvature.
(2) The right gastric artery arises from the proper hepatic artery. It anastomoses with the left
gastric artery and supplies the part of the stomach along its lesser curvature.
(3) The left gastroepiploic artery arises from the splenic artery and anastomoses with the
right gastroepiploic artery. It supplies the part of the stomach along its greater
curvature.
(4) The right gastroepiploic artery arises from the gastroduodenal artery and supplies the
part of the stomach along its greater curvature.
(5) The short gastric arteries arise from the splenic artery and supply the fundus of the
stomach.
4. From dorsal venous rete the cephalic vein passes upwards on the radial side and the basilic vein
on the ulnar side of forearm. The cephalic vein runs upwards and laterally to the front of the
elbow, while the basilic vein runs upwards and slightly medially to the front of the elbow. In
front of the elbow a large median cubital vein runs obliquely upwards and medially from the
cephalic vein to the basilic vein. Above the elbow the cephalic vein ascends along the lateral
border of the biceps brachii, and pierces through the deep fascia to lie between the deltoid and
the pectoralis major. It then curves upwards and medially, and just below the clavicle it pierces
through the clavipectoral fascia to enter the axillary vein. The basilic vein ascends along the
medial side of the biceps brachii and about the middle of the arm pierces the deep fascia to enter
the brachial vein or the axillary vein. Occasionally, a large median antebrachial vein ascends
in the middle of the forearm and ends into the median cubital vein, sometimes it divides into
two branches which join the cephalic vein and the basilic vein respectively.
5. The small saphenous vein arises from the lateral of the side of the dorsal venous arch, passes
behind the lateral malleolus, ascends first on the lateral side of the tendo calcaneus, and then
along the middle of the back of the leg to end in the popliteal fossa by piercing the deep fascia
and draining into the popliteal vein. The great saphenous vein arises on the medial side of the
dorsal venous arch, and ascends in front of the medial malleolus. It passes up the leg along the
medial border of the tibia, goes behind the medial surface of the tibia and femur, then ascends in
the thigh somewhat obliquely to pierce the deep fascia at the saphenous hiatus, about 3-4 cm
below and lateral to the pubic tubercle. The great saphenous vein receives the superficial medial
femoral, the superficial lateral femoral, the external pudendal, the superficial epigastric and the
superficial iliac circumflex veins.
6. The hepatic portal vein is formed by the junction of the superior mesenteric and splenic veins.
Through these two veins and their tributaries the hepatic portal vein receives the venous blood
from most of the digestive tract in the abdomen, the spleen and the pancreas.
and in front of the inferior vena cava, and then ascends in the hepatoduodenal ligament. Here the
common bile duct and the proper hepatic artery are in front of the hepatic portal vein, the duct
being to the right of the artery. The hepatic portal vein finally reaches the porta hepatis, where it
divides into right and left branches for the corresponding lobes of the liver.
The superior mesenteric vein ascends to the right of the superior mesenteric artery and ends
behind the neck of the pancreas by joining the splenic vein. The superior mesenteric vein
conveys blood from the jejunum and ileum, the caecum, appendix, ascending colon, most of the
transverse colon, pancreas and duodenum.
The splenic vein lies below the splenic artery and behind the upper part of the pancreas. On
reaching the neck of the pancreas the vein joins the superior mesenteric vein to form the hepatic
portal vein. The splenic vein drains the spleen, part of the pancreas and stomach. In addition,
the splenic vein receives the inferior mesenteric vein.
The inferior mesenteric vein arises from the superior rectal vein, ascends behind the peritoneum
to the left of the inferior mesenteric artery, and then passes behind the duodenojejunal flexure to
enter the splenic vein. The inferior mesenteric vein receives tributaries from the sigmoid colon,
the descending colon and the left colic flexure. In addition to the superior mesenteric and
splenic veins, the hepatic portal vein receives the left and right gastric veins from the lesser
curvature of the stomach, the cystic vein from the gallbladder, and the small paraumbilical veins
passing along the ligamentum teres hepatis.
Between the tributaries of the hepatic portal vein and those of the superior and inferior vena
cavae, there are a number of anastomoses. These are normally small and unimportant, but
when there is increased resistance to the flow of blood through the hepatic portal vein or liver,
these communications enlarge and take some of the portal blood into the tributaries of the
superior and inferior vena cavae directly, thus by-passing the liver. The chief sites of
communications are. (1) the esophageal venous plexus at the junction of the esophagus and
stomach, where the tributaries of the left gastric vein communicate with the tributaries of the
esophagus veins which drain into the azygos vein; (2) the periumbilical venous rete around the
umbilicus, where the paraumbilical veins connect the hepatic portal vein with the veins of the
anterior abdominal wall by four epigastric routes to superior and inferior vena caval systems; (3)
the rectal venous plexus at the anorectal junction, where the superior rectal veins communicate
with the inferior rectal and anal veins. In portal venous obstruction these communications
enlarge and become varicose. The varices of the esophageal venous plexus and the rectal
venous plexus may rupture and give rise to hemorrhage.
(Wang Haijie)
Multiple Choice Questions
A1
1. Which of the following structures is not the lymphatic organ?
A. spleen
B. tonsils
C. thymus
D. lymph nodes
E. lymphoid follicles
2. The thoracic duct
.
A. receives lymphatic vessels of the spleen
B. lies first between the azygos vein and the thoracic aorta
C. clines to the left at the level of the third thoracic vertebra
D. ascends to the right of the esophagus at the upper part of the thorax
E. opens in the vertebral vein
3. Right lymphatic duct
.
A. receives right bronchomediastinal trunk
B. opens to the external jugular vein
C. receives the efferent lymphatic vessels of the apical lymph nodes
D. drains lymph of 3/4 area of the body
E. receives the efferent lymphatic vessels of Virchow lymph nodes
4. Which of the following groups of lymph nodes drains lymph of the larynx?
A. submandibular lymph nodes
B. parotid lymph nodes
C. superior lateral cervical lymph nodes
D. submental lymph nodes
E. paratracheal lymph nodes
5. Lymph from the breast drains primarily into which of the following nodes?
A. apical lymph nodes
B. pectoral lymph nodes
C. parasternal lymph nodes
D. supraclavicular lymph nodes
E. lateral lymph nodes
6. Which of the following statements is correct?
A. The pectoral lymph nodes lie along the lower border of the pectoralis major.
B. The infraclavicular lymph nodes is the upper group of the axillary lymph nodes.
C. The subscapular lymph nodes lie along the lower margin of the posterior wall of the axilla.
D. The lateral lymph nodes lie lateral to the cephalic vein.
E. The apical lymph nodes lie along the medial side of the axillary vein.
7. The lymphatic vessels of the lung drain into
.
A. parasternal lymph nodes
B. posterior intercostal lymph nodes
C. paratracheal lymph nodes
D. bronehopulmonary hilar lymph nodes
E. posterior mediastinal lymph nodes
8. The superior superficial inguinal lymph nodes
.
A. are arranged along the upper end of the greater saphenous vein
B. receive superficial lymphatics from the perineum
C. are not palpable
D. drain lymph of the thigh
E. lie in the femoral canal
9. Which of the following description is correct?
A. Below the umbilicus the superficial lymphatic vessels of the abdominal wall drain to the
deep inguinal lymph nodes.
B. Above the umbilicus the superficial lymphatic vessels of the abdominal wall converge to
the axillary lymph nodes.
C. In the upper part of the anterior abdominal wall the deep lymphatics pass to the posterior
intercostal lymph nodes.
D. In the lower part of the anterior abdominal wall the deep lymphatics pass to the deep
inguinal lymph nodes.
E. The deep lymphatics of' the posterior abdominal wall reach the cisterna chili.
10. Which of the following lymph nodes receives lymphatic vessels of the liver?
A. superior mesenteric lymph nodes
B. lumbar lymph nodes
C. celiac lymph nodes
D. parasternal (Internal thoracic) lymph nodes
E. pyloric lymph nodes
11. The lymphatic vessels of the uterine pass to the following lymph nodes except for
.
A. internal iliac lymph nodes
B. external iliac lymph nodes
C. superior superficial inguinal lymph nodes
D. sacral lymph nodes
E. lumbar lymph nodes
12. The spleen lies
.
A. deep to the left ninth, tenth and eleventh ribs
B. in front of the fundus of the stomach
C. above the tail of the pancreas
D. below the left costal arch
E. behind the left colic flexure
13. Which of the following ligaments does not support the spleen?
A. gastrosplenic ligament
B. splenorenal ligament
C. phrenicosplenic ligament
D. gastrocolic ligament
E. phrenicocolic ligament
A2
1. An acute infection involving the skin of the scrotum most likely leads to an enlargement of
which of the following lymph nodes?
A. lumber nodes
B. internal iliac nodes
C. common iliac nodes
D. superficial inguinal nodes
E. external iliac nodes
2. A surgeon of Department of Oncology observed that the submandibular lymph nodes of a
75-year-old man were enlarged. Which of the following organs most likely suffers from tumor?
A. thyroid gland
B. tongue
C. larynx
D. eye
E. external ear
B1
A. superior mesenteric lymph nodes
B. popliteal lymph nodes
C. Virchow lymph nodes
D. submandibular lymph nodes
E. retropharyngeal lymph nodes
1. The nasopharyngeal carcinoma may metastasize to
.
2. The gastric cancer may metastasize to
.
True or False Questions
1. The pectoral lymph nodes, lying on the anterior surface of the pectoralis minor, in relation to the
long thoracic nerve.
(
)
2. The efferent lymphatic vessels of the parasternal, anterior mediastinal and paratracheal lymph
nodes pass to the bronchomediastinal trunk.
(
)
3. The superficial lymphatics of the the foot and the leg ascend along the posterior tibial vein, then
piece the deep fascia of the popliteal fossa to reach the popliteal lymph nodes.
(
)
4. The superior superficial inguinal lymph nodes receive superficial lymphatics from both the
anterior and posterior aspects of the abdominal wall below the level of the umbilicus, from the
gluteal region and the perineum.
(
)
Explanation of terms
1. regional lymph node
2. supraclavicular lymph node
3. cisterna chyli
4. right lymphatic duct
Answer the following Questions
1. Explain location and drainage of the thoracic duct.
2. Describe groups and drainage of the axillary lymph nodes.
3. Describe lymph drainage of the breast.
4. Explain lymph drainage of the lung.
ANSWERS
Multiple Choice Questions
A1
1. E 2. B
3. A
4. E
5. B
6. C
7. D
8. B
9. B 10. D
11. E
12. A 13. D
A2
1. D
2. B
B1
1. E
2. C
True or False Questions
1. F
2. T
3. F
4. T
Explanation of terms
1. The lymph nodes draining the given region or organ are called the regional lymph nodes or
sentinel lymph nodes. The lymph nodes play important roles in filtering lymph, producing
lymphocytes and in response to immunological stimulation. Therefore, enlargement of the
regional lymph nodes is significant in diagnosis of the disease of the region or organ.
2. The supraclavicular lymph nodes are round the transverse cervical vessels. The left scalenus
lymph nodes is also called Virchow's lymph nodes, into which cancer cells of the esophagus and
stomach may metastasize through the thoracic duct.
3. The cisterna chyli is about 5-8 mm long and lies in front of the body of the first lumbar vertebra,
to the right of the abdominal aorta. The cisterna chyli receives the left and right lumbar trunks
and the intestinal trunk.
4. The fight lymphatic duct is formed at the root of the neck on the right side by the union of the
right jugular trunk, the fight subclavian trunk and the fight bronchomediastinal trunk. The fight
lymphatic duct is 1-1.5 crn long, and opens into the fight venous angle.
Answer the following Questions
1. The thoracic duct commences in the abdomen, usually as a dilated sac, the cisterna chyli. The
cisterna chyli is about 5-8 mm long and lies in front of the body of the first lumbar vertebra, to
the right of the abdominal aorta. The cisterna chyli receives: (1) the left and right lumbar trunks,
conveying lymph from the lower limbs and the pelvic viscera; (2) the intestinal trunk, draining
most of the abdominal part of the alimentary canal, liver, pancreas and spleen. After passing
through the aortic hiatus of the diaphragm, the thoracic duct ascends into the thorax
immediately in front of the vertebral column. It lies first between the azygos vein on the right
and the thoracic aorta on the left, while esophagus is anterior to it. At the level of fifth thoracic
vertebra it clines to the left, ascending to the left of the esophagus, and to the right of the aortic
arch. Entering the neck, the thoracic duct curves laterally behind the left common carotid artery,
internal jugular vein and vagus nerve, finally turns down, in front of the first part of the left
subclavian artery, to open in the left venous angle. Close to its termination, the thoracic duct
receives the left jugular trunk from the left side of the head and neck, the left subclavian trunk
from the left upper limb, and the left bronchomediastinal trunk from the upper part of the thorax
on the left side.
2. The axillary lymph nodes may be divided into five groups. Anterior group, the pectoral lymph
nodes, lying along the lower border of the pectoralis minor, in relation to the lateral thoracic
vessels. These lymph nodes receive lymph from the anterior and lateral abdominal walls above
the umbilicus, and the anterior and lateral thoracic walls, including the lateral and central parts
of the breast. Lateral group, the lateral lymph nodes, lying lateral to and behind the axillary vein.
These lymph nodes receive lymph from the upper limb.
Posterior group, the subscapular lymph nodes, lying along the lower margin of the posterior
wall of the axilla in the course of the subscapular vessels. These lymph nodes receive lymph
from the posterior thoracic wall and the posterior aspect of the shoulder. Central group, the
central lymph nodes, embedded in the fat near the base of the axilla. This group of lymph nodes
receive lymph from the pectoral, lateral and subscapular lymph nodes.
Apical group, the apical lymph nodes, lying in the apex of the axilla along the medial side of the
axillary vein. This group of lymph nodes receive lymph from all of the other axillary lymph
nodes and the upper part of the breast. The efferent lymphatic vessels of the apical lymph nodes
form the subclavian trunk.
3. Lymphatics from the breast drain chiefly to the axillary lymph nodes, and in the female they are
especially numerous and well developed. Lymphatic vessels from the lateral and central parts of
the breast drain to the pectoral lymph nodes, those from the upper part of the breast drain to the
apical and supraclavicular lymph nodes, those from the medial part of the breast pass to the
parasternal (internal thoracic) lymph nodes. In addition, the superficial lymphatic vessels of the
medial part of the breast communicate with the lymphatics of the opposite breast across the
midline. The lymphatic vessels of the inferomedial part of the breast communicate with the
lymphatics of the liver through lymphatics of the anterior abdominal wall and lower aspect of
the diaphragm.
4. The lungs possess a rich network of lymphatic capillaries. Collecting lymphatics pass either to
the lung surface, where a subpleural plexus is formed, or else follow the course of the bronchi
or blood vessels. All the collecting trunks converge on the lung root, from which they emerge.
There are four groups of lymph nodes connecting with the pulmonary lymph drainage, the
pulmonary lymph nodes in the substance of the lung, the bronchopulmonary hilar lymph nodes
at the hilum of the lung, the tracheobronchial lymph nodes round the bifurcation of the trachea
and the paratracheal lymph nodes. Lymphatic vessels from the lower part of the superior lobe of
the left lung and some lymphatic vessels from the lower lobe drains into the right
tracheobronchial and paratracheal lymph nodes. The lymph nodes in the pulmonary lymph
drainage drain foreign particles in the inspired air. The particles may deposit in those lymph
nodes so that they become black. From the upper end of the paratracheal lymph nodes the
bronchomediastinal trunk emerges on either side of the trachea.
(Wang Haijie)
Multiple Choice Questions
A1
1. About the wall of the eyeball, which of the following descriptions is not correct?
A. The outmost layer is called fibrous tunic.
B. The middle layer is called vascular tunic.
C. The inner layer is called retina.
D. The anterior 1/6 of the fibrous tunic is the sclera.
E. The posterior 5/6 of the fibrous tunic is the sclera.
2. Which of the following are not extraocular muscles?
A. superior rectus
B. inferior rectus
C. superior obliquus
D. levator palpebrae superioris
E. smooth muscles of the iris and ciliary body
3. Of the neurons in the retina, which form the optic nerve?
A. cone cells
B. ganglion neurons
C. bipolar neurons
D. rod cells
E. cone and rod cells
4. Blocking the scleral venous sinus might result in
.
A. a sty
B. glaucoma
C. conjunctivitis
D. chalazion
E. keratitis
5. Nearsightedness is more properly called
.
A. myopia
B. hyperopia
C. presbyopia
D. astigmatism
E. diplopia
6. Which lies closest to the exact posterior pole of the eyeball?
A. cornea
B. macula lutea
C. optic disc
D. ciliary body
E. lens
7. Paralysis of a medial rectus muscle would affect
.
A. accommodation
B. pupil constriction
C. refraction
D. depth perception
E. convexity of lens
8. Which series of reactions occurs when you look at a nearby object?
A. Suspensory ligament becomes taut, lens becomes less thick.
B. Suspensory ligament has less tension, lens thickens.
C. Suspensory ligament becomes taut, lens thickens.
D. Ciliary muscles relax, lens becomes less thick.
E. Suspensory ligament has less tension, lens becomes less thick.
9. The optic disc is the site where
.
A. the macula lutea is located
B. more rods than cones occur
C. only cones occur
D. rods and cones aggregate
E. the optic nerve exits the eye
10. The order in which a light ray passes through the refractory media of the eye is
A. cornea, aqueous humor, lens, vitreous humor
B. vitreous humor, lens, aqueous humor, cornea
C. cornea, vitreous humor, lens, aqueous humor
D. lens, aqueous humor, vitreous humor, lens
E. vitreous humor, lens, cornea, aqueous humor
11. The part of the fibrous tunic that is white, tough, and opaque is
.
A. choroid
B. cornea
C. retina
D. sclera
E. iris
12. The extraocular muscle that does not originate at or near the apex of the orbit is the
A. inferior obliquus
B. inferior rectus
C. levator palpebrae superioris
D. superior obliquus
E. superior rectus
13. From lateral to medial the ossicles arranges in the order of
.
A. incus, stapes, malleus
B. incus, malleus
C. malleus, incus, stapes
D. stapes, malleus, incus
E. incus, malleus, stapes
.
.
14. About the tympanic membrane, which of the following descriptions is not correct?
A. Functionally, the tympanic membrane regenerates the vibrations of sound source.
B. The tympanic membrane is obliquely placed, facing downward, forward, and laterally.
C. The lower 3/4 of the tympanic membrane is called the tense part.
D. The upper 1/4 of the membrane is called the flaccid part.
E. The tympanic membrane and the second tympanic membrane are the same in function.
15. The structure that allows air pressure in the middle ear to be equalized with outside air
pressure is
.
A. mastoid air
B. cochlear duct
C. oval window
D. auditory tube
E. mastoid sinus
16. About the auditory tube, which of the following descriptions is not correct?
A. The auditory tube is a canal connecting the tympanic cavity to the nasopharynx.
B. The medial 2/3 of the tube is supported by bone.
C. It functions to balance the pressures on both sides of the tympanic membrane.
D. In a child, the auditory tube is shorter and relatively wider.
E. Its posterior third is bony, and its anterior two thirds is cartilaginous.
17. About the boundaries of the tympanic cavity, which of the following statement is not right?
A. Eardrum is the lateral boundary.
B. Bony wall of the inner ear is the medial boundary.
C. Posterior wall opens into the mastoid antrum.
D. Anterior wall opens into the mastoid antrum.
E. Roof is formed by a thin plate of bone, the tegmen tympani, which is part of the petrous
temporal bone.
18. Infection of the middle ear can spread along all the following pathways except which?
A. through the tegmen tympani to the middle cranial fossa
B. through the medial wall into the labyrinth
C. through the canal for the tensor tympani muscle into the internal carotid artery
D. through the floor into the internal jugular vein
E. through the aditus to the mastoid antrum into the mastoid air cells
19. About the bony labyrinth, which of the following statements is not correct?
A. It is an irregular cavity carved in the petrous bone.
B. From anteromedial to posterolateral, there are semicircular canals, vestibule, and cochlea.
C. Vestibule contains the utricle and the saccule.
D. It is filled with a clear fluid called perilymph.
E. The three semicircular canals-superior, posterior, and lateral-open into the posterior part of
the vestibule.
20. About the descriptions of the membranous labyrinth, which one is wrong?
A. It is filled with a clear fluid called endolymph.
B. It is suspended within perilymph contained in bony labyrinth.
C. The saccule is globular and is connected to the utricle.
D. The semicircular ducts are located in the semicircular canals.
E. The utricle and saccule are contained in the cochlea.
21. About the coiled cochlea, which of the following statements is not true?
A. It resembles a snail shell and opens into the anterior part of the vestibule.
B. It is divided into three parts (scalae) on its section.
C. Running through its center is the cochlear duct.
D. The scala vestibule lies superiorly to the cochlear duct.
E. It coils for about 3.5 circles around a pillar of bone called the modiolus.
22. The inferior oblique muscle of the eye is innervated by the
.
A. abducent nerve
B. trigeminal nerve
C. oculomotor nerve
D. facial nerve
E. trochlear nerve
23. The optic canal is an opening in the
.
A. lesser wing of the sphenoid bone
B. occipital bone
C. petrous part of the temporal bone
D. frontal bone
E. squamous part of the temporal bone
24. The muscles and nerves that are responsible for adducting the eyeball (rotating the cornea
medially) include the following except which?
A. the superior rectus
B. the medial rectus
C. the oculomotor nerve
D. the inferior obliquus
E. the inferior rectus
25. The outermost layer of the optic nerve sheath is a continuation of the
.
A. arachnoid membrane
B. meningeal dura
C. periosteal dura
D. pia mater
E. retina
26. The inner lining of the eyelid is called the
.
A. orbital septum
B. palpebral conjunctiva
C. periorbita
D. sclera
E. tarsal plate
27. The ducts of the lacrimal gland open into the
.
A. superior fornix of the conjunctiva
B. inferior fornix of the conjunctiva
C. lacrimal puncta
D. lacrimal canaliculi
E. lacrimal lake
28. Starting from a position gazing straight ahead, to direct the gaze downward, the inferior rectus
muscle must be active along with the
.
A. superior obliquus
B. inferior obliquus
C. medial rectus
D. lateral rectus
E. superior rectus
A2
1. You are testing the extraocular muscles and their innervation in a patient who periodically
experiences double vision. When you ask him to turn his right eye inward toward his nose and
look downward he is able to look inward, but not down. Which nerve is most likely involved?
A. abducent
B. nasociliary
C. oculomotor, inferior division
D. oculomotor, superior division
E. trochlear
2. What would the examining physician notice in the eye of a person who has taken a sympathetic
blocking agent?
A. exophthalmos and dilated iris
B. enophthalmos and dry eye
C. dry eye and inability to accommodate for reading
D. wide open eyelids and loss of depth perception
E.ptosis and miosis (pin-point iris)
3. A patient complains of loss of hearing in the fight ear. Examination reveals ankylosis
(otoselerosis) of the footplate of the stapes to the surrounding bone. Which part of the bony
labyrinth is involved?
A. aditus and antrum
B. cochlear (round) window
C. cochlear duct
D. internal acoustic meatus
E. vestibular (oval) window
4. You have a patient with a drooping fight eyelid. You suspect Homer's syndrome. Which of the
following signs on the fight side would confirm this diagnosis?
A. constricted pupil
B. dry eye (lack of tears)
C. exophthalmos
D. pale, blanched face
E. sweaty face
5. Following endarterectomy on the fight common carotid, a patient is found to be blind in the
fight eye. It appears that a small thrombus embolized during surgery and lodged in the artery
supplying the optic nerve. What artery would be blocked?
A. central artery of the retina
B. infraorbital
C. lacrimal
D. nasociliary
E. supraorbital
6. You are asked to check the integrity of the trochlear nerve in the right eye Of a patient. Starting
with the eyes directed straight ahead, you would have the patient look
.
A. inward, toward the nose and downward
B. inward, toward the nose and upward
C. toward the nose in a horizontal plane
D. laterally in a horizontal plane
E. outward, away from the nose and downward
7. An elderly patient with chronic otitis media (middle ear infection) might have all the following
complications except:
A. inability to chew food due to injury to the mandibular division of the trigeminal nerve (CN V)
B. loss of taste in the anterior part of the tongue due to injury to the chorda tympani nerve
C. mastoiditis
D. paralysis of facial muscles due an injury to the facial nerve (CN VII)
E. some degree of deafness due to damage to the ossicles
8. An adolescent boy suffers from severe acne. As is often the case he frequently squeezed the
pimples on his face. He subsequently develops a fever and deteriorates into a confused mental
state and drowsiness. He is taken to his physician and after several tests a diagnosis of
cavernous sinus infection and thrombosis is made. The route of entry to the cavernous sinus
from the face was most likely the
A. carotid artery
B. mastoid emissary vein
C. middle meningeal artery
D. ophthalmic vein
E. parietal emissary vein
BI
A. outwards
B. inwards
C. down and outwards
D. up and outwards
E. up and inwards
1. Superior rectus muscle will move the eye
.
2. Lateral rectus muscle will move the eye
.
3. Superior obliquus muscle will turn the eye
.
4. Inferior obliquus muscle will turn the eye
.
A. auditory (nasopharyngeal) tube
B. cochlea
C. external acoustic meatus
D. internal acoustic meatus
E. sacculus
5. The entry of bacteria through which space could lead to an infection in the mastoid air cells?
6. The spiral organ lies in
.
True or False Questions
1. The central artery of the retina runs in the substance of the optic nerve and enters the eyeball at
the center of the optic disc. Here, it divides into branches, which may be studied in a patient
through an ophthalmoscope. The branches are end arteries.
(
)
2. The optic nerve enters the orbit from the middle cranial fossa by passing through the
superior orbital fissure.
(
)
3. The transparent cornea is largely responsible for the refraction of the light entering the eye. It is
in contact posteriorly with the aqueous humor.
(
)
4. The lens is situated behind the iris and in front of the vitreous body and is encircled by the
ciliary processes.
(
)
5. To accommodate the eye for close objects, the ciliary muscle relaxes and let the ciliary body
forward and inward so that the radiating fibers of the suspensory ligament are relaxed. (
)
6. The internal acoustic meatus in the skull is located in the body of the sphenoid bone. (
)
7. The middle ear communicates in front through the auditory tube with the nasopharynx and
behind with the mastoid antrum.
(
)
8. The bony labyrinth consists of three parts, the vestibule, the semicircular canals, and the cochlea.
They are lined by endosteum and contain a clear fluid, the endolymph.
(
)
Explanation of Terms
1. cornea
2. zonular fibers
3. ora serrata
4. fovea centralis
5. optic disc
6. sinus venosus sclerae
7. cone of light
8. fenestra vestibule
9. fenestra cochleae
10. prominence of the facial canal
11. spiral organ
12. macula sacculi
13. crista ampullaris
Answer the Following Questions
1. How many tissue layers (tunics) comprise the wall of eyeball and how are the tunics arranged?
2. What important structures are located on the inner surface of the posterior retinal wall?
3. What substances are contained within the space anterior to the lens and within the space
posterior to the lens respectively?
4. How does the lens focus light?
5. Give a simple description of the middle ear.
6. What is the structural feature and the function of the auditory tube?
7. What is the difference, if any, between a semicircular canal and semicircular duct?
ANSWERS
Multiple Choice Questions
A1
1. D 2. E
3. B
4. B
5. A
6. B
7. A
8. B
9. E
10. A
11. D 12. A 13. C 14. E
15. D
16. B
17. D
18. C 19. B 20. E
21. E 22. C 23. A
24. D
A2
1. E
2. E
3. E
4. A
B1
1. E
2. A
3. C
25. B
.A
26. B
6. E
27. A
28. A
7. A
8. D
4. D
5. A
6. B
True or False Questions
1. T 2. F 3. T 4. T 5. F 6. F 7. T 8. F
Explanation of Terms
1. The transparent tissue constituting the anterior sixth of the outer wall of the eye, with a 7.7-mm
radius of curvature as contrasted with the 13. 5 mm of the sclera.
2. Delicate fibers that pass from the equator of the lens to the ciliary body, collectively known as
the ciliary zonule.
3. The serrated extremity of the optic part of the retina, located a little behind the ciliary body and
marking the limits of the percipient portion of the membrane.
4. A depression in the center of the macula lutea containing only cones and lacking blood vessels.
5. An oval area of the ocular fundus devoid of light receptors where the axons of the retinal
ganglion cell converge to form the optic nerve head.
6. The vascular structure encircling the anterior chamber of the eye and through which the
aqueous is returned to the blood circulation.
7. A triangular area at the anterior inferior part of .the tympanic membrane, extending from the
umbo to the periphery, where there is a reflection of light.
8. An oval opening on the medial wall of the tympanic cavity leading into the vestibule, closed in
life by the foot of the stapes.
9. An opening on the medial wall of the middle ear leading into the cochlea, closed in life by the
secondary tympanic membrane.
10. The prominence on the medial wall of the tympanic cavity above the vestibular window
produced by the presence of the facial canal.
11. A prominent ridge of highly specialized epithelium in the floor of the cochlear duct
overlying the basilar membrane of cochlea, containing one inner row and three outer rows of
hair cells, or cells of Corti (the auditory receptor cells innervated by the cochlear nerve)
supported by various columnar cells.
12. The macula sacculi is placed vertically on the medial wall of the saccule. The hair cells in the
macula are innervated by fibers of the vestibular division of the vestibulocochlear nerve. It is
primarily static organs for signaling of the position of the head in space, and also responds to
quick tilting movement and to liner acceleration and deceleration.
13. An elevation on the inner surface of the ampulla of each semicircular duct; filaments of the
vestibular nerve pass through the crista to reach hair cells on its surface; the hair ceils are
capped by the cupula, a gelatinous protein-polysaccharide mass.
Answer the Following Questions
1. The wall of the eyeball has three tunics. The most external tunic, the fibrous tunic, consists of
the posterior sclera and the anterior cornea. The middle tunic, vascular tunic, consists of the
choroid, the ciliary body and the iris anteroposteriorly. The inner tunic, sensory tunic, consists
of an outer pigmented layer and an inner nervous layer.
2. Two important spots can be found on the posterior retinal wall. (1) the macula lutea with its
central fovea which is the area of highest visual acuity; (2) the optic disc, called blind spot,
which is the place where axons of ganglion cells form the optic nerve.
3. Within the space anterior to the lens aqueous humor is contained in both anterior and posterior
chambers, and the gel-like vitreous humor is contained within the space posterior to the lens.
4. The biconvex lens helps to focus light. It is suspended in the eye by the suspensory ligament
(zonule) attached to the ciliary body. Tension in the zonule resists the lens' natural tendency to
round up. A resting eye, with its lens stretched flat by tension in the suspensory ligament, is set
to focus the almost parallel rays from distant points. However the diverging rays from nearby
points must be bent more sharply if they are to focus on the retina, and this is accomplished by
releasing the tension on the suspensory ligament and rounding of the lens.
5. The middle ear is a small chamber within the temporal hone. Its boundaries are the eardrum
laterally, the bony wall of the inner ear medially, a bony roof superiorly, a posterior wall that
opens into the mastoid antrum, and an anterior wall that opens into the auditory tube.
6. The auditory tube consists of posterolateral bony part and inferomedial cartilaginous part. It
connects the tympanic cavity to the pharynx and equalizes air pressure across the eardrum.
7. The three semicircular canals are bony structures, but the semicircular ducts are membranous
structures. The special receptors that measure rotational acceleration of the head are located in
some particular area called membranous ampulla of semicircular duct.
(Dai Jibin, Pan Feng)
Multiple Choice Questions
A1
1. The human nervous system can be subdivided into
.
A. autonomic nervous system (ANS) and central nervous system (CNS)
B. autonomic nervous system (ANS) and peripheral nervous system (PNS)
C. central nervous system (CNS) and peripheral nervous system (PNS)
D. somatic nervous system and central nervous system (CNS)
E. somatic nervous system and peripheral nervous system (PNS)
2. The brain has a tiered structure and, from a gross point of view, can be subdivided into
.
A. the cerebrum, the brain stem, and the cerebellum
B. diencephalon, the brain stem, and the cerebellum
C. midbrain, pons, and medulla oblongata
D. telencephalon, diencephalon, and the cerebellum
E. diencephalon, pons, and medulla oblongata
3. Which is the basic structural and functional unit of the nervous system?
A. synapse
B. neuron
C. Nissl bodies
D. nerve fiber
E. neuroglia
4. Which nerves convey stimuli from the skin, mucous membranes, and deeper structures?
A. sympathetic nerves
B. parasympathetic nerves
C. motor nerves
D. sensory nerves
E. efferent nerves
5. What consists of rough endoplasmic reticulum and associated ribosomes?
A. Nissl body
B. neurofilaments
C. neurofibrils
D. neuroglia
E. microtubules
6. The conducting (propagating or transmitting) part of the nerve cell body is
A. soma
B. perikaryon
C. Nissl bodies
D. axon
E. dendrites
7. Which neuron appears T-shaped?
A. pseudounipolar neurons
B. unipolar neurons
C. bipolar neurons
D. tripolar neurons
E. multipolar neurons
8. The peripheral myelinated fiber consists of
.
A. axon and myelin sheath
B. axon and neurilemma
C. myelin sheath and neurilemma
D. axon, myelin sheath, and neurilemma
E. myelin sheath and membrane
9. Synaptic transmission permits information from
.
A. synaptic junction
B. gap junction
C. presynaptic element
D. synaptic cleft
E. postsynaptic element
10. In the CNS, the myelin is formed by
.
A. Schwann cells
B. astrocytes
C. oligodendrocytes
D. macroglia
E. microglia
11. In the PINS, the myelin is formed by
.
A. Schwann cells
B. astrocytes
C. oligodendrocytes.
D. macroglia
E. microglia
12. In the PNS, many nerve fibers assemble to form a
.
.
A. fiber bundles
B. white matter
C. nerve
D. tracts
E. fasciculi
True or False Questions
1. Functionally, the nervous system can be subdivided into two systems: somatic nervous system
and autonomic (visceral) nervous system.
(
)
2. Synapses are composed of three basic elements: the presynaptic element, the synaptic cleft, and
the postsynaptic element.
(
)
3. The gray matter on the surface of the brain is termed medulla.
(
)
4. In CNS, myelin is formed by Schwann cells.
(
)
5. Afferent fibers are involved in motor functions, such as the contraction of muscles or secretion
of glands.
(
)
6. Nissl bodies can usually be seen in nerve cell bodies stained with basophilic dyes, and consists
of rough endoplasmic reticulum and associated ribosomes.
(
)
7. The receptive part of the neuron is the dendrite.
(
)
8. Bipolar neurons transmit both sensory and motor nerve impulses, and are characteristic of the
brain, spinal cord, and peripheral autonomic nervous system.
(
)
9. The peripheral myelinated fiber consists of axon, myelin sheath, and neurilemma.
(
)
10. Classically, glial cells are grouped into .two categories in the CNS, macroglia and microglia.
(
)
Explanation of Terms
1. neuron
2. axon
3. dendrites
4. nerve fibers
5. neuroglia
6. gray matter
7. nucleus
8. ganglia
9. reticular formation
10. synapse
11. electrical synapses
Answer the Following Questions
1. What's the role of human nervous system?
2. What does the central portion of the nervous system consist of?
3. What does the peripheral nervous system consist of?
4. What is synapse?
5. How to subdivide the human nervous system?
6. How to classify the neurons?
7. How does impulse transmit at synaptic site?
8. What do the glial cells include?
ANSWERS
Multiple Choice Questions
1. C 2. A 3. B 4. D 5. A 6. C 7. A 8. D 9. B 10. C
11. A 12. C
True or False Questions
1. T
2. T
3. F
4. F
5. F
6. T
7. T
8. F
9. T
10. T
Explanation of Terms
1. It is the basic structural and functional unit of the nervous system. The functions of neurons are
to receive and integrate incoming information from sensory receptors or other neurons and to
transmit information to other neurons or effector organs.
2. It is a process extending from the nerve cell body. Most neurons give rise to a single axon. It's
the conducting (propagating or transmitting) part of the neuron, and has one or more collateral
branches.
3. The processes extend from the nerve cell body. The receptive part of the neuron is the dendrite,
or dendritic zone. Some dendrites give rise to dendritic spines, which are small
mushroom-shaped projections that act as fine dendritic branches and receive synaptic inputs.
4. In the nervous system, axons with their surrounding myelin sheath and/or nerve membrane are
usually termed nerve fibers.
5. Or glial cells, it outnumber neurons in the brain and spinal cord 10: 1. These cells appear to play
a number of important roles, including myelin formation, guidance of developing neurons,
maintenance of extracellular K+ levels, and reuptake of transmitters after synaptic activity.
Classically, glial cells are grouped into two categories in the CNS, macroglia and microglia. The
term macroglia refers to astrocytes and oligodendrocytes, in the broad sense, including
ependymal cells. And in the PNS, the glial cells include Schwann cells, and the satellite cells of
peripheral sensory ganglia.
6. In the CNS, some regions are relatively enriched in nerve cell bodies (e. g. , the central
portion of the spinal cord, the surface of the cerebral hemisphere) and are called gray matter.
7. In the CNS, out of cortex, nerve cell bodies serving a common function, often with a common
target, are frequently grouped together into nucleus.
8. In the PNS, nerve cell bodies with similar functions and connections that are grouped together
outside the CNS are called ganglia.
9. Among the more conspicuous tracts and nuclei throughout the CNS are extensive fields of
intermingled neurons and nerve fibers collectively termed the reticular formation.
10. Communication between neurons usually occurs from the axon terminal of the
transmitting neuron (presynaptic side) to the receptive region of the receiving neuron
(postsynaptic side). This specialized interneuronal complex is a synapse.
11. Current passes directly from cell to cell through specialized junctions called electrical
synapses, or gap junctions. Electrical synapses are most common in invertebrate nervous
systems, although they are found in a small number of sites in the mammalian CNS.
Answer the Following Questions
1. The human nervous system is a control system that regulates and coordinates all functions of
the organism. Besides, it is also the seat of all mental activity, including consciousness, memory,
and thinking. The human nervous system is the most complex and elegant computing device
that exists. It receives and interprets an immense array of sensory information, controls a variety
of simple and complex motor behaviors, and engages in deductive and inductive logic. The
nervous system performs these functions as an information processing system.
2. The central portion of the nervous system consists of the brain and the elongated spinal cord.
The brain can be subdivided into the cerebrum, the brain stem, and the cerebellum. The
cerebrum (forebrain) consists of the telencephalon and the diencephalon; the telencephalon
includes the cerebral cortex (gray matter), subcortical white matter, and the basal ganglia. The
major subdivisions of the diencephalon are the thalamus and hypothalamus. The brain stem
consists of the midbrain (mesencephalon), pons, and medulla oblongata. The cerebellum
includes the vermis and two hemispheres. The brain, which is hollow, contains a system of
spaces called ventricles; the spinal cord has a narrow central canal that is largely obliterated in
adulthood. These spaces are filled with cerebrospinal fluid.
3. The peripheral nervous system (PNS) consists of spinal nerves, cranial nerves, and their
associated ganglia. The nerves contain nerve fibers that conduct information to (afferent) or
from (efferent) the CNS. In general, efferent fibers, or motor nerves, are involved in motor
functions, such as the contraction of muscles or secretion of glands; afferent fibers, or sensory
nerves, usually convey sensory stimuli from the skin, mucous membranes, and deeper structures.
Visceral motor nerves are divided into sympathetic and parasympathetic nerves.
4. Communication between neurons usually occurs from the axon terminal of the transmitting
neuron (presynaptic side) to the receptive region of the receiving neuron (postsynaptic side).
This specialized interneuronal complex is a synapse, or synaptic junction. Synapses are
composed of three basic elements: the presynaptic element, the synaptic cleft, and the
postsynaptic element. Some synapses are located between an axon and a dendrite (axodendritic
synapses, which tend to be excitatory), whereas others are located between an axon and a nerve
cell body (axosomatic synapses, which tend to be inhibitory). Still other synapses are located
between an axon terminal and another axon; these axoaxonic synapses modulate transmitter
released by the postsynaptic axon. Synaptic transmission permits information from many
presynaptic neurons to converge on a single postsynaptic neuron. Some large cell bodies receive
several thousand synapses.
5. Anatomically, the human nervous system is a complex of two subdivisions.
(1) Central nervous system (CNS)-The CNS consists of the brain and spinal cord, both of them
are encased in bone and protected by coverings (meninges) as well as cerebrospinal fluid (CSF).
(2) Peripheral nervous system (PNS)--The PNS is formed by twelve pairs of cranial nerves and
31 pairs of spinal nerves.
Functionally, the nervous system can also be subdivided into two systems.
(1) Somatic nervous system--This innervates the structures of the body wall (muscles, skin, and
mucous membranes).
(2) Autonomic (visceral) nervous system (ANS)--The ANS contains portions of the central
and peripheral systems. It controls the activities of the cardiac muscles, smooth muscles and
glands of the internal organs (viscera) and the blood vessels and returns sensory information
to the brain.
6. According to the numbers of processes, neurons can be divided into three types:
pseudounipolar, bipolar, and multipolar neurons. Pseudounipolar neurons appear T- shaped, in
which the initial portion of the two processes approximate and eventually fuse. Such sensory
neurons convey nerve impulses from a variety of receptors. Bipolar neurons that have two
processes, are sensory in function and transmit impulses generated by sensory receptor endings.
Multipolar neurons transmit both sensory and motor nerve impulses, and are-characteristic of
the brain, spinal cord, and peripheral autonomic nervous system.
According to the function and the conducting direction, neurons can be grouped into three
categories: sensory, motor, and interneurons or association neurons. Afferent or sensory neurons
convey information from the periphery to the CNS, whereas efferent or motor neurons carry
impulses away from the CHIS. Interneurons, the remaining cells, are the most abundant
signaling elements in the CNS and transmit impulses locally.
7. Impulse transmission at most synaptic sites involves, the release of a chemical transmitter
substance; at other sites, current passes directly from cell to cell through
specialized
junctions called electrical synapses, or gap junctions. Electrical synapses are most common in
invertebrate nervous systems, although they are found in a small number of sites in the
mammalian CNS. Chemical synapses have several distinctive characteristics- synaptic vesicles
in the presynaptic element, a synaptic cleft, and a postsynaptic element. The dense of the cell
membrane is thick on both the presynaptic side and the postsynaptic side. Synaptic vesicles
contain neurotransmitters, and each vesicle contains a small packet, or quanta, of transmitter.
Synapses are very diverse in their shapes and other properties. Some are inhibitory and some
excitatory; in some, the transmitter is acetylcholine; in others, it is a catecholamine,
amino acid, or other substance. Some synaptic vesicles are large, some small; some have a
dense core, whereas others do not. Flat synaptic vesicles appear to contain an inhibitory
mediator; dense-core vesicles contain catecholamines. A large number of molecules act as
neurotransmitters at chemical synapses. These neurotransmitters are present in the synaptic
terminal (terminal bouton), a bulb at the end of an axon in which neurotransmitter molecules are
stored and released. The action of the neurotransmitters may be blocked by pharmacologic
agents. Some presynaptic neurons can release more than one transmitter; differences in the
frequency of nerve stimulation probably control which transmitter is released.
8. Glial cells or neuroglia outnumber neurons in the brain and spinal cord 10:1. These ceils appear
to play a number of important roles, including myelin formation, guidance of developing
neurons, maintenance of extracellular K+ levels, and reuptake of transmitters after synaptic
activity. Classically, glial cells are grouped into two categories in the CNS, macroglia and
microglia. The term macroglia refers to astrocytes and oligodendrocytes.
There are two broad classes of astrocytes, protoplasmic and fibrous. Protoplasmic astrocytes are
more delicate, confined to the grey matter and their many processes are branched. Fibrous
astrocytes are more fibrous, situated chiefly in the white matter, and their processes (containing
glial fibrils) are seldom branched. Astrocytic processes radiate in all directions from a small cell
body. They provide structural support to nervous tissue and act during development as
guidewires that direct neuronal migration. They also maintain appropriate concentrations of
ions such as K+ within the extracellular space of the brain and spinal cord. Astrocytes may also
play a role in synaptic transmission.
Oligodendrocytes predominate in white matter; they form myelin in the CNS and may provide
some nutritive support to the neurons they envelop. A single oligodendrocyte may wrap myelin
sheaths around many axons. An oligodendrocyte may myelinate up to 40 to 50 axons. In
peripheral nerves, by contrast, myelin is formed by Schwann cells. Each Schwann cell
myelinates a single axon and remyelination can occur at a brisk pace after injury to the myelin
in the peripheral nerves.
Microglial ceils (microglia) are the smallest of the glia cells, have an elongated nucleus; they
are the macrophages, or scavengers, of the CNS. When an area of the brain or spinal cord is
damaged or infected, microglias migrate to the site of injury to remove cellular debris. Some
microglias are always present in the brain, but when injury or infection occurs, others enter the
brain from blood vessels.
In the broad sense, the glial cells in the CNS should include ependymal cells, the cells lining the
ventricles of the brain, which also belong to macroglia. And in the PNS, the glial cells include
Schwann cells, and the satellite cells of peripheral sensory ganglia.
(Gao Xiulai)
Multiple Choice Questions
A1
1. The caudal extremity of the spinal cord in adults lies at the level of the inferior margin of the
lumbar vertebra.
A. first
B. second
C. third
D. fourth
E. fifth
2. The caudal extremity of the spinal cord in a new-born infant lies at the level of the inferior
margin of the
lumbar vertebra.
A. first
B. second
C. third
D. fourth
E. fifth
3. Which of the following statements is correct?
A. The spinal cord occupies the upper one-thirds of the adult spinal canal within the vertebral
column.
B. The spinal cord begins as a continuation of the medulla oblongata.
C. The length of the adult spinal cord ranges from 32 to 35 cm.
D. The diameter of the spinal cord is same at different levels.
E. The conus medullaris ends at the level of the intervertebral disc between the fourth and fifth
lumbar vertebra.
4. The filum terminale attaches to the
.
A. lumbar vertebra
B. first sacral vertebra
C. second sacral vertebra
D. coccyx
E. none of the above
5. Which of the following statements about the ilium terminale is wrong?
A. It is long about 20 cm.
B. It becomes invested by dura mater.
C. It attaches to the posterior surface of the coccyx.
D. It is invested by pia mater.
E. It consists mostly of pia mater.
6. The cervical enlargement of the spinal cord extends from
to
of the spinal
segments.
A. C2,T1
B. C6, T1
C. C1, T3
D. C6, C7
E. C4, T1
7. The lumbosacral enlargement of the spinal cord extends from
to
of the
spinal segments.
A. T12, L4
B. L2, Col
C. L2, S3
D. L2, L4
E. L3, L5
8. About the lumbosacral enlargement of the spinal cord, which of the following statements is
correct?
A. It extends from the second lumbar to the third sacral segments.
B. Nerves that supply the upper extremities arise from it.
C. It extends from the fourth cervical to the first thoracic segments.
D. It extends from the fourth lumbar to the first sacral segments.
E. None of the above.
9. Which of the following statements is correct?
A. A shallow anterior median fissure and a deep posterior median sulcus divide the spinal
cord into asymmetric right and left halves.
B. There are two pairs of the shallow anterolateral sulci.
C. There are two pairs of the shallow postlateral sulci.
D. The posterior intermediate sulci are present between the posterior median sulcus and the
postlateral sulci.
E. None of the above.
10. Which of the following statements is wrong?
A. The spinal cord has 8 cervical segments.
B. The spinal cord has 12 thoracic segments.
C. The spinal cord has 5 lumbar segments.
D. The spinal cord has 5 sacral segments.
E. The spinal cord has 3-4 coccygeal segments.
11. A spinal segment is defined as region of the spinal cord that
.
A. corresponds to a collection of nerves passing up or down within the white matter
B. corresponds to the region of the vertebral column (i. e. cervical, thoracic, lumbar, and
sacral) to which spinal nerves are sent
C. refers to a region of the spinal cord from which a pair of spinal nerves arises
D. underlines the neural arch of a particular vertebra in the adult
E. none of the above
12. Which of the following statements is correct?
A. The spinal cord is enlarged in the thoracic region for innervation of the upper limbs.
B. The spinal cord is enlarged in the lumbosacral region for innervation of the lower limbs.
C. In embryos, the spinal cord occupies only the superior two-thirds of the vertebral canal.
D. In adults, the spinal cord occupies the full length of the vertebral canal.
E. The cauda equina is composed of ventral but not dorsal roots.
13. Which of the following statements is not true in adults?
A. The tapering end of the spinal cord may terminate as high as T12 or as low as Ls.
B. The proportion of gray to white matter is greatest in the lumbar and cervical enlargements.
C. In the lumbar region, the posterior and anterior columns are broad and expanded.
D. The ilium terminal is the vestigial remnant of the caudal part of the spinal cord that was in
the tail of the embryo.
E. The spinal cord has a lumbosacral enlargement for the lower limb but no equivalent
enlargement for the smaller upper limb.
14. Which of the following statements is not true in respect to caudal epidural anesthesia?
A. A local anesthetic is injected into the sacral hiatus or the posterior sacral foramina.
B. The anesthetic acts on the coccygeal nerves.
C. The height to which the anesthetic travels is primarily limited by the amount of fat in the
epidural space.
D. The anesthetic acts on the cauda equina.
E. The sacral hiatus is located between the sacral cornua and inferior to the 4th sacral spinous
process or median sacral crest.
15. The sympathetic fibers in the nerves originate in which of the following spinal cord segments?
A. C5-T1
B. C1-T3
C. T1-L3
D. T9-S1
E. L1-S2
16. The segments of the C1-4 spinal cord match
of the vertebral bodies.
A. C1-4
B. C2-6
C.C3-7
D. C4-T1
E. none of the above
17. The segments of the C5-T4 spinal cord match
of the vertebral bodies.
A. C5-T4
B. C4-T3
C. C6-T4
D. C7-T5
E. T1-T5
18. The segments of the T5-8 spinal cord match
of the vertebral bodies.
A. T5-8
B.T1-7
C.T3-6
D.T6-12
E. none of the above
19. The segments of the T9-12 spinal cord match
of the vertebral bodies.
A. T3-8
B. L1-3
C. T1-6
D. T6-9
E. T9-12
20. The segments of the L1-5 spinal cord match
of the vertebral bodies.
A. T12-L3
B. L1-5
C. T8-12
D. L3-4
E. T10-12
21. The segments of the S1-Co1 spinal cord match the
vertebral body.
A. L1
B. L2
C. S1
D. S4
E. L3
22. Which of the following statements is not true?
A. The spinal cord consists of both gray and white matter.
B. A cross section of the spinal cord shows an H-shaped internal mass of white matter
surrounded by gray matter.
C. The gray matter is made up of two symmetric portions joined across the midline by
a transverse connection of gray matter, the gray commissure.
D. The anterior horn is the enlarged protrusion in the front of the gray matter of each side.
E. The ventral horn contains the cells of origin of the fibers of the anterior roots.
23. Which of the following statements is wrong?
A. The posterior horn reaches almost to the posterolateral sulcus.
B. The posterior horn can be divided into a head, a neck and a relatively broad base.
C. The lateral horn is the portion of gray matter between the anterior and posterior horn.
D. The posterior horn contains pregangionic cells for the autonomic nervous system.
E. The region between the anterior horn and posterior horn is the intermediate zone.
24. Which of the following statements is correct?
A. Within spinal segments C5-L1, preganglionic sympathetic neurons within the lateral column
give rise to sympathetic axons.
B. Within spinal segments S1-S5, there are sacral parasympathetic neurons within the lateral
column.
C. The form and quantity of the gray matter vary at different levels of the spinal cord.
D. In the cervical region, the posterior horn is more expansive than the anterior horn.
E. In the cervical region, the anterior horn is more narrow than the posterior horn.
25. Which of the following statements is right?
A. The spinal cord has three white funiculi on each half.
B. The anterior funiculus lies between the posterolateral sulcus and the anterior median fissure.
C. The posterior funiculus lies between the posterior median sulcus and the anterolateral sulcus.
D. The lateral funiculus lies between the anterior median fissure and the anterolateral sulcus.
E. Anterior to the gray commissure is a bundle of transverse fibers, the posterior white
commissure.
26. In which Rexed's lamina is the posteromarginal nucleus situated?
A. lamina Ⅴ
B. lamina Ⅳ
C. lamina Ⅲ
D. lamina Ⅱ
E. lamina Ⅰ
27. In which Rexed's lamina is the substantia gelatinosa located?
A. lamina Ⅰ
B. lamina Ⅱ
C. lamina Ⅲ
D. lamina Ⅳ
E. lamina Ⅴ
28. In which Rexed' s lamina is the nucleus propdus situated?
A. lamina Ⅰ
B. lamina Ⅱ
C. lamina Ⅲ
D. lamina Ⅴ
E. lamina Ⅵ
29. In which Rexed's lamina is the thoracic nucleus located?
A. lamina Ⅵ
B. lamina Ⅶ
C. lamina Ⅷ
D. lamina Ⅸ
E. lamina Ⅹ
30. In which Rexed's laminae is the medial motor neuron column situated?
A. lamina Ⅴ and Ⅵ
B. lamina Ⅵ and Ⅶ
C. lamina Ⅶ and Ⅷ
D. lamina Ⅷ and Ⅸ
E. lamina Ⅸand Ⅹ
31. In which Rexed's laminae is the lateral motor neuron column located?
A. lamina Ⅸ and Ⅹ
B. lamina Ⅷ and Ⅳ
C. lamina Ⅶ and Ⅷ
D. lamina Ⅵ and Ⅶ
E. lamina Ⅴ and Ⅵ
32. Which of the following statements is wrong?
A. The fasciculus gracilis is large proportion of the heavily myelinated fibers of the posterior
root enter the ipsilateral posterior funiculus of the spinal cord.
B. The fibers arising from the sacral, lumbar and lower eight thoracic segments make up the
fasciculus gracilis in the posterior funiculus.
C. The fibers of the fasciculus gracilis and fasciculus cuneatus are somatotopically organized
in their courses.
D. After reaching the pons, the fasciculus gracilis and the fasciculus cuneatus terminate upon
the nuclei gracilis and nuclei cuneatus respectively.
E. The fasciculus gracilis and the fasciculus cuneatus conduct the kinesthetic and the
discriminating tactile senses.
33. Which of the following statements is not true?
A. The spinothalamic tract includes the lateral and anterior spinothalamic tracts.
B. The lateral spinothalamic tract is closely outer to the anterior spinocerebellar tract.
C. The nucleus proprius receives the thin myelinated fibers associated with pain and thermal
senses from the dorsolateral fasciculus.
D. The anterior spinothalamic tract arises from the nucleus proprius.
E. The spinothalamic tract transmits the pain and thermal sensations of the trunk and limbs.
34. Which of the following statements is not correct?
A. The anterior corticospinal tract decussates in the medulla oblongata.
B. The lateral corticospinal tract extends to the most caudal part of the spinal cord.
C. The lateral corticospinal tract has somatotopical arrangement from the lateral to medial and
they are in sacral, lumbar, thoracic and cervical segmental order.
D. The anterior corticospinal tract occupies a strip adjacent to the anterior median fissure.
E. The anterior corticospinal tract normally extends only to the upper thoracic spinal segments.
35. Which of the following statements is right?
A. The rubrospinal tract lies posterior to the fibers of the lateral corticospinal tract.
B. The fibers of the rubrospinal tract arise from the vestibulae nucleus of the midbrain.
C. The function of the rubrospinal tract is to excite the extensor muscles of the trunk and limbs.
D. The vestibulospinal tract arises from the lateral vestibulae nucleus of the opposite side.
E. The function of the vestibulospinal tract is to excite the extensor muscles of the trunk and
limbs.
36. The function of the lateral corticospinal tract is
.
A. fine motor function
B. gross and postural motor function
C. muscle tone and posture
D. postural reflexes
E. reflex head turning
37. The function of the anterior corticospinal tract is
.
A. fine motor function
B. gross and postural motor function
C. muscle tone and posture
D. postural reflexes
E. reflex head turning
38. The function of the rubrospinal tract is
.
A. fine motor function
B. gross and postural motor function
C. muscle tone and posture
D. postural reflexes
E. reflex head turning
39. The function of the vestibulospinal tract is
.
A. fine motor function
B. gross and postural motor function
C. muscle tone and posture
D. postural reflexes
E. reflex head turning
40. The function of the tectospinal tract is
.
A. fine motor function
B. gross and postural motor function
C. muscle tone and posture
D. postural reflexes
E. reflex head turning
41. The function of the medial longitudinal fasciculus is
.
A. coordination of head and eye movements
B. gross and postural motor function
C. muscle tone and posture
D. postural reflexes
E. reflex head turning
42. Which of the following statements is not correct?
A. The stretch reflexes are also called superficial reflexes.
B. The stretch reflex is based on a monosynaptic reflex arc.
C. In the stretch reflex, only two-order neurons are involved.
D. The stretch reflex results in the contraction of a muscle when it is stretched.
E. One of the stretch reflexes is the knee-jerk.
43. Which of the following statements is not true?
A. The flexor reflexes are involved association neurons in addition to the sensory and motor
neurons.
B. Since more than two-order neurons are involved, there is more than one synapse in the
flexor reflex.
C. The flexor reflexes are polysynaptic reflex arcs.
D. The flexor reflex is protective in that it results in the movement of an extremity to avoid
pain.
E. None of the above.
44. About the simple reflex, which of the following statements is not true?
A. The receptor is a special sense organ, cutaneous end-organ, or muscle spindle,
whose stimulation initiates an impulse.
B. The afferent neuron transmits the impulse through a peripheral nerve to the central nervous
system.
C.. The interneurons relay the impulse to the efferent neuron.
D. The afferent neuron passes outward in the nerve and delivers the impulse to an effector.
E. The effector is the muscle or gland that produces the response.
45. Which of the following statements is not correct?
A. Reflexes that result in the contraction of skeletal muscles are known as somatic reflexes.
B. Reflexes that cause the contraction of smooth, or cardiac muscle, or secretion by glands
are visceral reflexes.
C. The stretch reflexes are also called tendon reflexes.
D. The tendon reflexes are the deep tendon reflexes.
E. None of the above.
46. About the stretch reflex, which of the following statements is not correct?
A. It provides a feedback mechanism for maintaining appropriate muscle tone.
B. It depends on specialized sensory receptors, muscle spindles.
C. It depends on specialized afferent nerve fibers, primarily Ia fibers.
D. It is based on a monosynaptic reflex arc.
E. None of the above.
47. The spinal cord transection results in a complete lose of the followings below the level of the
lesion, except
.
A. all somatic sensation
B. hearing
C. all visceral sensation
D. all reflex activity
E. all motor function
48. Hemisection of the spinal cord produces a Brown-Sequard syndrome. The patient with signs
and symptoms includes the followings, except
.
A. the ipsilateral motor neuron paralysis below the level of lesion
B. loss of the ipsilateral deep sensation below the level of lesion
C. loss of the contralateral superficial sensation below the level of lesion
D. loss of the contralateral deep sensation below the level of lesion
E. loss of the ipsilateral proprioceptive, vibratory and two-point discrimination sense below
the level of lesion
49. Lesion of anterior horn of the spinal cord causes the followings, except
.
A. ipsilateral lower motor neuron paralysis in the segments of the lesion
B. infantile paralysis
C. the upper motor neuron paralysis above the segments of the lesion
D. lower motor neuron paralysis of the same side in the segments of the lesion
E. problem when the patient runs
50. Lesion around the central gray matter can affect or cause the followings, except
.
A. the decussating fibers of the spinothalamic tract from both sides without affecting other
ascending tracts
B. dissociated sensory abnormalities with loss of pain and temperature sensibility in
appropriate dermatomes
C. the decussating fibers of the spinothalamic tract from both sides without affecting other
descending tracts
D. infantile paralysis
E. syringomyelia
51. The brainstern is formed by the structures as follows:
A. medulla oblongata, pons and cerebellum
B. medulla oblongata, pons, middle brain and cerebellum
C. pons, middle brain and cerebellum
D. medulla oblongata, pons, middle brain and thalamus
E. medulla oblongata, pons and middle brain
52. The structure in anterior surface of brainstem doesn't include:
A. superior medullary velum
B. basilar sulcus
C. cerebral peduncles
D. pyramids
E. interpeduncular fossa
53. The structure in anterior surface of pons includes:
A. cerebral peduncles
B. superior cerebellar peduncle
C. basilar sulcus
D. inferior cerebellar peduncle
E. olive
54. The structure belong to posterior surface of medulla oblongata is
.
A. colliculus of facial nerve
B. superior colliculus
C. locus ceruleus
D. superior medullary velum
E. cuneate tubercle
55. The nerve connecting to posterior surface of brainstem is
.
A. oculomotor nerve
B. trochlear nerve
C. hypoglossal nerve
D. facial nerve
E. abducent nerve
56. About the position of brainstem, which statement is right?
A. It locates in middle part of posterior cranial fossa.
B. It is related to clivus of occipital bone ventrally and cerebellum dorsally.
C. The cerebral hemisphere is above it.
D. It may extend to the foramen magnum downwards.
E. The thalamus is above it.
57. The cranial nerves related to medulla oblongata doesn't include:
A. hypoglossal nerve
B. vagus nerve
C. glossopharyngeal nerve
D. abducent nerve
E. accessory nerve
58. The cranial nuclei located in pons are
.
A. nucleus of facial nerve, motor nucleus of trigeminal nerve and nucleus ambiguus
B. nucleus of facial nerve, hypoglossal nucleus and vestibular nucleus
C. nucleus of facial nerve, motor nucleus of trigeminal nerve and abducent nucleus
D. nucleus of facial nerve, motor nucleus of trigeminal nerve and nucleus ceruleus
E. abducent nucleus, superior salivatory nucleus and superior olivary nucleus
59. Which group of the cranial nuclei belongs to medulla oblongata as follows?
A. solitary nucleus, nucleus ambiguus and dorsal nucleus of vagus nerve
B. hypoglossal nucleus, superior and inferior salivatory nuclei
C. solitary nucleus, superior salivatory nucleus and dorsal nucleus of vagus nerve
D.hypoglossal nucleus, dorsal nucleus of vagus nerve, inferior olivary nucleus
E. nucleus ambiguus, accessory nucleus, and abducent nucleus
60. Which nucleus doesn't belong to the parasympathetic nuclei as follows?
A. accessory nucleus of oculomotor nerve
B. superior salivatory nucleus
C. dorsal nucleus of vagus nerve
D. inferior salivatory nucleus
E. nucleus ambiguus
61. The parasympathetic nuclei in brainstem include:
A. accessory nucleus of oculomotor nerve, dorsal nucleus of vagus nerve, and superior
salivatory nucleus
B. dorsal nucleus of vagus nerve, hypoglossal nucleus, solitary nucleus
C. accessory nucleus of oculomotor nerve, superior salivatory nucleus, and
hypoglossal nucleus
D. inferior salivatory nucleus, dorsal nucleus of vagus nerve, and inferior olivary nucleus
E. accessory nucleus of oculomotor nerve, dorsal nucleus of vagus nerve, and sacral
parasympathetic nucleus
62. Which nucleus gives off parasympathetic preganglionic fibers to enter facial nerve in
brainstem?
A. inferior salivatory nucleus
B. nucleus ambiguus
C. superior salivatory nucleus
D. dorsal nucleus of vagus nerve
E. accessory nucleus of oculomotor nerve
63. The parasympathetic preganglionic fibers in facial nerve arise from
.
A. accessory nucleus of oculomotor nerve
B. superior salivatory nucleus
C. inferior salivatory nucleus
D. dorsal nucleus of vagus nerve
E. superior olivary nucleus
64. The parasympathetic preganglionic fibers in glossopharyngeal nerve arise from
.
A. dorsal nucleus of vagus nerve
B. inferior salivatory nucleus
C. hypoglossal nucleus
D. superior salivatory nucleus
E. nucleus ambiguus
65. Which nucleus receives the visceral sensory fibers in the brainstem?
A. cochlear nucleus
B. vestibular nucleus
C. inferior olivary nucleus
D. solitary nucleus
E. sensory nucleus of trigeminal nerve
66. Which group of cranial nuclei in brainstem belongs to the general somatic motor nucleus?
A. trochlear, abducent, hypoglossal nuclei and nucleus of oculomotor nerve
B. abducent nucleus, nucleus of facial nerve and nucleus ambiguus
C. nucleus of facial nerve, motor nucleus of trigeminal nerve and nucleus ambiguus
D. hypoglossal nucleus, accessory nucleus and nucleus ambiguus
E. abducent nucleus, motor nucleus of trigeminal nerve and nucleus of oculomotor nerve
67. The parasympathetic preganglionic fibers originated from superior salivatory nucleus may
enter
.
A. facial nerve
B. glossopharyngeal nerve
C. vagus nerve
D. oculomotor nerve
E. lacrimal nerve
68. The parasympathetic preganglionic fibers originated from inferior salivatory nucleus may
enter
.
A. facial nerve
B. vagus nerve
C. glossopharyngeal nerve
D. greater petrosal nerve
E. chorda tympani
69. Which one belongs to the special visceral motor nucleus?
A. abducent nucleus
B. nucleus of facial nerve
C. trochlear nucleus
D. nucleus of oculomotor nerve
E. hypoglossal nucleus
70. Which one doesn't belong to parasympathetic nucleus in brainstem?
A. accessory nucleus of oculomotor nerve
B. superior salivatory nucleus
C. inferior salivatory nucleus
D. nucleus ambiguus
E. dorsal nucleus of vagus nerve
71. About the gracile nucleus and cuneate nucleus, which statement is right?
A. They are cranial nuclei in the medulla oblongata.
B. They receive the somatic sensory fibers from the spinal nerves.
C. They are situated in upper part of medulla oblongata.
D. They receive the termination of fasciculus gracilis and fasciculus cuneatus.
E. They transmit the proprioceptive and fine touch sensations from opposite side of the body.
72. Following statements about the medial lemniscus, which one is incorrect?
A. The fibers of it arise from contralateral gracile and cuneate nuclei.
B. It conducts the proprioceptive and fine touch sensations from the contralateral trunk and
limbs.
C. It conducts the proprioceptive and fine touch sensations from ipsilateral trunk and limbs.
D. It is terminated in the ventral posterolateral nucleus of thalamus.
E. It is located dorsolateral to the red nucleus in the midbrain.
73. Which of the following tracts is relayed in the gracile and cuneate nuclei?
A. spinothalamic tract
B. anterior spinocerebellar tract
C. medial longitudinal fasciculus
D. posterior spinocerebellar tract
E. fasciculus gracilis and fasciculus cuneatus
74. Which group of nuclei is located in the midbrain?
A. nucleus of oculomotor nerve, trochlear nucleus, accessory nucleus of oculomotor nerve
B. nucleus of oculomotor nerve, nucleus of trochlear nerve, nucleus of abducent nerve
C. mesencephalic nucleus of trigeminal nerve, nucleus of oculomotor nerve, and nucleus of
abducent nerve
D. nucleus ceruleus, nucleus of trochlear nerve, and accessory nucleus of oculomotor nerve
E. motor nucleus of trigeminal nerve, nucleus of trochlear nerve, nucleus of oculomotor nerve
75. The following cranial nuclei are in the gray matter distributed on the surface of rhomboid
fossa except:
A. dorsal nucleus of vagus nerve
B. nucleus ambiguus
C. abducent nucleus
D. hypoglossal nucleus
E. vestibular nucleus
76. Which of the following statements concerning vestibular nucleus is correct?
A. The nucleus is situated in the reticular formation of the pons.
B. It is divided into medial vestibular nucleus and lateral vestibular nucleus.
C. It receives the afferent fibers of the vestibulocochlear nerve.
D. It is deep to the vestibular area of rhomboid fossa.
E. The nucleus belongs to the general somatic sensory nucleus.
77. The following statements concerning the inferior olivary nucleus complex are correct except:
A. It is deep to the olive.
B. It is divided into inferior olivary nucleus, medial accessory olivary nucleus and dorsal
accessory olivary nucleus.
C. It is important relay nucleus complex in the brainstem.
D. Most of the efferent fibers from it traverse into contralateral inferior cerebellar peduncle.
E. All of the efferent fibers from it traverse into contralateral inferior cerebellar peduncle.
78. Which of the following statements about the cochlear nuclei is correct?
A. It receives the afferent fibers from contralateral cochlear nerve.
B. The efferent fibers of it traverse into bilateral lateral lemniscus.
C. The efferent fibers of it traverse into ipsilatera[ lateral lemniscus.
D. The efferent fibers of it traverse into contralateral lateral lemniscus.
E. It receives the afferent fibers from ipsilateral vestibulocochlear nerve.
79. The accessory nucleus of oculomotor nerve gives off efferent fibers to control.
A. lacrimal gland
B. dilator pupillae
C. sphincter pupillae and ciliary muscle
D. superior obliquus
E. lateral rectus
80. Which structures are controlled by parasympathetic fibers from superior salivatory nucleus?
A. submandibular gland, sublingual gland and lacrimal gland
B. lacrimal gland, submandibular gland and parotid gland
C. lacrimal gland and parotid gland
D. submandibular gland, sublingual gland and parotid gland
E. lacrimal gland, sublingual gland and parotid gland
81. Which structures are controlled by parasympathetic fibers from inferior salivatory nucleus?
A. lacrimal gland
B. parotid gland
C. submandibular gland
D. sublingual gland
E. thyroid gland
82. The following structures are controlled by the parasympathetic fibers of dorsal nucleus of
vagus nerve except:
A. thyroid gland
B. liver
C. pancreas
D. stomach
E. sigmoid colon
83. Which of the following statements about the internal structure of brainstem is correct?
A. The gray matter is inside and white matter is on the surface of brainstem.
B. The pyramid contains-ascending and descending tracts.
C. Gracile and cuneate nuclei are cranial nuclei.
D. The reticular formation is situated between the gray matter and white matter.
E. The inner cavity of brainstem is the fourth ventricle.
84. Where is the red nucleus located?
A. in the diencephalon
B. in the midbrain
C. in the cerebellum
D. in the upper part of pons
E. in the junctional zone between pons and midbrain
85. The relay nucleus in brainstem doesn't include:
A. inferior olivary nucleus
B. gracile nucleus
C. substantia nigra
D. superior colliculus
E. vestibular nucleus
86. Which structure as follows forms the trapezoid body?
A. spinothalamic lemniscus
B. trigeminal lemniscus
C. medial lemniscus
D. the fibers from contralateral cochlear nucleus
E. the fibers from ipsilateral cochlear nucleus
87. The nucleus giving off somatic motor fibers of the glossopharyngeal nerve is:
A. nucleus of abducent nerve
B. inferior salivatory nucleus
C. superior salivatory nucleus
D. parasympathetic nucleus
E. inferior olivary nucleus
88. Which of the following statements concerning lateral lemniscus is correct?
A. It transmits the auditory sense from ipsilateral ear only.
B. It is formed by the fibers arising from contralateral cochlear nucleus.
C. It is formed by the fibers arising from ipsilateral cochlear nucleus.
D. All of the fibers of it terminate to medial geniculate body directly.
E. It is formed by the fibers arising from bilateral cochlear nucleus.
89. The fibers of lateral lemniscus arise from:
A. contralateral cochlear nucleus
B. bilateral cochlear nucleus
C. ipsilateral vestibular nucleus
D. ipsilateral vestibular and cochlear nuclei
E. ipsilateral cochlear nucleus
90. The relay nucleus in the midbrain doesn't includes:
A. red nucleus
B. substantia nigra
C. nucleus ceruleus
D. nucleus of inferior colliculus
E. superior colliculus
91. The following cranial nuclei receive the projection of bilateral corticonuclear tract except:
A. nucleus of hypoglossal nerve
B. nucleus of abducent nerve
C. nucleus of trochlear nerve
D. motor nucleus of trigeminal nerve
E. nucleus of oculomotor nerve
92. The nucleus receiving the contralateral corticonuclear tract only is
.
A. motor nucleus of trigeminal nerve
B. upper part of nucleus of facial nerve
C. dorsal nucleus of vagus nerve
D. lower part of nucleus of facial nerve
E. nucleus ambiguus
93. The following statements concerning spinothalamic lemniscus are correct except:
A. It arises from contralateral nucleus proprius of spinal cord.
B. The fibers stop into the ventral posteromedial nucleus of thalamus.
C. In the pons, it passes through the trapezoid body and lateral to the medial lemniscus.
D. The fibers stop into the ventral posterolateral nucleus of thalamus.
E. It conducts the pain, thermal and rough touch-pressure sensation from contralateral trunk
and limbs.
94. Which of the following statements concerning the corticospinal tract is wrong?
A. It arises from ipsilateral upper 2/3 of precentral gyrus and anterior part of paracentral lobule.
B. It passes downwards in the crus cerebri of midbrain.
C. It passes downwards in the pyramids of pons and medulla oblongata.
D. It controls the muscles of contralateral limbs and bilateral trunk.
E. Most of the fibers of the tract cross the midplane in pyramidal decussation.
95. The major nuclei of reticular formation in brainstem don't include:
A. nucleus raphes magnus
B. nucleus raphes dorsalis
C. nucleus raphes pallidus
D. lateral and medial parabrachial nuclei
E. superior olivary nucleus
96. The incorrect statement about the substantia nigra is:
A. It appears in mammals only especially it reaches its greatest development in man.
B. It is divided into a dorsal compact part and a ventral reticular part.
C. The dopaminergic neurons which contain the numerous melanin pigments are in the
compact part mainly.
D. It is situated between the tegmentum and rectum of midbrain.
E. It is situated between the tegmentum and crus cerebri of midbrain.
97. The following statements concerning the trigeminal lemniseus are incorrect except:
A. It is formed by the fibers arising from the ipsilateral trigeminal ganglion.
B. It is formed by the fibers arising from the contralateral trigeminal ganglion.
C. It is formed by the fibers arising from the ipsilateral pontine and spinal nuclei of
trigeminal nerve.
D. It is formed by the fibers arising from the contralateral pontine and spinal nuclei of
trigeminal nerve.
E. It is formed by the fibers arising from the contralateral mesencephalic nucleus of
trigeminal nerve.
98. The tract transmitting the pain, thermal and touch-pressure sensation of contralateral head
and face is:
A. medial lemniscus
B. trigeminal lemniscus
C. spinothalamic lemniscus
D. medial longitudinal fasciculus
E. lateral lemniscus
99. The incorrect statement about red nucleus is:
A. It is situated in the center of tegmentum of midbrain.
B. It has pinkish colour in fresh specimens.
C. The afferent fibers of it are entirely derived from the cerebellar nuclei via the superior
cerebellar peduncle.
D. It is an important relay station of the somatic motor pathway.
E. The nucleus is composed of the magnocellular and parvocellular parts.
100. The fiber bundle connecting the brainstem and cerebellum doesn't include:
A. superior cerebellar peduncle
B. rubrospinal tract
C. anterior spinocerebellar tract
D. middle cerebellar peduncle
E. inferior cerebellar tract
101. Which of the following combinations is not correct?
A. corpus callosum-diencephalon
B. corpora quadrigemina-midbrain
C. cortex-cerebellum
D. thalamus-diencephalon
E. vermis-cerebellum
102. Cerebellar nuclei doesn't include:
A. dentate nucleus
B. globose nucleus
C. emboliform nucleus
D. fastigial nucleus
E. red nucleus
103. Which nucleus is related to the cerebellar cortex directly?
A. vestibular nucleus
B. red nucleus
C. substantia nigra
D. ventral posterolateral nucleus of thalamus
E. none of the above
104. Dentate nucleus projects fiber to
.
A. globus pallidus
B. substantia nigra
C. spinal cord
D. pons
E. dorsal thalamus
105. Which Is correct about the vestibulocerebellum?
A. receives fibers from spinal cord
B. receives fibers from pontocerebellar
C. receives fibers from vestibular nuclei and vestibular nerves
D. receives fibers from midbrain
E. receives fibers from cerebrum
106. Lesion of left cerebellar hemisphere will result in
.
A. consciousness proprioceptive sense loss at right side of body
B. consciousness proprioceptive sense loss at left side of body
C. locomotor ataxia at left limbs
D. hard paralysis at left limbs
E. tendon hyperreflexia at left limbs
107. Which is correct about the cerebellum?
A. located at middle cranial fossa
B. flocculonodular lobe belongs to neocerebellum
C. tonsil of cerebellum locates behind cerebellar hemisphere
D. tonsil of cerebellum nears to pons
E. connection with pons by the middle cerebellar peduncle
108. Hypothalamus is a part of the
.
A. basal nuclei
B. diencephalon
C. cerebrum
D. cerebellum
E. telencephalon
109. Which fibers are received by the ventral posteromedial nucleus?
A. trigeminal lemniscus
B. medial lemniscus
C. spinal lemniscus
D. optic radiation
E. acoustic radiation
110. Lateral geniculate body is the subcortical center of
.
A. audition
B. vision
C. pain and thermal
D. deep sensation
E. visceral sensation
111. Medial geniculate body is the subcortical center of
.
A. audition
B. vision
C. algesia and thalposis
D. deep sensation
E. visceral sensation
112. Which of the following combinations is not correct?
A. corpus callosum-diencephalon
B. corpora quadrigemina-midbrain
C. cortex-cerebellum
D. thalamus-diencephalon
E. vermis-cerebellum
113. Which is correct about dorsal thalamus?
A. located lateral to internal capsule
B. the minimum gray matter in diencephalon
C. its function is related to motion conduction
D. all of sensory pathways terminate here
E. receives deep and superficial sensation of the body
114. Which is wrong about lateral geniculate body?
A. belongs to metathalamus
B. receive lateral lemniscus
C. laminate structurelly
D. relay station of vision pathway
E. project to occipital cortex
115. Which is wrong about medial geniculate body?
A. belongs to metathalamus
B. receives lateral lemniscus
C. receives medial lemniscus
D. relay station of acoustic pathway
E. project to transverse temporal gyri
116. Which nucleus does belong to specific relaying nuclei?
A. nucleus dorsomedialis
B. dorsal lateral nucleus
C. pulvinar
D. intralaminar nucleus
E. ventral posterolateral nucleus
117.Which structure is damaged, the symptoms occur at the opposite side?
A.ventral posterolateral nucleus
B.anterior horn of spinal cord
C.posterior spinocerebellar tract
D.fasciculus gracilis
E.spinal tract of trigeminal nerve
118. Which structure does belong to hypothalamus?
A. inferior colliculus
B. tuber cinereum
C. pineal body
D. red nucleus
E. optic nerve
119. Which sensation pathway doesn't relay in diencephalon?
A. audition
B. vision
C. algesia and thalposis
D. deep sensation
E. olfactory
120. Which structure does belong to dorsal thalamus?
A. pulvinar
B. habenular trigone
C. pineal body
D. mammillary body
E. lateral geniculate body
121. Which specific relaying nucleus is related to motion?
A. medial geniculate body
B. lateral geniculate body
C. ventral anterior nucleus
D. ventral posterior nucleus
E. intralaminar nuclei
122. Which structure doesn't belong to epithalamus?
A. pineal body
B. habenular trigone
C. habenular commissure
D. thalamic medullary stria
E. anterior commissure
123. About the lateral sulcus of the cerebral hemisphere, which statement is right?
A. It separates the precentral gyrus and the postcentral gyrus.
B. It separates the temporal lobe and the frontal lobe.
C. The temporal lobe is below the sulcus.
D. The temporal is behind the sulcus.
E. None of above.
124. The angular gyrus situates in
.
A. frontal lobe
B. temporal lobe
C. parietal lobe
D. occipital lobe
E. insula lobe
125. The lower part of the precentral gyms is controlled of
.
A. the movement of the head the face
B. the movement of the trunk
C. the movement of the lower limb
D. the movement of the upper limb
E. the movement of the body
126. Which is none of the language area?
A. the motor area
B. the motor speech area
C. the writing area
D. auditory speech area
E. visual speech area
127. When the somatic motor area in one side is hurt,
can be found.
A. paralysis in the same side of the body
B. paralysis in contralateral side of the body
C. paralysis in the whole body
D. paralysis only in the lower limb
E. none of above
128. The nucleus on the auditory pathway forming a swelling beneath the pulvinar is
A. medial geniculate body
B. anterior nucleus
C. lateral geniculate body
D. ventral posterior
E. none of above
129. The left ventral lateral nucleus of the thalamus receives input mainly from the
.
.
A. right dentate nucleus
B. left globus pallidus
C. left dentate nucleus
D. right globus pallidus
E. substantia nigra
130. The frontal lobe does not include:
A. the premotor area
B. orbita gyri
C. the uncus
D. the motor area
E. Broca's area
131. The main projection of the general sensory cortex is from the
A. ventral posterior nucleus of thalamus
B. motor and premotor cortical areas
C. medial thalamic nucleus
D. dorsal tier of thalamic nuclei
E. parietal association cortex
132. The auditory area lies in the
.
A. precentral gyrus
B. postcentral gyrus
C. superior frontal gyrus
D. transverse temporal gyrus
E. inferior temporal gyrus
133. The auditory speech area lies in the
.
A. transverse temporal gyrus
B. posterior part of superior temporal gyrus
C. anterior part of middle temporal gyrus
D. posterior part of inferior temporal gyrus
E. anterior part of superior temporal gyrus
134. The writing area locates in the
.
A. angular gyrus
B. supramarginal gyrus
C. posterior part of the middle frontal gyrus
D. anterior part of the superior frontal gyrus
E. posterior part of the inferior frontal gyrus
135. The gustatory area situates in the
.
A. inferior part of the precentral gyrus
B. inferior part of the postcentral gyrus
C. superior parietal lobule
D. middle frontal gyrus
E. inferior frontal gyrus
136. The internal medullary lamina divides the thalamus into
A. 'two
B. five
.
gray masses.
C. three
D. six
E. four
137. The sequential relations of certain hemispherical structures in a medial to lateral
direction, are as followings:
A. internal capsule, putamen, globus pallidus, extreme capsule, claustrum, external capsule
B. internal capsule, globus pallidus, putamen, external capsule, claustrum, extreme capsule,
insula
C. internal capsule, globus pallidus, putamen, extreme capsule, claustrum, insula
D. internal capsule, globus pallidus, putamen, claustrum, external capsule, extreme capsule
E. internal capsule, globus pallidus, external capsule, putamen, claustrum, external capsule,
insula
138. Which of the following combinations of structures include only telencephalic components of
the limbic system of the brain?
A. hippocampus, corpus striatum, cingulated gyms, insula
B. cingulated gyms, amygdaloid nucleus, claustrum
C. cingulated gyms, hippocampus, amygdaloid nucleus, dentate gyrus
D. hippocampus, dentate gyms, mammillary body, amygdaloid nucleus
E. cingulated gyrus, anterior thalamic nucleus, claustrum, hippocampus
139. Ablation of the arm region of the first motor area of the left hemisphere, or destruction of this
region by natural muses, results in
A. voluntary muscle paralysis of the right arm, without flaccidity of the affected muscle
B. impairment of voluntary movements in both arms, with the more pronounced motor deficit
on the right side
C. progressive atrophy of the muscles of the right arm
D. insignificant motor deficits from the start, owing to compensation by intact pyramidal and
extrapyramidal neurons
E. permanent spastic paralysis of most of the musculature of the right arm
140. About the telencephalon, which is incorrect?
A. It is the highest level of the central nervous system.
B. It is composed of cerebral hemisphere.
C. There are many gyri on the surface of the telencephalon.
D. There are some nuclei of cranial nerves in it.
E. There are many fibers in it.
141. Which does not pass the posterior limb of the internal capsule?
A. corticospinal tract
B. corticorubral tract
C. frontopontine tract
D. corticopontine tract
E. central thalamic radiation
142. The internal capsule lies among
.
A. thalamus, caudate nucleus and lenticular nucleus
B. lenticular nucleus, caudate nucleus and thalamus
C. caudate nucleus, lenticular nucleus and thalamus
D. thalamus, lenticular nucleus and caudate nucleus
E. All of the above are wrong.
143. The following statements concerning the caudate nucleus are correct except:
A. It is C-shaped.
B. It lies laterally to the thalamus.
C. It has a head, a body and and a tail.
D. The body lies in the root of the lateral ventricle.
E. The tail terminates anteriorly in the amygdaloid nucleus.
144. Concerning the first motor area:
A. It controls the contraction of groups of muscles.
B. It is concerned with the performance of specific movements on the opposite side of the
body.
C. The movement areas of the body are represented in an inverted form in the precentral
gyrus.
D. There is an extensive area representing the movements of the thumb and fingers.
E. All of the above are correct.
145. The insula of the cerebrum is located in which area?
A. medial to the corpus stratum
B. in the longitudinal fissure
C. in the parietal lobe
D. deep to the lateral fissure between the parietal and the temporal lobes
E. between the temporal and occipital lobes
146. Lesions of the posterior part of the parietal lobe of the right hemisphere may result in
.
A. paralysis of the left side of the face
B. anesthesia of the left arm
C. receptive aphasia
D. paralysis of the right leg
E. inability to recognize objects in the left visual half-fields
147. The olfactory association cortex includes the
.
A. parahippocampal gyrus
B. superior temporal gyrus
C. hypothalamus
D. paracentral gyrus
E. cuneus
148. The corticospinal tracts originate from
.
A. piriform cells
B. pyramidal cells
C. horizontal cells
D. stellate cells
E. Golgi type cells
149. The cerebral cortex is an essential component of the pathway included in
.
A. pupillary light reflex
B. consensual light reflex
C. corneal reflex
D. Gag reflex
E. accommodation reflex
150. Stimulation of the middle frontal lobe of the cerebral cortex would most likely produce
.
A. contraction of the muscles of the contralateral leg
B. visual agnosia
C. auditory hallucination
D. conjugate eye movements
E. fasciculations in the muscles of the ipsilateral hand
151. Which of the following regions of the body has the largest representation in Brodmarm' s area 4?
A. trunk muscles
B. muscles of forearm
C. muscles of mastication
D. muscles of the thumb
E. muscles of the foot
152. Representation of the body parts (somatotopic representation) is found in the
.
A. postcentral gyrus
B. angular gyrus
C. primary visual area
D. Wernicke's area
E. hippocampal gyrus
153. The following statements concerning the position of the basal nuclei are correct except:
A. The lentiform nucleus is related medially to the internal capsule.
B. The putamen lies medial to the globus pallidus.
C. The head of the caudate nucleus lies medial to the internal capsule.
D. The head of the caudate nucleus is connected to the putamen.
E. The amygdaloid nucleus is situated in the temporal lobe of the cerebral hemisphere.
154. The following statements concerning the basal nuclei are correct except:
A. The insula forms part of the basal nuclei.
B. The corpus striatum is made up of the caudate nucleus and the lentiform nucleus.
C. The pulvinar of the thalamus is not part of the basal nuclei.
D. The neostriatum is formed by the caudate nucleus and the putamen.
E. The basal nuclei are formed of masses of gray matter buried deep in the white matter of the
cerebral hemispheres.
A2
1. The patient had found some things when he waked up in the morning: disappeared left
nasolabial groove, the mouth deviates rightward, the frontal wrinkles exist; the tongue deviated
leftward when it was extended; the limbs were normal. What structure is injured by the lesion?
A. left facial nucleus
B. left hypoglossal nucleus
C. right corticonuclear tract
D. right corticospinal tract
E. left motor nucleus of trigeminal nerve
2. After stroke a patient has been found suddenly: the right eye is deviated rightward and dilated
pupil, the tongue is deviated leftward, mimetic paralysis below the eye on the left side of the
face, and the left limbs are rigidity paralysis. The question is what structure isn't included in the
injured structures possible?
A. right corticonuclear tract
B. right corticospinal tract
C. right cerebral peduncle
D. right oculomotor nerve
E. right trochlear nerve
3. One body was ill. Clinical examinations showed: the left limbs are anesthesia to passive
movement, but the pain and thermal sensibility exist. No additional sighs and symptoms.
What structure is involved by the lesion?
A. right gracile and cuneate fasciculi in spinal cord
B. left gracile and cuneate fasciculi in middle segments of spinal cord
C. left medial lemniscus in brainstem
D. right medial lemniscus in brainstem
E. right gracile nucleus
4. A patient had suffered from hemorrhage of brainstem. Physical examinations revealed: right
mimetic paralysis, the mouth deviated leftward, frontal wrinkles exist; the tongue deviated
rightward when it is extended; difficult to active movement of right limbs, tendon hyperreflexia.
Where is damaged most possible?
A. crus cerebri of midbrain
B. ventral part of pons
C. pyramid of medulla oblongata
D. ventral part of medulla oblongata
E. dorsal part of pons
5. Male, 63 years old, suddenly coma two days ago, recovering from coma two days later, but the
limbs on the right side are paralysis, the sensory for the whole body is normal. According to the
case above, what this patient may be suffering from?
A. Posterior limb of internal capsule on the left side is hurt.
B. Precentral gyrus on the left side is hurt.
C. Posterior limb of internal capsule on the right side is hurt.
D. Pyramidal tract on the on right side is hurt.
E. Corticonuclear tract on the left side is hurt.
6. A 71-year-old man was seen by his physician because of tremor of the left arm. The patient also
noticed that the muscles of his limbs sometimes felt stiff. Examination revealed that the patient
rarely smiled and spoke with a weak voice. At rest, the fingers of the left hand alternately
contracted and relaxed and there was a fine tremor of the wrist and elbow. There was no sensory
or motor loss. A diagnosis of Parkinson' s disease was made. The following statements
concerning this patient are correct except:
A. The stiffness of the muscles of the limbs, the unsmiling face, and the weak voice are due to
loss of influence of the basal nuclei on the motor areas in the brainstem.
B. The absence of sensory and motor losses is due to the fact that the ascending sensory
pathways and the main descending motor tracts are unaffected by this disease.
C. Neuronal degeneration is probably present in the substantia nigra and to a lesser extent in
the globus pallidus, putamen, and caudate nucleus.
D. The release of dopamine within the corpus striatum is reduced.
E. The dopamine receptors in the postsynaptic neurous in the striatum are unchanged in
Parkinson's disease.
7. A 64-yesr-old man was examined by a doctor for a suspected brain tumor, with the patient's
eyes closed, a spoon was placed in her right hand and she was asked to recognize the object.
After moving the spoon around in her hand, she was unable to recognize it. However, on
opening her eyes the patient was embarrassed and immediately recognized what was in her
hand. This patient was demonstrating astereognosis. In this condition the following facts
could be correct except:
A. This patient has a lesion involving the left parietal lobe.
B. The tumor involves the left superior parietal lobule.
C. This area of the cerebral cortex is known as the somesthetic association area.
D. It is unnecessary for the patient to move the spoon around in her right hand in order to
recognize the object.
E. The association area is where the sensations of touch, pressure, and proprioception are
integrated.
8. A 71-year-old man was walking along the street when he suddenly collapsed. On admission to
the Emergency Department of the local hospital he was found to have paralysis on the right side
of his body, mainly involving the right leg. There was also some sensory loss of the right foot
and ankle. The sudden onset of herniplegia and hemianesthesia made the diagnosis of a
cerebrovascular accident most probable. The following considerations in this patient are correct
except:
A. The lesion may involve the left cerebral hemisphere.
B. The right side hemiplegia most probably involves the left precentral gyrus.
C. Since the signs of the paralysis are limited to the right leg and foot, the vascular defect
probably involves the upper part of the precentral gyrus and the paracentral lobule.
D. The loss of cutaneous sensation on the right foot and ankle may be due to alesion of the left
postcentral gyrus and the paracentral lobule.
E. The right middle cerebral artery or one of its branches may be blocked by a thrombus or an
embolus.
9. A 47-year-old man was diagnosed as having a rapidly growing intracranial tumor. One of the
clinical findings was paralysis and sensory loss of the right foot and ankle. In this condition
which following fact could be correct?
A. The location of the intracranial tumor is near the paracentral lobule on the left side.
B. The location of the intracranial tumor is near the paracentral lobule on the right side.
C. The location of the intracranial tumor is near lower part of the precentral gyrus and
postcentral gyrus on the left side.
D. The location of the intracranial tumor is near lower part of the precentral gyrus and
postcentral gyrus on the right side.
E. The location of the intracranial tumor is near supramarginal gyrus on the left side.
10. A 15-year-old boy was seen by a neurologist. Examination also revealed writing loss. The
sensory and motor for the whole body are normal. An MRI revealed a tumor in brain. This
patient may be suffering from:
A. Postcentral gyrus on the right side is hurt.
B. Precentral gyrus on the right side is hurt.
C. Visual area is hurt.
D. Writing area is hurt.
E. Visual speech area is hurt.
11. A 35-year-old man, two days ago he suddenly can not read the news paper, he don't
understand what the paper meanings, an examination is given, no visual block revealed, what
may be the matter with the patient?
A. Visual area is hurt.
B. Auditory area is hurt.
C. Writing area is hurt.
D. Visual speech area is hurt.
E. Auditory speech area is hurt.
12. A 42-year-old man was involved in a automobile accident resulting in a fracture of skull. 2
clays later he recovered consciousness, the patient was found to have some sensory loss on
the left side of the body, the motor for the whole body is normal. In this condition which
following fact could be correct?
A. The location of the fracture of skull is near postcentral gyrus on the right side.
B. The location of the fracture of skull is near preeentral gyrus on the right side.
C. The location of the fracture of skull is near inferior frontal gyrus on the right side.
D. The location of the fracture of skull is near superior temtx)ral gyms on the right side.
E. The location of the fracture of skull is near supramarginal gyrus on the right side.
13. A 48-year-old man was examined by a neurologist for his face paralyzed. Examination
revealed that his left angle of his mouth was sag, saliva tended to dribble from the left corner
of his mouth and had difficulty exposing his teeth fully on the left side. In this condition the
following facts could be correct except:
A. Left facial nerve is hurt.
B. Nucleus of facial nerve on the left side is hurt.
C. Genu of internal capsule on the right side is hurt.
D. Genu of internal capsule on the left side is hurt.
E. Precentral gyrus on the right side is hurt.
B1
A. superior salivatory nucleus
B. inferior salivatory nucleus
C. dorsal nucleus of vagus nerve
D. accessory nucleus of oculomotor nerve
E. nucleus ambiguus
1. Which nucleus is related to the secretion of lacrimal gland?
2. Which nucleus is related to the secretion of parotid gland?
3. Which nucleus supplies the thyroid gland?
A. motor nucleus of trigeminal nerve
B. nucleus of abducent nerve
C. solitary nucleus
D. nucleus of cochlear nerve
E. sensory nucleus of trigeminal nerve
4. Which nucleus belongs to the special visceral motor nucleus?
5. Which nucleus belongs to the general visceral sensory nucleus?
6. Which nucleus belongs to the special somatic sensory nucleus?
A. lateral lemniscus
B. medial lemniscus
C. trigeminal lemniscus
D. corticonuclear tract
E. spinothalamic lenmiscus
7. Which tract conducts the deep sensation of trunk and limbs on contralateral side of the body?
8. Which tract conducts the pain and thermal sensations of the headand face?
9. Which tract conducts the pain and thermal sensations of the contralateral side of trunk and
limbs?
A. corticonuclear tract
B. lateral corticospinal tract
C. medial longitudinal fasciculus
D. anterior corticospinal tract
E. rubrospinal tract
10. Which tract innervates the movement of head and face?
11. Which tract innervates the movement of limbs?
12. Which tract innervates the voluntary movement of trunk?
A. inferior olivary nucleus
B. substantia nigra
C. nucleus of inferior colliculus
D. nucleus gracilis
E. pontine nucleus
13. Which nucleus situated in pons gives off the middle cerebellar peduncle?
14. Which nucleus situated in medulla oblongata can relay the proprioception of the trunk and
limbs?
15. Which nucleus situated in midbrain can relay the auditory sense?
A. solitary nucleus
B. nucleus ambiguous
C. hypoglossal nucleus
D. accessory nucleus of oculomotor nerve
E. cochlear nucleus
16. When the nucleus is injured, it can result in the dysacusia of one ear.
17. When the nucleus is injured, it can result in the paralysis of half of the tongue and the
deviation when it is extended.
18. When the nucleus is injured, it can result in the platycoria of one eye.
19. When the nucleus is injured, it can result in the dysgeusia possible.
A. vestibular area
B. acoustic tubercle
C. facial colliculus
D. cerebral peduncle
E. pyramid
20. The cochlear nucleus is deep to
.
21. The abducent nucleus is deep to
.
22. The pyramidal tract is deep to
.
A. spinal cord
B. medulla oblongata
C. pons
D. midbrain
E. internal capsule
23. Which structure is crushed by hippocampal hernia first?
24. Which structure is crushed by hernia of cerebellar tonsil first?
A. precentral gyrus
B. postcentral gyrus
C. lingual gyrus
D. transverse temporal gyrus
E. inferior frontal gyrus
25. primary somatic sensory area
26. primary somatic motor area
27. auditory area
A. motor speech area
B. gustatory area
C. auditory speech area
D. writing area
E. visual speech area
28. angular gyrus
29. middle frontal gyrus
A. cingulate gyrus
B. precentral gyrus
C. cuneus
D. dentate gyrus
E. paracentral lobule
30. archicortex
31. paleocortex
A. cingulum
B. corpus callosum
C. anterior commissure
D. fornix
E. internal capsule
32. projection fibers
33. association fibers
True or False Questions
1. The spinal cord extends from the foramen magnum of the occipital bone to the inferior margin
of the L1 level of the vertebral column in adults.
(
)
2. The length of the adult spinal cord ranges from 32 to 35 cm.
(
)
3. The cervical enlargement extends from the fourth cervical to the first thoracic segments.
(
)
4. The lumbosacral enlargement extends from the second lumbar to the first sacral segments.
(
)
5. The conus medullaris ends at the inferior margin of the L1 level of the vertebral column in
adults.
(
)
6. The filum terminale is long about 20 cm and consists mostly of dura mater.
(
)
7. The spinal cord is divided into 31 segments: 8 cervical segments, 12 thoracic segments, 5
lumbar segments, 5 sacral segments and 1 coccygeal segment.
(
)
8. The spinal segment refers to a region of the spinal cord from which two pair of spinal nerves
arises.
(
)
9. The C1-4 segments of the spinal cord match the C1-4 vertebral bodies.
(
)
10. The T5-8 segments of the spinal cord match the T6-9 vertebral bodies.
(
)
11. The T10-12 vertebral bodies match the L1-5 segments of the spinal cord.
(
)
12. The T10-12 vertebral bodies match the S1-Co1 segments of the spinal cord.
(
)
13. Clinically, cerebrospinal fluid is removed from the subarachnoid space in the inferior lumbar
region of the spinal cord. The procedure is normally performed between the third and fourth
or fourth and fifth lumbar vertebrae.
(
)
14. The lateral horn of the spinal cord is a prominent lateral triangular projection in the whole
thoracic and lumbar regions.
(
)
15. Within spinal segments S2-S4, there are sacral parasympathetic neurons within the lateral
column.
(
)
16. There is no reticular formation in the spinal cord.
(
)
17. Lamina Ⅰ contains the posteromarginal nucleus.
(
)
18. Substantia gelatinosa is situated in Lamina Ⅲ.
(
)
19. Lamina Ⅶ is a large zone that contains the cells of the thoracic nucleus.
(
)
20. Lamina Ⅳ and Ⅶ contain the medial motor neuron column and the lateral motor neuron
column.
(
)
21. The brain stem is consists of thalamus, midbrain, pons and medulla obl0ngata from rostral to
caudal.
(
)
22. The pyramids are consisted of the bundles of the nerve fibers of corticospinal tract mainly,
which arises from the precentral gyri of the cerebral cortex.
(
)
23. The middle cerebellar peduncle (brachium pontis) is formed by the fibers arising from the
ipsilateral pontine nucleus.
(
)
24. The General somatic motor nuclei include paired nuclei of oculomotor, trochlear, abducent and
hypoglossal nerves, motor nucleus of trigeminal nerve, nucleus of facial nerve, nucleus
ambiguus and nucleus of accessory nerve.
(
)
25. Deep to the vestibular area of rhomboid fossa, there are vestibular nuclei and cochlear nuclei.
(
)
26. The fourth ventricle is situated anterior to the cerebellum and posterior to the pons and the
superior part of medulla oblongata.
(
)
27. The crus cerebri contains the descending fibers of the corticospinal and corticonuclear tracts.
(
)
28. The nucleus of facial nerve lies deep to the facial colliculus.
(
)
29. The nucleus ambiguus sends the efferent fibers to join the glossopharyngeal, vagus and
accessory nerves respectively.
(
)
30. The general visceral motor nuclei in brainstem are all parasympathetic nuclei giving off the
parasympathetic preganglionic fibers to innervate the smooth muscles, glands and cardiac
muscle in the body.
(
)
31. The solitary nucleus only relays the general sensory stimuli from the viscera by the
glossopharyngeal and vagus nerves.
(
)
32. The gracile and cuneate nuclei give off the fibers to form ipsilateral medial lemniscus.
(
)
33. The nucleus of inferior colliculus is an important relay station of the auditory pathway.
(
)
34. The efferent fibers from the magnocellular part of red nucleus descend to form the ipsilateral
tectospinal tract.
(
)
35. The substantia nigra plays a key role in motor control. Degeneration of the substantia nigra can
produce the movement disorder, such as Parkinson's disease.
(
)
36. The medial lemniscus is the important tract for conducting the proprioceptive and fine touch
sensations of the contralateral trunk and limbs.
(
)
37.The spinothalamic lemniscus transfers the pain, temperature and rough touch-pressure
sensation of the ipsilateral trunk and limbs.
(
)
38. The fibers of the corticonuclear tract are terminated to all of the bilateral general somatic
motor nuclei and special visceral motor nuclei.
(
)
39. The ascending reticular activating system and ascending inhibitory system of the reticular
formation can regulate the conscious state of cerebral cortex.
(
)
40. In lower end of pyramid all of the fibers cross the midplane in pyramidal decussation and
continuously descend to form the lateral corticospinal tract.
(
)
41. Lesion of left cerebellar hemisphere will result in subconsciousness proprioceptive sense loss
at right side of body.
(
)
42. The efferents of cerebrocerebellum, after relayed in the dentate nucleus, end in ipsilateral red
nucleus and ventrolateral nuclei of thalamus.
(
)
43. The cerebellar cortex consists of three layers from the surface to the white matter: the
molecular layer, the piriform cell layer and the granular layer.
(
)
44. Cerebellar nuclei include fastigial nucleus, globose nucleus, emboliform nucleus and dentate
nucleus.
(
)
45. Archicerebellum lesion can result in dysequilibrium and standing unsteadily.
(
)
46. Paleocerebellum injury can result in ataxia, nystagmus, intention tremor and
hypomyotonia.
(
)
47. In the inferior surface of the cerebellum, the posterolateral fissure separates the anterior lobe
and the flocculonodular lobe.
(
)
48. The fastigial nucleus belongs to arehicerebellum, the globose and emboliform nuclei belong to
paleocerebellum, the dentate nucleus belongs to neocerebellum.
(
)
49. The inferior cerebellar peduncle, also called the restiform body, connects with the medulla
oblongata, contains partly afferents and efferents of cerebellum.
(
)
50. The superior cerebellar peduncle, also called brachium conjunctivum, connects between
cerebellum and midbrain.
(
)
51. Supraoptic nucleus, paraventricular nucleus and infundibular nucleus are related to
neuroendocrine.
(
)
52. The ventral posteromedial nucleus receives fibers of medial lemniscus and spinal lemniscus.
(
)
53. Ventral posteromedial nucleus, ventral posterolateral nucleus, lateral geniculate body and
medial geniculate body are specific relaying nuclei.
(
)
54. The ventral posteromedial nucleus receives trigeminal lemniscus; the ventral posterolateral
nucleus receives the fibres of the medial lemniscus and spinal lemniscus.
(
)
55. Epithalamus includes the pineal body, habenular trigone, habenular commissure, thalamic
medullary stria and posterior commissure.
(
)
56. The fornix is the largest afferents of the hypothalamus.
(
)
57. The subthalamus includes the subthalamic nucleus and the upper ends of the red nucleus and
substantia nigra.
(
)
58. Hypothalamic sulcus separates the dorsal thalamus dorsally and the hypothalamus inferiorly.
(
)
59. The ventral posteromedial nucleus, which relays sensory information of the head and face,
composes the central thalamic radiations and further projects into the superior part of
postcentral gyrus.
(
)
60. Vasopressin and oxytocin are secreted by the neurosecretory neurons within the supraoptic
nucleus and paraventricular nucleus, then transmitted to the anterior lobe of hypophysis.
(
)
61. The central nervous system includes brain and spinal cord.
(
)
62. The cerebral cortex is the highest level of the central nervous system.
(
)
63. Most areas of the cerebral cortex possess five layers.
(
)
64. The somatic motor area lies in the preeentral gyrus and posterior part of the paracentral lobule.
(
)
65. The somatic sensory area locates in the postcentral gyrus and posterior part of the paracentral
lobule.
(
)
66. The muscles of the trunk and limbs of the body in one side are controlled by the contralateral
side of the somatic motor area.
(
)
67. The gustatory area locates in the inferior part of the postcentral gyrus.
(
)
68. The motor speech area is in the posterior part of the middle frontal gyrus.
(
)
69. When the motor speech area hurt, the patient can say nothing.
(
)
70. The auditory speech area is in the posterior part of the superior temporal gyms.
(
)
71. When the auditory speech area is hurt, the patient can hear every thing.
(
)
72. The writing area locates in the anterior part of the middle frontal gyrus.
(
)
73. The visual speech area lies in the angular gyms.
(
)
74. When the visual speech area is hurt, the patient can see nothing.
(
)
75. The medial genicular body is concerned with seeing.
(
)
76. The lateral genicular body is concerned with hearing.
(
)
77. The frontopontine tract passes through the genu of the internal capsule.
(
)
78. The anterior thalamic radiation passes through the anterior limb of the internal capsule.
(
)
79. The corticospinal tract passes through the posterior limb of the internal capsule.
(
)
80. A lesion of internal capsule in one side can cause paralysis in contralateral side of the body.
( )
81. The supramarginal gyrus is the portion of the inferior parietal lobule that arches above the end
of the superior temporal sulcus.
(
)
82. The lingual gyrus lies between the calcarine and parietooccipital sulcus.
(
)
83. The medial occipitotemporal gyrus lies between the hippocampal sulcus and occipitotemporal
sulcus.
(
)
84. The first somatic sensory area is located in the precentral gyrus (Broadmann areas 3, 1, and 2).
(
)
85. The auditory speech area is located in the transverse temporal gyrus.
(
)
86. Anatomically, the basal ganglia include the caudate nucleus and the lenticular nucleus, the
claustrum, and the amygdaloid body.
(
)
87. The lateral ventricle communicates with the cavity of the fourth ventricle through the
interventricular foramen.
(
)
88. The neocortex contains three to six well-defined layers of cells.
(
)
89. The association fibers connect the various portions of a cerebral hemisphere and permit the
cortex to function as a coordinated whole.
(
)
90. The hippocampal formation includes the dentate gyrus, the hippocampus and parahippocampal
gyrus.
(
)
Explanation of Terms
1. conus medullaris
2. filum terminale
3. cervical enlargement
4. lumbosacral enlargement
5. anterior horn
6. posterior horn
7. lateral horn
8. anterior median fissure
9. posterior median sulcus
10. posteromarginal nucleus
11. substantia gelatinosa
12. nucleus proprius
13. nucleus thoracicus
14. medial motor neuron column
15. lateral motor neuron column
16. intermediolateral nucleus
17. anterior funiculus
18. posterior funiculus
19. lateral funiculus
20. anterior white commissure
21. decussation of pyramid
22. middle cerebellar peduncle
23. corpus quadrigemina
24. rhomboid fossa
25. genu of facial nerve
26. trapezoid body
27. medial lemniscus
28. pretectal area
29. substantia nigra
30. nucleus of hypoglossal nerve
31. tonsil of cerebellum
32. archicerebellum
33. paleocerebellum
34. neocerebellum
35. cerebellomedullary cistern
36.diencephalon
37. third ventricle
38. hypothalamic sulcus
39. medial geniculate body
40. pineal body
41. internal capsule
42. cerebral cortex
43. basal ganglia
44. projection fibers
45. septal area
46. fornix
47. corpus callosum
48. Wernicke's area
49. association cortex
50. paleocortex
Answer the Following Questions
1. Describe the location, origin, termination, and function of the fasciculus gracilis and
the fasciculus cuneatus.
2. Describe the location, origin, termination, and function of the posterior spinocerebellar tract.
3. Describe the location, origin, termination, and function of the anterior spinocerebellar tract.
4. Describe the location, origin, termination, and function of the lateral spinothalamic tract.
5. Describe the location, origin, termination, and function of the anterior spinothalamic tract.
6. Explain the location, origin, termination, and function of the lateral corticospinal tract.
7. Explain the location, origin, termination, and function of the anterior corticospinal tract.
8. Explain the location, origin, termination, and function of the rubrospinal tract.
9. Explain the location, origin, termination, and function of the tectospinal tract.
10. Explain the location, origin, termination, and function of the medial longitudinal fasciculus.
11. Describe the location and external features of the spinal cord.
12. Describe the internal structure of the spinal cord.
13. List the location, origin, termination, and function of the principal ascending tracts of the
spinal cord.
14. List the location, origin, termination, and function of the principal descending tracts of the
spinal cord.
15. Explain the stretch reflexes.
16. The origin, termination and function of medial lemniscus?
17. What are the parasympathetic nuclei which located in brainstem and their relations to the
cranial nerves?
18. Which nuclei are related to the movement of skeletal muscles in head and face in brainstem?
19. What is the visceral sensory nucleus in the brainstem? Which cranial nerves are related to it?
20. Which nuclei give off the fibers to extraocular muscles in brainstem?
21. Which important structures can be seen on the transverse section at middle part of olive?
22. Which important structures can be seen on the horizontal section at middle part of the pons?
23. Which important structures can be seen on the horizontal section at inferior colliculus of the
middle brain?
24. What kind of fibers are there in the facial nerve? Which cranial nuclei are related to them
respectively?
25. A patient may present cough, hoarseness and dysphagia when the medulla oblongata is lesion.
Why?
26. What is the main function of the cerebellum?
27. Please answer the functional divisions of the cerebellum?
28. What are the typical signs after the cerebellum injury?
29. Please describe the fiber connections and functions of vestibulocerebellum?
30. Please describe the fiber connections and functions of spinocerebcllum?
31. Please describe the fiber connections and functions of cerebrocerebellum?
32. Which kinds of nuclear groups are classified about thalamic nuclei?
33. What is the function of specific relaying nuclei of the thalamic nuclei?
34. Please answer the afferent connections of the hypothalamus.
35. What are composed of the cerebral hemispheres?
36. How many language areas are there in the cerebral cortex? What are the names of these
language areas?
37. What is the striatum and what is the function of it?
38. What types are comprised the cortex of the cerebrum?
39. What is the limbic system and what is its main function?
40. What layers of cells are composed of the archicortex?
41. What parts can be divided into on the lateral ventricle?
42. The location and the divisions of the internal capsule? What important fibers pass through
each division of the internal capsule?
43. What is the composition and the function of the basal forebrain?
44. Please describe the location and the fibrous connection of the the hippocampus?
45. Please describe the cytoarchitecture of the neocortex.
46. How many parts are composed of the white matter of the cerebral hemisphere? What are they?
ANSWERS
Multiple Choice Questions
A1
1. A 2. C 3. B 4. D
5. D
6. E
7. C
8. A
9. D 10. E 11. C
12. B 13. E 14. C 15. C 16. A 17. B 18. C 19. D 20. E 21. A 22. B
23. D
24. C
25. A
26. E
27. B
28. C
29. B
30. D
31. B
32. D 33. B 34. A 35. E 36. A 37. B 38. C 39. D 40. E
41. A 42. A
43. E
44. D
45. E 46. E
47. B
48. D
49. C
50. D 51. E
52. A
53. C
54. E 55. B
56. B 57. D 58. C
59. A
60. E 61.
A
62. C
63. B
64. B 65. D
66. A 67. A
68. C 69. B 70. D 71. D 72. C 73. E 74. A 75. B 76. D 77. E
78. B 79. C 80. A 81. B 82. E 83. D 84. B 85. E 86. D 87. A
88. E
89. B
90. C
91. A
92. D 93. B
94. C
95. E
96. D
97. D
98. B 99. C
100. B
101. A 102. E
103. A 104. E 105. C
106. C
107. E
108. B
109. A
110. B
111. A
112. A 113. E
114. B
115. C
116. E
117. A
118. B
119. E
120. A
121. C
122. E 123. C
124. C
125. A
126. A
127. B
128. A
129. A
130. C 131. A
132. D
133. B 134. C 135. B 136. B 137. B
138. C
139. A 140. D 141. C 142. A 143. D 144. E 145. D 146. E 147. A
148. B 149. E 150. D 151. D
152. A 153. B 154. A
A2
1. C 2. E
3. D
4. A
5. B
6. E 7. D
8. E
9. A
10. D 11. D
12. A 13. D
B1
1. A
2. B
3. C
4. A
5. C
6. D
7. B
8. C
9. E
10. A 11. B
12. D 13. E 14. D 15. C 16. E
17. C 18. D 19. A 20. B
21. C 22. E
23. D 24. B
25. B
26. A
27. D
28. E 29. D
30. D
31. A
32. E 33. A
True or False Questions
1. T 2. F 3. T 4. F 5. T 6. F 7. T 8. F 9. T 10. F
11. T 12. F 13. T 14. F 15. T 16. F 17. T 18. F 19. T 20. F
21. F 22. T 23. F 24. F 25. F 26. T 27. F 28. F 29. T 30. T
31. F 32. F 33. T 34. F 35. T 36. T 37. F 38. F 39. T 40. F
41. F 42. F 43. F 44. T .45. T 46. F 47. F 48. T 49. T 50. T
51. T 52. r 53. T 54. T 55. T 56. T 57. T 58. T 59. T 60. T
61. T 62. T 63. F 64. F 65. T 66. T 67. T 68. F 69. F 70. T
71. T 72. F 73. T 74. F 75. F 76. F 77. F 78. T 79. T 80. T
81. F 82. F 83. F 84. F 85. F 86. T 87. F 88. F 89. T 90. F
Explanation of Terms
1. Below the lumbar enlargement, the spinal cord tapes to form a conical portion known as the
conus medullaris.
2. Filum terminale is a long about 20cm nonnervous fibrous tissue and consists mostly of pia
mater. It extends inferior to attach to the coccyx.
3. Cervical enlargement extends from the fourth cervical to the first thoracic segments. Nerves that
supply the upper extremities arise from it.
4. Lumbosacral enlargement extends from the second lumbar to the third sacral segments. Nerves
that supply the lower extremities arise from it.
5. Anterior horn is an enlarged protrusion in the front of gray matter of the spinal cord on each
side. It contains the cells of origin of the fibers of the anterior roots.
6. Posterior horn is the narrow protrusion nearly reaching to the posterolateral sulcus. The
posterior horn can be divided into a head, a neck and a relatively broad base. It represents the
sensory part of the gray matter.
7. Lateral horn is a prominent lateral triangular projection of gray matter between the anterior and
posterior horn or column in the thoracic and upper lumbar regions. It contains preganglionic
cells for the autonomic nervous system.
8. Anterior median fissure is a deep fissure on the anterior surface of the spinal cord.
9. Posterior median sulcus is a shallow sulcus on the posterior surface of the spinal cord.
10. Posteromarginal nucleus is situated in lamina Ⅰ and receives the afferent fibers of posterior
roots and sends axons to the contralateral spinothalamic tract.
11. Substantia gelatinosa is located in lamina Ⅱ and is made up of small neurons and forms a
well delimited band around the apex of the posterior horn.
12. Nucleus proprius is situated in laminas Ⅲ and Ⅳ. Their main input is from fibers that
convey position and light touch sense.
13. Nucleus thoracicus is located in lamina Ⅶ and contains cells that give rise to the posterior
spinocerebellar tract.
14. Medial motor neuron column is located in laminas Ⅷ and Ⅸ. It contains the lower motor
neurons that innervate axial musculature.
15. Lateral motor neuron column is situated in laminas Ⅷ and Ⅸ. It contains lower motor
neurons for the distal muscles of the arm and leg.
16. Intermediolateral nucleus is located in lamina Ⅶ in thoracic and upper lumbar regions (T1
through L3). Preganglionic sympathetic fibers project from cells in this nucleus, via the anterior
roots and white rami communicates, to sympathetic ganglia. The intermediomedial nucleus
extends virtually the entire length of the spinal cord and lies in the most medial part of the
lamina. It may receive visceral afferent fibers and serve as an intermediary relay in transmission
of impulses to visceral motor neurons.
17. Anterior funiculus is a portion of the white matter, which lies between the anterolateral sulcus
and the anterior median fissure.
18. Posterior funiculus is a portion of the white matter, which lies between the posterior median
sulcus and the posterolateral sulcus.
19. Lateral funiculus is a portion of the white matter, which lies between the posterolateral
sulcus and the anterolateral sulcus.
20. Anterior white commissure is a bundle of transverse fibers, anterior to the gray commissure.
21. Near the lower end of medulla oblongata, most descending fibers of the pyramidal tract cross
over the midplane; these decussating fibers are called the decussation of pyramid.
22. The transverse fibers of the pons which across the median plane of it and converge to form the
lateral part of the pons, is called the middle cerebellar peduncle.
23. On the dorsal aspect of the midbrain four rounded eminences are called the corpora
quadrigemina, which include a pair of superior colliculus and a pair of inferior colliculus.
24. The rhomboid fossa is formed by the dorsal surface of both the pons and the upper part of
medulla oblongata. It is the floor of fourth ventricle and rhomboidal in shape. The superolateral
borders of the fossa are the superior cerebellar peduncles; the inferolateral borders are the
gracile tubercle, cuneate tubercle and inferior cerebellar peduncles from medial to lateral.
25. The nucleus of facial nerve gives off efferent fibers which firstly proceed dorsomedially and
round the nucleus of abducent nerve to form the genu of facial nerve.
26. Most of the efferent fibers of cochlear nuclei cross the median plane of lower part of pons to
form the trapezoid body.
27. The medial lemniscus is formed by the secondary-order sensory fibers arising from the
contralateral gracile and cuneate nuclei. The fibers firstly pass forwards, laterally round the
central gray matter and then decussate in the midplane with the corresponding fibers of the
opposite side. After that the fibers turn upwards to form the medial lemniscus conducting the
deep sense of the body.
28. The pretectal area is at the junction of midbrain and diencephalon, just rostral to the superior
colliculus. The pretectal area receives the fibers from the ipsilateral optic tract, visual cortex and
the lateral geniculate body. The efferent fibers project to the bilateral accessory nucleus of
oculomotor nerve.
29. The substantia nigra is situated between the tegmentum and crus cerebri of midbrain, extends
through the whole length of the midbrain and into the caudal diencephalon. In section the
substantia nigra is divided into two parts, a dorsal compact part and a ventral reticular part.
30. The nucleus of hypoglossal nerve is in the hypoglossal triangle. It receives the afferent fibers
from the contralateral corticonuclear tract and gives off efferent fibers to form the hypoglossal
nerve which innervates the ipsilateral muscles of the tongue.
31. It is a prominence of cerebellar hemispheres adjacent to the great occipital foramen. The
cerebellar tonsillar hernia will occur if tonsil of cerebellum is protruded to the great occipital
foramen because of intracranial hypertension. This condition can pose a threat to life since the
respiratory and cardiovascular centers in the medulla could be compressed in the protruded
cerebellar tonsil.
32. The archicerebellum consists of the flocculonodular lobe. It is also known as the
vestibulocerebellum because of connecting with the vestibular nuclei and vestibular nerves. It
regulates muscle movement of trunk, playing an important role in the maintenance of body
equilibrium.
33. The paleocerebellum is composed of the vermis (medial zone) and intermedial zone of the
hemispheres, receives the afferents from the spinal cord, also called the spinocerebellum.
It modulates the muscular tension and motor coordination of extremities.
34. The neocerebellum consists of the lateral zone, connects with the cerebral cortex, also termed
as the cerebrocerebellum. It affects the modulation of initiation, planning and coordination of
limbs fineness movement, including determination of movement strength, direction and
amplitude.
35. It is the largest of the subarachnoid cisterns between the undersurface of the cerebellum and
the dorsal surface of the medulla oblongata. Subarachnoid space stabbing can be carried out
here in clinic.
36. The diencephalon is located between the midbrain and telencephalon, develops from the
forebrain vesicle. It is mostly surrounded by the cerebral hemispheres, left the optic chiasm,
optic tracts, tuber cinereum, infundibulum, hypophysis and mamillary body exposed ventrally.
The diencephalon is subdivided into five parts: dorsal thalamus, epithalamus, hypothalamus,
subthalamus and metathalamus.
37. The third ventricle is a narrow midplane cavity within the diencephalon. It is bordered
superiorly by the choroid plexus of third ventricle, inferiorly by the optic chiasm, gray tubercle,
and mamillary, anteriorly by the lamina terminalis, laterally by the dorsal thalami and
hypothalami. Inferoposteriorly, the third ventricle is communicated with the fourth ventricle via
the cerebral (mesencephalic) aqueduct.
38. It is a small groove and extends from the interventricular foramen to the cerebral aqueduct on
the lateral wall of the third ventricle, separating the dorsal thalamus dorsally and the
hypothalamus inferiorly.
39. The medial geniculate body, belonging to metathalamus, a relay station of the auditory
conducting pathway, receives acoustic fibers from the inferior colliculus and the brachium of
inferior colliculus. It projects fibers, via the acoustic radiation, to the auditory center of
temporal cortex.
40. The pineal body is an endocrine gland that secretes a hormone, melatonin, which plays an
important role in the control of sexual glands and biological clock of the body. After the age of
16, the pineal body is gradually calcified, hence usually taken as an imageological mark
clinically. Pineal body is also belongs to epithalamus.
41. The internal capsule is a broad band of myelinated fibers that separates the lentiform nucleus
from the medial caudate nucleus and thalamus. It has an anterior limb, genu, and a posterior
limb.
42. The cerebral cortex is the highest level of the central nervous system. It is composed of gray
matter and forms complete covering of the cerebral hemisphere. The surface area of the cortex
has been increased by throwing it into convolution, or gyri, separated by fissures or sulci.
43. The basal ganglia refers to masses of gray matter deep within the cerebral hemispheres.
Anatomically, the basal ganglia include the corpus striatum (caudate nucleus and lentiform
nucleus), the claustrum, and the amygdaloid body.
44. The projection fibers connect the cerebral cortex with lower portions of the brain or the spinal
cord. They are composed of afferent and efferent nerve fibers, form a compact band known as
the internal capsule, which is flanked medially by the caudate nucleus and the thalamus and
laterally by the lentiform nucleus.
45. The septal area is an area of gray matter lying above the lamina terminalis near and around the
anterior commissure. It includes paraterminal gyms and paraolfactory area.
46. The fornix is composed of myelinated nerve fibers and constitutes the efferent system of the
hippocampus that passes to the mammillary bodies of the hypothalamus.
47. The corpus callosum comprises the largest bundle of myelinated and nonmyelinated fibers, the
great white commissure that crosses under the longitudinal cerebral fissure and interconnects
the hemisphere& It is the largest of the interhemispheric commissures and is largely responsible
for coordinating the activities of the two cerebral hemispheres.
48. The Wernicke's area is localized in the left "dominant" hemisphere, mainly in the superior
temporal gyms, with extensions around the posterior end of the lateral sulcus into the parietal
lobe. It permits the understanding of the written and spoken language and enables a person to
read a sentence with understanding and loudly.
49. The primary sensory areas and the primary motor areas form only a small part of the total
cortical surface area. The remaining areas are referred to as association cortex. These areas of
the cortex have multiple inputs and outputs and are very much concerned with behavior,
discrimination, and interpretation of sensory.
50. The forms the transition between the archicortex and neocortex. It contains three to six layers
and is found in such regions as the subicular complex, the cingulate gyrus and the insula.
Answer the Following Questions
1. Their receptors of the fasciculus gracilis and the fasciculus cuneatus are situated in the skin,
joints and tendons. The two fasciculi are large proportion of the heavily myelinated fibers of
posterior root that enter the ipsilateral posterior funiculus of the spinal cord. The fibers arising
from the sacral, lumbar and lower eight thoracic segments make up the fasciculus gracilis, while
the fibers arising from the upper four thoracic and cervical segments make up the fasciculus
cuneatus. The fibers of these two tracts are somatotopically organized in their course, i.e., from
the lateral to medial they are in cervical, thoracic, lumbar and sacral segmental order. After
reaching the medulla oblongata, the fasciculus gracilis and the fasciculus cuneatus terminate
upon the nuclei gracilis and nuclei cuneatus respectively. These two tracts conduct the
kinesthetic senses (sense of position and movement) and the discriminating tactile (to recognize
the size, shape and texture).
2. The posterior spinocerebellar tract is situated along the posterolateral periphery of the lateral
funiculus of the spinal cord. It arises from the ipsilateral nucleus thoracicus and ascends through
the spina! cord to the medulla oblongata in which it becomes incorporated in the inferior
cerebellar peduncle. It conveys the subconscious proprioceptive impulses (controls muscular
tension and coordination).
3. The anterior spinocerebellar tract is located along the lateral periphery of the lateral funiculus of
the spinal cord, anterior to the posterior spinocerebellar tract. Its fibers arise from the lateral
part of the laminae Ⅴ-Ⅸ and most of the fibers cross the spinal cord and ascend through the
spinal cord, medulla oblongata and pons, then enter the cerebellum by coursing along the dorsal
surface of the superior cerebellar peduncle. Its function is the same as that of the posterior
spinocerebellar tract.
4. The lateral spinothalamic tract is closely inner to the anterior spinocerebellar tract. The nucleus
proprius receives the thin myelinated fibers associated with pain and thermal senses from the
dorsolateral fasciculus and gives rise to most of the axons that cross in the anterior white
commissure and ascend in the opposite lateral funiculus as lateral spinothalamic tract. This tract
passes through the brain stem and ends directly in the thalamus. Fibers of the lateral
spinothalamic tract are somatotopically organized in their course, i.e. from the lateral to medial
they are in sacral, lumbar, thoracic and cervical segmental order.
5. The anterior spinothalamic tract arises from the nucleus proprius, crossing the anterior white
commissure, and ascending contralaterally anterior to the lateral spinothalamic tract. A small
number of uncrossed fibers may ascend in the ipsilateral anterior spinothalamic tract. The
spinothalamic tract transmits the pain and thermal sensation of trunk and limbs.
6. The lateral corticospinal tract arises from the motor and premotor area of the cerebral cortex and
decussates in the medulla oblongata and descends medially to the posterior spinocerebellar tract
in the spinal cord. The tract extends to the most caudal part of the spinal cord and progressively
diminishes in size as more and more fibers leave to terminate in the motor neurons (lateral
motor neuron column) of the anterior horn of the gray matter. This tract has somatotopical
arrangement from the lateral to medial, that is in sacral, lumbar, thoracic and cervical segmental
order.
7. The anterior corticospinal tract arises from the motor and premotor area of the cerebral cortex
and occupies a strip adjacent to the anterior median fissure and normally extends only to the
upper thoracic spinal segments. Most of these fibers decussate in the anterior white commisure
before they terminate in the motor neurons (medial motor neuron column) of the anterior horn.
8. The rubrospinal tract lies anterior to and partially intermingled with the fibers of the lateral
corticospinal tract. The fibers of this tract arise from the red nucleus of the midbrain, and cross
the midbrain raphe immediately, and then to the spinal level. Their terminals contact cells in the
posterior horn and intermediate zone through which they facilitate flexor motor neurons and
control the muscular tone of the flexor muscle group.
9. The tectospinal tract arises from the neurons in the superior collieulus, and then decussates to
the opposite side and descends through the pons and medulla oblongata to reach the cervical
spinal segments. In the spinal cord, it's in the anterior part of the anterior funiculus near the
anterior median fissure. The fibers of this tract innervate the motor neurons of anterior horn and
excite the muscles of the neck of the opposite side and inhibit the ipsilateral cervical muscles.
10. The medial longitudinal fasciculus is formed by a small compact tract of nerve fibers, situated
close to the medial plane and ventral to the hypoglossal nucleus. It is continuous upward
throughout the pons and the midbrain and downwards in the anterior funiculus of the spinal
cord. This fasciculus is the main pathway that connects the vestibular and cochlear nuclei with
the nuclei controlling the extraocular muscles and the cervical anterior gray matter, principally
the neurons innervating the muscles of the neck. It's chief function is to ensure the coordinate
movement of the eyes and head response to stimulation of the vestibular-cochlear nerve.
11. The spinal cord occupies the upper two-thirds of the adult spinal canal within the vertebral
column. It begins as a continuation of the medulla oblongata, and extends from the foramen
magnum of the occipital bone to the inferior margin of the L1 level of the vertebral column in
adults, and to the one of the L3 level in new-born. The length of the adult spinal cord ranges
from 42 to 45cm.
The spinal cord is a long cylindrical structure that is slightly flattened anteriorly and
posteriorly. The diameter of the spinal cord varies at different levels. When the spinal cord is
viewed externally, two conspicuous enlargements can be seen. The enlargements of the
spinal cord contain increased numbers of motor neurons and provide the origins of the nerves
of the upper and lower extremities. The superior enlargement, the cervical enlargement,
extends from the fourth cervical to the first thoracic segments. Nerves that supply the upper
extremities arise from the cervical enlargement. The inferior enlargement, called the
lumbosacral enlargement, extends from the second lumbar to the third sacral segments. Nerves
that supply the lower extremities arise from the lumbosacral enlargement. Below the
lumbosacral enlargement, the spinal cord tapes to form a conical portion known as the conus
medullaris. Arising from the conus medullaris is the filum terminale, a nonnervous fibrous
tissue of the spinal cord that extends inferior to attach to the coccyx. The filum terminale is
long about 20 cm and consists mostly of pia mater, the innermost of three membranes that
cover the spinal cord and brain. It becomes invested by dura mater, the outermost of three
membranes that cover the spinal cord and brain, at the level of the second sacral vertebra and
attaches to the posterior surface of the coccyx. On the surface of the spinal cord, six
longitudinal grooves can be observed. A deep anterior median fissure and a shallow posterior
median sulcus divide the spinal cord into symmetric right and left halves. There are two pairs
of the shallow anterolateral sulci and postlateral sulci. The ventral roots and the dorsal roots of
the spinal nerves are attached to the spinal cord along the grooves respectively. Besides the
six longitudinal grooves, in the cervical and upper thoracic segments, a pair of smaller, less
definite posterior intermediate sulci are present between the posterior median sulcus and the
postlateral sulci.
The spinal cord is divided into 31 segments, 8 cervical segments, 12 thoracic segments, 5
lumbar segments, 5 sacral segments and 1 coccygeal segment. The spinal segment refers to a
region of the spinal cord from which a pair of spinal nerves arises.
12. The spinal cord consists of both gray and white matter. A cross section of the spinal cord
shows an H-shaped internal mass of gray matter surrounded by white matter. The gray matter
consists primarily of nerve cell bodies, unmyelinated axons and dendrites of association and
motor neurons. The white matter surrounds the gray matter and consists of bundles of
myelinated axons of motor and sensory neurons. In the center of the gray matter, the gray
commissure forms the cross bar of the H connecting the right and left portions. In the center
of the gray commissure is a small space called the central canal, which runs the length of the
spinal cord. It contains cerebrospinal fluid and is enlarged in the conus medullaris to form the
8-10cm long terminal ventricle. The anterior gray commissure and posterior gray commissure
occupy the anterior and posterior halves of the gray commissure. The anterior (ventral)
horn or anterior column is the enlarged protrusion in the front of the gray matter of each side.
It contains the cells of origin of the fibers of the anterior roots, including alpha and gamma
motor neurons. The posterior (dorsal) horn or posterior column is the narrow protrusion
nearly reaching to the posterolateral sulcus. The posterior horn can be divided into a head, a
neck and a relatively broad base. It represents the sensory part of the gray matter. The region
between the anterior horn and posterior horn is the intermediate zone. The lateral horn or
lateral column is a prominent lateral triangular projection of gray matter between the anterior
and posterior horn or column in the thoracic and upper lumbar regions. It contains
preganglionic cells for the autonomic nervous system. Within spinal segments T1-L3,
preganglionic sympathetic neurons within the lateral column give rise to sympathetic axons
that leave the spinal cord within the anterior roots and then travel to the sympathetic ganglia
via the white rami communicantes. Within spinal segments S2-S4, there are sacral
parasympathetic neurons within the lateral column. These neurons give rise to preganglionic
parasympathetic axons that leave the spinal cord within the sacral anterior roots.
The form and quantity of the gray matter vary at different levels of the spinal cord. The
proportion of gray to white matter is greatest in the lumbar and cervical enlargements. In the
cervical region, the posterior column is comparatively narrow and the anterior column is
broad and expansive, especially in the four lower cervical segments. In the thoracic region,
both the posterior and anterior columns are narrow, and there is a lateral column. In the
lumbar region, the posterior and anterior columns are broad and expanded. In the conus
medullaris, the gray matter looks like two oval masses, one in each half of the spinal cord,
connected by a wide gray commissure. The spinal cord has three white funiculi anterior
(ventral), lateral and posterior (dorsal) funiculus around the spinal gray columns. The anterior
funiculus lies between the anterolateral sulcus and the anterior median fissure. The
posterior funiculus lies between the posterior median sulcus and the posterolateral sulcus. In
the cervical and upper thoracic regions, the posterior funiculus is divided into a medial
portion (the fasciculus gracilis) and a lateral portion (the fasciculus cuneatus). The posterior
intermediate sulcus is the boundary between the fasciculus gracilis and fasciculus cuneatus.
The lateral funiculus lies between the posterolateral sulcus and the anterolateral sulcus.
Anterior to the gray commissure is a bundle of transverse fibers, the anterior white
commissure.
The reticular formation is a field of intermingled gray and white matter collectively and is
present in the area between lateral side of the base of the posterior horn and the white matter,
especially obvious in the cervical segments. It is the important integration center for the vital
activity.
13. Ascending tracts of the spinal cord
Name
Location
Origin
Termination
Function
Fasciculus
Posterior Skin, joints,
Nucleus gracilis and nucleus Finetouch
gracilis
& funiculus tendons
cuneatus, Second-order
proprioception, twofasciculus
neurons project to contralateral point discrimination
cuneatus
thalamus (cross in medulla at
lemniscal decussation)
Posterior
Lateral
Muscle spindle,Golgi Cerebellar paleocortex ( via Movement and
spinocerebellar funiculus tendon organs touch ipsilateral inferior cerebellar position mechanisms
tract
and pressure receptors peduncle)
(via nucleus
dorsalis column)
Anterior
Lateral
As above
Cerebellar paleocortex ( via As above
spinocerebellar funiculus
contralateral and ipsilateral
tract
superior cerebellar peduncle)
Lateral
Lateral
Skin
Posterior bom Second-order Sharp
spinothalamic
funiculus
neurons project to contralateral pain, Temperature
tract
thalamus (cross in spinal cord
close to level of entry)
Anterior
Anterior
Skin
As above
Crude touch
spinothalamic funiculus
tract
14. Descending tracts of the spinal cord
Name
Location
Origin
Termination
Function
Lateral
Lateral funiculns
Motor and premotor Anterior horn Free motor function
corticospinal (crosses in medulla at area of cerebral vortex cells(intemeurons (controls distal
tract
pyramidal decnssation)
and lower motor musculature)
neurons)
Modulation of sensory
functions
Anterior
Anterior funiculus
Motor and premotor
Anterior horn Gross and postural
corticospinal
(uncrossed in
area of
cells(interneurons motor function
tract
medulla, but crosses
cerebral vortex
and lower motor (proximal and axial
to opposite side of
neurons)
masculature)
spinal cord)
Rubrospinal
Lateral funiculns
Red nucleus
Anterior horn
Muscle tone
tract
(crosses to opposite
cells (intemeurons) and posture
side of spinal cord)
Vestibulospinal Anterior funiculus
Lateral and medial Anterior horn Postural reflexes
tract
vestibular nucleus
interneurons
Tectospinal Anterior funiculus Superior colliculus
tract
(crosses to opposite
side of spinal cord)
Medial longitudinal Anterior funiculus Vestibular nuclei
Fasciculus
and motor
neurons for
extensors)
Anterior horn
Reflex head turning
interneurons
Cervical
gray matter
Coordination of
head and
eye movements
15. The stretch reflexes are also called tendon reflexes or deep tendon reflexes. The stretch reflex
is based on a monosynaptie reflex arc. Only two neurons are involved, and there is only one
synapse in the pathway. This reflex results in the contraction of a muscle when it is stretched.
Slight stretching of a muscle stimulates receptors in the muscle called neuron musclar
spindles. The spindles monitor changes in the length of the muscle. Once the spindle is
stimulated, an impulse is sent along a sensory neuron to the spinal cord. The sensory neuron
lies in the posterior root of a spinal nerve and synapses with a motor neuron in the anterior
gray horn. The sensory neuron generates an impulse at the synapse that is transmitted along
the motor neuron. The motor neuron lies in the anterior root of the spinal nerve and
terminates in a skeletal muscle. Once the impulse reaches the stretched muscle, it contracts.
Thus the stretch is counteracted by contraction. One such reflex is the knee-jerk, or patellar
reflex. Stretch reflexes provide a feedback mechanism for maintaining appropriate muscle
tone. The stretch reflex depends on specialized sensory receptors (muscle spindles), afferent
nerve fibers (primarily Ia fibers) extending from these receptors via the dorsal roots to the
spinal cord, two types of lower motor neurons (alpha and gamma motor neurons) that
project back to muscle, and specialized inhibitory interneurons (Renshaw cells).
16. The medial lemniscus is formed by the secondary-order sensory fibers arising from the
contralateral gracile and cuneate nuclei. After the decussation of medial lemniscus, the fibers
turn upwards to form the medial lemniscus and ascend through the medulla oblongata, pons
and midbrain, terminate into the ventral posterolateral nucleus of thalamus. The medial
lemniscus is the important tract for conducting the proprioceptive and fine touch sensations
of the contralateral trunk and limbs.
17. There are four pairs of parasympathetic nuclei in brainstem. Accessory nucleus of oculomotor
nerve is also called the Edinger-Westphal nucleus that is situated in the median plane and
dorsomedial to the nucleus of oculomotor nerve. The nucleus gives off the parasympathetic
preganglionic fibers that pass through the oculomotor nerve and are relayed in the ciliary
ganglion; the postganglionic fibers innervate the ciliary muscle and the sphincter pupillae.
Superior salivatory nucleus is in the reticular formation of lower pons and surrounds the
caudal portion of nucleus of facial nerve. The nucleus sends the parasympathetic
preganglionic fibers into the facial nerve to relay in the pterygopalatine ganglion and
submandibular ganglion respectively. The postganglionic fibers regulate the secretion of the
lacrimal gland, submandibular gland and sublingual gland mainly.
Inferior salivatory nucleus is situated in the reticular formation of the medulla, just below the
superior salivatory nucleus. This nucleus sends the parasympathetic preganglionic fibers
into the glossopharyngeal nerve to relay in the otic ganglion; the postganglionic fibers control
the secretion of parotid gland. Dorsal nucleus of vagus nerve is located within the vagus
triangle. It gives off the parasympathetic preganglionic fibers, which are along the branches
of the vagus nerve into the parasympathetic ganglions in the visceral organs of neck, thorax
and most of the abdomen to relay. The postganglionic fibers innervate the actions of those
organs.
18. Motor nucleus of trigeminal nerve is located in the reticular formation of the pons;
ventromedial to the pontine nucleus of the trigeminal nerve. The efferent fibers of the nucleus
run ventrolaterally into the mandibular division of the trigeminal nerve to innervate the
masticatory muscles and the tensor tympani of middle ear etc. Nucleus of facial nerve lies in
the ventrolateral portion of the reticular formation of lower pons. The fibers from upper part
of the nucleus only innervate the expression muscles above the eye; the fibers from lower
part of the nucleus just innervate the expression muscles of the face below the eye.
Nucleus ambiguus is situated within the reticular formation of medulla oblongata. The
nucleus receives the afferent fibers from the bilateral corticonuclear tracts and sends the
efferent fibers that join the glossopharyngeal, vagus and accessory nerves respectively and
supply the muscles of pharynx and larynx.
Accessory nucleus includes two parts: the cranial part is connected with lower part of the
nucleus ambiguus, but the caudal part is called the accessory nucleus which extends
downward as low as the level of the fifth cervical segment of spinal cord. The efferent fibers
of it control the trapezius and sternocleidomastoid mainly.
19. The visceral sensory nucleus in the brainstem is solitary nucleus. This nucleus is composed of
two parts. The minor upper part is considered the gustatory nucleus by some authors, which
relays the specialized taste stimuli from the tongue and epiglottis by the facial,
glossopharyngeal and vagus nerves. The major lower part relays the general sensory stimuli
from the viscera by the glossopharyngeal and vagus nerves.
20. Nucleus of oculomotor nerve is situated in the ventral portion of the central gray matter at the
level of the superior colliculi of midbrain. Its efferent fibers form the oculomotor nerve to
innervate ipsilateral most of the extraocular muscles, except the lateral rectus and the superior
obliquus, and the contralateral levator palpebrae superioris.
Nucleus of trochlear nerve lies in the ventral area of the central gray matter at the level of the
inferior colliculi of the midbrain. The nucleus sends the efferent fibers to innervate the
contralateral superior obliquus. Nucleus of abducent nerve occupies the inferior portion of
the pons just within the facial colliculus. It's efferent fibers form the abducent nerve to
innervate the lateral rectus of the ipsilateral eye. Some cells of the nucleus also control the
medial rectus of the contralateral eye.
21. The most evident characteristics of this section are as follows: the central canal has been
spread to form the lower part of fourth ventricle; and the olive with the inferior olivary nuclei
are presented lateral to the pyramid very clearly. A median sulcus and a pair of sulcus
limitans are appeared on the rhomboid fossa. The nucleus of hypoglossal nerve, dorsal
nucleus of vagus nerve and vestibular nuclei are situated in the gray matter of the floor of
fourth ventricle from medial to lateral. The portion ventrolateral to the vestibular nuclei is the
inferior cerebellar peduncle. The pyramidal tracts are still located in the pyramids. Closely
dorsal to the pyramidal tracts and by the midplane are the medial lemniscus, tectospinal tract
and medial longitudinal fasciculus successively from ventral to dorsal. The extensive area
between the inferior cerebellar peduncle and the inferior olivary nuclei is the reticular
formation in which some cranial nuclei, such as nucleus of solitary tract, spinal nucleus and
tract of trigeminal nerve and nucleus ambiguus are situated. The other long tracts are located
in the lateral border of the medulla oblongata.
22. In the center of the section, the transverse fibers of dumb-bell shaped trapezoid body are
presented. The basilar part is bulged ventrally and contains the transverse fibers, longitudinal
bundles of fibers and numerous dispersed neurons called as the pontine nucleus. The
transverse fibers arising from the pontine nucleus cross the midplane to form the contralateral
middle cerebellar peduncle. The longitudinal fibers are belong to the pyramidal tracts and
separated to several small bundles by transverse fibers of pons. The fourth ventricle is shrunk.
The fibers of superior and middle cerebellar peduncles are located on the lateral wall of
fourth ventricle. In the lateral portion of tegmentum, the motor nucleus and pontine nucleus
of trigeminal nerve are situated at both sides of the fibers of trigeminal nerve which run
ventrolaterally and emerge through the ventrolateral surface of pons. The other long tracts are
still in the original places.
23. The central canal is called the cerebral aqueduct and surrounded by the periaqueductal gray
matter. The eminences dorsolateral to the periaqueductal gray matter are inferior colliculi
which contain the corresponding nuclei. The mesencephalic nucleus of trigeminal nerve
appears at the lateral border of periaqueductal gray matter, while the nucleus of trochlear
nerve is in the ventral portion of it and close to the midplane. The medial longitudinal
fasciculus is ventral to the nucleus of trochlear nerve. The most ventral portion of the cerebral
peduncle is the crus cerebri which is formed by the pyramidal tract in the middle 3/5,
frontopontine tract in the medial 1/5 and parieto- occipito-temporo-pontine tracts in lateral
1/5. The portion between the substantial nigra and periaqueductal gray matter is the
tegmentum, which is formed by the reticular formation mainly. The medial, trigeminal and
spinal lemnisci are in the ventrolateral portion of the tegmentum.
24. There are three kinds of fibers in the facial nerve. The special visceral motor fibers come from
the nucleus of facial nerve, the general visceral motor fibers origin from the superior
salivatory nucleus, and the special visceral sensory fibers conducting the taste from anterior
2/3 of tongue enter the upper part of solitary nucleus.
25. If the medulla oblongata is lesion, it can damage to the nucleus ambiguus. The nucleus sends
the efferent fibers that join the glossopharyngeal, vagus and accessory nerves respectively to
innervate the movement of the muscles of soft palatine, pharynx and larynx. So, the cough,
hoarseness and dysphagia may present if the lesion involved to the nucleus ambiguus.
26. The main function of the cerebellum is to maintain posture (equilibrium), to regulate
muscle tone, and to coordinate skilled voluntary and programmed movements.
27. The cerebellum is divided into three main functional areas. The archicerebellum consists of the
flocculonodular lobe. It is also known as the vestibulocerebellum because of connecting with
the vestibular nuclei and vestibular nerves. The paleocerebellum is composed of the vermis
(medial zone) and intermedial zone of the hemispheres, receives the afferents from the spinal
cord, also called the spinocerebellum. The neocerebellum consists of the lateral zone,
connects with the cerebral cortex, also termed as the cerebrocerebellum.
28. Typical signs of the cerebellum injury: (1) ataxia, incoordination of voluntary movements; (2)
nystagmus; (3) intention tremor; (4) hypomyotonia.
29. Vestibulocerebellum mainly receives the axonal terminals from the ipsilateral vestibular
ganglion and vestibular nuclei. These mossy fibres enter in flocculonodular lobe via the
inferior cerebellar peduncle, and conduct the messages of topognosis of head to the
cerebellum. The efferents of vestibulocerebellum relay at the ipsilateral vestibular nuclei via
the inferior cerebellar peduncle, then modulate the activity of motor neurons of spinal medial
nuclear group and cranial nuclei innervating the extraocular muscles through the
vestibulospinal tract and medial longitudinal bundle respectively. These pathways regulate
muscle movement of trunk, playing an important role in the maintenance of body
equilibrium.
30. Its afferents mainly consists of the spinocerebellar tract. The spinocerebellar tract transmits
various movement information of body to the anterior lobe, medial and intermedial areas of
posterior lobe via the inferior and superior cerebellar peduncles. The spinocerebellum also
receives the visual, auditory, vestibular sensations and the input from sensory and motor
cortex of cerebrum (relayed via pons). The efferents originated from spinocerebellum first
relay at the fastigial and interposed nuclei, then depart from the cerebellum. The efferents of
vermis project to ipsilateral vestibular nuclei and reticular formation via the inferior cerebllar
peduncle after relayed at the fastigial nucleus. By innervating ipsilateral spinal medial motor
neurons, it modulates the muscular tension and motor coordination of trunk and proximal
limb muscles through the vestibulospinal and reticulospinal tracts. The efferents of
intermedial cortex of posterior lobe relay at the interposed nuclei, project to contralateral red
nucleus and ventrolateral nuclei of thalamus respectively via decussation of superior
cerebellar peduncle. The relayed fibres of the ventrolateral nuclei of thalamus project to
contralateral cerebral cortex. The red nuclei and cerebral cortex respectively send out the
rubrospinal and corticospinal tracts to ipsilateral spinal lateral motor neurons, to modulate the
muscular tension and motor coordination of extremities.
31. The cerebrocerebellum receives massive inputs of contralateral extensive cerebral cortex,
including sensory, motor and association cortex. The fibres originated from extensive
cerebral cortex terminate in ipsilateral pontine nuclei, then project to contralateral
cerebrocerebellum via the middle cerebellar peduncle. The efferents of cerebrocerebellum,
after relayed in the dentate nucleus, end in contralateral red nucleus and ventrolateral nuclei
of thalamus via decussation of superior cerebellar peduncle. Further, they project to the motor
areas of cerebral cortex. The motor centre of cerebral cortex sends out corticospinal tract to
contralateral spinal lateral motor neurons via decussation of pyramid. Through this
cerebellum-cerebrum feedback circuit, the cerebr0cerebellum affects the modulation of
initiation, planning and coordination of limbs fineness movement, including determination
of movement strength, direction and amplitude. The lesion of neocerebellum may usually
involve the paleocerebellum, with symptoms of lower muscular tension, decrease of tendon
reflex, incoordination and intention tremor. For example, inability of finger-nose test,
dysdiadochokinesia, etc.
32. Non-specific projecting nuclei include the midline nuclei, reticular nuclei and intralaminar
nuclei. Specific relaying nuclei include the ventral anterior nucleus, ventral intermediate
nucleus and ventral posterior nucleus. Internuncial nuclei include the medial nuclear group,
anterior nuclear group and dorsal layer of the lateral nuclear group.
33. Specific relaying nuclei include the ventral anterior nucleus, ventral intermediate nucleus
and ventral posterior nucleus. The ventral anterior nucleus and ventral lateral nucleus receive
axonal terminals of corpus striatum, dentate nucleus and substantia nigra and project into the
cerebral motor cortex to regulate body movement. The ventral posteromedial nucleus
receives the trigeminal lemniscus and taste fibres of the solitary nucleus; the ventral
posterolateral nucleus receives the fibres of the medial lemniscus and spinal lemniscus. The
ventral posterior nucleus contains a well-ordered topographic representation of the body,
which includes from lateral to medial, the sacral, lumbar, thoracic, cervical and cranial
segments. The ventral posteromedial nucleus, which relays sensory information of the head
and face, composes the central thalamic radiations and further projects into the superior part
of postcentral gyrus.
34. The medial forebrain bundle is a loose grouping fiber pathway which runs mostly
longitudinally through the hypothalamus, connecting forebrain autonomic and limbic
structures with the hypothalamus and brainstem, receiving and giving small tracts throughout
its course. It contains many different kinds of fibres: descending afferents from the septal
area and orbitofrontal cortex; ascending afferents from the brainstem; efferents from the
hypothalamus.
The fornix is the largest afferents of the hypothalamus. It originates from the hippocampus
and ends in the mamillary nucleus, preoptic region, lateral zone and posterior hypothalamic
nucleus. The amygdalohypothalamic fiber, which forms terminal stria, originates from the
amygdaloid complex and terminates in the medial preoptic nucleus, anterior hypothalamic
nucleus and supraoptic nucleus.
35. The cerebral hemispheres are composed of the cerebral cortex, the cerebral medullary matter,
the basal ganglia, and the lateral ventricles.
36. The language areas include the motor speech area, the writing area, the auditory speech area
and the auditory speech area.
37. The caudate nucleus and putamen develop together and contain similar cells and, collectively,
are termed the striatum. Functionally, the striatum and their interconnections and
neurotransmitters form the extrapyramidal system, which includes midbrain nuclei such as
the substantia nigra, and the subthalamic nucleus.
38. The cortex of the cerebrum is considered to comprise two types: archicortex and neocortex.
The archicortex is found predominantly in the limbic system cortex and contains three layers,
while the neocortex is more commonly found in most of the cerebral hemisphere and
contains six layers. The paleocortex forms the transition between the archicortex and
neocortex.
39. The limbic lobe includes the subcallosal gyrus, cingulate gyrus, parahippocampal gyrus,
hippocampal formation, insula and temporal pole. The limbic system includes the limbic lobe,
the amygdaloid body, septal nucleus, the anterior nuclear group of thalamus and
hypothalamus. Its basic functions include feeding behavior, "fight-or-flight" responses,
aggression,' and the expressions of emotion and of the autonomic, behavioral, and endocrine
aspects of the sexual response.
40. The archicortex consists of up to three layers of cells. They are the molecular layer, the
pyramidal layer, and the multiform layer.
41. The lateral ventricle may be divided into a central part, which occupies the parietal lobe, and
from which anterior, posterior, and inferior horns extend into the frontal, occipital, and
temporal lobes, respectively.
42. The internal capsule is a broad band of myelinated fibers that separates the lentiform nucleus
from the medial caudate nucleus and thalamus. It has an anterior limb, genu, a posterior limb.
The anterior limb contains the anterior thalamic radiation, thalamocortical and
corticothalamic tracts, the frontopontine tracts. The genu is usually regarded as containing
corticonuclear tract mainly from precentral gyms and terminating in the largely contralateral
motor nuclei of cranial nerves.
The posterior limb contains major ascending and descending pathways. It has the
thalamolentiform, retrolentiform and sublentiform parts. The thalamolentiform part includes
the corticospinal tract, the frontopontine and corticorubral tracts, the central thalamic
radiation. The retrolentiform part includes the occipitopontine, posterior thalamic radiation
and also the optic 'radiation. The sublentiform part contains the temporopontine tract and
acoustic radiation.
43. The basal forebrain comprises several cellular groups of different sizes that are situated in the
rostral and ventral aspect of the telencephalon. The nuclei usually considered together here
are the paraolfactory cortex, diagonal band of Broca, nuclei of Meynert, septal nuclei,
amygdaloid body, and the preoptic region of the hypothalamus.
The basal forebrain is comprised of a neurochemically heterogeneous population of neurons,
including cholinergic, r-Aminobutyric Acid (GABA ergic) and peptidergic neurons that
project to the cerebral cortex, hippocampus, thalamus, posterior hypothalamus and brain stem.
This highly complex brain region has been implicated in attention, motivation, and memory
as well as in a number of neuropsychiatric disorders such as Alzheimer's disease, Parkinson's
disease, and schizophrenia.
44. The hippocampus is a primitive cortical structure that has been "folded in" and "rolled up" so
that it is submerged deep into the parahippocampal gyrus. It extends the length of the floor of
the inferior horn of the lateral ventricle and becomes continuous with the fornix below the
splenium of corpus callosum. The hippocampus has been divided into CA1, CA2, CA3 and
CA4 areas.
The hippocampus receives input from many parts of the neocortex, especially the temporal
neocortex. The fomix is an arched white fiber tract extending from the hippocampal
formation to the mamillary bodies of the hypothalamus and septal area. The hippocampal
efferent axons travel in the fornix and synapse on neurons in the mamillary bodies. These
neurons project axons, within the mamillothalamic tract, to the anterior thalamus. The
anterior thalamus projects, in turn, to the cingulate gyms, which contains a bundle of
myelinated fibers, the cingulum, that curves around the corpus callosum to reach the
parahippocampal gyms. Thus, the following circuit is formed parahippocampal gyrus→
hippocampus → fornix → mamillary bodies → anterior nuclear group of thalamus →
cingulate gyrus→ parahippocampal gyrus. This circuit, called the Papez circuit, ties together
the cerebral cortex and the hypothalamus.
45. The neocortex consists of up to six well-defined layers of ceils. The organization of these
layers is referred to as cytoarchitecture. The outermost molecular layer contains non-specific
afferent fibers that come from within the cortex or from the thalamus. The external granular
layer is a rather dense layer composed of small cells. The external pyramidal layer contains
pyramidal cells, frequently in row formation. The internal granular layer is usually a thin
layer with cells similar to those in the external granular layer. These cells receive specific
afferent fibers from the thalamus. The internal pyramidal layer contains, in most areas,
pyramidal cells that are fewer in number but larger in size than those in the external
pyramidal layer. These cells project to distal structures. The multiform layer consists of
irregular fusiform cells whose axons enter the adjacent white matter.
46. The white matter of the cerebral hemisphere is deep to the cerebral cortex and consists of
myelinated nerve fibers of many sizes. These fibers form connections within the brain. The
white matter contains the commissural fibers, the association fibers, and the projection fibers.
The association fibers connect the various portions of a cerebral hemisphere and permit the
cortex to function as a coordinated whole. They include the arcuate fibers, the uncinate
fasciculus, the cingulum, the superior longitudinal fasciculus, the interior longitudinal
faseiculus.
The commissural fibers interconnect the two cerebral hemispheres which include the corpus
callosum, the anterior commissure and the commissure of the fornix. The projection fibers
connect the cerebral cortex with lower portions of the brain or the spinal cord. They are
composed of afferent and efferent nerve fibers, which form a compact band known as the
internal capsule. The internal capsule is a broad band of myelinated fibers that separates the
lentiform nucleus from the medial caudate nucleus and thalamus. It has an anterior limb,
genu, a posterior limb. In horizontal section, it presents a V-shaped appearance, with the genu
(apex) pointing medially.
(Shao Xujian, Hu Haitao, Li Yueying, Luo Xuegang, Peng Yingji, He Hongwen)
Multiple Choice Questions
A1
1. Which of the following nerves may be injured alter fracture of the medial epicondyle of the
humerus?
A. median nerve
B. ulnar nerve
C. radial nerve
D. musculocutaneous nerve
E. axillary nerve
2. Which of the following nerves may be injured after fracture of the fibular neck?
A. common peroneal nerve
B. tibial nerve
C. sciatic nerve
D. sural nerve
E. posterior femoral cutaneous nerve
3. The tibialis anterior muscle is supplied by
.
A. tibial nerve
B. superficial peroneal nerve
C. deep peroneal nerve
D. sciatic nerve
E. femoral nerve
4. Which of the following nerve supplies the first lumbrical muscle of hand?
A. radial nerve
B. median nerve
C. ulnar nerve
D. musculocutaneous nerve
E. axillary nerve
5. About the course of phrenic nerve, all of the following statements are not true except that the
nerve
.
A. descends on the front of the prevertebral fascia
B. passes behind the first part of the subclavian artery
C. passes behind the root of the lung
D. descends on the front of scalenus anterior
E. emerge from the scalene fissure
6. The serratus anterior is supplied by
.
A. thoracodorsal nerve
B. lateral pectoral nerve
C. medial pectoral nerve
D. dorsal scapular nerve
E. long thoracic nerve
7. Which of the following nerves supplies the brachioradialis?
A. median nerve
B. ulnar nerve
C. radial nerve
D. musculocutaneous nerve
E. axillary nerve
8. The deltoid muscle is supplied by
.
A. radial nerve
B. axillary nerve
C. musculocutaneous nerve
D. dorsal scapular nerve
E. thoracodorsal nerve
9. The skin on the plane of the sternal angle is mainly distributed by
.
A. anterior branches of the first pair of thoracic nerves
B. anterior branches of the second pair of thoracic nerves
C. anterior branches of the third pair of thoracic nerves
D. anterior branches of the fourth pair of thoracic nerves
E. anterior branches of the sixth pair of thoracic nerves
10. Which of the following nerves supplies the triceps brachii?
A. musculocutaneous nerve
B. axillary nerve
C. median nerve
D. ulnar nerve
E. radial nerve
11. Which of the following nerves may be injured, if a patient was unable to extend knee?
A. femoral nerve
B. superior gluteal nerve
C. inferior gluteal nerve
D. sciatic nerve
E. obturator nerve
12. The biceps brachii is supplied by
A. ulnar nerve
B. radial nerve
C. musculocutaneous nerve
D. median nerve
E. axillary nerve
13. About femoral nerve, which of the following statements is not true?
A. It arises from the lumbar plexus.
B. It descends between the psoas major and iliacus.
C. It enters the femoral triangle behind the inguinal ligament and lateral to the femoral vessels.
D. Its muscular branches supply the muscles of medial group of the thigh.
E. Its longest branch is the saphenous nerve.
14. Which of the following nerves supplies the gluteus maximus?
A. superior gluteal nerve
B. inferior gluteal nerve
C. sciatic nerve
D. pudendal nerve
E. posterior femoral cutaneous nerve
15. About sciatic nerve, which of the following statements is not true?
A. It is the largest nerve in the body.
B. It leaves the pelvis through the greater sciatic foramen inferior to the piriformis.
C. It leaves the buttock region by passing deep to the short head of the biceps femoris.
D. It divides into the tibial and common peroneal nerves.
E. It gives off muscular branches to supply the muscles of back of the thigh.
16. Which of the following nerves supplies the adductor pollicis?
A. median nerve
B. musculocutaneous nerve
C. radial nerve
D. ulnar nerve
E. axillary nerve
17. The following structures pass behind the flexor retinaculum of the wrist extant __
.
A. flexor digitorum superficialis tendons
B. flexor pollicis longus tendon
C. ulnar nerve
D. median nerve
E. anterior interosseous nerve
18. Which kind of cranial fibers are found in olfactory nerves?
A. general somatic afferent (GSA) fibers
B. general somatic efferent (GSE) fibers
C. special somatic afferent (SSA) fibers
D. general visceral afferent (GVA) fibers
E. special visceral afferent (SVA) fibers
19. Which kind of cranial fibers are found in optic nerves?
A. general somatic afferent (GSA) fibers
B. general somatic efferent (GSE) fibers
C. special somatic afferent (SSA) fibers
D. general visceral afferent (GVA) fibers
E. special visceral afferent (SVA) fibers
20. Where synapses of olfactory nerve are made?
A. mucosa of the superior nasal conchae
B. bipolar neuron
C. olfactory bulb
D. olfactory tract
E. primary olfactory area in the cerebral cortex
21. Which nerve emerge dorsally from the brainstem?
A. optic nerve
B. oeulomotor nerve
C. trochlear nerve
D. abducent nerve
E. hypoglossal nerve
22. Trochlear nerve supplies the
.
A. levator palpebrae superioris
B. superior rectus
C. superior obliquus
D. inferior rectus
E. inferior obliquus
23. Abducent nerve supplies the
.
A. levator palpebrae superioris
B. superior rectus
C. inferior rectus
D. medial rectus
E. lateral obliquus
24. Which of the following is not a result of sympathetic nerve stimulation?
A. constriction of pupil
B. stimulation of hormone secretion
C. heart rate and force increased
D. bronchial muscle contracted
E. relaxation of ciliary muscle
25. Atropine blocks parasympathetic nerve effects It would therefore result in
.
A. dilation of the pupils
B. a decrease in mucus secretion
C. a decrease in GI tract movement
D. an increase in heart rate
E. all of the above
26. The area of the brain that is most directly involved in the reflex control of the autonomic
system is
.
A. the hypothalamus
B. the cerebral cortex
C. the medulla oblongata
D. the cerebellum
E. brainstem
27. The cell bodies of preganglionic parasympathetic neurons are located in the
.
A. cervical and thoracic spinal cord
B. brain and lumbar spinal cord
C. thoracic and lumbar spinal cord
D. brain and sacral spinal cord
E. hypothalamus and cerebral cortex
28. Which of the following cranial nerves contains parasympathetic fibers?
A. abducent
B. optic
C. olfactory
D. vestibulocochlear
E. none of the preceding
29. Regarding the sympathetic division of the ANS, which statement is right?
A. Acetylcholine is secreted by some sympathetic postganglionic fibers.
B. Sympathetic postganglionic neurons are found in spinal segments from T1 to L3.
C. The sympathetic chain extends from the thoracic to the sacral regions of the spinal cord.
D. Most sympathetic preganglionic fibers secrete norepinephrine (noradrenaline).
E. Sympathetic preganglionic neurons are found in whole spinal segments.
30. Regarding the parasympathetic division of the ANS, which statement is right?
A. Parasympathetic preganglionic fibers are found in all cranial nerves.
B. Parasympathetic vasoconstrictor fibers are present in the salivary glands.
C. Parasympathetic postganglionic neurons are found in spinal ganglia from T1 to L3.
D. Parasympathetic postganglionic fibers secrete acetylcholine onto their target organs.
E. Parasympathetic postganglionic neurons are located in brainstem.
31. Sympathetic preganglionic fibers are characterized as
.
A. being short in length and unmyelinated
B. being short in length and myelinated
C. being long in length and myelinated
D. being long in length and unmyelinated
E. being long in length and gray communicating branches
32. All preganglionic autonomic fibers release
and the effects are always
.
A. norepinephrine; inhibitory
B. norepinephrine; excitatory
C. acetylcholine; excitatory
D. acetylcholine; inhibitory
E. norepinephrine; inhibitory
33. Postganglionic fibers of autonomic neurons are usually
.
A. myelinated
B. unmyelinated
C. larger than preganglionic fibers
D. shorter than preganglionic fibers
E. located in the spinal cord
34. The autonomic division of the nervous system directs
.
A. voluntary motor activity
B. conscious control of skeletal muscles
C. conscious control of smooth muscles
D. all behavioral activities
E. processes that maintain homeostasis
35. Dual innervation refers to situations in which
.
A. visceral organs receive instructions from both sympathetic and parasympathetic fibers
B. sympathetic and parasympathetic fibers have similar effects
C. the atria and ventricles of the heart receive autonomic stimulation from the same nerves
D. sympathetic fibers always stimulate organs and parasympathetic fibers always inhibit
organs
E. sympathetic division conserves the body's energy through cholinergic effects
36. What type of effector is not under autonomic regulation?
A. glands
B. skeletal muscle
C. smooth muscles
D. cardiac muscles
E. two of the preceding
37. The actions of sympathetic and parasympathetic neurons are cooperative in
.
A. the heart
B. the reproductive system
C.the digestive system
D. the eyes
E. the lungs
38. Which of the following neurons release norepinephrine?
A. preganglionic parasympathetic neurons
B. postganglionic sympathetic neurons in the heart
C. postganglionic parasympathetic neurons
D. postganglionic parasympathetic neurons in sweat glands
E. all of the above
39. The prevertebral ganglia contain which kind of cell bodies?
A. preganglionic parasympathetic
B. postganglionic parasympathetic
C. preganglionic sympathetic
D. postganglionic sympathetic
E. visceral sensory ganglia
40. Which of the following is the best way to describe how the ANS is controlled?
A. completely under control of voluntary cerebral cortex
B. entirely controls itself
C. completely under control of brain stem
D. completely under control of spinal cord
E. less control from cerebrum, more control by hypothalamus, amygdala, spinal and
peripheral reflexes
41. All of the following statements regarding the anatomy of the superior hypogastric plexus are
true except
.
A. it lies anterior to Ls vertebra
B. it lies just inferior to the aortic bifurcation
C. it lies right of midline
D. it branches left and right and descends to form the inferior hypogastric plexus
E. it receives sympathetic fibers from lumbar ganglia of sympathetic trunk
42. A celiac plexus block is effective in reducing pain originating from all of the following
organs except
.
A. pancreas
B. transverse colon
C. gall bladder
D. descending colon
E. kidney
43. Which of the following statements about the carotid plexuses is not correct?
A. branches of the superior cervical sympathetic ganglion
B. project to the salivary and lacrimal glands
C. around the internal and external carotid arteries
D. distribution of sympathetic fibers to the head
E. constrict the pupil
44. The hypogastric plexus are found
.
A. in front of the fifth lumbar vertebra
B. on either side of the rectum
C. extending along the visceral branches of the hypogastric artery
D. to the pelvic viscera and genitals
E. all of the above
45. Which of the following statements about the afferent visceral fibers is not fight?
A. The fibers are involved in reflexes that regulate blood pressure and heart rate.
B. The fibers carry a variety of sensations to the cerebellum.
C. Visceral afferent axons in the glossopharyngeal nerve and vagus nerve.
D. The cell bodies of visceral sensory neurons are located in spinal ganglia.
E. They accompany the autonomic motor fibers to the visceral organs.
46. Which is concerning the pelvic splanchnic nerves?
A. preganglionic parasympathetic fibers
B. innervate lower half of the large intestine
C. the rectum
D. urinary and reproductive systems
E. all of the above
47. Damage to the anterior roots of the first five thoracic nerves on the fight side would interfere
with
.
A. the ability to dilate the right pupil
B. the ability to dilate the left pupil
C. the ability to constrict the right pupil
D. the ability to contract the left biceps brachii muscle
E. the ability to contract the right biceps brachii muscle
48. Under which set of circumstances would the diameter of peripheral blood vessels be the
greatest
.
A. increased sympathetic stimulation
B. decreased sympathetic stimulation
C. increased parasympathetic stimulation
D. decreased parasympathetic stimulation
E. none of the above
49. Injury to the neurons of a prevertebral ganglion would affect the function of the
.
A. heart
B. pupils
C. sweat glands
D. digestive tract
E. lungs
50. Regarding the role of the autonomic innervation, which statement is fight?
A. Stimulation of the sympathetic nerves to the eyes causes pupillary constriction.
B. Activation of the sympathetic system causes vasoditatation in the skin.
C. Activation of the sympathetic system causes vasoconstriction in the viscera and
vasodilatation in skeletal muscle.
D. Stimulation of the vagus nerves speeds up the heart.
E. Stimulation of parasympathetic nerves to the bronchi causes inhibition of
secretion.
51. Which of the following does not characterize the ANS?
A.independent and without a person's conscious control
B. preganglionic cell bodies in the CNS
C. presence of postganglionic cell bodies in ganglia
D. It is composed of portions of both the CNS and PNS.
E. The somatic motor system is organized more diffusely than the ANS.
52. In the autonomic nervous system,
.
A. the lower motor neurons directly innervate effector organs
B. there is always a synapse between the CNS and the effector organ
C. motor neurons do not synapse but are connected by gap junctions
D. all preganglionic sympathetic fibers release acetylcholine
E. innervation of skeletal muscle
53. In the ANS,
.
A. preganglionic fibers are mostly unmyelinated
B. postganglionic fibers are myelinated
C. all parasympathetic preganglionic neurons have their cell bodies in the brainstem
D. acetylcholine is released at all ganglia
E. one preganglionic sympathetic neuron can synapse in several postganglionic ganglia
54. Sympathetic nerves arise from
and
regions.
A. cervical; lumbar
B. cranial; sacral
C. cranial; thoracic
D. thoracic; lumbar
E. thoracic; sacral
55. The longest autonomic nerve is the
.
A. phrenic
B. spinal accessory
C. vagus
D. thoracic
E. lumbar
56. The autonomic nervous system
.
A. contains somatic motor fibers
B. operates reflexly
C. contains ganglia
D. all of the preceding
E. both B and C
57. The cell bodies of postganglionic autonomic fibers are located in
.
A. the lateral horn of the spinal cord
B. the effector organ
C. the anterior horn of the spinal cord
D. the medulla oblongata
E. ganglia
58. Which cranial nerves contain parasympathetic fibers?
A. Ⅲ, Ⅴ,Ⅳ, and Ⅶ
B. Ⅶ, Ⅸ, Ⅷ, and Ⅹ
C. Ⅲ, Ⅶ, Ⅸ, and Ⅹ
D. Ⅶ, Ⅴ, Ⅷ, and Ⅹ
E. Ⅴ, Ⅶ, Ⅸ, and Ⅹ
59. Which is concerning the sympathetic division of the ANS?
A. intramural ganglia
B. increases motility of stomach
C. otic and ciliary ganglia
D. secretion of lacrimal and salivary glands
E. increases heart rate, respiratory rate, and blood pressure
60. Which is concerning the parasympathetic division of the ANS?
A. gray communicating branch
B. cervical ganglia of the sympathetic trunk
C. craniosacral outflow
D. synapse in celiac ganglion
E. short preganglionic fibers, long postganglionic fibers
61. Neurons of the intermediolateral nucleus of the spinal cord make direct synaptic connection
with
.
A. paravertebral sympathetic ganglia
B. prevertebral ganglia
C. ganglia of sympathetic trunk
D. all of the above
E. none of the above
62. The lesser splanchnic nerve is formed by
.
A. T5-T9
B. T1-L2
C. L3-L5
D. S1-S4
E. T10-T12
63. Splanchnic nerves
.
A. are formed by parasympathetic postganglionic fibers
B. include preganglionic fibers that go to prevertebral ganglia
C. control sympathetic function of structures in the head
D. connect one postganglionic neuron with another
E. include postganglionic fibers
64. Preganglionic fibers of the sympathetic nervous system that carry motor impulses to targets in
the body wall or thoracic cavity synapse in a
.
A. intramural ganglion
B. spinal ganglion
C. ganglion of sympathetic trunk
D. adrenal ganglion
E. prevertebral sympathetic ganglia
65. Preganglionic fibers of the ANS sympathetic division originate in the
.
A. cerebral cortex of the brain
B. medulla oblongata
C. brain stem and sacral spinal cord
D. thoracolumbar spinal cord
E. ganglion of sympathetic trunk
66. The greater splanchnic nerves contain which kind of fibers?
A. preganglionic sympathetic
B. postganglionic sympathetic
C. preganglionic parasympathetic
D. postganglionic parasympathetic
E. thoracic nerves
67. The white communicating branches contain what kind of fibers?
A. preganglionic parasympathetic
B. postganglionic parasympathetic
C. preganglionic sympathetic
D. postganglionic sympathetic
E. spinal nerves
68. Prevertebral ganglia are involved with the innervation of the
.
A. abdominal organs
B. thoracic organs
C. head
D. arrector pili
E. all of above
69. Which of the following statements about the superior mesenteric ganglion is true?
A. It is a parasympathetic ganglion.
B. It is a paravertebral sympathetic ganglion.
C. It is located in the head.
D. It contains postganglionic sympathetic neurons.
E. It is a preganglionic sympathetic ganglion.
70. The preganglionic neurons of the sympathetic division of the autonomic nervous system
originate in
.
A. the midbrain and the medulla oblongata
B. the entire spinal nerve complex
C. the first cervical (C1) to the first lumbar (L1) segments
D. the first thoracic (T1) to the third lumbar (L3) segments
E. the brain and spinal cord
71. The pterygopalatine, ciliary, submandibular, and otic ganglia are
.
A. prevertebral ganglia
B. cervical sympathetic ganglia
C. postganglionic sympathetic neurons
D. parasympathetic ganglia that receive neurons from the vagus nerves
E. parasympathetic ganglia that receive neurons from the third, seventh, and ninth cranial
nerves
72. Parasympathetic ganglia are located
.
A. in a trunk parallel to the spinal cord
B. in the prevertebral ganglia
C. near to or within the organs innervated
D. in the brain
E. in the posterior roots of spinal nerves
73. Which of the following is not a result of parasympathetic nerve stimulation?
A. increased movement of the G1 tract
B. increased mucus secretion
C. dilation of the pupils
D. constriction of visceral blood vessels
E. increased urine secretion
A2
1. A patient suffered from a short period of paralysis of the diaphragm on one side on a lung
operation, which of the following nerves may be blocked?
A. vagus nerve
B. phrenic nerve
C. anterior branches of the sixth thoracic nerve
D. anterior branches of the eighth thoracic nerve
E. sympathetic trunk
2. An old man slipped and fell A glass pierced the skin on the front of his left wrist. The palmaris
longus tendon had been severed. The thumb was laterally rotated and adducted, and the man
can not oppose his thumb to the other fingers. The skin sensation over the lateral half of the
palm and the palmar side of the lateral three and one-half fingers was lost. The following facts
concerning this patient are correct except:
A. The palmar cutaneous branch of the median nerve was severed.
B. Sensory loss of the distal part of the dorsal surfaces of the lateral three and one-half fingers
was experienced.
C. The median nerve lies radial side to the palmaris longus proximal to the flexor retinaculum
and was severed by the piece of glass.
D. The median nerve lies in the interval between the tendons of flexor digitorum superficialis
and flexor carpi radialis muscles just proximal to the wrist joint.
E. Adduction of the thumb was produced by the contraction of the adductor pollicis muscle,
which is supplied by the ulnar nerve.
3. A 60-year-old woman fell down the stairs with severe left arm pain. On examination, the
woman was unable to extend the wrist and fingers, and the wrist was flexed. The skin sensation
of the back of the arm and forearm was also lost, which of the following nerves may be injured?
A. axillary nerve
B. musculocutaneous nerve
C. ulnar nerve
D. median nerve
E. radial nerve
4. A young man was involved in a fight. During the brawl he received a deep knife wound to the
front of his right thigh. On examination in the Emergency Department, the patient had weak
flexion of the knee, skin sensation was lost over the anterior and medial sides of the thigh,
which of the following nerves may be injured?
A. obturator nerve
B. femoral nerve
C. sciatic nerve
D. lateral femoral cutaneous nerve
E. genitofemoral nerve
5. This is a 61-year old turn with pain and severe ischaemia to the lower limb due to peripheral
vascular disease. Lumbar sympathectomy was advised. The mechanisms by which this works
are
.
A. visceral sensory fibres travel in the lumbar sympathetic chain
B. vasomotor fibres travel in the lumbar sympathetic chain
C. somatic sensory fibres travel in the lumbar sympathetic chain
D. somatic motor fibres travel in the lumbar sympathetic chain
E. both sensory and vasomotor fibres travel in the lumbar sympathetic chain
6. Trauma to the neck as with a strong jerk from a collar or straining against a leash can also
produce Homer's syndrome. The symptoms of Homer's syndrome include, a constricted pupil
(miosis), drooping of the upper eyelid (ptosis), absence of sweating of the face (anhidrosis), and
sinking of the eyeball into the bony cavity that protects the eye (enophthalmos). Homer' s
syndrome may occur with injury or lesions of
.
A. cervical sympathetic chain
B. anterior roots of T1 or T2
C. white communicating branches from T1 or T2
D. brainstem stroke
E. all of the above
BI
A. posterior cord
B. lateral cord
C. both medial and lateral cords
D. upper trunk
E. lower trunk
1. Axillary nerve arises from
.
2. Median nerve arises from
.
A. median nerve
B. radial nerve
C. dorsal cutaneous branch of ulnar nerve
D. superficial branch of ulnar nerve
E. palmar cutaneous branch of ulnar nerve
3. Skin sensation of the dorsal surface of root of thumb is innervated by
.
4. Skin sensation of the medial side of palmar aspect of ring finger is innervated by
A. deltoid muscle
B. brachialis
C. serratus anterior
D. triceps brachii
E. pronator teres
5. The axillary nerve supplies
.
6. The long thoracic nerve supplies
.
A. gluteus medius
B. gluteus maximus
C. gracilis
D. biceps femoris
E. quadratus femoris
7. The obturator nerve innervates
.
8. The superior gluteal nerve innervates
.
A. below the inguinal ligament
B. through the obturator canal
C. through the greater sciatic foramen superior to the piriformis
D. through the greater sciatic foramen inferior to the piriformis
E. through the popliteal fossa
9. The pudendal nerve leaves the pelvis by passing
.
10. The sciatic nerve leaves the pelvis by passing
.
A. the reproductive system
B. the heart
C. the eyes
.
D. most blood vessels
E. answers B and C
11. The actions of sympathetic and parasympathetic neurons are 'cooperative in
.
12. The actions of sympathetic and parasympathetic neurons are antagonistic in
.
13. The actions of only sympathetic innervation are in
.
A. parasympathetic ganglia
B. preganglionic sympathetic neurons
C. paravertebral sympathetic ganglia
D. prevertebral sympathetic ganglia
E. sensory ganglia
14. Superior mesenteric ganglion is
.
15. Pterygopalatine ganglia are
.
16. Inferior ganglion is
.
17. Cervical sympathetic ganglia are
.
A. lateral horn located at spinal cord segments S2-4
B. lateral horn located at spinal cord segment T6
C. lateral horn located at spinal cord segment T11
D. middle cervical ganglion
E. lateral horn located at spinal cord segment L1
18. Which is the location of postganglionic sympathetic nerve cell bodies that innervate the heart?
19. Which is the location of preganglionic sympathetic nerve cell bodies that supply the liver?
20. Which is the location of preganglionic parasympathetic nerve cell bodies that supply the
bladder?
True or False Questions
1. Nerves Ⅲ, Ⅳ, and Ⅵ control movements of the eye and pupillary constriction.
(
)
2. Nerves Ⅲ, Ⅴ, Ⅸ, and Ⅹ carry parasympathetic fibers.
(
)
3. Special somatic afferent (SSA) fibers are found in nerves Ⅰ , Ⅱ and Ⅷ.
(
)
4. General visceral afferent (GVA) fibers are found in nerves Ⅸ and Ⅹ.
(
)
5. Olfactory nerves are composed of bipolar neurons that function as chemoreceptors, responding
to volatile chemical particles breathed into the nasal cavity.
(
)
6. The opticnerve conducts impulses from the ganglion cells in the retina.
(
)
7. Damage to the mandibular nerve impairs chewing.
(
)
8. Auriculotemporal nerve is responsible for the referral of pain from the ternporomandibular joint.
( )
9. The vestibulocochlear nerve arises from spiral and vestibular ganglia in the labyrinth of the
inner ear.
(
)
10. Pressure over the carotid sinus normally produces slowing of the heart rate and a fall in blood
pressure.
( )
11. The sympathetic ganglionic trunks (chains) extend from the base of the skull to the coccyx,
along the lateral sides of bodies of the vertebral column.
(
)
12. Sympathetic ganglia include 22-24 pairs of paravertebral ganglia and unpaired prevertebral
ganglia.
(
)
13. Some of the pregangtionic fibers synapse with postganglionic neurons located at their same
level in the ganglia of sympathetic trunk.
(
)
14. Many preganglionic axons ascend or descend in the sympathetic trunk to synapse in a ganglion
at another body level.
(
)
15. The preganglionic fibers synapse in the ganglia of sympathetic trunk and the postganglionic
fibers run in white communicating branches to the spinal nerves.
(
)
16. In the head, most preganglionic sympathetic fibers associate with a large artery that distributes
them to the glands and smooth musculature of the head.
(
)
17. Many postganglionic sympathetic fibers to the heart, however, descend from the cervical
ganglia in the neck.
(
)
18. In the abdominal organs, the postganglionic sympathetic fibers run in splanchnic nerves to
synapse in prevertebral ganglia on the aorta.
(
)
19. In the pelvic organs, postganglionic sympathetic fibers travel through the most inferior
hypogastric plexuses to the pelvic organs.
(
)
20. Four cranial nerves convey postganglionic parasympathetic fibers.
(
)
21. Each parasympathetic ganglion has a sympathetic, parasympathetic, and sensory roots.
(
)
22. The sacral preganglionic parasympathetic fibers form distinct lumbar splanchnic nerves.
(
)
23. Most parasympathetic neurons travel within spinal nerves.
(
)
24. Mass activation of the sympathetic division prepares the body for intense physical activity in
emergencies; the heart rate increases, blood glucose rises.
(
)
25. In the cardiac baroreceptor reflex, preganglionic neurons found in the intermediolateral
column of the spinal cord are inhibited when blood pressure increases
(
)
26. All motor neurons that control the eye muscles are located in the nucleus of oculomotor nerve
in the brainstem.
(
)
27. General visceral sensory neurons monitor temperature, pain, irritation, chemical changes, and
stretch in the visceral organs.
(
)
28. Visceral sensory fibers run within the autonomic nerves, especially within the vagus and the
sympathetic nerves.
(
)
29. A simplified description of most visceral sensory pathways to the brain is the following:
sensory neurons to spinothalamic tract to thalamus to cerebral cortex.
(
)
30. The sympathetic division conserves the body's energy through cholinergic effects.
(
)
31. The autonomic :nervous system is concerned with control of target tissues: the skeletal muscle
and the glands.
(
)
32. The autonomic nervous system helps maintain a constant internal body, environment
(homeostasis).
(
)
33. The sympathetic and somatic divisions are the two divisions of the autonomic nervous
system.
(
)
34. The autonomic motor pathway involves two neurons in the motor transmission of impulses.
(
)
35. The cell body of the first neuron for the ANS is located within ,the central nervous system and
is called the postganglionic neuron.
(
)
36. Preganglionic fibers are usually myelinated fibers. Most postganglionic fibers are
unmyelinated fibers.
(
)
37. The somatic motor system is organized more diffusely than the ANS.
(
)
38. The cell body of the first-order neuron for the ANS is located within the PNS.
(
)
39. Most organs are innervated by fibers from both divisions of the ANS.
(
)
40. The preganglionic sympathetic cell bodies are in the lateral horns of the spinal cord from the
levels of C1 to L3.
(
)
Explanation of Terms
1. nerve point of neck
2. brachial plexus
3. saphenous nerve
4. ciliary ganglion
5. pterygopalatine ganglion
6. submandibular ganglion
7. otic ganglion
8. chorda tympani
9. greater petrosal nerve
10. tympanic nerve
11. recurrent laryngeal nerve
Answer the Following Questions
1. Narrate the composition of the spinal nerve, the types and distribution of nerve fibers in the
spinal nerve.
2. Which nerve may be injured after fracture of the surgical neck of the humerus, and analyze the
clinical features of the nerve lesion.
3. Which nerve may be injured after kicking on the fibular neck, and analyze the clinical features
of the nerve lesion.
4. Injure the olfactory bulb, what will happen?
5. What will happen when the oculomotor nerve is damaged?
6. What will happen when abducent nerve is damaged?
7. Which branches of facial nerve pass through the bony facial canal?
8. What will happen when the vestibulocochlear nerve is lesion?
9. Damage the recurrent laryngeal nerve, what will happen?
10. What will happen when hypoglossal nerve injured?
11. Describe the division and innervations of the oculomotor nerve.
12. Describe the division and innervations of the trigeminal nerve.
13. What will happen when facial nerve is injured?
14. Describe the innervations of the eye.
15. Describe the innervations of the tongue.
16. Describe the division and innervations of the trigeminal nerve.
17. Lesions of the vagus nerve, what will happen?
ANSWERS
Multiple Choice Questions
A1
1. B
2. A
3. C 4. B
5. D
6. E 7. C
8. B 9. B 10. E
ll. A
12. C 13. D 14. B 15. C 16. D 17. C
18. E 19. A 20. C 21. C
22. C 23. C 24. D 25. E
26. E 27. D 28. E 29. A 30. D 31.
B
32. C
33. B 34. E
35. A
36. B
37. B
38. B
39. D 40. E
41. C 42. D 43. E 44. E 45. B 46. E 47. A 48. B 49. D 50. C 51. E
52. D 53.
E 54. D 55. C 56. E 57. E 58. C 59. E
60. C 61. D
62. E 63. B
64. C 65. D 66. A 67. C
68. A 69. D 70. D 71.
E
72. C 73. D
A2
1. B
2. D
3. E
4. B
5. E
6. E
BI
1. A
2. C
3. B
4. D 5. A
6. C 7. C
8. B
9. D
10. D Il. A
12. E
13. D
14. D
15. A
16. E 17. C 18. D 19. B 20. A
True or False Questions
1. T 2. F
3. F
4. T
5. T
6. F
7. T
8. T
9. T
10. T 11. T
12. T 13. T 14. T 15. F 16. F 17. T
18. F
19. T 20. F
21. T
22. F 23. F 24. T 25. T 26. F 27. T 28. T 29. T 30. F 31. F 32. T 33. F 34. T
35. F 36. T 37. F 38. F 39. T 40. F
Explanation of Terms
1. Superficial branches of the cervical plexus emerge around the middle of the posterior border of
the sternocleidomastoid muscle, the midpoint is called nerve point of neck. In a cervical plexus
block, the anesthetic agent is injected here.
2. Brachial plexus is formed by the union of the anterior rami of the four lower cervical nerves and
great part of the anterior ramus of the first thoracic nerve. It extends downward and laterally
through the scalene fissure, then passes behind the clavicle, and finally enters the axilla.
Structurally, the brachial plexus is divided into roots, trunks, divisions and cords. Each brachial
plexus innervates the entire upper extremity of one side, as well as a number of muscles of the
shoulder and neck.
3. It is the longest cutaneous nerve of femoral nerve. It runs downward and passes through the
adductor canal. It then emerges on the medial side of the knee and descends along the medial
side of the leg. It is distributed to the skin of the medial side of the leg and foot.
4. It is small, flat, irregular-shaped ganglion measuring between 1 and 2mm in diameter. It lies
between the optic nerve and the lateral rectus. It has three roots. The parasympathetic root,
derived from the branch of the oculomotor nerve to the inferior obliquus, consists of
preganglionic fibers from the E-W nucleus, which relay in the ganglion, the postganglionic
fibers traveling in the short ciliary nerves to the sphincter pupillae and ciliary muscle. The
sympathetic root is a branch from the internal carotid plexus, which either passes directly to the
ganglion or joins the sensory root to reach it indirectly; it consists of postganglionic fibers from
the superior cervical ganglion, which traverse the ganglion without synapse, to emerge into the
short ciliary nerve. They are distributed to the dilator pupillae. The sensory root is a
communicating branch of the nasociliary nerve, containing sensory fibers from the eyeball
which reach the ganglion in short ciliary nerves and traverse it without synapse. The branch
leaves the ganglion posteriorly and runs backwards to join the nasociliary nerve near its
orbital entry.
5. It is flattened, reddish-grey and lies deeply in the pterygopalatine fossa just below the maxillary
nerve as it crosses. It has three roots. The motor or parasympathetic root is the nerve of the
pterygoid canal, consists of preganglionic fibers, which arise from special lacrimatory nucleus
and run in greater petrosal nerve, the postganglionic fibres join the maxillary nerve, supplying
secretomotor fibers to the lacrimal gland. The sympathetic root is also incorporated in the nerve
of the pterygoid canal. Its postganglionic fibers arise in the superior cervical ganglion and
travel via the internal carotid plexus and deep petrosal nerve. The sensory roots are maxillary
division sensory fibers which pass through the ganglion without synapsing. The branches of
the ganglion include the orbital, palatine, nasal and pharyngeal.
6. It is a small, fusiform body which lies on the upper part of the hyoglossus. It is superior to the
deep part of the submandibular gland and inferior to the lingual nerve, suspended from the latter
by anterior and posterior filament. It has three roots. Its motor, parasympathetic root conveys
preganglionic fibers from the superior salivatory nucleus traveling in the facial, chorda tympani
and lingual nerves to the ganglion, which they synapse. The postganglionic fibers are
secretomotor to the submandibular and sublingual salivary glands. The sympathetic root is
derived from the plexus' on the facial artery, which traverse the ganglion without synapsing.
The sensory root derived from the sensory fibers of lingual nerve. The branches from the
ganglion supply the submandibular gland and its duct, the sublingual and anterior lingual gland.
7. It is a small, oval, flat reddish-grey ganglion situated just below the foramen ovale, lateral to the
mandibular nerve. It has four roots. The parasympathetic root is the lesser petrosal nerve,
conveying preganglionic fibers from the glossopharyngeal nerve, postganglionic fibers passing
by a communicating branch to the auriculotemporal nerve and so to the parotid gland. The
sympathetic root is from the plexus on the middle meningeal artery. The motor root derived
from the motor fibers of mandibular nerve. The sensory root derived from the auriculotemporal
nerve.
8. It is a branch of facial nerve. It leaves the facial nerve about 6mm above the stylomastoid
foramen. It contains efferent preganglionic parasympathetic (secretomotor) fibres which enter
the submandibular ganglion, from which postganglionic fibres are relayed to the submandibular
and sublingual glands. However, most of its fibres are afferent from the mucosa of the anterior
two-thirds of the tongue, except the vallate papillae, and it constitutes the nerve of taste for this
lingual region. Before uniting with the lingual nerve, the chorda tympani is joined by a small
branch from the otic ganglion.
9. It is a branch of facial nerve. It passes forward and joins with a deep petrosal nerve from the
sympathetic carotid plexus to form the nerve of the pterygoid canal. This nerve passes through
the canal of the pterygoid process to the pterygopalatine ganglion where parasympathetic fibers,
originating in a lacrimal nucleus in the brain stem, relay and reach the lacrimal gland through
the zygomatic branches of the maxillary nerve.
10. It is a branch of glossopharyngeal nerve. It leaves the inferior ganglion, ascends to the
tympanic cavity through the inferior tympanic canaliculus and divides in the cavity into
branches forming the tympanic plexus. This plexus supplies a branch to the greater
superficial petrosal branch of the facial nerve; branches to the mucosa of the tympanic cavity,
auditory tube and mastoid air cells and a branch to the lesser petrosal nerve. The lesser
petrosal nerve contains parotid secretomotor fibers. It enters a canal inferior to that for
tensor tympani, reaches the anterior surface of the petrous bone, via the foramen ovale to join
the otic ganglion.
11. The branches of the vagus nerve, they are differs, in origin and course, on the two sides. On
the right it arises from the vagus nerve anterior to the subclavian artery, curving backwards
below and then behind it to ascend obliquely to the side of the trachea behind the common
carotid artery. On the left, the nerve arises from the vagus nerve on the left of the aortic arch,
curves below it and ascends to the side of the trachea. On both sides the recurrent laryngeal
nerve ascends in or near a groove between the trachea and esophagus, passes closely related
to the medial surface of the thyroid gland, enters the larynx behind the cricothyroid joint. It
supplies all laryngeal muscles, except the cricothyroid; it communicates with the internal
laryngeal nerve, supplying sensory filaments to the laryngeal mucosa below the vocal folds.
Answer the Following Questions
1. A spinal nerve is a mixed nerve and composed of by an anterior (ventral) root and a posterior
(dorsal) root. The anterior root includes somatic and visceral motor fibers, the posterior root
includes somatic and visceral sensory fibers. Their distribution are as following :
(1) Somatic motor fibers innervate the skeletal muscles.
(2) Visceral motor fibers innervate the smooth muscles, cardiac muscle and glands.
(3) Somatic sensory fibers are distributed to the skin, skeletal muscles, tendons and joints.
(4) Visceral sensory fibers are distributed to the viscera, heart, blood vessels and glands.
2. The axillary nerve may be injured, because it winds round the surgical neck of the humerus. It
supplies teres minor and deltoid, and skin over the deltoid. Therefore, injury to the axillary
nerve results in paralysis of deltoid muscle and causes the "quadrate" shoulder.
3. The common peroneal nerve may be injured, because it winds around the neck of the fibula. It
divides into superficial and deep peroneal nerves. The superficial peroneal nerve supplies the
peroneal muscles and the skin on the lower part of the anterior surface of the leg, the dorsum of
the foot and toes. And the deep peroneal nerve supplies the muscles of the anterior group of the
leg and the first web of the foot. Therefore, its injury results in paralysis of the muscles in the
anterior and lateral groups of the leg. The eversion of the foot and dorsiflexion of the ankle
joint are lost. As a result, the opposing muscles cause the foot to be plantar flexed and inverted,
This is referred to as "foot-drop" or "talipes equinovarus". Sensation is also lost on the anterior
and lateral surfaces of the leg and dorsum of the foot.
4. The tiny olfactory nerves and bulbs can be injured as a result of head trauma. The location of
the olfactory bulb and tract, below the frontal lobe, predisposes them to compression from
frontal lobe tumors and olfactory groove meningiomas.
5. A person whose oculomotor nerve is damaged may have a drooping upper eyelid or dilated
pupil, or be unable to move the eyeball in the directions permitted by the four extrinsic muscles
innervated by this nerve.
6. Because the abducent nerve has a long intracranial course, it is often stretched when intracranial
pressure rises. If it is damaged, not only will the patient be unable to move the eyeball laterally,
but because of the lack of muscle tonus to the lateral rectus muscle, the eyeball will be pulled
medially.
7. (1) Chorda tympani leaves the facial nerve about 6mm above the stylomastoid foramen. It
contains efferent preganglionic parasympathetic (secretomotor) fibres which enter the
submandibular ganglion,
from which postganglionic fibres are
relayed to
the
submandibular and sublingual glands. However, most of its fibres are afferent from the mucosa
of the anterior two-thirds of the tongue, except the vallate papillae, and it constitutes the nerve
of taste for this lingual region. Before uniting with the lingual nerve, the chorda tympani is
joined by a small branch from the otic ganglion.
(2) Greater petrosal nerve passes forward and joins with a deep petrosal nerve from the
sympathetic carotid plexus to form the nerve of the pterygoid canal. This nerve passes through
the canal of the pterygoid process to the pterygopalatine ganglion where parasympathetic fibers,
originating in a lacrimal nucleus in the brain stem, relay and reach the lacrimal gland through
the zygomatic branches of the maxillary nerve.
(3) Stapedial nerve reaches the stapedius and supplies this muscle.
8. Although the vestibular and cochlear nerves are essentially independent, peripheral lesions often
produce concurrent clinical effects because of their close relationship. Hence, lesions of the
vestibulocochlear nerve may cause tinnitus (ringing or buzzing in ears), vertigo (dizziness, loss
of balance), and impairment or loss of hearing. Central lesions may involve either the cochlear
or vestibular divisions of the vestibulocochlear nerve.
9. Damage to the recurrent laryngeal nerve, which arises from the vagus nerve, can occur as a
result of invasion or compression by tumor or as a complication of thyroid surgery. It may be
accompanied by hoarseness or hypophonia but can be asymptomatic.
10. Injury to the hypoglossal nerve paralyzes the ipsilateral half of the tongue. After some time,
the tongue atrophies, making it shrunken and wrinkled. When the tongue is protruded, its tip
deviates toward the paralyzed side because of the unopposed action of the genioglossus on the
normal side of the tongue.
11. The oculomotor nerve is a motor nerve that arises from nuclei within the midbrain, pierces the
dura, and runs in the lateral wall of the cavernous sinus. It divides into superior and inferior
branches as it passes through the superior orbital fissure in the orbit. The superior branch
innervates the superior rectus, which moves the eyeball superiorly, and the levator palpebrae
superioris, which raises the upper eyelid. The inferior branch innervates the medial rectus,
inferior rectus, and inferior obliquus for medial, inferior, and superolateral movement of the
eyeball, respectively. In addition, fibers from the inferior branch of the oculomotor nerve also
carry presynaptic autonomic fibers from the accessory nucleus of oeulomotor nerve
(Edinger-Westphal's nucleus) to the ciliary ganglion, where the parasympathetic fibers
synapse. Postsynaptic fibers from this ganglion pass to the eyeball in the short ciliary nerves
and supply the ciliary muscle (accommodation of lens) and sphincter pupillae (constriction of
pupil).
12. Three large branches arise from the trigeminal ganglion: the ophthalmic nerve enters the orbit
through the superior orbital fissure, the maxillary nerve extends through the foramen
rotundum, and the mandibular nerve passes through the foramen ovale. The smaller motor
root consists of motor fibers of the trigeminal nerve that accompany the mandibular nerve
through the foramen ovate and innervate the muscles of mastication and certain muscles in
the floor of the mouth.
(1) Ophthalmic nerve
The ophthalmic nerve consists of sensory fibers from the anterior half of the scalp, skin of the
forehead, upper eyelid, surface of the eyeball, lacrimal gland, side of the nose, and upper
mucosa of the nasal cavity.
1) Frontal nerve divided about midway between the apex and base of the orbit into a small
supratrochlear and a large supraorbital branch, the former supplies the skin of upper eyelid
and the skin near the midline, while the latter supplies the skin of the forehead.
2) Lacrimal nerve enters and supplies the lacrimal gland and the adjoining conjunctiva. It
pierces the orbital septum and ends in the upper eyelid, where it joins filaments of the facial
nerve.
3) Nasociliary nerve: The infratrochlear nerve branches from the nasociliary nerve and
supplies the skin of the eyelids and the side of the nose above the medial angle of the eye, the
conjunctiva, lacrimal sac and lacrimal caruncle. The anterior ethmoidal nerve and posterior
ethmoidal nerve mainly supply the ethmoidal and sphenoidal sinuses.
(2) Maxillary nerve
The maxillary nerve is composed of sensory fibers from the lower eyelid, lateral and inferior
mucosa of the nasal cavity, palate and portions of the pharynx, teeth and gums of the upper
jaw, upper lip, and skin of the cheek.
1) Infraorbital nerve passes through a groove, a canal and a large infraorbital foramen before
appearing on the face; here it branches to supply the skin of the maxillary process, namely
the lower eyelid, the side of the nose and the upper lip.
2) Zygomatic nerve gives off the secretory branch to the lacrimal nerve for the lacrimal gland,
and divides into two branches which perforate the zygomatic bone to supply the cheek and
temple.
3) Superior alveolar nerves. Before entering the inferior orbital fissure the maxillary nerve
gives off posterior superior alveolar branches; in the maxilla it joins the anterior and middle
superior alveolar branches, which arise in the infraorbital canal, to form a superior alveolar
plexus for the teeth of the upper jaw.
4) Pterygopalatine nerves arise from the pterygopalatine ganglion.
(3) Mandibular nerve
Sensory fibers of the mandibular nerve transmit impulses from the teeth and gums of the
lower jaw, anterior two-thirds of the tongue (no taste), mucosa of the mouth, auricle of the ear,
and lower part of the face.
1) Auriculotemporal nerve supplies the upper portions of the auricle and the temple. It is
responsible for the referral of pain from the temporomandibular joint.
2) Buccal nerve supplies the skin over the anterior part of the buccinator and the buccal
mucous membrane, together with the posterior part of the buccal gingivae adjacent to the
second and third molar teeth.
3) Lingual nerve mediates common sensation from the anterior two-thirds of the tongue.
4) Inferior alveolar nerve runs forward in the mandibular canal, sending a branch of supply to
each tooth, to finally the mental nerve emerging from the mental foramen and turn upward to
supply the lower lip.
5) Nerves for masticatory muscles supply the medial pterygoid, masseteric, deep temporal,
and the lateral pterygoid muscles.
13. The complete destruction of the facial nucleus itself or its branchial efferent fibers (facial
nerve proper) paralyzes all ipsilateral face muscles; this is equivalent to a peripheral facial
lesion. Peripheral facial paralysis (Bell's palsy) can occur as an idiopathic condition, but it is
seen as a complication of diabetes and can occur as a result of tumors, sarcoidosis. When an
attempt is' made to close the eyelids, the eyeball on the affected side may turn upward.
The symptoms and signs depend on the location of the lesion. A lesion in or outside the
stylomastoid foramen results in a flaccid paralysis (lower-motor-neuron type) of all the
muscles of facial expression in the affected side; this can occur from a stab wound or from
swelling of the parotid gland (e. g,. as seen in mumps). A lesion in the facial canal involving
the chorda tympani nerve results in reduced salivation and loss of taste sensation from the
ipsilateral anterior two-thirds of the tongue. A lesion higher up in the canal can paralyze the
stapedius muscle. A lesion in the middle ear involves all components of nerve Ⅶ, whereas a
tumor in the internal auditory canal (e. g,. a schwannoma) can cause dysfunction of nerves
Ⅶ and Ⅷ.
14. The optic nerve conducts impulses from the photoreceptors (rods and cones) in the retina of
the eye.
The ophthalmic nerve consists of sensory fibers from the eye.
The oculomotor nerve innervates the levator palpebrae superioris, the superior rectus, the
medial rectus, the inferior rectus, and the inferior obliquus. The parasympathetic fibers in
oculomotor nerve supply the ciliary muscle (accommodation of lens) and sphincter pupillae
(constriction of pupil).
The trochlear nerve supplies the superior obliquus.
The abducent nerve supply the lateral rectus.
15. The tongue receives its sensory innervation through multiple pathways: Three cranial nerves
contain taste fibers (nerve Ⅶ for anterior two-thirds of tongue; nerve Ⅸ for posterior
one-third of tongue; nerve Ⅹ for epiglottis), and the general sensory afferent fibers are
mediated by nerve Ⅴ.
The hypoglossal nerve innervates the muscles of the tongue.
16. (1) Branches in the neck
1) Superior laryngeal nerve issues from the inferior ganglion. It descends alongside the
pharynx, first posterior, and then medial to the internal carotid artery, and divides into the
internal and external laryngeal nerves. The internal laryngeal nerve is sensory to the laryngeal
mucosa down to the level of the vocal folds. It supplies the mucosa of the pharynx, the
epiglottic vallecula, laryngeal vestibule, the aryepiglottic fold and the mucosa on the back of
the arytenoid cartilage. The external laryngeal nerve is smaller than the internal, descends
posterior to the sternothyroid with the superior thyroid artery, supplying the cricothyroid.
2) Cervical cardiac branches arise from the vagus nerve at superior and inferior cervical
levels. The small superior branches join sympathetic cardiac branches and reach the deep
cardiac plexus. The inferior branches arise from the root of the neck; the right branches join
the deep cardiac plexus, the left branches join the superficial cardiac plexus.
3) Auricular branch arises from the superior ganglion, containing somatic afferent nerve
fibers. It distributed to the skin of part of the external acoustic meatus and adjoining part of
the outer surface of the tympanic membrane.
4) Pharyngeal branches emerge from the inferior ganglion and consist chiefly of filaments
from the cranial accessory nerve. They are divided into numerous filaments which join
branches of the sympathetic trunk and glossopharyngeal and external laryngeal nerves to
form a pharyngeal plexus by which vagal fibers are distributed to the pharyngeal and palatal
muscles, except the tensor veli palatini.
5) Meningeal branches start from the superior ganglion and are distributed to the dura mater
in the posterior cranial fossa.
(2) Branches in the thorax
1) Recurrent laryngeal nerve differs, in origin and course, on the two sides. On the right it
arises from the vagus nerve anterior to the subclavian artery, curving backwards below and
then behind it to ascend obliquely to the side of the trachea behind the common carotid artery.
On the left, the nerve arises from the vagus nerve on the left of the aortic arch, curves below
it and ascends to the side of the trachea. On both sides the recurrent laryngeal nerve ascends
in or near a groove between the trachea and esophagus, passes closely related to the medial
surface of the thyroid gland, enters the larynx behind the cricothyroid joint. It supplies all
laryngeal muscles, except the cricothyroid; it communicates with the internal laryngeal nerve,
supplying sensory filaments to the laryngeal mucosa below the vocal folds.
2) Bronchial and esophageal branches arise from both vagus nerves. The bronchial branches
form the pulmonary plexus with sympathetic rami, supplying the constrictor muscles and
other tissues of the pulmonary tree. The esophageal branches form the esophageal plexus,
supplying the esophagus and the back of the pericardium.
(3) Branches in the abdomen
1) Anterior gastric branches arise from the left vagus nerve and supply the anterior surface
of the stomach.
2) Hepatic branches arise from the anterior vagal trunks join the hepatic plexus and supply
the liver.
3) Posterior gastric branches arise from the right vagus nerve and supply the posterior
surface of the stomach.
4) Celiac branches arise from the posterior vagal trunks join the celiac plexus.
17. Lesions of the vagus nerve may be intramedullary or peripheral. Vagus nerve lesions near the
skull base often involve the glossopharyngeal and accessory nerves and sometimes the
hypoglossal nerve as well. Complete bilateral transection of the vagus nerve is fatal.
Unilateral lesions of the vagus nerve, within the cranial vault or dose to the base of the skull,
produce widespread dysfunction of the palate, pharynx, and larynx. The soft palate is weak
and may be flaccid so the voice has a nasal twang. Weakness or paralysis of the vocal cord
may result in hoarseness. There can be difficulty in swallowing, and cardiac arrhythmias may
be present.
Damage to the recurrent laryngeal nerve, which arises from the vagus nerve, can occur as a
result of invasion or compression by tumor or as a complication of thyroid surgery. It may be
accompanied by hoarseness or hypophonia but can be asymptomatic.
(Gao Xiulai, Su Baogui, Pan Sanxiang, Chen Mingfeng, Ding Wenlong)
Multiple Choice Questions
A1
1. Which of the following is the first-order neuron for the conscious proprioceptive pathway?
A. spinal ganglion
B. gracile nucleus
C. cuneate nucleus
D. ventral posterolateral nucleus of thalamus
E. ventral posteromedial nucleus of thalamus
2. Which of the following is the second-order neuron for the conscious proprioceptive pathway?
A. spinal ganglion
B. gracile and cuneate nuclei
C. nucleus proprius
D. ventral posterolateral nucleus
E. ventral posteromedial nucleus
3. Which of the following is the third-order neuron for the conscious proprioceptive pathway of
trunk and limbs?
A. spinal ganglion
B. nucleus gracilis
C. nucleus cuneatus
D. ventral posterolateral nucleus
E. ventral posteromedial nucleus
4. The conscious proprioceptive pathway mediates except
.
A. fine touch
B. stereognosis
C. proprioception
D. vibratory sensations
E. pain and temperature
5. Which of following is the second order fiber for the conscious proprioceptive pathway?
A. fasciculus gracilis
B. fasciculus cuneatus
C. medial lemniscus
D. dorsolateral fasciculus
E. lateral lemniscus
6. Which of the following is the first-order neuron for the unconscious deep sensory pathway?
A. spinal ganglion
B. nucleus proprius
C. gracile and cuneate nuclei
D. ventral posterolateral nucleus
E. ventral posteromedial nucleus
7. Which of the following is the second-order neuron for the unconscious deep sensory pathway?
A. spinal ganglion
B. gracile and cuneate nuclei
C. thoracic nucleus and posterior horn
D. ventral posterolateral nucleus
E. ventral posteromedial nucleus
8. Which of the following is the third-order neuron for the unconscious deep sensory pathway?
A. spinal ganglion
B. neuron of the cerebellar cortex
C. nucleus proprius
D. ventral posterolateral nucleus
E. ventral posteromedial nucleus
9. What does the lateral spinothalamic tract mediate for the limbs and trunk?
A. fine touch
B. stereognosis
C. proprioception
D. vibratory sensations
E. pain and temperature
10. Which of following are the second order fibers for the spinothalamic tract?
A. fasciculus gracilis and cuneatus
B. medial lemniscus
C. dorsolateral fasciculus
D. lateral lemniscus
E. lateral and anterior spinothalamic tracts
11. Which of the following is the first-order neuron for the superficial sensory pathway of head
and face?
A. spinal ganglion
B. trigeminal ganglion
C. spinal nucleus of trigeminal nerve
D. ventral posterolateral nucleus
E. ventral posteromedial nucleus
12. Which of the following is the second-order neuron for the superficial sensory pathway of head
and face?
A. spinal ganglion
B. trigeminal ganglion
C. spinal nucleus of trigeminal nerve
D. ventral posterolateral nucleus
E. ventral posteromedial nucleus
13. Which of the following is the third-order-neuron for the superficial sensory pathway of head
and face?
A. spinal ganglion
B. trigeminal ganglion
C. spinal nucleus of trigeminal nerve
D. ventral posterolateral nucleus
E. ventral posteromedial nucleus
14. What does the superficial sensory pathway mediate for head and face?
A. joint position
B. stereognosis
C. proprioception
D. vibratory sensations
E. pain and temperature
15. Which Of the following is the first-order neuron for the visual pathway?
A. rod cell
B. cone cell
C. bipolar cell
D. ganglion cell
E. lateral geniculate body
16. Which of the following is the second-order neuron, for the visual pathway?
A. rod cell
B. cone cell
C. bipolar cell
D. ganglion cell
E. lateral geniculate body
17. Which of the following is the third-order neuron for the visual pathway?
A. rod cell
B. cone cell
C. bipolar cell
D. ganglion cell
E. lateral geniculate body
18. Which of the following are the photoreceptors for the visual system?
A. rod cell and cone cell
B. bipolar cell
C. ganglion cell
D. medial geniculate body
E. lateral geniculate body
19. Which of following is the afferent nerve for the pupillary light reflex?
A. oculomotor nerve
B. optic nerve
C. abducent nerve
D. trigeminal nerve
E. trochlear nerve
20. Which of following is the central area for the papillary light reflex?
A. lateral geniculate body
B. medial geniculate body
C. pretectal area
D. superior colliculus
E. inferior colliculus
21. Which of following arises from the upper two-thirds of the primary somatic motor area?
A. corticospinal tract
B. corticobulbar tract
C. tectospinal tract
D. rubrospinal tract
E. vestibulospinal tract
22. Which of following arises from the lower one-third of the primary somatic motor cortex?
A. corticospinal tract
B. corticobulbar tract
C. tectospinal tract
D. rubrospinal tract
E. vestibulospinal tract
23. Concerning the corticospinal tract, which is wrong?
A. The corticospinal tract arises from the upper two-thirds of precentral gyrus.
B. The corticospinal tract passes through the posterior limb of internal capsule.
C. The corticospinal tract crosses at the caudal end of the medulla.
D. After crossing, motor fibers descend into the lateral corticospinal tract.
E. Upper motor neuron lesions result in atrophy and fasciculation.
24. Concerning the eorticobulbar tract, which is wrong?
A. The corticobulbar tract arises from the lower one-third of precentral gyms.
B. The corticobulbar tract passes through the genu of internal capsule.
C. The corticobulbar tracts decussated at the midplane in the brainstem, whereas others have
not.
D. Upper portion of face receives bilateral corticobulbar tract.
E. The lesion of the upper motor neuron, all the facial muscles of one side will be paralyzed.
25. What is not included in the conscious proprioceptive pathway for trunk and limbs?
A. the fasciculus gracilis
B. the fasciculus cuneatus
C. the medial lemniscus
D. the ventral posterolateral nucleus of thalamus
E. the lateral lemniscus
26. What is included in the spinothalamic pathway for trunk and limbs?
A. the fasciculus gracilis
B. the fasciculus cuneatus
C. the medial lemniscus
D. the lateral spinothalamic tract
E. the lateral lemniscus
27. What is not included in the visual pathway?
A. optic nerve
B. optic chiasm
C. optic tract
D. medial geniculate nucleus
E. optic radiation
28. Concerning the visual pathway, which is wrong?
A. Fibers from the nasal half of the retina decussate in the optic chiasm.
B. Each lateral geniculate body receives information from the homonymous (left or right)
halves of both eyes.
C. Lesion of the optic tract affects both sides of the visual field for one eye.
D. The primary visual cortex is at the calcarine cortex in the occipital lobe.
E. Optic radiation passes through the posterior limb of internal capsule.
29. Axons of the corticospinal tract synapse at
A. motor nuclei of cranial nerves
B. motor neurons in ,the anterior horns of the spinal cord
C. gracile nucleus
D. nucleus proprius
E. spinal ganglion
30. Axons of corticobulbar tract synapse at
A. motor nuclei of cranial nerves
B. motor neurons in the anterior horns of the spinal cord
C. gracile nucleus
D. nucleus proprius
E. spinal ganglion
31. Where does the lateral spinothalamic tract cross?
A. in the medulla
B. in the pons
C. in the midbrain
D. in the spinal cord
E. in the telencephalon
32. The pyramidal system consists of descending motor tracts, which of the following except?
A. spinocerebellar tract
B. corticobulbar tract
C. lateral corticospinal tract
D. anterior corticospinal tract
E. corticonuclear tract
33. Which of the following statements is false?
A. The conscious proprioceptive pathway carries highly localized joint sensation, two- point
discrimination, and vibration sense from receptors to the cortex.
B. The axons, with their first-order neurons in the spinal ganglion, reach the CNS through the
posterior roots of spinal nerves and collect sensory information from different body areas
for the conscious proprioceptive pathway.
C. Axons from the posterior roots of spinal nerves ipsilaterally ascend within the fasciculus
gracilis or the fasciculus cuneatus in the spinal cord.
D. The fasciculus gracilis transmits deep sensation from the lower half of the body. This
includes the afferent fibers entering the posterior funiculus from the sacral to midthoracic
section of the spinal cord.
E. The fasciculus cuneatus fibers carry "crude" sensation from the upper body and enter the
spinal cord above the midthoracic level.
34. Damage to the pyramidal cells of the cerebral cortex would directly affect
.
A. perception of pain
B. voluntary motor activity
C. the ability to see
D. the ability to hear
E. the ability to read
35. Which of the following statements is false?
A. The visual pathways also contain three orders of neurons.
B. The first-order neurons, the bipolar cells in the retina, synapse the visual receptors, rod
cells and cone cells.
C. Bipolar cells synapse within the layer of ganglion cells, the second-order neurons.
D. Axons from ganglion cells converge on the optic disc form the optic nerve, then the optic
nerve course through the optic canal of the skull to form the optic chiasm.
E. The optic tract fibers synapse with third-order neurons in the medial geniculate body of the
thalamus, whose axons project ipsilaterally by way of the optic radiation passing through
the posterior limb of the internal capsule to the calcarine cortex in the occipital lobe.
36. Which of the following statements is false?
A. The corticospinal tract travels through the posterior limb of internal capsule of the
forebrain, then descends through the intermediate 3/5 of the cerebral crus of the midbrain,
later it runs through the basilar part of the pons and continues to the pyramids of the
medulla oblongata
B. About 75% ~ 90% of the corticospinal tract crosses pyramidal decussation at the caudal
end of the medulla.
C. After crossing, the lateral corticospinal tract descends in the lateral funiculus of the spinal
cord. These fibers terminate on the interneuron and lower motor neurons in the ipsilateral
posterior horns of the spinal cord and mainly control musculature of the limbs.
D. The corticonuclear tracts extend through the genu of the internal capsule to the brainstem.
E. The corticobulbar tracts terminate the bilateral motor nuclei of the cranial nerves
(oculomotor, trochlear, motor nucleus of trigeminal nerve, ambiguous, accessory nuclei
and superior part of the facial nucleus), and the contralateral hypoglossal nucleus and the
inferior part of the facial nucleus.
37. What is not the clinical feature of the lesion of the lower motor neurons?
A. muscle tone decreased
B. myotatic (deep) reflex decreased or absent
C. cutaneous (superficial) reflex absent
D. pathological reflex (e. g. , Babinski sign) existed
E. muscle size pronounced atrophy
38. Damage to the oculomotor nerve would affect
.
A. bitemporal hemianopsia
B. monocular blindness
C. nasal hemianopsia
D. homonymous hemianopsia
E. pupillary light reflexes
39. Damage to the medial lemniscus would directly affect
.
A. perception of joint position, two-point discrimination, and vibration sensation
B. voluntary motor activity
C. the ability to see
D. the ability to hear
E. the ability to read
40. Damage to the lateral spinothalamic tract would directly affect
.
A. perception of pain and temperature
B. voluntary motor activity
C. the ability to see
D. the ability to hear
E. the ability to read
41. Damage to the anterior spinothalamic tract would directly affect
.
A. perception of pain and temperature
B. perception of crude touch and pressure
C. the ability to see
D. the ability to hear
E. the ability to read
42. Damage to the optic tract would produce
.
A. bitemporal hemianopsia
B. monocular blindness
C. nasal hemianopsia
D. homonymous hemianopsia
E. all of the above
43. Damage to the midplane of optic chiasm would produce
.
A. bitemporal hemianopsia
B. monocular blindness
C. nasal hemianopsia
D. homonymous hemianopsia
E. all of the above
44. Damage to the optic nerve would produce
.
A. bitemporal hemianopsia
B. monocular blindness
C. nasal hemianopsia
D. homonymous hemianopsia
E. all of the above
45. Damage to the lateral edges of the optic chiasm would produce
.
A. bitemporal hemianopsia
B. monocular blindness
C. nasal hemianopsia
D. homonymous hemianopsia
E. all of the above
46. Damage to the medial geniculate body would produce
.
A. bitemporal hemianopsia
B. monocular blindness
C. nasal hemianopsia
D. homonymous hemianopsia
E. none of the above
47. Damage to the lateral geniculate body would produce
.
A. bitemporal hemianopsia
B. monocular blindness
C. nasal hemianopsia
D. homonymous hemianopsia
E. all of the above
48. Damage to the optic radiation would produce
.
A. bitemporal hemianopsia
B. monocular blindness
C. nasal hemianopsia
D. homonymous hemianopsia
E. all of the above
A2
1. A patient complains of unsteadiness. Examination shows a marked diminution of position sense,
vibration sense, and stereognosis of all extremities. He is unable to stand without wavering for
more than a few seconds when his eyes are closed. There are no other abnormal findings. The
lesion most likely involves the
.
A. anterior spinothalamic tracts, bilaterally
B. inferior cerebellar peduncles, bilaterally
C. fasciculi gracilis and cuneatus of the spinal cord, bilaterally
D. lateral spinothalamic tracts, bilaterally
E. corticospinal tracts
2. A building worker complained to his physician about puncture wounds for the hack, alter injury,
the position sense in all lower limb is lost, further examination revealed that fine touch and
position sense is lost below the 12 thoracic segment. There are no other abnormal findings. He is
unable to stand when his eyes are closed. The lesion most likely involves the
.
A. corticospinal tract
B. fasciculus gracilis
C. fasciculus cuneatus
D. thalamocortical tract
E. anterior corticospinal tract
3. A 41-year-old woman complained of gradual deficit of pain and temperature in upper
limbs for more than four yeas. There are no other abnormal findings. Diagnosis:
syringomyelia, the lesion most likely involves the
.
A. fasciculus gracilis
B. fasciculus cuneatus
C. anterior spinothalamic tract
D. anterior white commissure
E. anterior gray commissure
4. As a result of calcification of the internal carotid artery, the calcification impinges upon the
lateral half of the light optic nerve prior to its entrance to the brain d a 55-year-old man,
resulting in certain visual deficits. The most likely visual deficits will be
.
A. total blindness of the right eye
B. right nasal hemianopsia
C. right homonymous hemianopsia
D. right bitemporal hemianopsia
E. left homonymous hemianopsia
5. It is discovered that a 35-year-old woman has a tumor pressing on the base of the brain where it
is impinging upon the optic chiasm. She discovers that her field of vision is now seriously
affected. The defect present in tiffs individual is
.
A. total blindness of both eyes
B. bitemporal hemianopsia
C. right homonymous hemianopsia
D. binasal hemianopsia
E. left homonymous hemianopsia
6. A 49-year-old woman developed a severe headache for the last two days, and accompanied by
vomiting and bitemporal hemianopsia for the visual field, CT scanning shows a pituitary tumor,
which of the following structure for lesion owing to a pituitary tumor results in bitemporal
hemianopsia for the visual field?
A. ophthalmic nerve
B. optic chiasma
C. trochlear nerve
D. optic nerve
E. oculomotor nerve
7. A routine magnetic resonance imaging (MRI) reveals the presence of a tumor situated in the left
optic tract proximal to the lateral geniculate body. The patient complained of having a reduction
in his field of vision. The likely visual deficit can be characterized as
.
A. total blindness of the left eye
B. bitemporal hemianopsia
C. right homonymous hemianopsia
D. left homonymous hemianopsia E. right nasal hemianopsia
8. A routine eye examination reveals the presence of inflammation limited to the left optic disk,
probably due to neuritis of this region. The likely visual deficit resulting from this disorder
is
.
A. total blindness of the left eye
B. left homonymous hemianopsia
C. left heteronymous hemianopsia
D. left enlargement of the blind spot
E. left nasal hemianopsia
9. Examination of a patient revealed a drooping left eyelid, together with weakness of adduction
and elevation of the left eye, loss of the papillary light reflex in the left eye, and weakness of the
limbs and lower facial muscle on the right side. A single lesion most likely to produce all these
signs would be located in the
.
A. left pontomedullary junction
B. left cerebral peduncle
C. pons on the left side
D. medulla on the left side
E. spinal cord on the left side
10. Paralysis of the right side of the lower face, right spastic paralysis of the limbs, deviation of
the tongue to the right with no atrophy, and no loss of taste from any region of the tongue will
likely result from a lesion of the
.
A. right internal capsule
B. left internal capsule
C. left pontine tegmentum
D. base of the medulla on the right side
E. base of the medulla on the left side
B1
A. the spinal cord
B. the medulla
C. the pons
D. the midbrain
E. the telencephalon
1. The second-order fibers from the gracile and cuneate nuclei travel anteromedially and cross
the midplane just above the pyramid in
.
2. About 75% to 90% of the corticospinal tract crosses pyramidal decussation at
.
A. medial lemniscus
B. lateral lemniscus
C. fasciculi gracilis and cuneatus.
D. anterior spinothalamic tract
E. lateral spinothalamic tract
3. After crossing, the internal arcuate fibers, the second-order fibers from the gracile and cuneate
nuclei form
.
4. The second order fibers ascend from the cochlear nuclei on both sides, the crossing fibers pass
through the trapezoid bodies, and some of them synapse in the superior olivary nuclei. The
ascending fibers course in
.
A. bitemporal hemianopsia
B. monocular blindness
C. nasal hemianopsia
D. right homonymous hemianopsia
E. left homonymous hemianopsia
5. Damage to the left optic tract would produce
.
6. Damage to the midplane of optic chiasm would produce
.
7. Damage to the optic nerve would produce
.
A. perception of joint position, two-point discrimination, and vibration
B. perception of pain and temperature
C. perception of crude touch and pressure
D. voluntary motor activity
E. the ability to read
8. Damage to the anterior spinothalamic tract would directly affect
.
9. Damage to the lateral spinothalamic tract would directly affect
.
10. Damage to the pyramidal cells of the cerebral cortex would directly affect
.
True or False Questions
1. The conscious proprioceptive pathway carries highly localized joint sensation, two-point
discrimination, and vibration sense from receptors to the cortex.
(
)
2. The axons, with their first-order neurons in the spinal ganglion, reach the CNS through the
posterior roots of spinal nerves and collect sensory information from different body areas for
the conscious proprioceptive pathway.
(
)
3. Axons from the posterior roots of spinal nerves ascend ipsilaterally within the fasciculus
gracilis or the fasciculus cuneatus in the spinal cord.
(
)
4. The fasciculus gracilis transmits pain sensation from the lower half of the body. This includes
the afferent fibers entering the posterior funiculus from the sacral to midthoracic section of
the spinal cord.
(
)
5. The fasciculus cuneatus fibers carry "crude" sensation from the upper body and enter the
spinal cord above the midthoracic level.
(
)
6. The fasciculi gracilis and cuneatus ascend ipsilaterally the spinal cord posterior funiculus
toward the brainstem and terminate at the gracile and cuneate nuclei.
(
)
7. The internal arcuate fibers, the second order fibers from the gracile and cuneate nuclei, travel
ventromedially and cross the midplane just above the pyramid in the medulla.
(
)
8. After crossing, the internal arcuate fibers, the second-order fibers from the nuclei gracilis and
cuneatus form the lateral lemniscus.
(
)
9. The medial lemniscus fibers ascend along the midplane and posterior to pyramidal tract through
the medulla.
(
)
10. The medial lemniscus fibers are horizontally arranged in the pontine tegmentum. Then they
migrate dorsolaterally in the midbrain to enter the ventral posteromedial nucleus of thalamus.
(
)
11. Any lesion implicating fibers at the spinal or low medullary level will result in the loss of
two-point discrimination, position sense and vibration sense in the half-body contralateral to
the site of interruption.
(
)
12. The fibers from spinal and pontine nuclei of trigeminal nerve, second-order neurons, cross
midplane and form contralateral trigeminal lemniscus terminating the ventral posterolateral
nucleus of thalamus, the third-order neurons.
(
)
13. The lateral spinothalamic tract mediates the sensations of pain and temperature.
(
)
14. The anterior spinothalamic tract mediates crude touch and pressure.
(
)
15. The spinothalamic fibers terminate on the third-order neurons in the ventral posteromedial
nucleus of thalamus.
(
)
16. Thalamocortical fibers carry the information from the thalamus to the sensory cortex through
the anterior limb of the internal capsule, then project to the upper two-thirds of the postcentral
gyrus located in the parietal lobe and the posterior part of the paracentral lobule.
(
)
17. The visual pathways also contain three orders of neurons.
(
)
18. The first-order neurons, the bipolar cells in the retina, synapse the visual receptors, rod cells
and cone cells.
(
)
19. Bipolar cells synapse within the layer of ganglion cells, the second-order neurons. (
)
20. Axons from ganglion cells converge on the optic disc and form the optic nerve, then the optic
nerve course through the optic canal of the skull to form the optic chiasm.
( )
21. The optic tract fibers synapse with third-order neurons in the medial geniculate body of the
thalamus, whose axons project ipsilaterally by way of the optic radiation passing through the
posterior limb of the internal capsule to the calcarine cortex in the occipital lobe.
(
)
22. The pretectal area is the central portion of the pupillary light reflex.
(
)
23. Efferent impulses pass along parasympathetic fibers of the oculomotor nerve to the orbit
where they synapse in the ciliary ganglion. Postganglionic fibers pass to the eyeball to supply
sphincter pupillae, which reduces the size of the pupil when it contracts.
(
)
24. The corticospinal tract travels through the posterior limb of internal capsule of the
telencephalon, and then descends through the intermediate 3/5 of the cerebral peduncle of the
midbrain. Later it runs through the basilar part of the pons and continues to the pyramid of
the medulla in the medulla.
(
)
25. About 75%~ 90% of the corticospinal tract crosses pyramidal decussation at the caudal end of
the medulla.
(
)
26. After crossing, the lateral corticospinal tracts descend in the lateral funiculus of the spinal cord.
These fibers terminate on the interneuron and lower motor neurons in the ipsilateral posterior
horns of the spinal cord.
(
)
27. The corticonuclear tracts extend through the posterior limbs of the internal capsule to the
midbrain.
(
)
28. The corticobulbar tracts terminate the bilateral motor nuclei of the cranial nerves (oculomotor,
trochlear, motor nucleus of trigeminal nerve, ambiguous, accessory nuclei and superior part
of the facial nucleus), and the contralateral hypoglossal nucleus and the inferior part of the
facial nucleus.
(
)
29. The first-order central processes, with cell bodies in trigeminal ganglion, geniculate ganglion,
superior ganglion of glossopharyngeal nerve and vagus nerve, enter the brain stem. They turn
downward, terminating in the spinal nuclei of the trigeminal nerve mediating pain and
temperature sensation and in the pontine nuclei of the trigeminal nerve mediating touch
sensation.
(
)
Explanation of Terms
1. upper motor neurons
2. lower motor neurons
3. corticospinal tract
4. corticobulbar tract
5. visual field
6. bitemporal hemianopsia
7. nasal hemianopsia
8. homonymous hemianopsia
9. pupillary light reflex
10. optic radiation
11. medial lemniscus
12. fasciculus gracilis
13. fasciculus cuneatus
14. lateral and anterior spinothalamic tract
15. trigeminal lemniscus
Answer the Following Questions
1. Discuss three-order neurons of the conscious proprioceptive pathway for trunk and limbs.
2. What side of the body will lose position sense and two point discriminative touch after a left
pontine lesion involving medial lemniscus? Why?
3. Discuss three-order neurons of the spinothalamic tract.
4. Describe functions of anterior and lateral spinothalamic tracts.
5. Discuss three-order neurons of the superficial sensory pathway of head and face.
6. Describe the upper motor neurons and lower motor neurons for the pyramidal system.
7. Describe clinical features of the upper motor neurons lesion.
8. Describe clinical features of the lower motor neurons lesion.
9. Describe the neuronal components and functions of the conscious proprioceptive pathway.
10. Specially discuss the visual field representation on the optic nerve, optic chiasm, lateral
geniculate body, and visual cortex.
11. Describe the pathway of pupillary light reflex.
12. Outline course of pyramidal tract from motor cortex to spinal cord; differentiate between
course and decussation point of the corticospinal and corticobulbar tracts.
ANSWERS
Multiple Choice Questions
A1
1. A
2. B 3. D 4. E 5. C 6. A 7. C 8. B
9. E
10. E 11. B 12. C
13. E 14. E 15. C 16. D
17. E 18. A 19. B 20. C 21. A
22. B.
23.
E
24. E 25. E 26. D
27. D
28. C
29. B
30. A
31. D
32. A 33. E 34. B 35. E 36. C 37. D 38. E 39. A
40. A
41. B
42. D
43.
A
44. B
45. C
46. E
47. D
48. D
A2
1. C
2. C
3. D
4. B
5. B
6. B
7. C
8. D
9. B
10. B
B1
1. B 2. B 3. A 4. B
5. D 6. A
7. B
8. C
9. B
10. D
True or False Questions
1. T 2. T 3. T 4. F 5. F 6. T
7. T
8. F 9. T
10. F 11. F 12. F
13. T 14. T 15. F 16. F 17. T 18. T 19. T 20. T 21. F 22. T 23. T
24. T 25. T 26. F 27. F 28. T 29. T
Explanation of Terms
1. The upper motor neurons are composed of the giant pyramidal cells (Betz cells) and other
pyramidal cells in the precentral gyrus and anterior part of the paracentral lobule, and their
axons (the corticonuclear tract and the corticospinal tract).
2. The lower motor neurons include the cranial nerve nuclei of the brain stem and motor nuclei of
the anterior horn of the spinal cord, and their axons (cranial nerves and spinal nerves).
3. The corticospinal tract originates the pyramidal cells of the superior and middle parts of the
precentral gyrus and anterior part of the paracentral lobule. It travels through the posterior limb
of internal capsule of the telencephalon, and then descends through the intermediate 3/5 of the
cerebral peduncles of the midbrain. Later it runs through the basilar part of the pons and
continues to the pyramids in the medulla oblongata. About 75% to 90% of the corticospinal
tract crosses pyramidal decussation at the caudal end of the medulla oblongata. After crossing,
these fibers descend into the lateral corticospinal tract in the lateral funiculus of the spinal cord.
These fibers terminate on the interneuron and lower motor neurons in the ipsilateral anterior
gray horns of the spinal cord, and mainly control musculature of the limbs. About 10% to 15%
of the corticospinal tract does not cross in the pyramidal decussation but descends in the
anterior funiculus of the spinal cord (the anterior corticospinal tract). These fibers cross or
uncross above the level of mid-thoracic segments, close to the anterior horns of spinal cord, and
control the skeletal muscles of trunk. Up to 3 % of the decending fibers in the lateral
corticospinal tract are uncrossed. The ipsilateral decending projections mainly control
musculature of trunk.
4. Some axons of upper motor neurons (the pyramidal cells in the inferior part of the precentral
gyrus) that conduct impulses for the control of skeletal muscles in the head extend through the
genu of the internal capsule to the midbrain, where they join the corticobulbar tracts in the right
and left of cerebral peduncles. Some of the axons in the corticobulbar tracts decussated at the
midplane in the brainstem, whereas others have not. The axons terminate the bilateral motor
nuclei of the cranial nerves (nucleus of oculomotor nerve, nucleus of trochlear nerve, motor
nucleus of trigeminal nerve, nucleus ambiguus, accessory nucleus, superior part of the facial
nucleus), and the contralateral hypoglossal nucleus and the inferior part of the facial nucleus.
5. The visual field is the area located external to the eyes. The visual field for each is divided into
two half fields, the nasal and the temporal halves. Each of these for each eye is also divided into
upper and lower quadrants. Retinal representation of the visual field for each eye is also divided
into nasal and temporal halves, which are further divided into upper and lower quadrants. The
image in the visual field image is projected to the retina in reversed and inverted form. Light
rays from the temporal half of the visual field project to the nasal half of the retina, and rays
from the top of the object strike the lower retina, and rays from the bottom of the object
strike the upper retina.
6. A chiasmatic lesion (often owing to a pituitary tumor or a lesion around the sella turcica) can
injury the decussating axons of retinal ganglion cells within the optic chiasm. These axons
originate from the nasal halves of the two retinas. Thus, this type of lesion produces bitemporal
hemianopsia, characterized by blindness in the lateral or temporal half of the visual field for
each eye.
7. A pathology encroaching on one of the lateral edge of the optic chiasm selectively interrupts the
fibers from the ipsilateral temporal portion of the retina, resulting in nasal hemianopsia in the
corresponding eye.
8. Lesions behind the optic chiasm cause a field defect in the temporal field of one eye, together
with a field defect in the nasal (medial) field of the other eye. The result is a homonymous
hemianopsia in which the visual field defect is on the side opposite the lesion.
9. Light shone on the retina of one eye causes both pupils to constrict normally. The response in
the eye stimulated is called the direct papillary light reflex, while that in the opposite eye is
known as the indirect (or consensual) papillary light reflex.
10. The fibers of the optic tract synapse with third-order neurons in the lateral geniculate body of
the thalamus, whose axons project ipsilaterally by way of the optic radiation passing through
the posterior limb of the internal capsule to the calcarine cortex in the occipital lobe.
11. The internal arcuate fibers, the second order fibers from the nuclei gracilis and cuneatus, travel
anteromedially and cross the midplane just above the pyramid in the medulla. After crossing,
the internal arcuate fibers form the medial lemniscus.
12. Axons from the spinal ganglia ipsilaterally ascend within the fasciculus gracilis in the spinal
cord. The fasciculus gracilis transmits deep sensations from the lower half of the body (below
the fifth thoracic segment). These fibers ipsilaterally ascend in the posterior funiculus of spinal
cord toward the brainstem and terminate at the gracile nucleus, the second-order neuron.
13. Axons from the spinal ganglia ipsilaterally ascend within the fasciculus cuneatus in the spinal
cord. Fasciculus cuneatus fibers carry deep sensations from the upper body and enter the spinal
cord above the level of the fourth thoracic segment. These fibers ipsilaterally ascend in the
posterior funiculus of spinal cord toward the brainstem and terminate at the cuneatus nucleus,
the second-order neuron.
14. The second-order neurons in Rexed lamina Ⅰ, Ⅳ and Ⅴ. After traveling upl-2 spinal
segments, the second order fibers cross the midline in the anterior white commissure to ascend
in the lateral and anterior spinothalamic tracts. The lateral spinothalamic tract mediates the
sensations of pain and temperature. The anterior spinothalamic tract mediates rough touch and
pressure.
15. Those fibers from the spinal and pontine nuclei of trigeminal nerve, the second-order neurons,
cross midplane and form contralateral trigeminal lemniscus running upwards and terminating
the ventral posteromedial nucleus of dorsal thalamus, the third neurons.
Answer the Following Questions
1. (1) First-order neurons
-are located in spinal ganglia at all levels.
-give rise to the fasciculus gracilis from the lower extremity.
-give rise to the fasciculus cuneatus from the upper extremity.
-give rise to axons that ascend in the dorsal columns and terminate in the gracile and cuneate
nuclei of the medulla.
(2) Second-order neurons
-are located in the gracile and cuneate nuclei of the caudal medulla.
-give rise to axons, internal arcuate fibers that decussate and form a compact fiber bundle,
the medial lemniscus. The medial lemniscus ascends through the contralateral brainstem to
terminate in the ventral posterolateral (VPL) nucleus of the thalamus.
(3) Third-order neurons
-are located in the VPL nucleus of the thalamus.
-project via the posterior limb of the internal capsule to the postcentral gyrus, the
somatosensory cortex (Broadmann's areas 3, 1, and 2).
2. The right side of the body will lose position sense and two point discriminative touch after a left
pontine lesion involving medial lemniscus. Because the internal arcuate fibers, the second order
fibers from the nuclei gracilis and cuneatus, travel anteromedially and cross the midline just
above the pyramid in the medulla. After crossing, the internal arcuate fibers form the medial
lemniscus. The medial lemniscus ascends along the midline and posterior to pyramidal tract
through the medulla oblongata. The medial lemniscus is horizontally arranged in the pontine
tegmentum. Then it posterolaterally migrates in the midbrain to enter the ventral posterolateral
nucleus o{ the dorsal thalamus, the third-order neurons. Damage to the medial lemniscus that
represents crossed fibers will lose position sense and two point discriminative touch on the
contralateral half-body.
3. (1) First-order neurons
-are found in spinal ganglia at all levels.
-project axons into the central process to second-order neurons in the dorsal horn.
-synapse with second-order neurons in the dorsal horn.
(2) Second-order neurons
-are located in the dorsal horn.
-give rise to axons that decussate in the ventral white commissure and ascend in the
contralateral anterior and lateral funiculi.
-terminate contralaterally in the VPL nucleus of the thalamus.
(3) Third-order neurons
-are found in the VPL nucleus of the thalamus.
-project via the posterior limb of the internal capsule and corona radiata to the postcentral
gyrus, (Broadmann's areas 3, 1, and 2).
4. The first-order neurons, with their nuclei in the spinal ganglia, carry sensations from the
receptors and enter the posterolateral sulcus of the spinal cord. After entering the spinal cord,
these fibers travel up or down a few spinal segments in the dorsolateral fasciculus (Lissauer's
tract), and synapse within the posterior horns, the second-order neurons in Rexed lamina Ⅰ,
Ⅳ and Ⅴ. After traveling upl-2 spinal segments, the second order fibers cross the midline in
the anterior white commissure to ascend in the lateral and anterior spinothalamic tracts. The
lateral spinothalamic tract mediates the sensation of pain and temperature. The anterior
spinothalamic tract mediates rough touch and pressure.
5. The cell bodies of the first-order neurons are situated in the trigeminal ganglion, which receive
the superficial sensation of head and face. After the central branches of the trigeminal ganglion
enter the pons, the pain and temperature sensation terminate in the spinal nucleus of the
trigeminal nerve, while the tactile and pressure sensations terminate in the pontine nucleus of
the trigeminal nerve. Those fibers from the spinal and pontine nuclei of trigeminal nerve, the
second-order neurons, cross midplane and form contralateral trigeminal lemniscus running
upwards and terminating the ventral posteromedial nucleus of dorsal thalamus, the third
neurons. Third order sensory fibers from thalamus travel through the posterior limb of internal
capsule and terminate in lower third of postcentral gyms, the primary somesthetic area of
parietal lobe.
6. (1) Upper motor neurons (UMNs)
-are cortical neurons that give rise to corticobulbar or corticospinal tracts.
-are found in brainstem nuclei that influence lower motor neurons (LMNs).
-terminate directly on or via interneurons on LMNs.
(2) Lower motor neurons (LMNs)
-are neurons that directly innervate skeletal muscles.
-are found in the ventral horns of the spinal cord.
-are found in the motor nuclei of cranial nerves Ⅲ-Ⅶ and Ⅸ-Ⅻ.
7. Upper motor neuron (UMN) lesions
-are caused by damage to the neurons (or their axons) that innervate lower motor neurons
(LMNs).
(1) Acute stage lesions
-result in transient spinal shock, including:
a. Flaccid paralysis
lx Areflexia
c. Hypotonia
(2) Chronic stage lesions
-result in:
a. Spastic paralysis
b. Hypertonia
-occurs with increased tone in antigravity muscles (i.e., flexors of the arms and extensors of
the legs).
c. Reduction or loss of superficial abdominal and cremasteric reflexes
d. Extensor toe response (Babinski's sign)
e. Clonus
-is a repetitive, sustained MSR (e. g. , ankle clonus).
8. Lower motor neuron (LMN) lesions
-result from damage to motor neurons or their peripheral axons.
-result in:
(1) Flaccid paralysis
(2) Arefiexia
(3) Muscle atrophy
(4) Fasciculations and fibrillations
9. The proprioceptive pathway for trunk and limbs carries highly localized deep sensations (body
posture, movement, vibration and pressure as well as two-point discrimination) from receptors
to the cortex. The axons, with their first-order neurons in the spinal ganglia, reach the CNS
through the posterior roots of spinal nerves and collect sensory information from different body
areas. Axons from the spinal ganglia ipsilaterally ascend within the fasciculus gracilis or
fasciculus cuneatus in the spinal cord. The fasciculus gracilis transmits deep sensations from the
lower half of the body (below the fifth thoracic segment). Fasciculus cuneatus fibers carry deep
sensations from the upper body and enter the spinal cord above the level of fourth thoracic
segment. These fibers ipsilaterally ascend in the posterior funiculus of spinal cord toward the
brainstem and terminate at the gracile nucleus and cuneatus nucleus, the second-order neurons.
The internal arcuate fibers, the second order fibers from the nuclei gracilis and cuneatus, travel
anteromedially and cross the midline just above the pyramid in the medulla. After crossing, the
internal arcuate fibers form the medial lemniscus. The medial lemniscus ascends along the
midline and posterior to pyramidal tract through the medulla oblongata. The medial lemniscus
is horizontally arranged in the pontine tegmentum. Then it posterolaterally migrates in the
midbrain to enter the ventral posterolateral nucleus of the dorsal thalamus, the third-order
neurons. The axons carrying the information from the thalamus to the sensory cortex
through the anterior posterior limb of the internal capsule are collectively known as projection
fibers, central thalamic radiations. They terminate in the upper two-thirds of the postcentral
gyrus, the primary somesthetic area in the parietal lobe.
10. What effects would lesion in the optic nerve have on monocular blindness in the eye? A
chiasmatic lesion (often owing to a pituitary tumor or a lesion around the sella turcica) can
injure the decussating axons of retinal ganglion cells within the optic chiasm. These axons
originate in the nasal halves of the two retinas. Thus, this type of lesion produces bitemporal
hemianopsia, characterized by blindness in the lateral or temporal half of the visual field for
each eye. A pathology encroaching on the lateral edge of the optic chiasm selectively interrupts
the fibers from the ipsilateral temporal portion of the retina, resulting in nasal hemianopsia in
the corresponding eye. Lesions behind the optic chiasm cause a field defect in the temporal field
of one eye, together with a field defect in the nasal (medial) field of the other eye. The result is
a homonymous hemianopsia in which the visual field defect is on the side opposite the lesion.
11. The light stimulus acting upon the retinal photoreceptors gives rise to activity in retinal
ganglion cells, the axons of which form the optic nerve. Activity is conducted through the optic
chiasma and along the optic tract, and the majority of fibers end in the lateral geniculate nucleus
of the thalamus. However, a small number of fibers leave the optic tract before it reaches the
thalamus and synapse in the pretectal area by short neurons which synapse bilaterally with
preganglionic parasympathetic neurons in the accessory nucleus of oculomotor nerve
(Edinger-Westphal's nucleus) complex in the rostal midbrain. Efferent impulses pass along
parasympathetic fibers of the oculomotor nerve to the orbit where they synapse in the ciliary
ganglion. Postganglionic fibers (short ciliary nerve) pass to the eyeball to supply sphincter
pupillae, which reduces the size of the pupil when it contracts. So light shone on the retina of
one eye causes both pupils to constrict normally. The response in the eye stimulated is called the
direct papillary light reflex,, while that in the opposite eye is known as the indirect (or
consensual) papillary light reflex.
12. The corticospinal tract originates the pyramidal cells of the superior and middle parts of the
precentral gyrus and anterior part of the paracentral lobule. It travels through the posterior limb
of internal capsule of the telencephalon, and then descends through the intermediate 3/5 of the
cerebral peduncles of the midbrain. Later it runs through the basilar part of the pons and
continues to the pyramids in the medulla oblongata. About 75% to 90% of the corticospinal
tract crosses pyramidal decussation at the caudal end of the medulla oblongata. After crossing,
these fibers descend into the lateral corticospinal tract in the lateral funiculus of the spinal cord.
These fibers terminate on the interneuron and lower motor neurons in the ipsilateral anterior
gray horns of the spinal cord, and mainly control musculature of the limbs. About 10% to 15 %
of the corticospinal tract does not cross in the pyramidal decussation but descends in the
anterior funiculus of the spinal cord (the anterior corticospinal tract). These fibers cross or
uncross above the level of mid-thoracic segments, close to the anterior horns of spinal cord, and
control the skeletal muscles of trunk. Up to 3% of the decending fibers in the lateral
corticospinal tract are uncrossed. The ipsilateral decending projections mainly control
musculature of trunk. Some axons of upper motor neurons (the pyramidal cells in the inferior
part of the precentral gyrus) that conduct impulses for the control of skeletal muscles in the
head extend through the genu of the internal capsule to the midbrain, where they join the
corticobulbar tracts in the right and left of cerebral peduncles. Some of the axons in the
corticobulbar tracts decussated at the midplane in the brainstem, whereas others have not. The
axons terminate the bilateral motor nuclei of the cranial nerves (nucleus of oculomotor nerve,
nucleus of trochlear nerve, motor nucleus of trigeminal nerve, nucleus ambiguus, accessory
nucleus, superior part of the facial nucleus), and the contralateral hypoglossal nucleus and the
inferior part of the facial nucleus.
(Li Feng, Ding Wenlong)
Multiple Choice Questions
A1
1. An obstruction in the interventricular foramen would interfere with the flow of cerebrospinal
fluid into the
.
A. lateral ventricle
B. third ventricle
C. fourth ventricle
D. subarachnoid space of the spinal cord
E. central canal of the spinal cord
2. The cerebral aqueduct links the
.
A. lateral ventricles
B. lateral ventricles and 3rd ventricle
C. 3rd and 4th ventricles
D. lateral ventricles and 4th ventricle
E. 3rcl ventricle and central canal of the spinal cord
3. Which is not associated with cerebrospinal fluid?
A. cerebral aqueduct
B. lateral ventricle
C. choroid plexus
D. subarachnoid space
E. epidural space
4. The region around the spinal cord that contains cerebrospinal fluid is the
.
A. central canal
B. subarachnoid space
C. epidural space
D. 4th ventricle
E. 3rd ventricle
5. Which of the following is true statement relating to the meninges?
A. The dura mater is composed of a dense fibrous connective tissue.
B. The dura mater around the brain is single-layered
C. Venous sinuses occur between the two dural layers around the spinal cord.
D. The most superficial meninx is the arachnoid membrane.
E. The epidural space contains cerebrospinal fluid.
6. Which of the following arteries is not included in the circle of Willis?
A. the anterior communicating artery
B. the anterior cerebral artery
C. the middle cerebral artery
D. the posterior communicating artery
E. the posterior cerebral artery
7. Regarding the meninges, which of the following is false?
A. The dura mater is sensitive to pain.
B. Rupture of an aneurysm of a cerebral artery leads to subarachnoid hemorrhage.
C. A torn middle meningeal artery bleeds into the space between the endosteal dura and the
bones of the cranium.
D. The cerebellomedullary cistern is a subarachnoid space between the medulla and the cerebellum.
E. The diaphragma (tentorium) sellae is a double layer of arachnoid mater.
8. The meninges, from the outermost to the innermost, are
.
A. the dura mater, the pia mater, the arachnoid
B. the pia mater, the dura mater, the arachnoid
C. the arachnoid, the pia mater, the dura mater
D. the dura mater, the arachnoid, the pia mater
E. the pia mater, the arachnoid, the dura mater
9. Lumbar punctures are performed into the
space in order to remove cerebrospinal
fluid.
A. subdural
B. subarachnoid
C. ventricles
D. epidural
E. central canal
10. The cerebrospinal fluid is secreted by
in the ventricles.
A. arachnoid granulations
B. choroid plexus
C. pia mater
D. arteries
E. veins
11. CSF is mainly produced within the
.
A. central canal
B. dural sinuses
C. brain ventricles
D. cerebral aqueduct
E. subarachnoid space
12. Which is false about the veins of brain?
A. The cerebral veins do not run together with the arteries.
B. It divided into superficial and deep groups.
C. The superficial vein empty into the straight sinus.
D. The superficial vein drain the blood from cerebral cortex~
E. The deep vein empty into the straight sinus.
13. The
allow(s) CSF to flow from the third ventricle to the fourth ventricle.
A. dural sinuses
B. arachnoid granulations
C. choroid plexuses
D. cerebral aqueduct
E. median and lateral apertures
14. The
is/are directly involved in the re. absorption of CSF.
A. central canal
B. arachnoid granulations
C. choroid plexuses
D. cerebral aqueduct
E. median and lateral apertures
15. Which of the following is not associated with the cranial meninges?
A. epidural space containing fats and veins
B. dura mater continuous with the periosteal layer
C. venous sinuses
D. pia mater containing blood vessels
E. arachnoid granulations
16. Which statement about blood vessels of the brain is true?
A. The middle cerebral artery is a branch of the internal carotid artery.
B. The great cerebral vein (of Galen) drains into the superior sagittal sinus.
C. The posterior cerebral artery is a branch of the vertebral artery.
D. The basilar artery enters the cranial cavity through the foramen magnum.
E. The anterior communicating artery connects the two middle cerebral arteries.
17. What is not there in the cavernous sinus?
A. the internal carotid artery
B. the internal jugular vein
C. the abducent nerve
D. the oculomotor nerve
E. the trochlear nerve
18. What are the main sources of cerebrospinal fluid?
A. the choroid plexuses
B. the cerebral pia mater
C. the internal carotid artery
D. the internal jugular vein
E. the vertebral artery
19. Which is true about the middle cerebral artery?
A. It supplies little of the dorsolateral surface of the cerebral hemisphere.
B. It arises from the subclavian artery.
C. It forms the cerebral arterial circle.
D. It runs horizontally to the lateral sulcus.
E. It descends to the lateral sulcus.
20. Which is true about the brain barrier?
A. Its function is to stable the environment surrounding the neurons.
B. It is composed of four parts.
C. It is just in between blood and neuron in the brain and spinal cord.
D. It consists the capillary endothelium homogenous basement and neuroglial membrane.
E. All areas in brain have a blood-brain barrier.
21. A person develops a cavernous sinus thrombosis. Because of its relationship to the sinus,
which cranial nerve might be affected?
A. abducent
B. facial
C. mandibular V3
D. olfactory
E. optic
22. The inferior sagittal sinus is found in the free edge of what structure?
A. diaphragma sellae
B. cerebellar falx
C. cerebral falx
D. ilium terminale
E. tentorium of cerebellum
23. Which statement about ventricles is false?
A. They contain cerebrospinal fluid.
B. They contain the choroid plexus.
C. The inferior horns of the lateral ventricles project into the temporal lobes.
D. The caudate nucleus contributes to the walls of the lateral ventricle.
E. The interventricular foramen connects the third with the fourth ventricle.
24. Which statement about the epidural space is false?
A. It is a space between the dura and the periosteum of the vertebral canal.
B. It contains a quantity of loose areolar tissue.
C. It contains lymphatic vessels and venous plexuses.
D. The spinal nerves on each side pass through it.
E. There is cerebrospinal fluid in the space.
25. Which statement about the cerebral dura mater is true?
A. It is the inner periosteum of the skull.
B. It is in closely connected with the calvaria.
C. It is loosely attached at the base of skull.
D. Cerebrospinal fluid may leak out from the nose or ear with a fracture of the base of skull.
E. Sometimes, it is also the arachnoid.
26. The most likely source of blood in a patient with an epidural hemorrhage is the
.
A. vertebral artery
B. middle meningeal artery
C. superior cerebral veins
D. anterior cerebral artery
E. circle of Willis
27. The patient's ventricle was injected into trypan blue and observed that the CSF turns blue but
the brain tissue does not. This represents an example of
.
A. blood-brain barrier
B. blood-CSF barrier
C. CSF-brain barrier
D. CSF-blood barrier
E. blood-blood barrier
28. Which of the following is/are true of the blood brain barrier (BBB)?
A. The BBB is located around the cerebral arterial circle and cerebral arteries.
B. The BBB is located on the walls of the brain ventricles and is involved in the production of
cerebrospinal fluid.
C. The BBB is composed of neuroglial cells called astrocytes.
D. The BBB controls the passage of chemical substances into the neurons of the brain.
E. The BBB is permeable to oxygen, carbon dioxide, and glucose.
29. Which artery is not formed the cerebral arterial circle?
A. the anterior cerebral arteries
B. the posterior inferior cerebellar arteries
C. the posterior cerebral arteries
D. a short segment of internal carotid arteries
E. the posterior communicating arteries
30. The direction of the blood flow in the sinuses of dura mater is as follows
.
A. inferior sagittal sinus-transverse sinuses-confluence of the sinuses-internal jugular vein
B. cavernous sinus--inferior petrosal sinus-internal jugular vein
C. superior sagittal sinus-inferior petrosal sinus-internal jugular vein
D. sigmoid sinus--transverse sinuses-confluence of the sinuses-internal jugular vein
E. straight sinus--superior petrosal sinus--internal jugular vein
31. The internal capsule is mainly supplied by the
.
A. direct branches of the internal carotid arteries
B. direct branches of the middle cerebral arteries
C. direct branches of the cerebral arterial circle
D. direct branches of the posterior cerebral arteries
E. direct branches of the basilar artery
32. Which is not the branch of the basilar artery?
A. the anterior inferior cerebellar artery
B. the middle cerebral arteries
C. the posterior cerebral arteries
D. the artery to the labyrinth
E. the superior cerebellar artery
33. Which arteries supply blood to the medial portions of the frontal and parietal lobes?
A. the anterior inferior cerebellar artery
B. the anterior cerebral arteries
C. the posterior cerebral arteries
D. the middle cerebral arteries
E. the superior cerebellar artery
34. The occipital lobe is supplied by the
.
A. middle cerebral artery
B. posterior cerebral artery
C. anterior inferior cerebellar artery
D. superior cerebellar artery
E. anterior cerebral artery
35. The auditory and language cortical areas is supplied by the
.
A. posterior communicating artery
B. middle cerebral artery
C. posterior cerebral artery
D. superior cerebellar artery
E. anterior cerebral artery
36. All of the following statements pertain to the spinal pia mater except:
A. It forms one of the boundaries of the subarachnoid space.
B. It forms the filum terminale.
C. It is adherent to the spinal cord.
D. It supports the anterior and posterior spinal arteries.
E. It terminates at the foramen magnum.
37. Cerebrospinal fluid enters the subarachnoid space at the
.
A. arachnoid granulations
B. choroids plexuses
C. median and lateral apertures of the fourth ventricle
D. interventricular foramen
E. mesencephalic aqueduct
38. Cerebrospinal fluid enters the venous system
.
A. at arachnoid granulations
B. at the cisterna magna
C. through subarachnoid veins
D. via capillaries in the central nervous system
E. through mesencephalic aqueduct
39. All of the following arteries are branches of the internal carotid artery except the
A. anterior cerebral arteries
.
B. middle cerebral arteries
C. ophthalmic arteries
D. posterior cerebral arteries
E. posterior communication arteries
A2
1. Blockage of the flow of cerebrospinal fluid (CSF) within the cerebral aqueduct (of Sylvius)
normally would result in the enlargement of all of the following ventricular spaces except
the
.
A. third ventricle
B. fourth ventriclee
C. right lateral ventricle
D. interventricular foramen
E. left lateral ventricle
2. During childbirth, an excessive anteroposterior compression of the head may tear the anterior
attachment of the cerebral falx from the tentorium of cerebellum. The bleeding that follows is
likely to be from which of the following venous sinuses?
A. occipital sinus
B. sigmoid sinus
C. straight sinus
D. superior sagittal sinus
E. transverse sinus
3. An infant was found to have hydrocephalus. Studies revealed that the hydrocephalus was caused
because CSF could not get out of the third ventricle. The blocked passage was the
.
A. central canal
B. cerebral aqueduct
C. interventricular foramen
D. lateral foramen
E. medial foramen
4. A patient is suspected of having bacterial meningitis. A lumbar puncture is performed to remove
cerebrospinal fluid for analysis. The fluid would be removed from the
.
A. epidural space
B. subarachnoid space
C. intervertebral foramen
D. spinal canal
E. subdural space
5. It is decided to image the spinal cord and spinal nerve rootlets by doing a myelogram (injection
of a radio-opaque dye into the subarachnoid space followed by a radiograph). In order to inject
the dye without injury to the spinal cord, the injection is usually done below which vertebral
level?
A. C7
B. T12
C. IA
D. S2
E. L1
6. You have been asked to assess the neurological deficit that might exist in a patient diagnosed
with cavernous sinus thrombosis. You will focus your examination on cranial nerves related to
the sinus that includes all the following except
.
A. abducent (CN Ⅵ)
B. facial (CN Ⅶ)
C. oculomotor (CN Ⅲ)
D. ophthalmic division of the trigeminal nerve (CN Ⅵ)
E. trochlear (CN Ⅳ)
7. While riding her bicycle on campus without a helmet a student is hit by a car and falls, hitting
her head on the pavement. She is brought to the Emergency Room in an unconscious state with
signs of a closed head injury. Tests reveal blood in her cerebrospinal fluid taken from a spinal
tap. Diagnosis is of torn cerebral veins as they pass from the brain to the superior sagittal sinus.
From which of the following was the bloody fluid taken?
A. cavernous sinus
B. epidural space
C. subarachnoid space
D. subdural space
E. vertebral venous plexus
8. A 35-year-old man was admitted to the hospital complaining of double vision (diplopia),
inability to see close objects, and blurred vision in the fight eye. A vertebrohasilar angiogram
revealed an aneurysm of the superior cerebellar artery close to its origin on the right side. The
doctor attributed the symptoms to the compression of an adjacent cranial nerve by the aneurysm
The compressed nerve is the
.
A. abducent (CN Ⅵ)
B. oculomotor (CN Ⅲ)
C. optic (CN Ⅱ)
D. trigeminal (CN Ⅴ)
E. trochlear (CN Ⅳ)
9. During an intramural baseball game a player is hit on the head, between the eye and the ear. He
immediately loses consciousness, wakes up momentarily and then becomes comatose. He is
rushed to the ER and immediately given a CT scare The scan shows a skull fracture and an
accumulation of blood between the dura and the cranial bone on the side of his head,
compressing his cerebrum. He is rushed to surgery where a hole is bored into his skull to relieve
the pressure. After a few tense hours, he regains consciousness and has an uneventful recovery.
The hemorrhage from the fracture would be described as
A. epidural
B. intracerebral
C. subaponeurotic
D. subarachnoid
E. subdural
10. An 84-year old woman suffers a stroke, with paralysis on the right side of her body.
Neurological tests show that the intracerebral hemorrhage has interrupted the blood supply to
the posterior part of the frontal, the parietal and medial portions of the temporal lobes of the
left cerebral hemisphere. Which vessel was involved?
A. anterior cerebral artery
B. great cerebral vein
C. middle cerebral artery
D. middle meningeal artery
E. posterior cerebral artery
B1
A. contains cerebrospinal fluid
B. innermost layer of the spinal meninges
C. contains a quantity of loose areolar tissue
D. sensitive to pain
E. dense fibrous connective tissue
1. pia mater
2. subarachnoid space
A. the posterior communicating artery
B. the anterior cerebral artery
C. the middle cerebral artery
D. the anterior choroidal artery
E. the posterior cerebral artery
3. Which is not included in the circle of Willis?
4. Which is not the branch of internal carotid arteries?
A. the arachnoid granulations
B. the denticulate ligament
C. the subarachnoid space
D. the choroids plexuses
E. the tentorium of cerebellum
5. Which is formed by the cerebral dura mater?
6. Which is formed by the arachnoid?
7. Which is formed by the spinal pia mater?
A. the superior sagittal sinus
B. the straight sinus
C. the transverse sinuses
D. the inferior sagittal sinus
E. the cavernous sinus
8. Which lies along the superior border of the cerebral falx?
9. Which lies along the posterior margin of the tentorium of cerebellum?
10: Which is situated laterally to the body of the sphenoid bone?
True or False Questions
1. A blockage in the cerebrospinal fluid circulation leads to hydrocephalus.
(
)
2. The lateral ventricles are located near the cerebellum.
(
)
3. Blockage of the flow of cerebrospinal fluid (CSF) within the cerebral aqueduct (of Sylvius)
normally would result in the enlargement of the fourth ventricle.
(
)
4. The presence of blood in a spinal tap taken from an individual with a closed head injury signals
arterial bleeding into the subarachnoid space.
(
)
5. The spinal dura mater terminates at the level of the second lumbar vertebra.
(
)
6. The vertebral venous plexus is found in the subarachnoid space.
(
)
7. Epidural hematoma is usually caused by leakage from a cerebral vein.
(
)
Explanation of Terms
1. epidural space
2. Sinuses of the dura mater
3. subarachnoid space
4. cerebral arterial circle
5. blood-brain barrier (BBB)
6. superior petrosal sinus
Answer the Following Questions
1. How does the pia mater differ from the arachnoid mater in covering the brain?
2. What is the arterial circle of Willis?
ANSWERS
Multiple Choice Questions
A1
1. B 2. C 3. E 4. B 5. A 6. C 7. E 8. D 9. B
10. B 11. C 12. C
13. D 4. B 15. A 16. A 17. B 18. A 19. D
20. A 21. A 22. C
23.
E 24. E
25. D 26. B 27. C
28.
E 29. B 30. B 31.
B
32. B 33. B 34. B 35. B 36. E 37. C 38. A
39. D
A2
1. B 2. C
3. B
4. B
5. C 6. B
7. B
8. B
9. A
10. C
BI
1. B 2. A 3. D 4. E 5. E 6. A 7. B 8. A 9. C 10. E
True or False Questions
1. T 2. F 3. F 4. T 5. F 6. F 7. F
Explanation of Terms
1. The space between the walls of the vertebral canal and the dura mater of the spinal cord. Within
it there is fat, loose connective tissue, and vertebral venous plexuses.
2. The sinuses of the dura mater are venous channels which drain the blood from the brain; they
are devoid of valves, and are situated between the two layers of the dura mater and lined by
endothelium continuous with that which lines the veins.
3. The space between the arachnoid and pia mater, filled with cerebrospinal fluid. The large blood
vessels supplying the brain and spinal cord lie in the subarachnoid space.
4. The cerebral arterial circle on the ventral aspect of the brain; formed by the anterior
communicating artery, the two anterior cerebral, the two internal carotid, the two posterior
communicating, and the two posterior cerebral arteries.
5. The BBB is just between the blood and neurons in the brain and spinal cord, and consists of the
capillary endothelium, capillary basement membrane and the neuroglial membrane formed by
the perivascular processes of the astrocytes.
6. The superior petrosal sinus connects the posterior end of the cavernous sinus to the bend
marking the transition between the transverse and sigmoid sinuses. It receives cerebellar and
inferior cerebral veins and veins from the tympanic cavity.
Answer the Following Questions
1. The pia mater on the brain is a delicate, intimate, areolar investment of brain and spinal cord
that enmeshes the blood vessels on their surfaces. It is a vascular membrane. On the other hand,
the arachnoid is a delicate transparent membrane composed of a blend of collagenous and
elastic fibers and squamous mesenchymal epithelial cells. It is not vascular and is not attached
directly to the brain or spinal cord.
2. The cerebral arterial circle, is an important anastomosis at the base of the brain between the
following arteries:
Posterior cerebral arteries;
Posterior communicating arteries;
Internal carotid arteries;
Anterior cerebral arteries;
Anterior communicating arteries.
The various components of the cerebral arterial circle give many small branches to the brain.
(Wang Jun)
Multiple Choice Questions
A1
1. The endocrine system
.
A. releases chemicals into the bloodstream for distribution throughout the body
B. releases hormones that alter the metabolic activities of many different tissues and organs
C. produces effects that can last for hours, days, or even longer
D. control the body processes with nervous system
E. all of the above
2. Another system that works closely with the endocrine system to control body processes is
the
.
A. circulatory system
B. nervous system
C. urinary system
D. digestive system
E. respiratory system
3. The endocrine system controls homeostasis
.
A. rapidly via nerve impulses
B. gradually via nerve impulses
C. rapidly by secreting hormones
D. gradually by secreting hormones
E. rapidly by secreting enzymes
4. Endocrine glands are often referred to as
.
A. ducted glands
B. digestive glands
C. enzyme-producing glands
D. ductless glands
E. respiratory glands
5. Endocrine glands that compose the endocrine system
.
A. produce hormones that are secreted into the digestive tract .
B. release hormones into the bloodstream
C. release hormones as rapidly as nerve impulses are transmitted
D. are present only in humans
E. are present in the internal cavity
6. Secretions from ductless glands are called
.
A. enzymes
B. digestive fluids
C. hormones
D. excretory fluids
E. interstitial fluid
7. Which of the following exerts control over all the others?
A. adrenal cortex
B. thyroid gland
C. parathyroid
D. hypophysis
E. testis
8. The gland, formerly called the "master gland" that controls many other endocrine glands in
the
.
A. thymus
B. thyroid gland
C. suprarenal gland
D. hypophysis
E. testis
9. The sella turcica that supports the hypophysis is located in which bone?
A. the ethmoid bone
B. the frontal bone
C. the sphenoid bone
D. the occipital bone
E. the temporal bone
10. The two basic divisions of the hypophysis are
.
A. neurohypophysis and infundibulum
B. pars distalis and pars intermedia
C. supraoptic and paraventricular nuclei
D. adenohypophysis and neurohypophysis
E. pars distalis and pars tuberalis
11. Oxytocin is secreted by the
.
A. adenohypophysis
B. neurohypophysis
C. zona glomerulosa
D. pars intermedia .
E. cervix
12. Reabsorption of water from the kidneys is regulated by hormones produced in the
.
A. adenohypophysis
B. thyroid
C. pancreas
D. hypothalamus
E. pineal body
13. The thyroid gland
.
A. is located above the larynx
B. consists of the lateral and pyramidal lobes
C. isthmus crosses the middle line in front of fifth to sixth tracheal rings
D. is surrounded by a fibrous capsule and a synovial capsule
E. secretes thyroxine and calcitonin
14. The parathyroid glands are located
.
A. below the thyroid, hence the name "para"
B. above the thyroid, hence the name "para"
C. near the thyroid, hence the name "para"
D. imbedded in the posterior surface of the thyroid gland
E. distant from the thyroid but named because there are two and they resemble the thyroid
glands.
15. The suprarenal glands consist of
.
A. the inner and outer layer of the kidney
B. the medulla and the cortex
C. the lower and upper sections
D. the anterior and posterior sections
E. the left and right lobe
16. The suprarenal cortex does not secrete
.
A. mineralocorticoids
B. progesterone
C. glucocorticoids
D. androgens
E. gonadocorticoids
17. The inner portion of the suprarenal glands
.
A. produces hormones that help the body deal with long-term stress
B. is called the suprarenal cortex
C. is the source of cortisol
D. produces epinephrine and norepinephrine
E. produces androgens
18. The
gland produces the hormone melatonin.
A. hypophysis
B. pineal
C. thyroid
D. pancreatic
E. thymus
19. The endocrine gland responsible for the body' s circadian rhythm is the
.
A. thymus
B. pineal
C. parathyroid
D. hypophysis
E. suprarenal
20. What gland is part of both the endocrine system and the digestive system?
A. hypophysis
B. pancreas
C. testis
D. thymus
E. suprarenal gland
21. The pancreatic islets produce
.
A. thyroxin
B. cortisone
C. insulin
D. somatotropin
E. calcitonin
22. The
gland is known for playing a major role in immunity.
A. thyroid
B. pineal
C. thymus
D. suprarenal
E. hypophysis
23. The thymus aids the differentiation of
cells.
A. red blood
B.B
C.T
D. cancerous
E. nerve
24. Androgens are produced by the
.
A. ovary
B. testis
C. thyroid
D. hypophysis
E. thymus
25. Which of the following is mismatched?
A. oxytocin-uterus
B. parathyroid hormone-bones
C. ADH-kidneys
D. insulin-hypothalamus
E. thyroxine-hypophysis
26. Two glands are especially noted for regressing after childhood, namely the
A. thymus and pineal gland
B. suprarenal cortex and medulla
C. thyroid and parathyroid
D. thymus and thyroid
E. testis and ovary
A2
1. Someone who has suffered damage to the pancreas might
.
.
A. have trouble walking a straight line
B. be unable to remember a recent event
C. have periods of very low energy
D. have trouble making a fist
E. be always cold
2. Dwarfism may result from a bloodstream that has
.
A. too much epinephrine
B. too little growth hormone
C. too little antidiuretie hormone
D. too much thyroxine
E. too much insulin
3. Failure of the pituitary to stop producing growth hormone after the body growth is completed
results in
.
A. acromegaly
B. giantism
C. tetany
D. kidney failure
E. hyperthyroidism
B1
Match the hormone with the primary agent that stimulates its secretion.
A. epinephrine
B. TSH
C. calcitonin
D. ACTH
E. growth hormone
1. thyroxine
2. corticosteroids
Match each item with the most closely related item.
A. parathyroid gland
B. thymus gland
C. suprarenal cortex
D. endocrine pancreas
E. hypophysis
3. secretes androgens, mineralocorticoids, and glucocorticoids
4. atrophies by adulthood
True or False Questions
1. The endocrine system is quicker than the nervous system.
(
)
2. Hormones are secreted by exocrine glands and affect nearby ceils.
(
)
3. Endocrine glands secrete hormones into the bloodstream for transport to target organs. ( )
4. It is possible to find all the hormones in a blood sample taken from the arm.
(
)
5. The hypothalamus is controlled by the pituitary gland.
(
)
6. The hypophysis is located within the hypophyseal fossa of the ethmoid bone, and is attached to
the thalamus by the infundibulum.
(
)
7. The hypophysis is divided into two portions: the cortex pituitary and the medulla pituitary.
(
8. Hyper-secretion of growth hormone may lead to "gigantism."
9. The thyroid gland is attached to the trachea just above the larynx.
10. The adrenal medulla is an endocrine gland.
11. Melatonin is involved with circadian rhythms.
12. Insulin is a hormone secreted by the adrenal cortex.
13. The pancreas has both exocrine and endocrine tissue.
14. The pancreatic endocrine tissues are called pancreatic islets.
15. The thymus increases in size with aging.
16. Male sex hormones can be produced by the testes or adrenal glands.
(
Explanation of Terms
1. endocrine glands
2. adenohypophysis
3. neurohypophysis
4. anterior lobe of hypophysis
5. posterior lobe of hypophysis
6. thyroid capsule
7. isthmus of thyroid gland
8. suprarenal cortex
9. suprarenal medulla
10. pancreatic islets
Answer the Following Questions
1. Which organ and tissue consist of the endocrine system?
2. Differentiate between endocrine and exocrine glands.
3. Describe the location and lobes of the hypophysis.
4. Describe the location, lobes and capsule of the thyroid gland.
5. Distinguish between the adrenal medulla and the adrenal cortex.
6. Describe the location and general function of the pineal gland.
7. Describe the location and structure of the pancreatic islets.
8. Describe the location and general function of the thymus gland.
9. Describe the endocrinal secretions produced by testis or ovary.
ANSWERS
Multiple Choice Questions
A1
1. E 2. B
3. D 4. D 5. B
6. C
7. D
8. D
9. C
10. D
12. D 13. B 14. D 15. B 16. B 17. D 18. B
19. B
21. C
22. C
23. C
24. B
25. D
26. A
Al
1. C 2. B 3. A
B1
1. B
2. D
3. C
4. B
True or False .Questions
1. F 2. F 3. T 4. T 5. F 6. F 7. F 8. T 9. F 10. T 11. T
13. T 14. T
15. F
16. T
)
(
(
(
(
(
(
(
(
)
)
)
)
)
)
)
)
)
11. B
20. B
12. F
Explanation of Terms
1. Endocrine glands are ductless; they secrete specific chemicals called hormones directly into the
blood or surrounding interstitial fluid; their action is relatively slow and prolonged.
2. The adenohypophysis includes the pars distalis, the pars tuberalis, and the pars intermedia.
3. The neurohypophysis consists of the pars nervosa and the pars infundibulum.
4. Anterior lobe of hypophysis consists of the pars distalis and the pars tuberalis and secretes
growth hormone, somatotropin, thyrotropin, adrenocorticotropin and gonadotropin, etc.
5. Posterior lobe of hypophysis constitutes in the pars intermedia and the pars nervosa. The lobe
stores and releases the vasopressin and oxytocin.
6. The thyroid gland is surrounded by a fibrous capsule (true capsule) external to this is a sheath of
pretracheal fascia (false capsule). Between the true and false capsules, there are parathyroid
glands and blood vessels and nerves of the thyroid.
7. The isthmus is a narrow portion which connects anteriorly thyroid right and left lobes. The
isthmus crosses the middle line in front of second to fourth tracheal ring.
8. Suprarenal cortex is an outer portion of suprarenal gland on section and produces
mineralocorticoids, glucocorticoids, and gonadocorticoids.
9. Suprarenal medulla is an inner portion of suprarenal gland on section and secretes epinephrine
and norepinephrine.
10. Pancreatic islets are most common in the tail of the pancreas and endocrine portion of the
pancreas which consists of scattered clusters of cells.
Answer the Following Questions
1. The glands and tissues of endocrine system include the pituitary body, the thyroid, the
parathyroids, the suprarenals, pancreas, the pineal body, the thymus and the gonads etc.
2. In contrast to exocrine glands, endocrine glands are ductless; they secrete specific chemicals
called hormones directly into the blood or surrounding interstitial fluid.
3. The hypophysis is located on the inferior aspect of the brain in the region of the diencephalon
and is attached to the end of the infundibulum of the brain and resting in the hypophyseal fossa.
The hypophysis is divided into an adenohypophysis and a neurohypophysis. The
adenohypophysis includes the pars distalis, the pars tuberalis, and the pars intermedia. The
neurohypophysis consists of the pars nervosa and the pars infundibulum.
4. The thyroid gland is located in the anterior neck, just below the larynx. It consists of right and
left lateral lobes, thyroid isthmus. The thyroid gland is surrounded by a fibrous capsule (true
capsule) external to this is a sheath of pretracheal fascia (false capsule). Between the true and
false capsules, there are parathyroid glands and blood vessels and nerves of the thyroid.
5. On section, the suprarenal cortex is an outer portion and the suprarenal medulla is an inner
portion. The suprarenal cortex produces mineralocorticoids, glucocorticoids, and
gonadocorticoids. The suprarenal medulla secretes epinephrine and norepinephrine.
6. The pineal body lies in the depression between the superior colliculi. It is attached to the roof of
the third ventricle near its junction with the mid-brain. Secretion of its principal hormone is
melatonin which is thought to affect the hypothalamus by stimulating the secretion of certain
releasing factors. These factors in turn affect the secretion of gonadotrophin and the ACTH
from the adenohypophysis.
7. Pancreatic islets are most common in the tail of the pancreas and consist of scattered clusters of
cells. The endocrine function of the pancreas is to produce and secrete the hormones glucagon
and insulin.
8. The thymus is a bilobed organ positioned in the upper mediastinum, in front of the aorta and
behind the manubrium of the sternurm The thymus is associated with the lymphatic system in
maintaining body immunity through the maturation and discharge of T cells. The thymus also
secretes a hormone called thymosin, which is believed to stimulate the T cells after they leave
the thymus.
9. The interstitial cells of the testes produce and secrete the male sex hormone testosterone.
Estrogens are produced in the ovarian follicles and corpus luteum of the ovaries. Progesterone
is produced by the corpus luteum.
(Xu Jiajun)